Summative Exam
A 55-year-old travel journalist presents with a 2-day history of hemoptysis. He has had a chronic cough, which he attributed to his 40-pack-year history of cigarette smoking. He also reports that he has been bruising easily. On examination, he has moon facies and truncal obesity, with wasting of his shoulder muscles. He has several red-purple striae over his anterior abdominal wall. The lungs are clear to auscultation. Laboratory results are as follows: Hemoglobin 12g/dl, white blood cell count 8 x 103/mm3, platelets 160 x 103/mm3, sodium 138mmol/L, potassium 3.0 mmol/L. Tuberculin skin test is negative. Sputum cytology is positive for carcinoma. What is the most likely diagnosis? 1. Lambert-Eaton myasthenic syndrome 2. Hypertrophic pulmonary osteoarthropathy 3. Horner's syndrome 4. Ectopic adrenocorticotropic hormone (ACTH) secretion 5. Syndrome of inappropriate antidiuretic hormone secretion (SIADH)
Correct answer: Ectopic adrenocorticotropic hormone (ACTH) secretion Cushing's syndrome may develop as a result of the small cell carcinomas secreting ectopic adrenocorticotropic hormone (ACTH). Patients may complain of bruising easily. On examination, they may have red-purplish striae, especially over their abdomen, in addition to the typical moon facies and truncal obesity. They may have muscle wasting, especially of the proximal limb girdle muscles. Laboratory investigations reveal hypokalemia and high plasma ACTH, as well as increased serum and urine cortisol. The syndrome of inappropriate antidiuretic hormone secretion (SIADH) is a paraneoplastic syndrome associated with bronchogenic carcinoma. There is excessive water reabsorption by the kidneys due to the elevated antidiuretic hormone (ADH). Patients may complain of feeling weak and lethargic. Other symptoms include confusion and coma. Laboratory tests reveal hyponatremia, high urine sodium, low serum osmolality, and high urine osmolality in a euvolemic patient. In cases of hypertrophic pulmonary osteoarthropathy (HPO), there is formation of new subperiosteal cancellous bone at the distal ends of long bones. Patients present with periarticular pain, polyarthralgia, and painful swellings at the wrist, knees, elbows, and ankles. On examination, there is clubbing of the fingers and toes. There may also be localized articular erythema, tenderness, swelling, and even effusions. X-rays may reveal subperiosteal bone formation with periosteal elevation that is seen as thickening and detachment of the periosteum. Lambert-Eaton myasthenic syndrome is an immune-mediated disorder of neuromuscular transmission. It results in impaired release of acetylcholine from nerve terminals. Patients complain of weakness of the scapular and pelvic girdles musclesand the resultant difficulties in performing activities such as climbing stairs. Other symptoms such as dry mouth, paresthesias, and sexual impotence due to autonomic dysfunction may also be present. On examination, there is proximal muscle weakness. Deep tendon reflexes are decreased or absent. Horner's syndrome is due to a bronchogenic carcinoma tumor mass extending to the sympathetic chain. Horner's syndrome symptoms include miosis, ptosis, enophthalmos, and hemianhidrosis. In addition, the patients may also have atrophy of hand muscles.
A 42-year-old woman was diagnosed with deep vein thrombosis of the left leg 3 weeks ago, and therapy was initiated with heparin. History includes smoking and birth control pills for endometriosis several years ago. On exam lungs show a decrease in air entry bilaterally. Which of the following is most likely?? 1. Lupus anticoagulant 2 Oral contraceptive induced thrombosis 3 Trousseau's syndrome 4 Heparin induced thrombosis 5 Paroxysmal nocturnal hemoglobinuria (PNH)
Trousseau's syndrome, or migratory thrombophlebitis, is a malignancy associated hypercoagulable state that is characterized by a recurrent thrombosis in a migratory pattern and involvement of superficial veins in unusual sites. It is generally associated with an occult neoplasm (50%), usually an adenocarcinoma. This patient has an extensive smoking history and possibly has an adenocarcinoma of the lung. Several mechanisms for enhanced thrombosis by tumor cells have been proposed: (1) release of a thromboplastic tissue factor, which activates the extrinsic clotting pathway (2) activated protein C resistance (3) tumor cell membranes causing direct activation of the platelets (4) tumor cells releasing a procoagulant, which activates factor X directly (5) the indirect effects on tissue factor, thrombomodulin, and other factors by cytokines derived or induced by tumor cells. Treatment is with heparin, since warfarin is ineffective, and is continued till the malignancy has been adequately eradicated.
A 48-year-old man with hypertension and coronary artery disease presents with protracted fever, fatigue, anorexia, weight loss, night sweats, and non-specific, non-radiating joint pains. These symptoms began insidiously following a routine dental cleaning, but they have progressed over the past 4 weeks. He denies any chills, myalgias, sore throat, palpitations, shortness of breath, pleurisy, cough, wheezing, abdominal pain, nausea, vomiting, diarrhea, peripheral edema, trauma, travel, insect bites, or sexual contact within the past year. His physical exam is remarkable for a fever of 101.3°F. His fundoscopic examination is notable for cytoid bodies and hemorrhages, while his oral mucosa displays conjunctival petechiae. There is a palpable purpuric skin rash of the lower extremities, reduced bilateral radial and ulnar pulsations, linear hemorrhages under the nails not reaching the nail margin, as well as tender, erythematous nodules occurring in the of the fingers. His cardiac exam demonstrates a soft, medium-pitched holosystolic murmur located at the apex with radiation to the axilla, while his foot exam reveals the findings in the attached image. A comparison to the patient's last physical exam reveals no abnormal physical exam findings. What pharmacotherapeutic agent is most appropriate for this patient? 1 Penicillin G 2 Rifampin 3 Linezolid 4 Doxycycline 5 Ampicillin
This patient's presentation is most consistent with native valve endocarditis caused by Viridans group streptococci ( -hemolytic streptococci). These are a frequent cause of community-acquired native valve endocarditis. Viridans streptococci are normal residents of the oropharynx and easily gain access to the circulation after dental or gingival trauma. Adult native valve endocarditis caused by penicillin-susceptible S. viridans, S. bovis, and other streptococci should be treated with one of the following regimens: penicillin G at 12 - 18 million U/d IV by continuous pump or in 6 equally divided doses for 4 weeks, ceftriaxone at 2 g/d IV for 4 weeks, or penicillin G or ceftriaxone and gentamicin at 1 mg/kg (based on ideal body weight) every 8 hours for 2 weeks. The emergence of methicillin-resistant S. aureus (MRSA) and penicillin-resistant streptococci has led to a change in empiric treatment with liberal substitution of vancomycin in lieu of a penicillin antibiotic.
A 12-year-old presents with an injury of his left arm and leg. He states that he felt dizziness during the 2nd mile of the long distance run organized by the school. He fell and lost the consciousness for several seconds, but after that he felt "normal". His father has been diagnosed with Emery-Dreifuss muscular dystrophy type 1. On examination, you find a few superficial excoriations; there is also symmetric humero-peroneal weakness involving the biceps, triceps, and peroneal muscles. There is also atrophy and contractures of Achilles-heel, elbows, and posterior neck. After taking care of his injuries, what test should you order? 1 CK 2 LDH 3. EKG 4. EEG 5. CT
Both family history and clinical presentation in this patient are consistent with Emery-Dreifuss muscular dystrophy. Cardiac disease in Emery-Dreifuss muscular dystrophy is nearly universal. It usually begins after the onset of weakness, and it manifests as syncope in the 2nd or 3rd decade of life; it can also be a cause of sudden cardiac death. Cardiac disease can take form of the bradycardia, atrial arrhythmias (including atrial fibrillation/flutter), AV conduction defect, or even atrial or ventricular cardiomyopathy. Pacemakers are often needed by the age of 30. Minimal follow up requirements in patients are annual cardiac assessment (ECG, Holter, echocardiography) and the monitoring of respiratory function.
A term infant is apneic at birth. After providing warmth and positioning and clearing the airway, the infant is still apneic and has central cyanosis; his heart rate is 80 beats per minute. What is the next appropriate step for resuscitation of this newborn? 1. Stimulate the infant by flicking the soles 2. Begin positive pressure ventilation 3. Start chest compressions 4 Administer epinephrine 5. Begin positive pressure ventilation and chest compressions
Correct answer: Begin positive pressure ventilation Explanation The initial steps like providing warmth, positioning, clearing the airway, drying, removing wet linen, and evaluating the infant takes about 30 seconds. If the infant is still apneic and the heart rate is less than 100/minute, the infant's breathing should be assisted by providing positive pressure ventilation with a bag and mask. When a newborn or fetus is deprived of oxygen, there is an initial period of rapid breathing followed by a period of primary apnea during which stimulation like drying or flicking the sole will cause resumption of breathing. If oxygen deprivation continues, the infant will have gasping irregular breathing efforts and will then go into secondary apnea. At this point, stimulation will not restart the infant's breathing; therefore, positive pressure ventilation must be provided for the respiration to start. This infant will require positive pressure ventilation because he is in secondary apnea. He will not respond to stimulation by flicking the soles or free flow of oxygen. After providing PPV for 30 seconds, evaluate the heart rate. If it is below 60 beats per minute, you should support circulation by providing chest compressions (CC). Since the heart rate is 80/min, chest compressions are not required at present. Epinephrine is administered if the heart rate remains below 60/min after giving PPV with bag and mask/endotracheal tube and bag for 30 seconds, followed by PPV and CC for another 30 seconds. At present, it is not required in the above infant; however, effectiveness of PPV and chest compressions should be checked frequently. Ventilation of the lungs is the single most important and most effective step in the cardiopulmonary resuscitation of the asphyxiated newborn. Once this is accomplished, heart rate, blood pressure, and pulmonary blood flow improve spontaneously. However if oxygen levels have become very low, cardiac output may have to be supported by chest compressions and by administering epinephrine.
A 29-year old patient presents with a 9-month history of recurrent hemoptysis. The patient suffered from cavitary tuberculous infection 5 years ago and was effectively treated with antituberculous drugs. A thin-walled cavity was seen over the right upper lobe when first diagnosed. At a later date, a progressive opacification of tuberculous cavity was seen. What organism is most likely responsible for such radiologic changes of a tuberculous cavity? 1. Secondary growth of staphylococci 2. Growth of Candida albicans might have filled the cavity 3. Growth of Aspergillus fumigatus might have filled the cavity 4. The cavity filled with granulation tissue and subsequent fibrosis 5. The cavity filled with desquamated epithelial cells of alveoli
Correct answer: Growth of Aspergillus fumigatus might have filled the cavity Explanation One of the complications of tuberculous cavity is the growth of Aspergillus fumigatus within the cavity (fungal ball). Diagnosis is made by isolation of Aspergillus hyphae in the sputum. IgG antibody against Aspergillus can be demonstrated in such patients.
A 72-year-old man presents to the outpatient clinic in follow-up for his dyspnea and cough. He reports shortness of breath, especially with activity, and a cough, which is non-productive. Symptoms have been present for 1 year, and they are getting worse. He initially went to the cardiologist for heart concerns, but no cardiovascular disease was found. A chest X-ray was ordered, and the patient reports it showed no masses in his chest. The patient denies any other symptoms, including fever, chills, night sweats, chest pain, and weight loss. The patient is a retired salesman; he fishes as a hobby. He lives at home with his wife; he denies use of tobacco, alcohol, and drugs. He denies any out of the country travel. On physical exam, the patient sits comfortably with normal respiratory effort. Auscultation of his lungs reveals fine crackles in both bases. A dry cough is noted a few times. Cardiovascular exam, including heart and extremities, is normal, except for clubbing of the fingers bilaterally. What test is the next most appropriate in evaluation of this patient's condition? 1. Echocardiogram 2. High-resolution computed tomography (CT) 3. Mantoux test or purified protein derivative (PPD) 4. Ultrasound of the thorax 5. Ventilation-perfusion (VQ) scan
Correct answer: High-resolution computed tomography (CT) Explanation This patient has a likely diagnosis of idiopathic pulmonary fibrosis (IPF). This is a relatively rare disease and can be difficult to diagnose. Patients typically have symptoms for 1 - 2 years prior to definitive diagnosis. IPF is a chronic, progressive restrictive pulmonary disease of the lung parenchyma. Unlike pneumoconiosis, there are no known occupational exposures (hence the term idiopathic). IPF presents with exertional dyspnea and a non-productive cough. A high-resolution computed tomography (CT) and/or a lung biopsy are the means for definitive diagnosis. The CT may show reticular opacities, honeycombing, and perhaps a 'ground glass' appearance in the lung tissue. There are heterogeneous areas of diseased lung interspersed with healthy tissue. An echocardiogram is not helpful in diagnosing IPF. It can be assumed that evaluation by a cardiologist would have determined if an echocardiogram was necessary. This patient was cleared by a cardiologist and has only pulmonary complaints (dyspnea and cough, with findings of lung crackles and clubbing). The CT should be done next, and if not consistent with IPF, further testing with an echocardiogram should be considered. The Mantoux test or purified protein derivative (PPD) is a skin test for tuberculosis. This patient's presentation is not consistent with tuberculosis (TB), so a PPD would not be recommended. Chronic cough, fever, night sweats, weight loss, along with TB risk factors (such as nursing home setting, foreign birth or travel, etc.) would indicate PPD. Ultrasound of the thorax would not be helpful in evaluating possible IPF. In fact, ultrasound of the thoracic structures (outside of echocardiography) is not often done. A plain chest film (X-ray) is a good starting point for pulmonary complaints, followed by a CT as the next appropriate test. A ventilation-perfusion scan is a test that is primarily used in the diagnosis of pulmonary embolism (PE). This patient is not presenting with a history consistent with a PE (which may include acute dyspnea and chest pain).
An 8-month-old infant presents with shortness of breath, wheezing, intercostal retractions, and respiratory rate of 50. What would further suggest a diagnosis of bronchiolitis? 1. Wheezing improved after nebulizer 2. Peak flow 300 prior to treatment 3. Liver and spleen palpable 4. Clubbing of the finger
Correct answer: Liver and spleen palpable Explanation Enlargement of the liver and spleen occurs due to hyperinflation of the lungs with bronchiolitis. With this diagnosis, a nebulizer that improves wheezing is not a specific criterion, but it is more so with asthma. A peak flow of 300 is good; therefore, it would likely be lower with bronchiolitis. Clubbing of the fingers is seen with prolonged COPD in adults, not in the pediatric population.
A 68-year-old man presents with long-standing dyspnea and non-productive cough. The patient has had the cough and progressively worsening dyspnea for about 1.5 years. He denies other symptoms, and he is not a smoker. He denies fevers, chills, and night sweats. He denies unusual travel, hobbies, and occupational exposures. The rest of his review of systems is negative. His past medical history is unremarkable. He had a workup through his family practice, and he was then sent to a cardiologist, who ruled out cardiovascular causes of his dyspnea. He was given trials of various antibiotics, inhalers, and steroids; there was no improvement in symptoms, despite good compliance. Several screening blood tests (including blood count, metabolic panel, HIV, and autoimmune markers) are done, revealing no abnormalities. A chest X-ray shows a few reticular opacities. A high-resolution computed tomography (HSCT) scan shows some scattered areas of reticular opacities and honeycombing. Your office runs a pulmonary function test (PFT), which shows some restrictive impairment and reduced perfusion of carbon monoxide. Physical exam is significant for fine inspiratory bibasilar crackles and clubbing in the fingers. An occasional dry cough is noted. What intervention is recommended for this patient's condition? 1. Avoidance of exertion 2. Daily inhaled steroids 3. Long-term oral steroid use 4. Lung transplant 5. Ventilation-perfusion scan
Correct answer: Lung transplant Explanation This patient is presenting with idiopathic pulmonary fibrosis (IPF), and the only intervention currently associated with improvements in mortality is lung transplant. IPF is a chronic, progressive restrictive pulmonary disease of the lung parenchyma. IPF presents with exertional dyspnea and non-productive cough; it also presents with the imaging characteristics described in this patient. Avoidance of exertion may prevent the exertional dyspnea in this patient, but it is not recommended. Respiratory therapy to improve exercise tolerance is recommended in cases of IPF. Daily inhaled steroids and long-term oral steroid use are not proven as effective in IPF. The pathophysiologic basis for IPF is much more a disorder of fibroblastic disease, in which there is an abnormal accumulation of extracellular matrix, and not inflammatory in nature. Anti-inflammatory medications do not alter the course of IPF. A ventilation-perfusion scan is a test that is primarily used in the diagnosis of pulmonary embolism (PE). This patient is not presenting with a history consistent with a PE (which may be acute dyspnea and chest pain). The imaging that has already been done (high-resolution CT) is sufficient for diagnosis of IPF. No further studies are recommended. If the provider needs further confirmation, a lung biopsy should be the confirmatory test of choice.
A 72-year-old man is evaluated at the bedside following hospital admission for a 1-year history of progressive dyspnea, weight loss, low-grade fevers, fatigue, and myalgias. His past medical history is remarkable for atrial fibrillation, for which he takes amiodarone, hypercholesterolemia, and recurrent urinary tract infections, for which his urologist prescribed nitrofurantoin on a chronic, prophylactic basis. He denies any cigarette use, history of murmurs or coronary artery disease, chest pain, wheezing, hemoptysis, syncope, abdominal pain, rashes, peripheral edema, diaphoresis, or vomiting. Bedside echocardiogram and electrocardiograms are unremarkable for abnormalities. A chest x-ray revealed peripheral reticular opacities at the lung bases and a generalized honeycombing pattern (refer to image). Which of these statements regarding the clinical presentation of this patient is correct? 1. Acute, severe shortness of breath is the most common presenting symptom. 2. Most patients are asymptomatic at the time of diagnosis. 3. The most common symptoms are fever and a productive cough. 4. Physical exam findings of pulmonary hypertension commonly occur. 5. The pulmonary exam routinely reveals generalized wheezing and rhonchi.
Correct answer: Physical exam findings of pulmonary hypertension commonly occur. Explanation This patient's most likely diagnosis is idiopathic pulmonary fibrosis. It is critical to obtain a complete history, including medication history, social history, occupational history, exposure history, and review of systems, to ensure other causes of interstitial lung disease are excluded. Amiodarone, bleomycin, and nitrofurantoin are notable medications associated with pulmonary fibrosis. Most patients with idiopathic pulmonary fibrosis present with a gradual onset, often for longer than 6 months duration. Dyspnea, which is the most prominent symptom in idiopathic pulmonary fibrosis (IPF), usually begins insidiously and is often progressive. Approximately 5% of patients have no presenting symptoms when idiopathic pulmonary fibrosis is diagnosed. The clinical symptoms of idiopathic pulmonary fibrosis are nonspecific; symptoms often precede the diagnosis by a median of 1-2 years. Most patients present with exertional dyspnea and a nonproductive cough. Pulmonary hypertension is a common comorbidity in patients with idiopathic pulmonary fibrosis, and an estimated 20-40% of patients with idiopathic pulmonary fibrosis who are evaluated or listed for lung transplantation have pulmonary hypertension at rest. Patients may have a loud P2 component of the second heart sound, a fixed split S2, a holosystolic tricuspid regurgitation murmur, and pedal edema. As right ventricular hypertrophy ensues, a right ventricular heave may be palpated at the lower left sternal border and increased right atrial pressure may cause elevation of the jugular venous pressure. The chest radiograph lacks diagnostic specificity for idiopathic pulmonary fibrosis. In most patients with idiopathic pulmonary fibrosis, the physical examination reveals fine bibasilar inspiratory crackles (Velcro crackles). Additionally, digital clubbing is seen in 25-50% of patients with IPF.
A 67-year-old man presents with a 3-week history of increasing shortness of breath; it occurs even while he is at rest. The patient was diagnosed with congestive heart failure in the past year, and he has been well controlled on oral medication. He has no history of tobacco use. He has gained 10 pounds since his last exam 2 months prior to presentation. On physical exam, there are diminished breath sounds and decreased tactile fremitus bilaterally at the base of the lungs. Dullness to percussion is also noted in the same area. He has 3+ bilateral pitting lower extremity edema. Based on the patient's physical exam and history, what is the most likely diagnosis? 1. Lung malignancy 2. Tuberculosis 3. Empyema 4. Spontaneous pneumothorax 5. Pleural effusion
Correct answer: Pleural effusion Explanation The correct answer is pleural effusion. This patient has a history of congestive heart failure, which is one of the most common causes of transudative pleural effusions. Pleural effusions in the setting of heart failure are usually right-sided or bilateral; in this patient's case, physical exam findings point to bilateral pleural effusion. Dyspnea is a common symptom seen in patients with a pleural effusion. Based on his pitting edema on exam, it is clear that this patient has uncontrolled heart failure. The physical exam findings are also common with pleural effusion. Lung malignancy is incorrect because he does not have a history of tobacco use, and because of his career as a post office worker, he has a low probability of being exposed to asbestos. Tuberculosis is incorrect because pleural effusions caused by tuberculosis are typically unilateral. Typically, a patient with TB would have a productive cough, night sweats, and weight loss as presenting symptoms. Empyema is incorrect because a patient would more than likely present with fever, productive cough, and pleuritic chest pain; however, a patient with empyema would have diminished breath sounds and decreased tactile fremitus. Empyema is more likely to cause a massive pleural effusion with mediastinal shift. Spontaneous pneumothorax is incorrect because it is unilateral; also, the patient complains of dyspnea and chest pain localized to the side of the pneumothorax. It is also acute onset. Physical examination shows hyperresonant percussion and decreased tactile fremitus that is localized to the side of the pneumothorax.
A 24-year-old man presents with a 1-week history of shortness of breath and a nonproductive cough. On physical exam, he is tachycardic, tachypneic, and febrile. He has lost weight without a change in dietary habits. Auscultation of this chest reveals bibasilar crackles. A chest X-ray is ordered and demonstrates diffuse interstitial infiltrates. You collect an arterial blood gas, and the results show moderate hypoxemia. A metabolic panel is ordered, and the only abnormality is an isolated elevated lactate dehydrogenase (LDH) enzyme. What is the most likely diagnosis? 1. Bowen's disease 2. Streptococcal pneumoniae 3. Mycoplasma pneumoniae 4. Stevens-Johnson syndrome 5. Pneumocystis jiroveci
Correct answer: Pneumocystis jiroveci Explanation The clinical picture is suggestive of Pneumocystis jiroveci pneumonia. Signs and symptoms include fever, dyspnea, and a nonproductive cough. Chest examination reveals bibasilar crackles on auscultation, and X-rays reveal diffuse interstitial infiltrates. Isolated elevated LDH is often a finding on serum chemistries. The infectious agent is Pneumocystis jiroveci. This type of pneumonia is common in AIDS patients. Bowen's disease is a form of early squamous cell carcinoma. Lesions can appear as solitary or multiple, and they are pink or red in color with a slightly scaling surface, small erosions, and possible crusting. Streptococcal pneumoniae and/or Mycoplasma pneumoniae pneumonia are community-acquired pneumonias that present similarly to Pneumocystis jiroveci pneumonia, except bibasilar crackles are not present and isolated LDH is not a finding. Steven-Johnson syndrome is a mucocutaneous drug-induced or idiopathic reaction of the skin that is characterized by skin tenderness and erythema of skin and mucosa, followed by extensive cutaneous and mucosal epidermal necrosis and sloughing.
A 72-year-old man presents with an 8-month history of progressive dyspnea, which has been accompanied by a dry and persistent hacking cough. While the dyspnea now occurs at rest, he denies fever, chills, palpitations, chest pain, or peripheral edema. He states that he has worked for many years at a local chemical plant. His physical exam is remarkable for digital cyanosis and clubbing, while his pulmonary exam reveals diffuse fine, dry inspiratory crackles. His cardiac exam was positive for a prominent pulmonary valve closure sound (P2) and an elevated jugular venous pressure of 6 cm. A chest x-ray noted small lung volumes, with increased densities in the lung periphery and a honeycombing pattern; pulmonary function testing measured reductions in TLC, FEV1, and FVC with a preserved FEV1/FVC ratio. What is the best treatment for this patient at this time? 1. Erythromycin 2. Furosemide 3. Lung transplantation 4. Prednisone 5. Cyclophosphamide
Correct answer: Prednisone Explanation This patient most likely has hypersensitivity pneumonitis (HP), also known as extrinsic allergic alveolitis. This is an inflammatory disorder of the lung involving alveolar walls and terminal airways that is induced by repeated inhalation of a variety of organic agents. The chronic form of HP typically results from low-grade or recurrent exposure over many months to years, and the lung disease may already be partially or completely irreversible. These patients are usually advised to avoid all possible contact with the offending agent. In addition to identification of the causative agent and its avoidance, institution of glucocorticoid treatment is indicated. Prednisone at a dosage of 1 mg/kg per day or its equivalent is continued for 7 - 14 days. It is then tapered to 0.25-0.5 mg/kg and is maintained at this level for an additional 4 - 12 weeks at a rate that depends on the patient's clinical status. While patients with chronic HP may gradually recover without therapy following environmental control, a trial of prednisone may be useful to obtain maximal reversibility of the lung disease. Following initial prednisone therapy (1 mg/kg per day for 2 to 4 weeks), the drug is tapered to the lowest dosage that will maintain the functional status of the patient. Many patients will not require or benefit from long-term therapy if there is no further exposure to the antigen. Improvement of lung function may continue over a few months to years. If the patient's condition continues to decline on glucocorticoids, a second agent should be introduced while lowering or maintaining the prednisone dose at 0.25 mg/kg per day. Glucocorticoid therapy is recommended for symptomatic interstitial lung disease (ILD) patients with eosinophilic pneumonias, cryptogenic organizing pneumonia, connective tissue diseases, sarcoidosis, hypersensitivity pneumonitis, acute inorganic dust exposures, acute radiation pneumonitis, diffuse alveolar hemorrhage, and drug-induced ILD. In organic dust disease, glucocorticoids are recommended for both the acute and chronic stages.
A 41-year-old woman presents due to worsening symptoms. She was diagnosed with idiopathic pulmonary hypertension about 2 years prior to presentation; she is on home oxygen therapy. She has longstanding fatigue and dyspnea, but she is now experiencing profound dyspnea with exertion; swelling in her ankles; some discomfort in her right, upper abdomen; and the inability to breathe well when lying down. She has always been thin, but her weight has increased by 10 pounds in the last month. She denies fever and chills. She recently had an electrocardiogram (ECG), but she has not seen a healthcare provider to discuss the results. The ECG report indicates peaked p waves, right axis deviation, and tall R wave in V1. What is the most appropriate intervention for her current condition? 1. Prescribe a calcium channel blocker, such as verapamil 2. Prescribe a diuretic, such as furosemide 3. Prescribe a fluoroquinolone, such as levofloxacin 4. Prescribe a lipase inhibitor, such as orlistat 5. Prescribe a thiazolidinedione, such as actose
Correct answer: Prescribe a diuretic, such as furosemide This patient is presenting with a progression of a primary pulmonary disease (pulmonary hypertension) into cor pulmonale, which is also known as pulmonary heart disease. Cor pulmonale, when moderate to severe, will present with signs and symptoms of right heart failure, such as the severely fluid overloaded state. In addition to treatment of the underlying pulmonary disorder, cor pulmonale is treated much like right-sided heart failure is treat. It would be most appropriate to prescribe a diuretic, such as furosemide, at this time. If possible, this patient should also be referred to a cardiopulmonary specialist. A calcium channel blocker, such as verapamil, can actually worsen right ventricular function, which is already diminished in this patient. The ECG findings indicate right ventricular hypertrophy. A fluoroquinolone, such as levofloxacin, would be an appropriate choice if this patient's dyspnea were due to bacterial pneumonia. However, she is afebrile, and her fluid overload symptoms and ECG findings are not typical in cases of pneumonia. A lipase inhibitor, such as orlistat, might be considered for weight loss in a patient with obesity and the inability to lose weight. However, this patient reports a relatively recent weight gain, which is attributable to fluids; therefore, orlistat would be inappropriate for this patient. A thiazolidinedione, such as actose, is contraindicated in a patient with symptomatic heart failure. In essence, this patient has right-sided heart failure due to her pulmonary hypertension.
A 56-year-old man presents for a routine follow-up regarding his positive HIV status. He is compliant with his medications and has been feeling well. In addition to his antivirals, he takes daily trimethoprim/sulfamethoxazole for Pneumocystis jiroveci pneumonia (PCP) prophylaxis. What detail of his history would warrant the prophylaxis? 1. CD4 cell count <400 2. History of previous PCP infection 3. CD4 cell count >200 4. History of any previous pneumonia infection 5. HIV viral load >100,000 copies/mL
Correct answer: History of previous PCP infection History of previous Pneumocystis jiroveci pneumonia (PCP) infection is the correct answer. Patients who are HIV positive should take antibiotic prophylaxis for PCP if they have a history of PCP infection, if their CD4 cell count is below 200, or if they have evidence of immunodeficiency, such as oral candidiasis. Prophylaxis for PCP is typically daily trimethoprim/sulfamethoxazole double strength, but it can also be done with daily dapsone or monthly aerosolized pentamidine. If patients are on antiretroviral therapy and their CD4 cell count remains above 200 for at least 3 months, then they can stop prophylaxis. If their CD4 cell count drops below 200 again, they need to restart prophylaxis. CD4 cell count <400 is not the correct answer. Patients who are HIV positive need to be on PCP prophylaxis for various reasons, one of which is a CD4 count <200. A CD4 cell count between 200 and 400 would not warrant prophylaxis, until the point that the count drops below 200. CD4 cell count >200 is not the correct answer. A CD4 cell count over 200 would actually be a reason NOT to have the patient on PCP prophylaxis, as one of the criteria to start prophyhlaxis is a CD4 cell count below 200. In addition, when a patient has been on prophylaxis and antiretrovirals and their CD4 cell count is over 200 for 3 months, they can stop the prophylaxis. History of any pneumonia infection is not the correct answer. Only patients who have had previous PCP infections need to be treated with prophylaxis for PCP. Having had other types of pneumonia does not warrant PCP prophylaxis. HIV viral load >100,000 copies/mL is not the correct answer. HIV viral loads over 100,000 warrant antiretroviral treatment of HIV, but PCP prophylaxis is not given based on viral load. Rather, it is given based on CD4 cell count, previous PCP infection, or evidence of immunodeficiency.
A 3-year-old boy presents for his annual well child check. He and his family moved to the United States from Africa 4 months ago. He is a thin boy; he is in no acute distress. His heart rate and rhythm are regular; his lungs are clear, and he has no hepatosplenomegaly. Due to his recent immigration, he is given a tuberculin skin test. He has no known chronic medical conditions and no known contacts with tuberculosis disease. The test shows an induration of 6 mm. What is the next step in the management of this patient? 1. Chest X-ray and isoniazid 2. Isoniazid, rifampin, pyrazinamide, and ethambutol 3. Hospitalization for further workup 4. Normal well child care 5. Gastric aspirate sampling
Correct answer: Normal well child care For a child who has immigrated from a high-prevalence area of the world but has no close contacts with tuberculosis disease, is not immunocompromised, and has no signs or symptoms of the disease, the induration must be 10 mm or greater to be positive. Therefore, this child has a negative test and should receive normal well child care. If the test were positive, he would require a chest X-ray and further workup. If he were found to have latent infection without active disease, he would be treated with isoniazid alone. If he were found to have a pulmonary or extrapulmonary disease, except meningitis, he would be treated with isoniazid, rifampin, pyrazinamide, and ethambutol. Gastric aspirate sampling is one way of isolating the organism if a patient has active tuberculosis disease.
A 30-year-old immunocompromised patient presents with a 2-week history of breathlessness and a nonproductive, dry cough. The patient is afebrile, pulse is 100, and BP is 110/70 mm Hg. On auscultation, scattered rales all over the chest are heard. A chest X-ray shows diffuse air space and interstitial shadowing in both lungs. The shadowing is more prevalent in the apical region. What is the most likely diagnosis? 1. Tuberculosis 2. Mycoplasma pneumoniae 3. Viral pneumonia 4. Bacterial pneumonia 5. Pneumocystis pneumoniae
Correct answer: Pneumocystis pneumoniae The patient being immunocompromised narrows the choices to Pneumocystis pneumoniae and tuberculosis; diffuse infiltrate is more common in Pneumocystis infections of the lung. The fungus Pneumocystis jirovecii causes pneumocystis pneumonia. Tuberculosis shows localized infiltrate; it is commonly seen at the apices with hilar lymphadenopathy. Bacterial pneumonia will have an acute history of 2 - 4 days with fever and a nonproductive cough. The X-ray will show lobar infiltrate; it is rarely found throughout the lung. Viral pneumonia will have fever before the onset of the respiratory symptoms. Diagnosis is usually by isolating the organisms as well as serological tests. The presence of cavities in the lung indicates bacterial causes, fungal (Mycoplasma) causes, tuberculous infection, or neoplasm.
On Thanksgiving day, a 5-month-old infant presents with wheezing, rapid respirations (>45 breaths/min), and chest retractions. The patient has a 2-day history of rhinorrhea and low-grade fever. Breath sounds are normal, and there is no cyanosis. What test can confirm the most likely diagnosis? 1. Chest X-ray 2. Immunofluorescence of nasal secretion 3. Gram stain of the sputum 4. Blood gas analysis 5. White blood cell count and differential
Explanation The correct answer is Immunofluorescence of nasal secretion. The clinical presentation, time of year, and age of the child strongly suggest bronchiolitis caused by respiratory syncytial virus. A chest X-ray may indicate interstitial pneumonia, but it can be relatively normal. Gram stain is needed in case of suspected bacterial infection, and sputum usually is difficult to obtain from children. Blood gas analysis may detect abnormalities when cyanosis is absent, but it cannot confirm a specific diagnosis. The white blood cell count and differential are usually normal; however, immunofluorescence of nasal secretion can lead to rapid viral identification.
Routine physical examination of a 55-year-old man demonstrates marked finger clubbing. Radiography of the hand shows new bone formation beneath the periosteum. With what disorder is this finding most strongly associated? 1. Chronic renal failure 2. Colon cancer 3. Endocrine adenomas 4. Intrathoracic cancer 5. Profound anemia
Explanation The correct response is intrathoracic cancer. Enlargement of the distal segments of the fingers and toes due to proliferation of connective tissue is known as clubbing. Clubbing may be associated with lung cancer, mesothelioma, bronchiectasis, and hepatic cirrhosis. In these conditions, the clubbing is accompanied by formation of new subperiosteal bone (hypertrophic osteoarthropathy). Chronic renal failure causes renal osteodystrophy, which may manifest as a combination of osteitis fibrosa cystica and osteomalacia. Multiple exostoses are sometimes associated with multiple colonic polyps and colon cancer in Gardner's syndrome. A variety of endocrine adenomas can be seen in polyostotic fibrous dysplasia. Profound anemia can be seen in diseases that destroy the marrow (e.g., osteopetrosis).
A 40-year-old man presents with atrial flutter with 2:1 atrioventricular (AV) conduction, giving him a pulse of 150 per minute, which is perfectly regular. His blood pressure is 70/40 mm Hg. He takes no medications regularly. You plan to provide him with urgent direct current cardioversion with conscious sedation. What would be an appropriate level of energy for cardioversion in order to restore sinus rhythm in this patient? 1. 10 Joules 2. 15 Joules 3. 50 Joules 4. 200 Joules 5. 300 Joules 6. 360 Joules
Of all of the arrhythmias, both supraventricular and ventricular, atrial flutter is the easiest to cardiovert back to a regular sinus rhythm. Direct cardioversion is usually successful with low energy - 25 to 100 Joules. There is no need to apply especially high energies such as 200 Joules, 300 Joules, or 360 Joules as the initial energy for cardioversion in case of atrial flutter, as higher energies have a greater probability of causing burns or broken bones. On the other hand, 10 or 15 Joules is unlikely to result in a successful cardioversion. *50*
A 66-year-old man with a past medical history of myocardial infarction 2 years ago, angina pectoris, aortic regurgitation, congestive heart failure, atrial fibrillation, and chronic obstructive pulmonary disease is presently being monitored in the hospital status-post admission for chest pain 1 day ago. Myocardial infarction has been ruled out. An EKG performed upon admission revealed a prolonged Qt interval and significant Q waves in the anterior leads. A diagnostic echocardiogram confirms moderate aortic and mitral valve regurgitation and a left-ventricular ejection fraction of 30%. He denies any complaints upon bedside evaluation. His physical exam reveals a blood pressure of 105/70 mm Hg and tachycardia. Continuous bedside ECG monitoring notes wide, monomorphic QRS complexes with a heart rate of 160 beats per minute that spontaneously resolve within 20 seconds, reverting to the pattern identified upon admission. What antiarrhythmic agent is considered to be the pharmacologic treatment of choice in the management of this patient? 1 Lidocaine 2 Procainamide 3 Verapamil 4 Sotalol 5 Amiodarone
The bedside electrocardiogram was remarkable for nonsustained, monomorphic ventricular tachycardia. The patient with hemodynamically stable VT in the setting of significant left ventricular dysfunction should be treated with intravenous amiodarone. When antiarrhythmic drug therapy is chosen to prevent recurrence or when VT is accompanied by hemodynamic instability, amiodarone is the treatment of choice. Patients in VT with hemodynamic compromise, congestive heart failure, chest pain, or ischemia should be treated promptly with DC cardioversion.
A 52-year-old patient, with a known case of renovascular hypertension, presents in the with poorly-controlled hypertension. He has been treated with both enalapril and nifedipine. He had been diagnosed with unilateral left renal artery stenosis, but recent tests have demonstrated mild changes in the right renal artery also. What should be the next step in management? 1 Diuretics 2 Reduction of NaCl consumption 3 Percutaneous transluminal angioplasty 4 Add atorvastatin and observe 5 Left nephrectomy
The correct choice would be percutaneous transluminal angioplasty because the patient is already on 2 antihypertensives, yet his hypertension is poorly controlled. Also, he is gradually developing bilateral renal artery stenosis. This will probably worsen his hypertension. If there are no contraindications for a surgical repair of the arteriosclerotic artery, it is the preferred course of action. It cures the hypertension without the need for medication. Percutaneous transluminal angioplasty is the procedure of choice in symptomatic stenosis. Additional stenting can also be done. This procedure has shown 90% success rates.
A 27-year-old woman presents with a 3-day history of "sharp", diffuse chest pain. She states the pain is worse with movement and deep breathing. On examination, it is noted that the patient prefers to sit upright and lean forward; she states, "I feel better in this position". Vital signs include a BP of 126/72 mm Hg; HR is 82, RR is 18, O2 sat is 96% RA, and temp is 101.3? F. On exam, you appreciate a friction rub. What set of diagnostics should you order? CXR, EKG, ECHO, DDimer, and BHCG 2 CBC, BMP, BHCG, and EKG 3 CBC, BHCG, EKG, ECHO, and CXR 4 CXR, EKG, DDimer, and BHCG 5 CXR, CBC, BMP, BHCG, EKG, and ECHO
The correct response is CXR, CBC, BMP, BHCG, EKG, and ECHO. This patient presents with acute pericarditis. Inflammation of the pericardium can occur as the result of an infectious source (including viruses, bacteria, TB, fungal, and parasitic sources), medications, systemic diseases (such as SLE), post MI (also known as Dressler's syndrome), uremia, or malignancy; there may also be no known cause.1 Pericarditis typically presents with substernal cheat pain that is aggravated by deep breathing and alleviated by leaning forward.1 The hallmark of pericarditis the finding of pericardial friction rub on exam.1 In addition to exam findings, patients with suspected pericarditis should undergo a CXR (may reveal cardiomegaly if a pericardial effusion is present), a CBC (often demonstrates leukocytosis), a BMP (to evaluate for uremia), a BHCG (necessary prior to imaging in a woman of childbearing age), EKG (may demonstrate diffuse ST segment elevation), and an ECHO (needed to evaluate for pericardial effusion and/or tamponade).1 A DDimer is not necessary in this patient; the possibility of a PE being the cause of her symptoms is low. Review of the patient's vital signs reveal a fever with a normal HR and O2 sat, making PE an unlikely cause.
A 4-year-old boy presents with poor weight gain, small size for his age, and dyspnea upon feeding. His mother notes that the child suffers from frequent upper respiratory tract infections. On physical exam, the child is underweight for his age. You note a precordial bulge, a prominent right ventricular cardiac impulse, and palpable pulmonary artery pulsations. You also find a widely split and fixed second heart sound as well as a mid-diastolic rumble at the left sternal border. What pharmacologic agent would be most appropriate in the medical management of this patient at this time? 1 Lasix (Furosemide) 2 Coumadin (Warfarin) 3 Procardia (Nifedipine) 4 Inderal (Propranolol) 5 Indocin (Indomethacin)
The correct response is Lasix (Furosemide). This patient's manifestations suggest a diagnosis of an atrial septal defect (ASD). ASD with moderate-to-large left-to-right shunts result in increased right ventricular stroke volume across the pulmonary outflow tract, creating a crescendo-decrescendo systolic ejection murmur. This murmur is heard in the second intercostal space at the upper left sternal border. Patients with large left-to-right shunts often have a rumbling middiastolic murmur at the lower left sternal border because of increased flow across the tricuspid valve. Definitive therapy for ASD includes closure of the defect, which is achieved surgically or through interventional catheterization. No specific or definitive medical therapy is available; however, patients with significant volume overload or atrial arrhythmias may require specific drug therapy. For patients with large shunts and heart failure, diuretics, digoxin, and ACE inhibitors should be used before surgery.
A 28-year-old man presents with a 2-hour history of chest tightness, coughing, and wheezing. The history determines that he has had many such attacks in recent years, usually brought on by emotional factors or exertion; the attacks are generally treatable by self-medication at home. He has a history of hay fever, and other members of the family have had similar symptoms. Physical examination reveals dyspnea, orthopnea, and cyanosis. High-pitched, sibilant rhonchi occur on inspiration and expiration, and some coarse crepitations are audible. Pulse is 130/min and regular. Coughing produces viscid sputum. An emergency arterial PO2 is 65 mm Hg. What will likely be shown by pulmonary function studies during an acute attack? 1. Decreased FEV1 2. Increased vital capacity 3. Decreased residual volume 4. Decreased airway resistance 5. Increased arterial PO2
The correct response is decreased FEV1. The patient suffers from extrinsic asthma, which is an obstructive lung disease. There is, therefore, reduction of both the vital capacity and the FEV1; there is more reduction of the latter, causing a reduction of the FEV1: FVC ratio. The total lung capacity (TLC) may, however, be increased due to air being trapped into the lung. This causes an increase in the residual volume(RV). Bronchial obstruction leads to an increase in airway resistance. In severe asthma, the patient first hyperventilates to maintain his PaO2, and as a result he has a low PaCO2. As he tires, he hypoventilates; this causes his PaCO2 to become normal, and it then rises. This elevation of the PCO2 is of grave significance, as it indicates that the resting ventilation is maximal and that reduction through fatigue may be lethal. Vital capacity is also decreased in restrictive lung diseases.
A 35-year-old woman presents for follow-up. She has a 6-month past medical history of hypertension; it has responded poorly to lifestyle approaches. She denies using any medications; she does not smoke or use illicit drugs. Her review of systems is notable for muscular weakness, paresthesias, headaches, polyuria, and polydipsia. On physical exam, her blood pressure is 155/95 mm Hg. She has generalized muscular weakness that is measured in all 4 extremities. The remainder of her exam is unremarkable. Laboratory analysis reveals hypokalemia and a hemoglobin A1c level of 5.5. What dx test result is most likely? 1 Metabolic alkalosis 2 Low serum aldosterone to plasma renin activity ratio 3 Hyponatremia 4 Increased erythrocyte sedimentation rate 5 Hypoglycemia
The correct response is metabolic alkalosis. This patient is demonstrating signs and symptoms consistent with primary hyperaldosteronism, which is most commonly caused by a unilateral adrenocortical adenoma (Conn's syndrome), but in a minority of patients, it is caused by adrenal hyperplasia.
A 73-year-old man with no significant past medical history presents with a 1-month history of light-headedness, dizziness, and near-faintness; it has been occurring in response to sitting up and standing from a supine position. He denies chest pain, palpitations, shortness of breath, cough, loss of consciousness, vision or speech changes, nausea or vomiting, numbness, tingling, paresthesias, and focal weakness. His physical exam is noteworthy for a drop of systolic blood pressure of 24 mm Hg from a supine to standing position. What test is most helpful in identifying the cause of this patient's symptoms? 1. Hemoglobin A1c 2. Tilt-table test 3. Cardiac enzymes 4. CT scan of the head 5. Urinalysis.
The correct response is tilt-table test. This patient is presenting with signs and symptoms consistent with orthostatic hypotension. It is defined as a reduction in systolic blood pressure of at least 20 mm Hg or diastolic blood pressure of at least 10 mm Hg within 3 minutes of standing or head-up tilt on a tilt table. It is a manifestation of sympathetic vasoconstrictor (autonomic) failure.
A patient is presenting with substernal crushing chest pain and shortness of breath for just 15 minutes prior to arriving to the emergency department. An EKG is obtained and appears suspicious for an acute myocardial infarction. What would most likely be seen in this patient's EKG? 1 S-T segment depression 2 T-wave changes 3 Ventricular bigeminy 4 Q-wave elongation 5 Hyperacute T-wave
The earliest presentation of an acute myocardial infarction is the hyperacute T wave. If found, this is treated the same as an ST segment elevation type of infarction (STEMI). The hyperacute T wave is considered very rare in clinical practice, as it only exists for 2-30 minutes after the onset of an infarction. Hyperacute T waves must also be distinguished from the characteristic peaked T waves, which commonly are seen with hyperkalemia.
A 25-year-old woman with a history of a childhood murmur has chronic exertional dyspnea associated with intermittent chest pain, hemoptysis, and lightheadedness. She denies smoking, a history of travel, medication usage, fever, chills, cough, palpitations, abdominal pain, and peripheral edema. Her physical exam reveals central cyanosis, a right ventricular parasternal heave, JVD of 4 cm, and an accentuated P2 heart sound. A loud, harsh holosystolic murmur in the left third and fourth interspaces was present along the sternum, with a systolic thrill. An electrocardiogram noted right atrial enlargement, RVH, and right axis deviation. A bedside echocardiogram confirmed RVH, right atrial enlargement, and tricuspid regurgitation with a bidirectional ventricular shunt. What health maintenance recommendations should be made to this patient? 1. The patient should avoid using oral contraceptives. 2. This patient should avoid foods containing iron. 3. Daily use of aspirin is recommended. 4. Regular, strenuous aerobic exercise should be strongly encouraged. 5. Routine CBC, EKG, and echocardiograms should not be made available.
The patient should avoid using oral contraceptives. This patient's presentation and diagnostic testing most likely reveals Eisenmenger's syndrome with a ventricular septal defect. Eisenmenger syndrome is a general term applied to pulmonary hypertension and shunt reversal in the presence of a congenital defect, including VSD, ostium primum ASD, AV canal defect, aortopulmonary window, or PDA. In the Eisenmenger VSD patient, chronic hypoxemia in cyanotic CHD results in secondary erythrocytosis due to increased erythropoietin production. Normal white cell counts with normal to decreased platelet counts are typical. Patients with decompensated erythrocytosis fail to establish equilibrium with unstable, rising hematocrits, and recurrent hyperviscosity symptoms. Iron-deficiency occurs from erythrocytosis and may be exacerbated by phlebotomy. Progressive symptoms after recurrent phlebotomy in decompensated patients with Eisenmenger's syndrome are usually due to iron depletion with hypochromic microcytosis. Iron depletion results in a larger number of smaller (microcytic) hypochromic red cells that are less capable of carrying oxygen and less deformable in the microcirculation; as such, iron-depleted erythrocytosis results in increasing symptoms due to decreased oxygen delivery to the tissues. Thus, this patient will benefit from increased iron supplementation. Higher RBCs counts relative to plasma volume will increase viscosity versus an equivalent hematocrit with fewer, larger, iron-replete, deformable cells. Owing to the risk of clot formation and vascular thrombosis due to hyperviscosity, oral contraceptives are often contraindicated for cyanotic women.
A 60-year-old man presents to your outpatient clinic for work up after having a syncopal episode during a walk 2 days prior to presentation. He mentions generalized fatigue for several months as well as "just not feeling well". He describes a non-specific mild angina that has been coming and going for the last several months; however, he has not sought medical care. Exam reveals a harsh systolic ejection murmur that radiates to the neck and is louder when the patient leans forward. Lung auscultation is clear throughout. He has no peripheral edema or calf pain. You obtain an EKG, which reveals left ventricular hypertrophy; a chest X-ray shows cardiac enlargement with small bilateral pleural effusions. What is the most appropriate treatment plan for this patient? 1. Admit to the hospital in order to undergo further testing 2 Order an outpatient echocardiogram 3 Start the patient on anticoagulation therapy 4 Order a troponin, BNP, TSH, and chest CT; based on those result, you should decide if a referral to cardiology is necessary 5 Arrange for the patient to undergo a cardiac stress test
The patient should be admitted to the hospital so that he might undergo further testing. The patients' history, exam, and diagnostics are consistent with a diagnosis of aortic stenosis. Due to the fact that the patient is symptomatic (syncope and angina) and there is diagnostic evidence of heart failure (pleural effusions, cardiomegaly, left ventricular hypertrophy), there is significant concern that this patient may deteriorate quickly. The best course of action at this time is hospital admission for continued diagnostic work up and treatment.
A 58-year-old woman presents with a 3-month history of postprandial abdominal pain. This crampy pain occurs 30 minutes after eating, every time. Due to these symptoms, the patient has lost 30 pounds and is afraid to eat. Her past medical history includes hypertension treated with enalapril, coronary artery disease for which she has undergone a right coronary artery stent, and she underwent a carotid endarterectomy for symptomatic carotid stenosis. She has smoked 2 packs of cigarettes a day for 30 years. What is the best initial test for this patient? 1 Mesenteric angiogram 2 Mesenteric duplex ultrasound 3 Computerized tomography (CT) 4 Magnetic resonance angiography (MRA) 5 Computerized tomography angiography (CTA)
The symptoms of chronic mesenteric ischemia have a typical presentation, including a cachectic, middle-aged patient with crampy abdominal pain after eating. The risk factors for chronic mesenteric ischemia are the same as those for atherosclerosis. Treatment aimed at restoration of mesenteric blood flow is required to restore blood supply, prevent bowel necrosis, and restore normal weight. *Mesenteric duplex ultrasound is an excellent screening test to detect chronic mesenteric ischemia;* however, angiography allows for confirmation of the diagnosis. Combined B-mode and color Doppler ultrasound analyze flow though the mesenteric arteries and identifies stenosis as an elevated velocity. Limiting factors include obesity and intestinal gas, which may obstruct the ability to obtain a good ultrasound image.
A 36-year-old woman presents with chronic dyspnea that is worse while lying prone. The patient reports progressive worsening of the symptoms. On physical examination, a heart murmur is detected upon cardiac auscultation, heard best with the bell over the apex. The murmur is a non-radiating, low-pitched diastolic rumble. A loud S1 and opening snap can also be heard in addition to an apical thrill and decreased pulse pressure. An EKG is done and shows an arrhythmia. What is the patient's most likely underlying condition? 1 Aortic regurgitation 2 Pulmonic stenosis 3 Mitral stenosis 4 Hypertrophic subaortic stenosis 5 Mitral valve prolapse
This patient has mitral stenosis. Dyspnea and orthopnea are symptoms that can be seen with mitral stenosis. The murmur of mitral stenosis is heard best over the apex area with the bell. It is a non-radiating, low-pitched diastolic rumble. A loud S1 and opening snap is consistent with mitral stenosis. Atrial fibrillation can sometimes be seen because of dilation of the left atrium. Mitral stenosis most often occurs after an episode of rheumatic fever.
A 68-year-old man with a past medical history of diabetes mellitus type II, hyperlipidemia, myocardial infarction 1 year ago, and congestive heart failure with left ventricular ejection fraction of 35% is rushed to his local emergency room by his wife after he collapsed and became unresponsive at their residence. He admitted to her that he had been experiencing severe chest pain and pressure, fatigue, palpitations, diaphoresis, and lightheadedness for several minutes prior to his collapse. His present medications include aspirin, atorvastatin, lisinopril, glipizide, and carvedilol. Upon physical exam, he is found to have a blood pressure of 60/palpable, is pulseless, and has gasping respirations. His troponin T level was found to be elevated at 0.2 ng/ml, and troponin I level elevated and measured to be 0.25 ng/ml. The admission ECG revealed bizarre, irregular, random waveform, no clearly identifiable QRS complexes or P waves, and a wandering baseline. Following appropriate stabilization, what is best next step for this patient? 1 Prophylactic lidocaine 2 Implanatable cardioverter-defibrillator (ICD) implantation 3 Long-term metoprolol use 4 Percutaneous coronary intervention (PCI) 5 Discontinue aspirin, atorvastatin, and lisinopril
This patient has most likely experienced ventricular fibrillation (VF) due to a myocardial infarction. External electrical defibrillation remains the most successful treatment of VF. Patients with depressed left ventricular function at least 40 days post-STEMI are referred for insertion of an implantable cardioverter/defibrillator (ICD) if the LVEF is <30-40% and they are in NYHA class II-III or if the LVEF is <30-35% and they are in NYHA class I functional status. Patients with preserved left ventricular function (LVEF >40%) do not receive an ICD regardless of NYHA functional class. Cardiac electrophysiologists should be involved in the care of these patients. ICDs, which effectively provide early defibrillation, are used for patients at high risk for recurrent VF. Survivors of VF have a recurrence rate on the order of 20-25% per year, making ICD placement important in most patients. The annual VF rate in patients with these devices has been reduced from 25% to 1-2%. Studies indicate that patients with VF arrest who receive ICDs have improved long-term survival rates compared with those receiving only medications. However, patients with ICDs may also require oral antidysrhythmic therapy to minimize recurrent device activation.
A 49-year-old man presents with chest pain. He has had this type of pain in the past, but it typically occurred with significant exertion, such as shoveling snow. Over the past 2 weeks, however, the pain has come on with progressively less activity, and today the pain persists despite rest. Although he has a history of hypertension, he admits to rarely taking his antihypertensive medication. On examination, his HR is 95 beats per minute, and his blood pressure is 212/100 mm Hg. An electrocardiogram is performed and reveals ST elevation in the anterolateral leads. What medication would be the most appropriate treatment? 1 Adenosine 2 Furosemide 3 Methyldopa 4 Nifedipine 5 Nitroprusside
This patient is presenting with a hypertensive emergency, marked by severely elevated blood pressure (>180/120 mm Hg), accompanied by signs of progressive organ dysfunction; in this case, he presents with unstable angina. The patient should receive immediate antihypertensive treatment with an intravenous agent. Several drugs are available for use in this context. Sodium nitroprusside is commonly used because of its immediate onset and short duration of action. Careful medication titration is indicated, as gradual blood pressure reduction is desired to avoid compromising end-organ perfusion.
A 22-year-old woman presents with an 8-month history of amenorrhea. Further questioning elicits additional pertinent positives of backaches, headaches, hirsutism, and acne. Physical examination reveals a female patient with a moon-shaped facies, multiple purple striae, and significant central obesity (body mass index of 36). Considering the likely diagnosis, what other abnormality would be expected? 1 Hyperkalemia 2 Hypotension 3 Hypertension 4 Exophthalmos 5 Carotid bruit
This patient likely is suffering from Cushing syndrome, which is also known as hypercortisolism. Consequences of excessive levels of circulating cortisol, no matter the etiology, will lead to signs and symptoms such as central obesity but thin extremities, a moon face, a buffalo hump, supraclavicular fat pads, protuberant abdomen, hirsutism; there may also be oligomenorrhea, amenorrhea, or in men, possible erectile dysfunction. Backaches, headaches, acne, purple striae, and impaired wound healing may also be found in these patients. The correct response is (secondary) hypertension; although secondary hypertension only accounts for 5 - 10% of all hypertension cases, Cushing's syndrome has been found to be a potential cause for these cases. Its main cause is via the mineralocorticoid effects of excess glucocorticoids.
A 25-year-old woman presents with intermittent palpitations that are associated with lightheadedness; she admits to a past medical history of having a self-described "hole in her heart". These seem to occur upon significant exertion and when she is "stressed out." She denies chest pain, shortness of breath, wheezing, hemoptysis, cough, syncope, abdominal pain, rashes, peripheral edema, diaphoresis, and vomiting. Her physical exam is remarkable for a mid-to-late systolic click; it is followed by a high-pitched, 'whooping' late systolic crescendo-decrescendo murmur, and it is heard best at the apex. The click occurs earlier with standing and upon Valsalva strain, and it also occurs later in the cardiac cycle with squatting and sustained handgrip. What is the most likely diagnosis? Aortic stenosis 2. Mitral valve prolapse 3 Ventricular septal defect 4 Aortic regurgitation 5 5 Mitral stenosis
This patient's diagnosis is mitral valve prolapse (MVP). It most commonly occurs in young women and in patients with heritable disorders of connective tissue, including Marfan's syndrome, osteogenesis imperfecta, and Ehlers-Danlos syndrome. Rarely, it occurs as a sequel to acute rheumatic fever, in ischemic heart disease, in various cardiomyopathies, as well as in 20% of patients with ostium secundum atrial septal defect. MVP varies in its clinical expression, ranging from only a systolic click and murmur with mild prolapse of the posterior leaflet to severe mitral regurgitation due to chordal rupture and leaflet flail; in North America, MVP is now the most common cause of isolated severe MR requiring surgical treatment. The most important finding is the mid- or late- (non-ejection) systolic click following S1, and it is thought to be generated by the sudden tensing of slack, elongated chordae tendineae or by the prolapsing mitral leaflet when it reaches its maximum excursion. Systolic clicks may be multiple, and representing mitral valve regurgitation, they may be followed by a high-pitched, late systolic crescendo-decrescendo murmur, occasionally 'whooping' or 'honking' and heard best at the apex. The click and murmur occur earlier with standing, during the strain phase of the Valsalva maneuver, and with any intervention that decreases LV volume, exaggerating the propensity of mitral leaflet prolapse. Conversely, squatting and isometric exercises, which increase LV volume, diminish MVP; the click-murmur complex is delayed, moves away from S1, and may even disappear. Patients with MVP most frequently have symptoms of autonomic dysfunction, including easy fatigability, dizziness, and atypical chest pain. Further symptoms include palpitations, light-headedness, and syncope.
A 74-year-old man with a past medical history of diabetes mellitus, hypertension, and hyperlipidemia presents with severe chest pain and dyspnea. On examination, he is confused, agitated, pale, apprehensive, and diaphoretic. His pulse is weak and tachycardic, with a systolic blood pressure of 80 mmHg. He has a narrow pulse pressure, tachypnea, a weak apical impulse, significant jugular venous distention, and pulmonary crackles. Bedside electrocardiogram reveals ST-segment elevations in the anterior and septal leads, while a portable chest X-ray notes diffuse pulmonary congestion. What is the most appropriate step in the management of this patient? 1 Crystalloid infusion 2 Initiate intravenous β-Blocker therapy 3 Begin phenylephrine 4 Nitrates and morphine 5 Emergent percutaneous coronary intervention
This patient's exhibits signs and symptoms of cardiogenic shock due to myocardial infarction with pulmonary edema. Treatment of cardiogenic shock includes general supportive measures of oxygen, aspirin, heparin, and "gentle" fluid challenges (250 cc) if there is no overt pulmonary edema. In cardiogenic shock, early revascularization with percutaneous coronary intervention (angioplasty) or coronary artery bypass graft is the treatment of choice. Survival from cardiogenic shock is highest with emergency coronary intervention, followed by intra-aortic balloon pump combined with thrombolytic therapy, and with thrombolytic therapy alone being least effective in reducing mortality. The greatest short-term benefit is reported in patients <75 years of age, those without previous MI, and those treated within 6 hours of symptom onset.
A 52-year-old man is hospitalized due to an acute myocardial infarction. Cardiac enzymes are as follows: Myoglobin: Normal CK-MB: Elevated Troponin T: Elevated Troponin I: Elevated Given the above information, when did the patient's myocardial infarction most likely occur? 1. Within 2 hours of when the cardiac enzymes were drawn 2. Within 12 hours of when the cardiac enzymes were drawn 3. Within 2 days of when the cardiac enzymes were drawn 4. Within 5 days of when the cardiac enzymes were drawn 5. Within 10 days of when the cardiac enzymes were drawn
Within 2 days of when the cardiac enzymes were drawn is the correct answer. The above patient has normal myoglobin with elevated CK-MB and troponins T and I. While myoglobin is a nonspecific marker for myocardial infarction, it typically appears within 1 - 4 hours of ischemia and normalizes by 1 day. CK-MB and troponins T and I all rise about 4 - 9 hours following myocardial infarction. CK-MB will normalize by 2 - 3 days, whereas the troponins take 1 - 2 weeks to normalize. Since the patient's myoglobin is negative, but his CK-MB is elevated, it suggests that the patient has had an acute myocardial infarction within the last 1 - 3 days. 'Within 2 days of when the cardiac enzymes were drawn' is the only answer that fits this window.
A 62-year-old man has a 15-year history of hypertension, hyperlipidemia, myocardial infarction, and congestive heart failure. He presents with a 10-day history of shortness of breath. He finds it difficult to walk short distances due to shortness of breath. He also notes cough, orthopnea, nocturnal dyspnea, and generalized abdominal discomfort. He has been taking large doses of his furosemide without relief. He denies cough, fever, chills, diaphoresis, anxiety, chest pain, pleurisy, nausea, vomiting, diarrhea, rashes, lightheadedness, and syncope. On physical examination, the patient is acutely dyspneic at rest. He is afebrile, but tachypnic and diaphoretic. There is a diminished first heart sound, S3 gallop, laterally displaced PMI; the lungs have bibasilar rales. The abdominal exam reveals distension with hepatomegaly in the right upper quadrant. There is 2+ pitting edema of the lower extremities to the level of the mid-calf. What is the most likely expected diagnostic test result in this case? 1 Hyponatremia 2 Reduced BUN levels 3 Hyperchloremia 4 Hyperalbuminemia 5 Hyperkalemia
his patient is experiencing an acute exacerbation of congestive heart failure. In cases of severe heart failure, prolonged, rigid sodium restriction, coupled with intensive diuretic therapy and the inability to excrete water, may lead to dilutional hyponatremia. This occurs because of a substantial expansion of extracellular and intravascular fluid volume and a normal or increased level of total body sodium.
A 60-year-old man, following up after a recent myocardial infarction, presents with sharp inspiratory chest pain. Other than his recent myocardial infarction, his past medical history is significant for peptic ulcer and renal insufficiency. You suspect Dressler's syndrome after seeing diffuse ST elevations on an electrocardiogram. What is the most appropriate treatment for this patient? 1. Indomethacin 2 Ibuprofen 3 A corticosteroid taper 4 Hydrocodone 5 Diclofenac sodium
Administering a corticosteroid taper in this patient is the safest option given his peptic ulcer history and renal insufficiency.
What is the mechanism of LMWH?
Both unfractionated heparin and low molecular weight Heparin act by *forming a complex with antithrombin III.*
A premature infant presents with jaundice and a hemoglobin level of 8 g/dL. The peripheral blood film reveals reticulocytosis; a vitamin deficiency is suspected. What is the most likely diagnosis? 1. Vitamin A deficiency 2. Pyridoxine deficiency 3. Vitamin C deficiency 4. Niacin deficiency 5. Vitamin E deficiency
Correct answer: Vitamin E deficiency Explanation Vitamin E deficiency may cause a hemolytic anemia in premature infants. Laboratory investigations reveal low plasma tocopherol levels, a low hemoglobin level, reticulocytosis, hyperbilirubinemia, and creatinuria. Causes of vitamin E deficiency in premature infants include limited placental transfer of vitamin E and the resultant low levels at birth; this is combined with its relative deficiency in the infant diet. Dietary sources for older children and adults include wheat germ, vegetable oils, egg yolk, and leafy vegetables. In cases of vitamin A deficiency, patients can present with night blindness or an inability to see well in dim light. There may also be conjunctival and corneal xerosis, as well as pericorneal and corneal opacities and Bitot's spots. Bitot's spots are a collection of keratin appearing as triangular, foamy spots on the conjunctiva. The patient may also have xeroderma, hyperkeratotic skin lesions, and increased susceptibility to infections. Causes include inadequate dietary intake and malabsorption. Dietary sources of vitamin A include fish, liver, egg yolk, butter, cream, dark green leafy vegetables, and yellow fruits and vegetables. In cases of pyridoxine (vitamin B6 deficiency), patients can present with peripheral neuropathy, seborrheic dermatosis, glossitis, and cheilosis. Laboratory analysis reveals anemia with lymphopenia. Causes include malabsorption, as well as medications (e.g., isoniazid and penicillamine). Dietary sources of vitamin B6 include liver, legumes, whole grain cereals, and meats. In cases of vitamin C (ascorbic acid) deficiency, patients can present with bleeding tendencies (as a result of weakened capillaries) and impaired wound healing due to the impaired formation of connective tissue. On examination, the gums may be swollen and friable; the teeth may be loose. There may also be multiple splinter hemorrhages on the nails and ecchymoses, especially over the lower limbs. Causes include inadequate dietary intake and certain conditions (e.g., pregnancy and lactation) which increase vitamin C requirements. Dietary sources of vitamin C include citrus fruits, (e.g., oranges, lemons, and tangerines), as well as tomatoes and potatoes. Niacin deficiency causes pellagra, which is characterized by: A symmetrical dermatitis, usually on parts of the body exposed to sunlight Scarlet glossitis and stomatitis Diarrhea Mental aberrations (e.g., memory impairment, depression, and dementia) which may appear alone or in combination Causes of niacin deficiency include inadequate dietary intake (especially in patients with corn-based diets or alcoholism). Dietary sources include legumes, yeast, meat, and enriched cereal products.
A 56-year-old man presents with abdominal pain, indigestion, weight loss, nausea, and vomiting, and gray colored stools for the past month. His past medical history is positive for alcoholism. Lab results demonstrate an elevated serum amylase and lipase, decreased trypsinogen, and a positive fecal fat test. What is the most likely diagnosis? 1. Acute pancreatitis 2. Chronic pancreatitis 3. Pancreatic carcinoma 4. Pacreatic abscess 5. Insulinoma
Correct answer: Chronic pancreatitis Pancreatitisis an inflammation or infection of the pancreas. Chronic pancreatitis, which is the correct response, is caused by alcohol abuse, hemochromatosis (a condition of excess iron in the blood), and other unknown factors. Inflammation and fibrosis cause the destruction of functioning glandular tissue in the pancreas. This results in an inability to properly digest fat caused due to a lack of pancreatic enzymes. The production of insulin is also affected. Symptoms include abdominal pain (mainly in the upper abdomen), nausea, vomiting, weight loss, and fatty stools. Additional symptoms may include swelling (overall), stools (clay colored), and abdominal indigestion. Tests should include serum lipase (may be elevated), serum amylase (may be elevated), serum trypsinogen (may be low), and fecal fat test (shows fatty stools). Abdominal ultrasound and CT may show an enlarged pancreas. Treatment of chronic pancreatitis includes reducing pancreas stimulation, alleviating fat indigestion, reducing pain, and treating diabetes. A reduced-fat diet, vitamin supplementation, no alcohol or caffeine, and regulation of blood sugar levels are indicated in the treatment. The chief causes of acute pancreatitis in adults are gallstones, other biliary disease, and alcohol use. Viral infection (mumps, Coxsackie B, mycoplasma pneumonia, and Campylobacter), injury, pancreatic or common bile duct surgical procedures, and certain medications (especially estrogens, corticosteroids, thiazide diuretics, acetaminophen, tetracycline) are other causes. After the triggering event, the process continues with autodigestion that causes swelling, hemorrhage, and damage to the blood vessels. An attack may last for 48 hours. Symptoms include abdominal pain (mainly located in the upper abdomen) nausea, vomiting, weakness, sweating, anxiety, fever, clammy skin, and mild jaundice. General examination may show a low blood pressure and a heart rate above 90. Most cases resolve within a week with supportive measures, such as analgesics and fluid replacement. However, some cases can be life threatening. Pancreatic abscess occurs in 5 to 10% of people with acute pancreatitis. An abscess may be caused by inadequate drainage of a pancreatic pseudocyst, which is a complication associated with pancreatitis. Symptoms include fever, chills, abdominal pain, and abdominal mass. Physical exam will show signs of pancreatitis, and tests should include an abdominal CT and ultrasound. Treatment will include laparotomy with drainage and possible resection of dead tissue. Pancreatic cancer is the 4th most common cancer causing death in the U.S. The disease is more common in men, especially those between 60 and 70 years. The cause is unknown; however the incidence is greater in smokers. A high fat diet and chemical exposures may increase the risk. Symptoms include weight loss, abdominal pain, loss of appetite, jaundice, nausea, weakness, fatigue, vomiting, diarrhea, indigestion, back pain, stools (clay colored), pallor, and depression. Tests should include a pancreatic biopsy, an abdominal CT scan, and abdominal ultrasound. Only 20% of the tumors are operable at the time of diagnosis. Palliation is generally the treatment, along with chemotherapy and radiation. Insulinomas are generally benign tumors of the insulin-secreting cells of the pancreas, which secrete excess amounts of insulin. Risk factors include a prior history of multiple endocrine neoplasia Type I (MEN I). Symptoms include sweating, tremor, rapid heart rate, anxiety, hunger, dizziness, headache, clouding of vision, confusion, behavioral changes, convulsions, and loss of consciousness. Surgery is the treatment of choice to remove the tumor. If the tumor is not found during surgery, diazoxide may be given. A diuretic is always given with this medication to avoid retaining too much salt.
A 24-year-old man presents with a painful ulcer on his left leg and systemic symptoms. The lesion started 1 week ago as a small pustule that developed at the site of mosquito bite, and it rapidly developed into a painful ulcer. Patient describes pain as "stabbing" and debilitating. He also develops symmetrical joints pain, muscle pain, fever, and malaise. His past medical history is significant for ulcerative colitis, which is currently in remission. On examination, you find a deep exudative ulceration, with a well-defined violet border and a worn, erythematous, and indurated edge. There are signs of pathergy as well. What will you recommend in addition to meticulous wound care? 1. Immunosuppressant (systemic corticosteroids) 2. Antiviral (systemic acyclovir) 3. Surgical debridement 4. Antifungal (systemic fluconazole) 5. Painkiller (systemic paracetamol)
Correct answer: Immunosuppressant (systemic corticosteroids) Your patient most probably has Pyoderma gangrenosum, which is a ulcerative skin lesion of an uncertain etiology; in more than 50% of cases, it is associated with systemic diseases, most commonly inflammatory bowel disease. It usually develops rapidly and can progress from a pimple to an ulcer in 1 or 2 days. In a process termed pathergy, new ulcerations may occur after trauma or injury to the skin. Pain is the predominant symptom, but symmetrical arthritis, myalgias, and malaise are also common. When the lesions heal, they usually leave the scars that are often cribriform. Immunosuppression is the mainstay of treatment; it is believed that dysregulation of the immune system (specifically, altered neutrophil chemotaxis) is involved. Most clinicians use both topical and systemic therapy. The most commonly used treatments include topical potent corticosteroids or tacrolimus to treat early lesions; systemic corticosteroids, TNF-α inhibitors, or other anti-inflammatories or immunosuppressants are used to treat more severe manifestations. Surgery or debridement is contraindicated because of the presence of pathergy. Even if there was no pathergy, surgery is contraindicated because skin trauma can trigger the pathergy. Acyclovir is not indicated; the clinical picture is not consistent with acute herpes infection and the patient is not immunocompromised. Fluconazole is not indicated; the clinical picture is not consistent with tinea (or any other fungal infection). Paracetamol alone probably will not relieve the debilitating pain. Sufficient pain medication in this case will probably include paracetamol and some other medication, sometimes opioids.
Which of the following complications is commonly associated with subarterial VSD? 1 Infective endocarditis 2 Pulmonary hypertension 3 Congestive cardiac failure 4 Cor pulmonale 5 Aortic insufficiency (AI)
In subarterial VSD, the defect occurs in the outlet septum and is also known as the supracristal, conoseptal, or outlet VSD. It is referred to as subarterial VSD, as the aortic and pulmonary valves are in fibrous continuity with the outlet septum. Subarterial VSDs are commonly associated with aortic insufficiency. The VSDs that are complicated by AI are restrictive with high velocity shunting through the VSD. This creates a low-pressure zone, which impacts the adjacent aortic valve cusp, resulting in aortic valve prolapse (AVP), and subsequent AI.
A 52-year-old woman presents for a routine checkup. She has 2 children, and she attained menopause 1 year prior to presentation. Pap smears, mammogram, and DEXA bone scan are normal. She is a non-smoker. Her previous biennial checkups were always normal. Her BP is 142/96 mm Hg; pulse is 72 bpm. Her lab values are below. Fasting Blood sugar: 112 Post Prandial Blood sugar: 138 Total cholesterol: 190 LDL: 102 TSH: Normal levels What is the next best step in the management of this patient? 1 Reassurance 2 Thiazide 3 Diet and exercise 4 Statin 5 Metformin
Diet and exercise is the correct answer. The patient has an elevated blood pressure and borderline blood sugar levels. Since all previous checkups have been normal, initiation of therapy at this stage is unwarranted.
A 62-year-old woman presents with extreme fatigue and shortness of breath. The symptoms began about 24 hours ago and have progressively worsened within the last 4 hours. Vital signs on arrival are as follows: HR 90 beats per minute; BP 165/72 mm Hg; RR16/min; SpO2 98% on 4L/min supplemental oxygen by nasal cannula. 12-lead ECG demonstrates ST-segment elevation of 2 mm in leads V4-V6. In addition to an aspirin tablet, what medication would be most appropriate in the emergency management of this patient? 1. Dobutamine 2. Dopamine 3. Morphine 4. Nitroglycerin 5. Vasopressin
Nitroglycerin The patient's presentation is consistent with acute myocardial infarction. Emergency department management of patients with acute coronary syndromes - which include acute myocardial infarction and unstable angina - should consist of supplemental oxygen to maintain SpO2 >90%, oral aspirin 160-325 mg, and sublingual nitroglycerin unless contraindicated (for example, in the context of hypotension).
An 80-year-old man was treated for ventricular arrhythmias. He presents 1 month later with joint pain. He also has an unusual mask-like rash over his face and body. Discontinuation of drug therapy causes the symptoms to abate. What drug was most likely administered to this patient? 1 Tocainide 2 Quinidine 3 Procainamide 4 Phenytoin 5 Propranolol
Procainamide - often prescribed for long-term control of arrhythmias. May cause lupus like SE.
A 65-year-old male presents with complaints of palpitations for 2 days, and a 3 hour history of severe, unrelenting, sharp mid-line abdominal pain. His past medical history includes hypertension, diabetes, and peripheral vascular disease. He underwent a femoral artery stent placement 3 years ago. He has also smoked 2 packs of cigarettes a day for 30 years. His physical exam consists of a blood pressure of 80/60 mmHg, pulse rate of 120 beats/ minute, and respiratory rate of 24 breaths/minute. He has minimal tenderness in the midline of the abdomen and hypoactive bowel sounds. His WBC count is 23/uL and serum lactate of 25/uL. His plain films of the abdomen are non-specific, and a CT demonstrates small bowel thickening. An aortogram and mesenteric angiogram demonstrate an abrupt cut-off of the superior mesenteric artery, just beyond its first arterial branch, with absent flow beyond this occlusion. The next treatment of this patient should include: Operative exploration 2 Thrombolysis 3 Antibiotics 4 Bowel transplantation 5 Appendectomy
Acute mesenteric ischemia due to superior mesenteric artery thromboemboli presents as sudden onset of severe periumbilical pain, disproportionate to findings on abdominal examination. Superior mesenteric artery thrombosis usually occurs at anatomic points of narrowing. Thrombi are typically observed within 2cm of the take-off of the superior mesenteric artery and may be associated with atherosclerotic disease. Superior mesenteric artery embolism usually has a cardiac source. Typically, there is also clinical evidence of other embolic events, such as to the kidneys, and the extremities. Non-cardiac sources include arterio-arterial emboli from atherosclerosis, and aortic aneurysms and iatrogenic from intra-arterial procedures. CT scan is non-specific for acute mesenteric ischemia, however, it may demonstrate intestinal wall thickening, intestinal distension, intraabdominal fluid, pneumatosis, portal venous air, or intraperitoneal free air. If intravenous contrast is utilized, arterial occlusion may be identified. Arteriography is the gold standard to diagnose acute mesenteric ischemia due to a superior mesenteric artery embolism. Also, if surgical treatment is required, knowledge of the extent and nature of the arterial occlusion may aid in performing a mesenteric bypass. Patients with a superior mesenteric artery embolism have successfully been treated with thrombolytics; however, this is usually combined with stenting, when bowel infarction is not suspected. This method has also been combined with laparoscopy to assess bowel viability. In this particular patient, bowel infarction is probable, and requires emergent surgical exploration. An embolectomy is performed via an arteriotomy in the superior mesenteric artery, allowing thrombectomy of the proximal and distal superior mesenteric artery. The thrombosis should also be milked out by sequentially compressing the artery up to the arteriotomy. If revascularization is necessary because of poor inflow from the proximal superior mesenteric artery, then prosthetic graft or saphenous vein, in the case of infarcted bowel, should be used for bypass.
A 40-year-old woman presents with a 1-month history of having at least 3 pasty, malodorous bowel movements each day; the bowel movements are hard to flush. The patient is not taking any medications. She states that her father died of a similar illness. On examination, she is of short stature, has mild pallor, and pedal pitting edema. Her BP is 110/60 mm Hg, and she has mild bowing of her lower limbs. Laboratory analysis reveals elevated alkaline phosphatase. What is the most likely diagnosis? 1 Celiac disease 2 Whipple's disease 3 Short bowel syndrome 4 Tropical sprue 5 Lactose intolerance
Correct answer: Celiac disease Explanation Celiac disease is a hereditary disorder caused by gluten intolerance. Gluten is found in wheat, rye, barley, and oats. Patients may be asymptomatic or may present with diarrhea, abdominal discomfort, distention, and steatorrhea (stools are pale, malodorous, and difficult to flush because they float on the toilet water). Patients may have anemia from iron and folate deficiencies, osteomalacia and bone pains from calcium deficiency, and edema from hypoproteinemia. Diagnosis is confirmed by a small intestine mucosal biopsy which shows a flat mucosa due to villous atrophy, and by subsequent improvement on a gluten-free diet. Specific treatment includes a gluten-free diet. Whipple's disease mainly affects men; it is caused by the bacterium Tropheryma whippelii. It is a multisystemic disease that affects the small intestines, joints, brain, heart, and eyes. Patients can present with diarrhea, steatorrhea, abdominal pain, weight loss, and joint pains. On examination, patients may be pale with lymphadenopathy. Histological examination of a small bowel mucosal biopsy reveals PAS-positive, foamy macrophages. Treatment is with trimethoprim-sulfamethoxazole or chloramphenicol. Short bowel syndrome is usually the result of surgical resection of the intestines or a jejunoileal bypass. The malabsorption is a result of inadequate absorptive surface. Malabsorption of vitamin B12 results in paraesthesias; malabsorption of calcium results in bone pain and carpopedal spasms. Tropical sprue is an acquired disease that affects both visitors and natives of tropical areas (e.g., the Caribbean and South India). Its etiology is unknown. Patients usually present with diarrhea and weight loss; they report that their stools are soft and bulky (steatorrhea). They may also develop deficiencies of folate and cobalamin. Stool microscopy should be done to look for cysts and trophozoites. Histological examination of a small bowel mucosal biopsy aids in making the diagnosis. Treatment is with tetracycline or oxytetracycline. Lactose intolerance results from a deficiency of lactose which is a disaccharidase in the mucosal cells of the small intestine; it splits the disaccharide lactose into glucose and galactose. Patients complain of borborygmi, flatulence, nausea, abdominal cramps, pain, and diarrhea after ingesting lactose-containing food (e.g., milk). Treatment includes following a lactose-free diet.
A 76-year-old Caucasian man with a 90 pack per year smoking history presents with progressive fatigue, tachypnea, exertional dyspnea, cough, and lower extremity edema. Inspection of his chest and abdomen reveals an increased chest diameter, labored respiratory efforts with retractions and cyanosis, left parasternal and subxiphoid heaves, hepatojugular reflux, and a pulsatile liver. Additionally, there is scattered wheezes and crackles in his lungs and bilateral lower extremity edema. What physical exam findings would be most consistent with the underlying diagnosis? 1. Distended neck veins with prominent a or v waves 2. Dullness to percussion and increased tactile fremitus of the lungs 3. Pericardial friction rub 4. Laterally displaced and enlarged point of maximal impulse 5. A systolic ejection murmur located at the aortic valve area
Correct answer: Distended neck veins with prominent a or v waves This patient's diagnosis is chronic cor pulmonale. Cor pulmonale is estimated to account for 6-7% of all types of adult heart disease in the United States, with chronic obstructive pulmonary disease (COPD) due to chronic bronchitis or emphysema the most common cause (the causative factor in more than 50% of cases). Cor pulmonale usually presents chronically; however, 2 main conditions can cause acute cor pulmonale: pulmonary embolism (more common) and acute respiratory distress syndrome (ARDS). In chronic cor pulmonale, right ventricular (RV) hypertrophy (RVH) generally predominates. Common symptoms are fatigue, tachypnea, exertional dyspnea, and cough. Other symptoms include anginal chest pain (due to right ventricular ischemia), hemoptysis, and, rarely, hoarseness due to compression of the left recurrent laryngeal nerve by a dilated pulmonary artery. Anorexia, right upper quadrant abdominal discomfort, and jaundice may occur due to passive hepatic congestion. Physical exam findings reflect the underlying lung disease or pulmonary hypertension, right ventricular hypertrophy (RVH), and RV failure. Expected signs include an increase in chest diameter, labored respiratory efforts with retractions of the chest wall, distended neck veins with prominent a or v waves, and cyanosis. Wheezes and crackles may be audible upon auscultation due to underlying lung disease. S2 heart sound splitting with an accentuated pulmonic component may be found early, while a systolic ejection murmur with sharp ejection click over the region of the pulmonary artery may be heard in advanced disease, along with a diastolic pulmonary regurgitation murmur. Other cardiac findings include third and fourth sounds and the systolic murmur of tricuspid regurgitation. RVH is characterized by a left parasternal or subxiphoid heave. Hepatojugular reflux and pulsatile liver are signs of RV failure with systemic venous congestion. On percussion, hyperresonance of the lungs may be a sign of underlying COPD; ascites can be seen in severe disease. Examination of the lower extremities reveals evidence of pitting edema. Dullness to percussion and increased tactile fremitus of the lungs suggests a lung consolidation, as in pneumonia, pulmonary edema, or pulmonary hemorrhage. Pericardial friction rubs suggest inflammation of the pericardial sac, while a laterally displaced and enlarged point of maximal impulse occurs with left ventricular hypertrophy, congestive heart failure, cardiomyopathy, and ischemic heart disease. The finding of a systolic ejection murmur located at the aortic valve area most likely suggests a diagnosis of aortic stenosis.
A 40-year-old Asian-American man presents with a 3-day history of nausea and vomiting. He notes his health is good. He was started on a new medication 1 month ago for a positive PPD. What is a true statement regarding this patient? 1. His kidney functions would be abnormal 2. His AST and ALT would be elevated 3. His GGT would be normal 4. He should be getting blood work at 3 and 9-month intervals for possible drug toxicity 5. He is suffering from gastroparesis from a drug-induced neuropathy
Correct answer: His AST and ALT would be elevated Isoniazid (INH) toxicity should be suspected in any patient who is on this medicine and presents with abdominal pain. INH is not metabolized by the kidney; therefore, it should not affect kidney function. GGT is a nonspecific liver marker that goes up in any liver inflammation, so you would expect it elevated in INH-induced hepatitis. Blood work is suggested for any patient who is older than 35. The intervals to check should be at 1, 3, 6, and 9 months. INH can cause peripheral neuropathy by blocking the metabolism of pyridoxine. Gastroparesis alone is not typical for a peripheral neuropathy presentation.
A 40-year-old man is a known case of glycogen storage disease Type IA (von Gierke's disease); he presents with constant pain in his right lower leg. The pain started shortly after lunch of that day, and it is located at the mid-tibia level. He denies a previous history of a similar pain, and he does not remember any recent trauma to the area. Examination reveals a well-developed man with obvious discomfort in his right leg. His vital signs are unremarkable. He has numerous tophi on the extensor surfaces of his limbs. Pertinent positive findings are confined to his swollen right lower leg; the patient holds it still. The leg has point tenderness to palpation and motion along the anterior tibia, approximately midway between the tibial tuberosity and the anterior ankle crease. Radiological examination of the leg shows a non-displaced fracture of the right tibia corresponding to the point of maximum tenderness. All bone elements show demineralization by radiological examination. Chronic metabolic acidosis is the cause for these bone changes in this patient. What is the cause of chronic metabolic acidosis in this case? 1. High lactate levels 2. Leaching of inorganic phosphates from bones 3. Increased uric acid levels in the blood 4. Large glycogen stores in the liver 5. Large glycogen stores in the kidneys
Correct answer: High lactate levels Explanation Chronic metabolic acidosis in von Gierke's disease is due to elevated lactic acid levels. In von Gierke's disease, there is a deficiency of glucose 6-phosphatase, which produces a block in the conversion of glucose 6-phosphate to glucose. Hence, glycogen breakdown, which normally produces glucose, diverts the glucose 6-phosphate into glycolysis. The clinical features include hepatomegaly, cherubic face, growth retardation, delayed onset of puberty, weakness, convulsions, bleeding tendencies due to altered platelet function, xanthomas, osteopenia, and fractures. The metabolic effects include hypoglycemia, hyperlactacidemia, hyperuricemia, and hyperlipidemia. Definitive diagnosis is by assessment of glucose 6-phosphatase activity in fresh frozen liver tissue specimen, which is obtained by liver biopsy. The aim of the treatment is prevention of hypoglycemia and metabolic acidosis by frequent small carbohydrate-rich meals and restriction of galactose and fructose intake, which increases the lactic acid levels. Persistent elevated uric acid levels may be treated with allopurinol. In von Gierke's disease, there is an excess of lactic acid and a chronic lactic acidemia. In an attempt to buffer the chronic metabolic acidosis, the calcium phosphate salts in the bones disassociate to free inorganic phosphate anions into the blood stream. The bones become demineralized, and they easily fracture from ordinary daily stress, as in the case of the above patient. In this disease, blood uric acid levels are increased. This is due to increased generation and decreased excretion of uric acid by the kidneys. It may be that the excess glucose 6-phosphate is also diverted into the pentose pathway and ultimately into the formation of phosphoribosyl-pyrophosphate, which is the starting substrate for the formation of pyrimidines. Because in this disease there is a bottleneck in glycogen breakdown at the glucose 6-phosphate step, glycogen stores build up in the liver and in the kidney; neither event in and of itself is a cause of the chronic acidosis.
A 50-year-old man presents to the emergency department with epigastric pain. Pain is sharp with radiation into the back and accompanied by nausea and vomiting. On exam, the patient exhibits tenderness to palpation of the upper right quadrant and upper left quadrant without rigidity or guarding. Laboratory findings reveal: WBC: 9.4 k/uL Hgb: 13.5 g/dL HCT: 41% PLT: 152 k/uL Na: 135 mEq/L K: 3.2 mEq/L Cl: 100 mmol/L CO2: 28 mg/dL BUN: 8 mg/dL Cr; 0.7 mg/dL Glucose: 115 mg/dL Calcium: 8.1 mg/dL Total Protein: 6 g/dL Albumin: 3 g/dL Total Bilirubin: 1.2 mg/dL Alkaline Phosphatase: 140 U/L AST: 200 U/L ALT: 155 U/L Amylase: 477 U/L Lipase: 625 U/L LDH: 30 Of the following, what is the most important component of the patient's treatment? 1. Oral creon 2. Oral omeprazole 3. Intravenous normal saline 4. Intravenous meropenem 5. Surgical debridement
Correct answer: Intravenous normal saline Explanation Intravenous normal saline is correct. The patient has acute pancreatitis. Recommended treatment includes bowel rest with intravenous fluids and intravenous pain medication. Dehydration may develop in pancreatitis due to leakage of fluid from the pancreas. IV hydration is critical to avoid morbidity and mortality from acute pancreatitis. Oral creon and oral omeprazole are incorrect. These therapies are used in the treatment of chronic pancreatitis. Intravenous meropenem is incorrect. Antibiotics are only indicated in acute pancreatitis when the patient has a poor predicted outcome using Ranson's criteria or signs of infection. Ranson's criteria states that the presence of the following factors indicate a worse prognosis: age greater than 55 years old, white blood cell count greater than 16,000, serum glucose greater than 200 mg/dL, serum lactate dehydrogenase greater than 350 units/L, and aspartate aminotransferase greater than 250 units/L. None of these factors are present in this patient, so antibiotics should not be given at this time. Surgical debridement is incorrect. Surgery is reserved as a last resort in acute pancreatitis.
A 60-year-old man presents with a 6-month history of pain in his knee joints. It has been gradually increasing, and now limits his activities. X-ray reveals osteoarthritic changes in both knee joints. You consider treating him with a non-steroidal anti-inflammatory drug (NSAID); however, you note that he has a history of gastric ulcers. In the past, the patient has not tolerated proton pump inhibitors well; he has had several episodes of severe headache, diarrhea, and dizziness when treated with them. What is the best way to reduce the risk of NSAID-induced gastric mucosal injury in this patient? 1. Misoprostol 2. Milk of Magnesia 3. Octreotide 4. Ranitidine 5. Sucralfate
Correct answer: Misoprostol Explanation Misoprostol is a prostaglandin analogue that acts on the gastric parietal cells to reduce gastric acid secretions. Proton pump inhibitors (PPI) are better tolerated than misoprostol and may be preferred over misoprostol, even in NSAID-induced gastric injury. Ideally, this patient should be treated with a selective COX-2 inhibitor NSAID with a PPI; however, considering his poor tolerance to PPI (which is rare), misoprostol is the next best choice. Misoprostol causes nausea, diarrhea, abdominal pain, and dysmenorrhea. It should be avoided in pregnancy, as it may cause an abortion. Milk of magnesia is an antacid that acts by neutralizing the acid in the stomach; it is commonly used in dyspepsia. Octreotide is a somatostatin analogue that inhibits gastric and pancreatic secretion. It is used in Zollinger-Ellison syndrome. H2 receptor blockers (e.g., ranitidine) are used to treat gastroesophageal reflux disease, as well as gastric, duodenal, and stress ulcers; however, they are not the 1st choice in NSAID-induced injury because a double dose would be required. Sucralfate is an ulcer protective; it forms a coating on the ulcer crater and acts as a barrier to acid and pepsin.
A 62-year-old woman has an unexplained weight loss. She has a vague, non-descriptive pain in her stomach accompanied by diarrhea. When her son sees that she has become yellow, he insists that she seek medical attention. Numerous tests are done, including endoscopic retrograde pancreatography, and she is told that she has pancreatic cancer. The tests reveal that the pancreatic cancer is located at the head of the pancreas. What percentage of pancreatic cancers are located at the head of the pancreas? 1 1-10 2 20-30 3 45-55 4 70-80 5 90-99
Correct answer: 70-80 About 70-80% of pancreatic cancers are located at the head of the pancreas. Despite the fact that pancreatic tumors that arise in the head of the pancreas may present with obstructive symptoms, the tumor still is usually advanced by the time of diagnosis. Other pancreatic cancers are located in the body and tail of the pancreas. Tumors that arise in the tail of the pancreas are even more insidious.
A 45-year-old man is admitted to the medical floor of a hospital with increasing jaundice, swollen legs, and episodes of disorientation for the last several weeks. His abdomen is distended and he looks ill. According to the family, he is an alcoholic and has recently been diagnosed with cirrhosis of liver. On examination, he has a temperature of 99 F, BP 100/72 mm Hg, and his pulse is 86/minute. Sclerae are icteric, and he has 2+ pitting pedal edema. Lungs are clear, and heart sounds are normal. Abdomen is distended with moderate ascites, caput medusae, and no tenderness. Liver and spleen are not palpable. He is awake but drowsy and oriented to person and place but not to time. He does have a fine tremor in his hands. Labs show WBC 8000/uL, platelets 100,000/uL, Hb 12g%, AST 76 U/L, ALT 56 U/L, AP 62 U/L, and ammonia is 124 mg/dL. At this time which of the following is most important to avoid full decompensation into hepatic encephalopathy? 1. Increase dietary protein 2. Prevent constipation 3. Sedate the patient to avoid injury from disorientation 4. Start IV antibiotics empirically anticipating infection 5. Add thiamine and folic acid for nutritional support
Correct answer: Prevent constipation Explanation This patient is already in the initial phases of hepatic encephalopathy due to alcoholic liver disease as evidenced by the disorientation and tremors, also known as asterixis. The primary cause of hepatic encephalopathy is unclear. Metabolic abnormalities due to liver dysfunction, resulting in a spectrum of neuropsychiatric signs and symptoms, are seen. High levels of ammonia are found in the blood. Constipation causes increased ammonia production and absorption due to prolonged intestinal contact and aggravates the condition. Lactulose should be administered frequently to eliminate the ammonia in the stool. It is an indigestible sugar that acts as an osmotic laxative by increasing the water content of the stool and promoting bowel movements. It is digested by the colonic bacteria, and the acidic remains convert ammonia into ammonium ions in the colon, which are then excreted in the stool. Oral antibiotics can lower blood ammonia levels by decreasing ammonia production and absorption. The commonly used ones are neomycin, metronidazole, vancomycin, and, lately, rifaximin. The last three are better tolerated than neomycin. However antibiotics have their side effects and can cause bacterial overgrowth syndromes. Their main use continues to be in patients who cannot tolerate disaccharides like lactulose. Acarbose and fermentable fiber can also cause decrease in intestinal ammonia production and absorption. Newer studies with sodium benzoate are ongoing. Benzoate and glycine react to form hippurate, and for every mole of benzoate utilized this way, one mole of nitrogen is excreted in the urine, thereby enhancing ammonia metabolism. This, however, still needs to be studied further to be used widely. Studies with ornithine-aspartate are also being done as a stimulator of ammonia metabolism. All the products mentioned are yet to replace lactulose as the first line of treatment but are potentially useful once more studies are done. Other precipitating factors for hepatic encephalopathy include azotemia, hypokalemia, gastrointestinal bleeding, high protein diet, alkalosis, infection, sedatives, and other hepatotoxic agents. High protein diet is a contraindication in this condition, as protein catabolism causes increase in ammonia levels. Daily protein should be restricted to 40 g/day. Sedation of the patient should also be avoided, since sedatives cause cerebral depression and worsening of encephalopathy. These drugs are also not metabolized adequately by the diseased liver. Though infection is an important precipitating factor for hepatic encephalopathy, empiric treatment is not recommended. However, early and adequate treatment of an infection should be done, especially for spontaneous bacterial peritonitis. In fact multiple randomized control trials have been done regarding antibiotic prophylaxis for SBP, and they have shown not only a decrease in bacterial infections but also a significant reduction in mortality. Prophylaxis is recommended in cirrhotic patients with risk factors for SBP like GI bleeding, prior history of SBP, and low ascitic fluid protein. Thiamine and folic acid should be added for nutritional support to all alcoholic patients, since they are malnourished and vitamin depleted. However, this will not change the outcome in hepatic encephalopathy as quickly or as much as avoiding constipation.
A 27-year-old man eats a meal consisting of ham casserole with potatoes and green beans. 3 hours later, he abruptly begins vomiting; he experiences profuse watery diarrhea, abdominal cramping, and headache. Examination determines that he is diaphoretic and salivating; he has normal vital signs. The patient is encouraged to rest and take copious fluids. The following evening his symptoms subside, and he is able to return to routine activities within 24 hours of becoming ill. This food-borne illness is most likely to be caused by what organism? 1 Clostridium perfringens 2 Clostridium difficile 3 Staphylococcus aureus 4 Shigella sonnei 5 Yersinia enterocolitica
Correct answer: Staphylococcus aureus Staphylococcal food poisoning results from the ingestion of heat-stable enterotoxin B. An acute clinical syndrome of salivation, vomiting, and abdominal cramps is followed by watery diarrhea. The onset of vomiting and nausea within the 1st hours following ingestion is characteristic of staphylococcal intoxication (refer to the table). S. aureus are tolerant of high-salt concentrations, making salted meats (e.g., ham) a common source of intoxication. Clostridium perfringens produces several enterotoxins that cause diarrhea. 1 of the most common agents of food-borne illness, it is transmitted primarily by meat and poultry. Symptoms include watery diarrhea, vomiting, and abdominal cramps (usually without fever). Unlike the rapid onset of S. aureus, symptoms of clostridial food poisoning occur 12 - 18 hours after ingestion. Clostridium difficile is also capable of producing an enterotoxin and a cytotoxin, and it is not associated with food-borne illness. This organism causes colitis frequently associated with antibiotic use. Patients, usually the elderly or hospitalized, present with mild to moderate diarrhea and abdominal cramps; they may develop more serious illnesses such as pseudomembranous colitis or an acute abdomen. Shigella causes enteritis within 12 - 18 hours after ingestion of as few as 200 organisms. This clinical syndrome consists of acute onset fever and abdominal cramps which are followed by an enterotoxin-mediated acute onset of copious watery diarrhea. Within 24 - 48 hours, dysentery occurs as a result of mucosal invasion. Shigella sonnei is the most common etiologic agent of shigellosis in industrialized countries. Yersinia enterocolitica is transmitted by milk, water, or solid foods; it is often outbreak-associated. Patients experience diarrhea, fever, and abdominal pain 16 - 72 hours after consuming food. Y. enterocolitica may mimic acute appendicitis as a result of mesenteric adenopathy or terminal ileitis. Refer to the information below. Incubation period (hours); clinical syndromes; likely organisms: 1 - 8; nausea and vomiting; Staphylococcus aureus, Bacillus cereus (emetic toxin), and Norwalk virus 8 - 12; diarrhea and abdominal cramps; Bacillus cereus (enterotoxin) and Clostridium perfringens 12 - 72; diarrhea, fever; mucus, dysentery, as well as neurologic and extraintestinal symptoms; Salmonella enteritidis, Escherichia coli, Campylobacter jejuni, Shigella, Clostridium botulinum, and Listeria monocytogenes
A 2-month-old male infant is brought by his mother for his 2 month well baby assessment. He had a completely normal birth history and has been growing well. The mother nursed him for about 3 weeks and then switched to a cow milk based formula after she developed nipple soreness. He took the formula well but a week later became very fussy, crying nonstop for more than 3 hours a day, usually in the evening. He does this just about every day according to mom. The crying does not seem to be related to feedings, and there is no vomiting or diarrhea. He has not been sick or congested and has had normal stools and wet diapers. He does seem gassier during these crying episodes, and he is inconsolable despite holding and rocking. The mother also has 2 toddlers at home and is becoming very frustrated and anxious. His vitals on exam are normal, as is the rest of his physical examination. What is most appropriate management for this infant? 1. Simethicone 2. Supportive measures 3. Hydrolyzed formula 4. Dicyclomine 5 Prone sleeping position
Correct answer: Supportive measures Explanation Colicky infants cry excessively (more than 3 hours per day, more than 3 days per week and for more than 3 weeks) without any identifiable need. It occurs in all socioeconomic, racial, and ethnic groups with no sex preference. Colic begins typically at 2 weeks of age, peaks at 6 weeks, and usually resolves completely by 16 weeks. Crying spells are episodic and unrelated to feeding with a diurnal pattern; crying is increased in the evening and night. Such infants are inconsolable for long intervals and cry with extreme intensity. They draw up their legs, have tense abdomens, arch their backs, and become 'gassy,' suggesting a GI etiology. However, aerophagia or swallowing of air will occur with excessive crying and the presentation of aerophagia and gas does not coincide with the timing of colic, thus the excessive crying accompanying colic usually leads to aerophagia and subsequent gas. It is believed to be multifactorial with behavioral, social, and neurodevelopmental components. Diagnosis is made by history along with a normal physical exam in a healthy and normally growing infant. Appropriate management should be directed toward supportive interventions, such as recommendations to simulate the fetal environment with tight swaddling, rapid swinging or juggling, non-nutritive sucking, and steady white noise. Holding for prolonged periods has been shown to decrease the intensity but not the duration of crying. Multiple formula changes or dietary changes for a breastfeeding mother have not been shown to benefit colic in otherwise healthy infants, though a subgroup of infants have colic that may be caused by allergies. If the history is suggestive for this, and emesis, diarrhea, blood in the stool, severe eczema, or urticaria are present, then a short trial of hydrolyzed formula may be appropriate. Dicyclomine, an anticholinergic, has been shown to decrease symptoms of colic; however, it has also caused sedation, apnea, seizures, and coma, causing manufacturers to contraindicate its use in infants younger than 6 months of age. Simethicone has also been suggested for gas and colic, but it has not been shown to be more effective than placebo in randomized controlled trials. Increased crying can be associated with gastroesophageal reflux (GER), however only 2% to 4% of infants with colic have been shown to have GER. Conservative measures of thickening feedings and trial of anti-reflux medication may be considered if symptoms are consistent with possible GER. Prone sleep positioning, once recommended, has the potential for increased risk of infant death syndrome. Lactose intolerance is uncommon in the first year of life and includes nonspecific symptoms that may be colic-like with bloating and gaseous distention, but also vomiting, spit-up, and nonspecific diarrhea, which are not exhibited here. There are very few indications for lactose-free formula; these include galactosemia and secondary to small intestinal injury not evident in this problem.
6 hours after eating raw fish and other seafood at a Japanese restaurant, a couple developed nausea and vomiting, watery diarrhea, abdominal cramps, and fever. What organism is likely involved in the patients' condition? 1. Salmonella enteritidis 2. Bacillus cereus 3. Clostridium perfringens 4. Staphylococcus aureus 5. Vibrio parahaemolyticus
Correct answer: Vibrio parahaemolyticus Explanation Bacillus cereus cause 2 syndromes: (1) one involves a short incubation period with nausea and vomiting and is similar to staphylococcal food poisoning; (2) the other involves a long incubation period (18 hours) with watery, non-bloody diarrhea and resembles clostridial gastroenteritis. Clostridium perfringens cause food poisoning with an 8- to 16-hour incubation period. It is characterized by watery diarrhea with cramps and little vomiting. It resolves in 24 hours. Staphylococcus aureus cause food poisoning characterized by vomiting being more prominent than diarrhea. The disease is due to ingestion of enterotoxin, which is performed in foods and hence has a short incubation period (1-8 hours). Vibrio parahaemolyticus are a marine organism transmitted by contaminated seafood, especially when raw fish is eaten. The clinical picture varies from mild to quite severe watery diarrhea, nausea and vomiting, abdominal cramps, and fever. The illness is self-limited, lasting about 3 days. Salmonella enteritidis cause enterocolitis after an incubation period of 6-48 hours. The disease begins with nausea and vomiting and then progresses to abdominal pain and diarrhea, which can vary from mild to severe and present with or without blood. Usually, the disease lasts a few days and is self-limited.
A 25-year-old Asian man presents with a 1-month history of an inability to see well while walking home from work at night. He also reports that he has been getting sore throats more frequently. He is a recent immigrant from his home country; he lives alone and eats mainly a rice-based diet. On examination, you note pericorneal and corneal opacities. What is the most likely diagnosis? 1. Vitamin A deficiency 2. Pyridoxine deficiency 3. Vitamin C deficiency 4. Niacin deficiency 5. Vitamin E deficiency
Correct answer: Vitamin A deficiency Explanation In cases of vitamin A deficiency, patients can present with inability to see well in dim light or night blindness. There may also be conjunctival and corneal xerosis, as well as pericorneal and corneal opacities, and Bitot's spots. Bitot's spots are a collection of keratin appearing as triangular foamy spots on the conjunctiva. The patient may also have xeroderma, hyperkeratotic skin lesions, and increased susceptibility to infections. Causes include inadequate dietary intake and malabsorption. Dietary sources of vitamin A include fish, liver, egg yolk, butter, cream, dark green leafy vegetables, yellow fruits, and vegetables. Vitamin E deficiency may cause a hemolytic anemia in premature infants. Laboratory investigations reveal low plasma tocopherol levels, a low hemoglobin level, reticulocytosis, hyperbilirubinemia, and creatinuria. Causes of vitamin E deficiency in premature infants include limited placental transfer of vitamin E and the resultant low levels at birth combined with its relative deficiency in the infant diet. Dietary sources for older children and adults include wheat germ, vegetable oils, egg yolk, and leafy vegetables. Niacin deficiency causes pellagra, which is characterized by: A symmetrical dermatitis usually on parts of the body exposed to sunlight Scarlet glossitis and stomatitis Diarrhea Mental aberrations, such as memory impairment, depression, and dementia. These may appear alone or in combination. Causes include inadequate dietary intake, especially in patients with corn-based diets and alcoholism. Dietary sources include legumes, yeast, meat, and enriched cereal products. In cases of vitamin C (or ascorbic acid) deficiency, patients can present with bleeding tendencies (as a result of weakened capillaries) and impaired wound healing due to impaired formation of connective tissue. On examination, the gums may be swollen and friable; the teeth may be loose. There may also be multiple splinter hemorrhages on the nails and ecchymoses, especially over the lower limbs. Causes include inadequate dietary intake and certain conditions, such as pregnancy and lactation, which increase vitamin C requirements. Dietary sources of vitamin C include citrus fruits, such as oranges, lemons, and tangerines, as well as tomatoes and potatoes. In cases of pyridoxine (or vitamin B6 deficiency), patients can present with peripheral neuropathy, seborrheic dermatosis, glossitis, and cheilosis. Laboratory investigations reveal anemia with lymphopenia. Causes include malabsorption as well as medications, such as isoniazid and penicillamine. Dietary sources of vitamin B6 include liver, legumes, whole grain cereals, and meats.
A 30-year-old man presents with a 1-year history of watery diarrhea. He has lost 5 kg. He has joint pains of 1-month duration. His stool examination for occult blood is negative. Stool culture is negative. No ova and cyst are seen in the stool. Laboratory tests fail to reveal rheumatoid factor or anti-nuclear antibodies. Serum calcium, phosphorous and alkaline phosphatase is 9.2 mg/dl, 4 mg/dl and 110 U/L respectively. A biopsy taken from the upper intestine shows distended macrophages in the lamina propria, which is found to be positive with PAS stain. What is the most likely diagnosis in this case? 1. Celiac sprue 2 Tropical sprue 3. Whipple disease 4. Disaccharidase deficiency 5. Amebic dysentery
Correct answer: Whipple disease Explanation Whipple disease is a rare condition that can involve any organ of the body but principally affects the intestine, central nervous system, and joints. It is caused by gram-positive actinomycetes called Tropheryma whippelii. Histologically, the small intestinal mucosa is laden with distended macrophages in the lamina propria. These macrophages contain periodic acid-Schiff positive granules and rod-shaped bacilli by electron microscopy. These patients usually present with a form of malabsorption, including diarrhea and weight loss, and generally respond to antibiotic therapy. Celiac sprue is a rare chronic disease in which there is a characteristic mucosal lesion of the small intestine in the form of blunting of the villi and an overall increase in plasma cells, lymphocytes, macrophages, and eosinophils in the lamina propria. Also known as the gluten-sensitive enteropathy, non-tropical sprue, and celiac disease, it is due to sensitivity to gluten, which is an alcohol soluble, water insoluble protein component called gliadin of wheat and other closely related grains. Detection of circulating anti-gliadin or anti-endomysial antibodies strongly favors the diagnosis. Both the symptoms and the mucosal histology improve on withdrawal of gluten from the diet. Tropical sprue is a celiac-like disease occurring in people living or visiting the tropics. Bacterial overgrowth by an enterotoxigenic organism is said to be the cause for this disease. Intestinal changes are similar to celiac sprue, but injury is seen at all levels of the small intestine in contrast to celiac sprue, where it is concentrated in the proximal small intestine. It is mainly treated by broad-spectrum antibiotics. The disaccharidases, of which the most important is lactase, are located in the apical cell membrane of the villous absorptive epithelial cells, the deficiency of which leads to the incomplete breakdown of the disaccharide lactose into its monosaccharides, glucose, and galactose, leading to diarrhea from the unabsorbed lactose. Bacterial fermentation of the unabsorbed sugars leads to increased hydrogen production, which is readily measured in the exhaled air by gas chromatography. There are both hereditary and acquired forms. Histologically in both, there is no abnormality of the mucosal cells of the bowel. Malabsorption is promptly corrected when exposure to milk and milk products is terminated. Amebic dysentery is caused by Entamoeba histolytica, a protozoan, and generally causes diarrhea with blood and mucus. A stool examination will show the cysts and trophozoites of Entamoeba histolytica, which is diagnostic.
A 30-year-old man presents with a 1-year history of watery diarrhea. He has lost 5 kg. He has joint pains of 1-month duration. His stool examination for occult blood is negative. Stool culture is negative. No ova and cyst are seen in the stool. Laboratory tests fail to reveal rheumatoid factor or anti-nuclear antibodies. Serum calcium, phosphorous and alkaline phosphatase is 9.2 mg/dl, 4 mg/dl and 110 U/L respectively. A biopsy taken from the upper intestine shows distended macrophages in the lamina propria, which is found to be positive with PAS stain. What is the most likely diagnosis in this case? 1. Celiac sprue 2. Tropical sprue 3. Whipple disease 4. Disaccharidase deficiency 5. Amebic dysentery
Correct answer: Whipple disease Whipple disease is a rare condition that can involve any organ of the body but principally affects the intestine, central nervous system, and joints. It is caused by gram-positive actinomycetes called Tropheryma whippelii. Histologically, the small intestinal mucosa is laden with distended macrophages in the lamina propria. These macrophages contain periodic acid-Schiff positive granules and rod-shaped bacilli by electron microscopy. These patients usually present with a form of malabsorption, including diarrhea and weight loss, and generally respond to antibiotic therapy. Celiac sprue is a rare chronic disease in which there is a characteristic mucosal lesion of the small intestine in the form of blunting of the villi and an overall increase in plasma cells, lymphocytes, macrophages, and eosinophils in the lamina propria. Also known as the gluten-sensitive enteropathy, non-tropical sprue, and celiac disease, it is due to sensitivity to gluten, which is an alcohol soluble, water insoluble protein component called gliadin of wheat and other closely related grains. Detection of circulating anti-gliadin or anti-endomysial antibodies strongly favors the diagnosis. Both the symptoms and the mucosal histology improve on withdrawal of gluten from the diet. Tropical sprue is a celiac-like disease occurring in people living or visiting the tropics. Bacterial overgrowth by an enterotoxigenic organism is said to be the cause for this disease. Intestinal changes are similar to celiac sprue, but injury is seen at all levels of the small intestine in contrast to celiac sprue, where it is concentrated in the proximal small intestine. It is mainly treated by broad-spectrum antibiotics. The disaccharidases, of which the most important is lactase, are located in the apical cell membrane of the villous absorptive epithelial cells, the deficiency of which leads to the incomplete breakdown of the disaccharide lactose into its monosaccharides, glucose, and galactose, leading to diarrhea from the unabsorbed lactose. Bacterial fermentation of the unabsorbed sugars leads to increased hydrogen production, which is readily measured in the exhaled air by gas chromatography. There are both hereditary and acquired forms. Histologically in both, there is no abnormality of the mucosal cells of the bowel. Malabsorption is promptly corrected when exposure to milk and milk products is terminated. Amebic dysentery is caused by Entamoeba histolytica, a protozoan, and generally causes diarrhea with blood and mucus. A stool examination will show the cysts and trophozoites of Entamoeba histolytica, which is diagnostic.
A 60-year-old man with a past medical history of COPD, hypertension, peptic ulcer disease, and hyperlipidemia is being evaluated for a 1-hour history of severe pain in the mid-abdominal region. His history includes abdominal surgery 6 months ago for a small intestinal obstruction, and his pain emanates from his incisional site. He also states that he has shaking chills, nausea, and vomiting, but denies chest pain, shortness of breath, wheezing, or abnormal bowel habits. Upon physical exam, he is found to be hypotensive, tachycardic, diaphoretic, and in acute painful distress. The abdomen is obese; bowel sounds are hypoactive; and rebound tenderness is noted. There is an exquisitely tender 2.0 cm ventral hernia located inferior to the umbilicus that is indurated, tender to palpation, and is not reducible. Auscultation of the mass reveals the presence of bowel sounds. What is the most appropriate therapeutic intervention for this patient at this time? 1. Broad spectrum antibiotics 2. Maintain a solid diet 3. Apply hot packs to the hernia 4. Oral motrin for pain 5. Attempt hernia reduction
Correct answer: Broad spectrum antibiotics This patient's presentation is significant of a strangulated hernia. Manifestations of hernia strangulation, a medical emergency, include an exquisitely tender mass associated with systemic signs and symptoms, such as intestinal obstruction, toxic appearance, peritonitis, or meeting sepsis criteria. General surgery should be consulted immediately.Additional interventions include the administering broad-spectrum IV antibiotics, such as cefoxitin, providing fluid resuscitation and adequate narcotic analgesia, and obtaining preoperative laboratory studies. If there is any concern for strangulation, do not attempt hernia reduction. The reintroduction of ischemic, necrotic bowel back into the peritoneal cavity can result in subsequent perforation and sepsis. Bedside ED US, using a linear high frequency probe with color or power Doppler of the hernia sac can be useful in these borderline cases to establish the presence or absence of blood flow.
A 36-year-old Jewish man presents with abdominal pain and diarrhea, as well as a 2-day history of a low-grade fever. He is initially treated with fluids and antibiotics, but his symptoms worsen. On evaluation, he is diagnosed with Crohn's disease. He is a nonsmoker, but has alcoholic drinks on alternate days; he underwent an appendectomy 2 years ago. His mother was diagnosed with irritable bowel syndrome (IBS) 10 years ago. What in his history is a risk factor for Crohn's disease? 1. His age 2. Alcohol consumption 3. His Jewish ancestry 4. Family history of IBS 5. Appendectomy
Correct answer: His Jewish ancestry Crohn's disease (CD) is more common in the Jewish population; it is more common in whites than African-Americans. Environmental factors may play a role in its etiopathogenesis because African-Americans seem to have the same risk as whites, but African blacks have a lower risk. The age of onset is between 15 to 30 years with a second peak between 60 to 80 years. CD is an inflammatory bowel disease affecting any part of the GIT from mouth to anus; the most commonly affected region is the ileocecal region. Symptoms include abdominal pain, diarrhea (which may be bloody), vomiting, fever, and weight loss. Extraintestinal symptoms (e.g., arthritis, uveitis) may also develop. Smoking, not alcohol, is a risk factor for CD. A family history of IBS is unrelated to CD; however, a family history of CD itself is a risk factor. An appendectomy neither increases the risk nor is protective in CD.
A 56-year-old woman presents with heartburn and dyspepsia. She was diagnosed with osteoarthritis 4 years prior to presentation, and for the past 18 months, she has been managing pain with naproxen. An upper gastrointestinal endoscopy shows several areas of gastric irritation and a single ulcer about 3 mm in diameter. The gastroenterologist suggests that the patient be tested for Helicobacter pylori infection. What is the gold standard for diagnosis of this infection? 1 Culture 2 Histology 3 Antigen test 4 Western blot serology 5 Urease breath test
Correct answer: Histology The long term use of non-steroidal antiinflammatory drugs (NSAIDS) in treatment of arthropathies carries about a 4-fold increase in the relative risk of duodenal or gastric ulcers. This risk is multifactorial; 1 risk factor for development of gastric ulcers in patients taking NSAIDS is infection with Helicobacter pylori. Histologic examination of gastric biopsies obtained during endoscopy is the gold standard for diagnosis of this infection based on sensitivity and specificity (refer to the table). This approach has the added benefit of allowing simultaneous assessment of the pathology of the lesion, which can rule out neoplasms. Culture of H. pylori is difficult and requires considerable microbiologic expertise. However, indicators in the cultivation medium can detect urease. Invasive procedures can be preceded by the urease breath test as a screening for the presence of the organism. Helicobacter serology by ELISA is limited to screening and detection of a past or present infection. An antigen test is also available that detects H. pylori protein in the stool.
A 2-month-old infant who is breastfeeding presents with low hemoglobin levels. The infant was born at home and the mother received no prenatal care; she did not, and does not, take any medications. Family history is unremarkable. On examination, the infant appears healthy. What is the underlying mechanism of low hemoglobin levels in this infant? 1. Folate deficiency 2. Vitamin K deficiency 3. Breastfeeding 4. Infection 5. Normal process
Correct answer: Normal process Intrauterine hypoxia stimulates erythropoietin. Infants are born with high levels of hemoglobin and erythrocytes in their blood which downregulates erythropoietin. A progressive drop in hemoglobin during the 1st 2 - 3 months of life is the signal to the infant's body to increase erythrocyte production. Called physiologic anemia of infancy, this is considered normal and no treatment is needed. Folate deficiency in the mother would more likely result in low birth weight, prematurity, and/or neural tube defects. Vitamin K deficiency is the cause of hemorrhagic diathesis; its manifestations would be obvious. Breastfeeding when the mother did not, and does not, take any drugs cannot be the cause of anemia in a 2-month-old infant. Common infections in a 2-month-old infant include the digestive tract, ear, skin, and upper respiratory tract; all will have relevant symptoms.
A 23-year-old Caucasian woman presents with weakness, fatigue, and occasional palpitations starting 6 months ago. She took a multivitamin supplement with transient improvement. In the last 3 months, blisters appeared on her elbows, which were associated with intense itching. Her only medication is an implantable contraceptive inserted 3 years ago. There is no relevant past medical history, and her family history is also unremarkable. Vital signs are BP 120 / 80 mm Hg, HR 94 bpm, RR 18 rpm temperature 36.3° C (97.2° F). On physical examination, she is alert and oriented, with no dyspnea or cyanosis. Examination of the skin reveals bullous lesions in her elbows. The remainder of the examination shows no abnormalities. A complete blood count shows hemoglobin 12 g/dL, hematocrit 31%, Mean Corpuscular Volume (MCV) 78 fl, Mean Corpuscular Hemoglobin Concentration (MCHC) 29 pg/L, Leukocytes 7.400 /mm3 w/normal differential, and platelets 218,000 /mm3. A skin biopsy reveals a neutrophilic infiltrate, with fibrin and microblisters, most dense at the tips of the dermal papillae and containing IgA on immunofluorescence staining. What finding is most probable on subsequent workup? 1 Positive transglutaminase antibodies 2 Positive ASCA antibodies 3 A warm spot on scintigraphy with 99mTc-labelled red cells 4 Positive stool guaiac test 5 Reduced serum erythropoietin
Correct answer: Positive transglutaminase antibodies Dermatitis herpetiformis produces erythematous, pruriginous bullous lesions on the extensor surfaces of the elbows, knees, scalp, neck, and buttocks. These lesions, sometimes called skin celiac disease, are found in 10% of patients with celiac disease, and are highly specific for it; nearly all patients with dermatitis herpetiformis have histologic evidence of celiac disease on small-bowel biopsy. However, in many cases the enteropathy is subclinical. Histopathologic examination of the bullae reveals deposits of anti-gliadin IgA antibodies in the basal membrane of the epidermis. In most cases, the skin lesions involute after the patient is put on a gluten-free diet. However, this can take several months. In patients with severe pruritus, the mainstay of treatment is dapsone, which is highly effective in relieving this symptom. Second-line options include sulfasalazine and sulfapyridine. Celiac disease can present with isolated nutrient deficiencies (e.g. iron, folate, B-complex vitamins), and therefore could explain this patient's iron deficiency anemia. As transglutaminase antibodies are found in 70% of cases of celiac disease, there is a high probability that this assay will be positive in this patient, whether or not the anemia is caused by celiac disease. The more florid presentation of celiac disease - with overt steatorrhea, weight loss, and malnutrition - is becoming less common in the United States. Therefore, the physician must be aware of the possibility of celiac disease in patients with milder symptoms, such as occasional diarrhea with bulky stools, bloating, or more rarely, isolated micronutrient deficiencies. Anti-Saccharomyces (ASCA) antibodies are associated with Crohn's disease (60-70% of patients are positive). Lesions in these patients include oral aphthous ulcers, pyoderma gangrenosum, and rheumatoid nodules, as well as signs of non-cutaneous disease (e.g. enterocutaneous fistulas, malnutrition, micronutrient deficiencies). A 99mTc-red cell scintigraphy could be positive because occult gastrointestinal bleeding is still a possibility. Causes of a negative iron balance include: Deficient intake, which is most prevalent in poor countries. Pregnancy. Chronic bleeding (gynecologic, gastrointestinal, hematuria). Iron deposition in tissues (e.g., cardiac hemosiderosis) The deficiency in hemoglobin synthesis in the bone marrow tends to cause a compensatory rise in erythropoietin levels, which produces erythroid hyperplasia in the bone marrow. Other findings include absence of sideroblasts and stainable iron in the bone marrow's reticular network. However, bone marrow biopsy has been largely abandoned for diagnosis of iron-deficiency anemia because of interobserver variability and the availability of less invasive tests of iron status, particularly serum ferritin and total iron binding capacity (TIBC).
How is a sigmoid volvulus treated? 1. Barium enema 2. Emergency colon resection 3. Rigid proctosigmoidoscopy 4. Oral purgatives 5. Bowel stimulants
Correct answer: Rigid proctosigmoidoscopy When sigmoid volvulus causing large bowel obstruction is suspected from the history and physical exam, or when it is seen on an abdominal radiograph, the diagnostic and therapeutic modality is rigid proctosigmoidoscopy with decompression and untwisting of the volvulus.
A woman is diagnosed with primary tuberculosis. She is 5 months pregnant. What drug should be avoided in this patient? 1. Isoniazid 2. Rifampin 3. Ethambutol 4. Streptomycin 5. Pyrazinamide
Correct answer: Streptomycin Tuberculosis in pregnancy is treated with an initial regimen of isoniazid, rifampin, and ethambutol. Because the teratogenicity of pyrazinamide is not determined, pyrazinamide is added only if a resistance to the other drugs is documented/suspected, and susceptibility to pyrazinamide is likely. Streptomycin is contraindicated in pregnancy because it may cause congenital defects. Isoniazid, rifampin, and ethambutol are considered safe to use during pregnancy.
A 35-year-old woman presents with vomiting and watery diarrhea that began 4 hours after eating a chicken sandwich at her workplace cafeteria. She reports that 2 other workmates have also developed similar symptoms. There are no leukocytes or cysts on examination of her stool. What is the causative organism? 1. Staphylococcus aureus 2. Clostridium perfringens 3. Escherichia coli serotype O157:H7 4. Giardia lamblia 5. Salmonella typhi
Correct answer: Staphylococcus aureus In cases of Staphylococcus aureus food poisoning, patients present with severe nausea and vomiting. They may also develop diarrhea, abdominal pain, and occasionally headache and fever. Stools are watery but not blood stained. The onset of symptoms is 1 to 6 hours after ingesting the contaminated food. Common culprits include processed meat, dairy products, and potato salads, especially when prepared by food handlers with skin infections that are left at room temperature. Several persons may be similarly affected. There are no fecal leukocytes or trophozoites on examination of the stool. Management includes intravenous replacement of fluid and electrolytes. In cases of Clostridium perfringens, food poisoning, patients present with watery diarrhea 8 to 24 hours after ingesting contaminated food like inadequately cooked meat or legumes. No fecal leukocytes or trophozoites are present on stool examination. In cases of Escherichia coli serotype O157:H7 food poisoning, patientspresent with watery diarrhea, which can become bloodstained. Sources of infection include undercooked beef. A stool examination reveals polymorphonuclear leukocytes. Giardia lamblia can be acquired by drinking contaminated water. Patients present with watery diarrhea. The incubation period is around 1 week. There are trophozoites in the stool, but no leukocytes. Patients with Salmonella typhi food poisoning usually present with bloody diarrhea. Sources of infection include beef, poultry, eggs, and dairy products. Examination of the stool reveals mononuclear leukocytes.
A 52-year-old man presents with heartburn, sometimes associated with reflux of sour-tasting material into the mouth. Some of these episodes are accompanied by increased salivation and coughing. He denies any bleeding or abnormalities in his stools. He is slightly overweight and has mild hypertension, which is well controlled with enalapril. Vital signs are within normal ranges and the physical examination is unremarkable. An upper endoscopy reveals coalescing linear erosions throughout the esophageal circumference and a 5.5-cm-long, circumferential cherry-red patch above the gastroesophageal junction. Biopsy of the patch reveals columnar metaplasia, but no dysplasia. What can improve the patient's symptoms most effiiciently? 1. Laparoscopic fundoplication 2 Start omeprazole 3 Esophagectomy 4 Radiofrequency thermal coagulation of the lower esophageal wall 5 Laser ablation of the Barrett's mucosa
Laparoscopic fundoplication Laparoscopic fundoplication leads to long-term relief for about 90% patients with gastroesophageal reflux disease (GERD). The most common technique is the Nissen procedure (i.e., suturing the gastric fundus to the lower esophagus throughout the full esophageal circumference). Side effects are uncommon and mostly minor: abdominal bloating, inability to belch, paraesophageal herniation, dysphagia, and diarrhea. Barrett's esophagus sometimes disappears after fundoplication, but this procedure does not eliminate the risk of dysplasia and carcinoma in all patients. Therefore, endoscopic screening after the procedure is mandatory. The other major therapeutic option for patients with GERD is long-term proton pump inhibitors (PPIs). Patients treated with these drugs have esophageal healing and symptom relief rates comparable to those of operated patients. However, about 30-40% of patients only improve after dose escalation. Therefore, PPI users take longer on average to achieve symptom control. Esophagectomy would be an option if there were an esophageal carcinoma or high-grade dysplasia, particularly in younger patients without significant comorbidities. The decision to operate must take into account the patient's general health and comorbidities, which are common given that Barrett's esophagus often occurs in elderly patients. However, advanced age by itself does not contraindicate esophagectomy. Several endoscopic techniques (e.g. mucosectomy, laser ablation, and photodynamic therapy) have had some success in the treatment of Barrett's esophagus. However, complication such as bleeding after extensive mucosectomies, laser perforations, and skin burns from photoporphyrin-induced photosensibility are common. Incomplete mucosectomies, after which small areas of high-grade dysplasia or even cancers remained in the esophageal wall, have also been documented. Radiofrequency thermal coagulation of the esophageal wall (Stretta procedure) consists of using a special catheter to induce controlled heating of the esophageal wall at the level of the gastroesophageal junction. This induces fibrosis and contraction of the gastroesophageal junction, which increases sphincter pressure and improves reflux. However, recurrence is high after this procedure (about 50%), and it cannot be performed when there is a hiatal hernia, as in this case.
A middle-aged woman presents with diarrhea and vomiting that started this afternoon. She does not remember eating any meat, chicken, pudding, or ice cream the day before. She lives alone and says she warmed up leftover rice for supper last night. She looks sick from vomiting, but she is not dehydrated. She has no fever, and her blood pressure and pulse are within normal limits. What would be the best next step? 1. Admit and observe 2. Start oral fluids and metronidazole 3. Reassure the patient and send home with oral rehydration 4. Send stool specimen to the lab and ask patient to come the next day for follow up 5. Admit for parenteral treatment
Reassure the patient and send home with oral rehydration This patient is suffering from acute food poisoning. The history gives us the clue that she acquired this food poisoning from reheated rice. The most common organism that might cause this condition in this case would be Bacillus cereus. Bacillus cereus is a Gram-positive, spore-forming rod. Spores on grains, such as rice, survive steaming and rapid frying. The spores germinate when rice is kept warm for many hours. B. cereus produces 2 enterotoxins. The mode of action of 1 of the enterotoxins is the same as that of cholera toxin ( i.e., ADP-ribosylates G protein, which stimulates adenylate cyclase and leads to an increased concentration of cyclic AMP within the enterocyte). The mode of the other enterotoxin is uncertain. Clinically, it is a self-limiting situation and requires supportive treatment only. The incubation period is about 18 hours. The best advice for this patient would be to reassure her that it will get better and give her some oral rehydration supplements.
You are examining a 6-month-old male infant who has not received any immunizations since birth. He was born at home, and he has suffered respiratory tract infections and chronic diarrhea since birth. His parents are not related. 2 of his older brothers died of pneumonia at the age of 8 and 12 months, respectively; however, his 4-year-old sister is healthy. His parents moved to United States from an underdeveloped country 3 months ago. They are worried because 1 of their neighbors' children, who is 6 years of age, developed a pruritic skin rash and fever 2 days ago; another child, who is 6 months of age, is in the hospital because of severe vomiting and diarrhea. The parents of your patient are worried. On examination, your patient's vitals are normal; his weight is <5th percentile, his height is at the 5th percentile, and his head circumference is at the 50th percentile. Eczematous skin rash and mucocutaneous changes suggestive to a fungal infection are noted. His thyroid, heart, lungs, abdomen, genitalia, and nervous system are within normal limits; you are not able to palpate lymph nodes, and there are no signs of hepatosplenomegaly. What vaccine should you recommend? 1. Rotavirus vaccine (RV) 2. Pneumococcal vaccine (PCV13) 3. Measles, mumps, and rubella vaccine (MMR) 4. Bacille Calmette-Guerin vaccine (BCG) 5. Variccella vaccine (Var
The correct response is pneumococcal vaccine (PCV13). Your patient suffers recurrent respiratory infections and diarrhea; he also has skin changes and a family history highly suggestive of an X-linked immune deficiency, most likely a severe combined immunodeficiency (SCID). SCID is a group of congenital diseases caused by different genetic mutations resulting in the severe deficiency of T and B- lymphocytes. Onset before age 6 months suggests a T-cell defect, as maternal antibodies are present for the 1st 6 to 9 months. Live vaccines are contraindicated in SCID because introducing an infectious agent, even a weakened one, can cause actual disease. However, pneumococcal vaccine PCV13 is considered risk-specific and is recommended in a 4-dose series at ages 2, 4, and 6 months (minimum age of 6 weeks), as well as at age 12 through 15 months. Rotavirus (RV) is a live vaccine and cannot be given to SCID patients. The measles, mumps, and rubella (MMR) vaccine is a live vaccine and cannot be given to patients with SCID. BCG, or Bacille Calmette-Guerin, is not generally recommended for use in the United States because of the low risk of infection with Mycobacterium tuberculosis, the variable effectiveness of the vaccine against adult pulmonary TB, and the potential interference with tuberculin skin test reactivity. More importantly, in regard to this case, it is a live vaccine and should not be given to any immunosuppressed person. Variccella (Var) is also a live vaccine and cannot be given to SCID patients.
A 62-year-old woman with a long-standing history of hypertension presents with severe headache; it started this morning and is rapidly worsening. During the interview, she suddenly collapses. Your brief examination shows that she responds with extensor posturing on external stimuli. Her deep tendon reflexes are 3, and you elicit Babinski bilaterally. You also notice that her breathing has a peculiar pattern: deep inspiration with a pause at full inspiration, followed by a brief insufficient release and the end-inspiration pause. How do you best describe her respiratory pattern? Answer Choices 1 Cheyne-Stokes 2 Apneusis 3 Ataxic 4 Cluster 5 Central neurogenic hyperventilation
Her breathing pattern is apneustic. Apneustic breathing pattern characterizes *deep, gasping inspiration with a pause at full inspiration followed by a brief, insufficient release and the end-inspiration pause before expiration.*
A 55-year-old woman is broughtin by her daughter, who claims that her mother has lost 4 pounds over the last month, has poor appetite, and complains of nausea. She has vomited twice over the past 1 week. History is significant for alcoholism for the past 15 years and hypertension and diabetes for the past 5 years, for which she takes no treatment. She has been drinking 2 pints of beer almost every day for the past 20 years and greater quantities on weekends. Vitals: BP 140/100 mm Hg, HR 102/min, RR 20/min, and Temp 99.8 degrees F. Physical exam reveals hepatomegaly. A liver biopsy reveals ballooning degeneration, spotty necrosis, and polymorphonuclear infiltration. What is the most likely diagnosis? 1. Alcoholic fatty liver 2. Alcoholic hepatitis 3. Alcoholic cirrhosis 4. Alcoholic siderosis 5. Hepatocellular carcinoma
Alcoholic hepatitis This patient most likely has alcoholic hepatitis, as her biopsy reveals ballooning degeneration, spotty necrosis, and polymorphonuclear infiltration. Other findings that may be seen are fibrosis of the perivenular and perisinusoidal space of Disse. Mallory bodies may be present. Alcoholic Liver Disease (ALD) begins with a fatty infiltration of the liver (fatty liver), which gradually progresses to alcoholic hepatitis. Alcoholic hepatitis is characterized by hepatocyte injury, as revealed by the biopsy findings in this patient. Lab findings include a moderately elevated AST, ALT, γ-Glutamyl transpeptidase, and alkaline phosphatase. Both AST and ALT are elevated by 2 - 7 fold, and AST is typically greater than ALT level (AST/ALT >1). Bilirubin is also elevated. Prothrombin time is >5 seconds, and macrocytic anemia may also be observed. Alcoholic cirrhosis refers to the irreversible injury caused to the liver parenchyma by excessive alcohol intake. Extensive fibrosis and formation of regenerative nodules is seen. Alcoholic siderosis is characterized by iron deposits in Kupffer cells in alcoholics. Alcoholic cirrhosis can progress to hepatocellular carcinoma, which is the primary malignant tumor of the liver.
A 35-year-old woman presents with fatigue and yellowish coloration of her eyes and skin that started several weeks after noneventful implantation of the prosthetic mechanical heart valve. Physical examination reveals the presence of regurgitant murmur and subicterus. Laboratory results are: hemoglobin 7.0 g/dL; reticulocytes 21%; white blood cells 11,500/µL; platelets 80,000/µL; and undetected levels of haptoglobin. In lactate dehydrogenase, the direct and indirect bilirubin levels are all elevated (3,100 U/L, 2.1 and 1.2 mg/dL, respectively). Peripheral blood smear shows burr and helmet cells (schistocytes) and polychromasia. Both direct and indirect Coombs' tests are negative. You suspect microangiopathic hemolytic anemia. What is the next step in management?? 1. Hemoglobin electrophoresis 2 Glucose-6-phosphate dehydrogenase deficiency 3 Echocardiography 4 Hepatitis B panel 5 Direct agglutination test
Anemia, reticulocytosis, and jaundice are the characteristics of hemolytic anemias. Red blood cells (RBC) survival is shortened; bone marrow increases erythroid production that results in the increased number of reticulocytes; and RBC breakdown manifests as increased unconjugated bilirubin and lactate dehydrogenase and decreased (undetectable) haptoglobin. Microangiopathic hemolytic anemia occurs when the red cell membrane is damaged in circulation, leading to intravascular hemolysis and the appearance of schistocytes (fragmented erythrocytes). Hemolytic anemia due to mechanical damage is seen with prosthetic mechanical heart valves. Your first and most important next step in the management of this patient will be to find out if there is a dysfunction of the prosthetic valve. Echocardiography will reveal the presence of significant leaking or valve dysfunction. Hemolytic anemias are generally caused by intrinsic (defects in erythrocytes membranes, enzyme systems, and hemoglobin; mostly hereditary) and extrinsic factors (immune and microangiopathic hemolytic anemias). Hemoglobin electrophoresis will be useful in the diagnosis of thalassemia or sickle cell anemia that is caused by intrinsic hemoglobin defect. Anemias in those disorders are usually hypochromic and microcytic.
A 56-year-old man presents with a 1-week history of palpitations and shortness of breath. He has a long-standing history of poorly controlled hypertension. Physical examination reveals an elevated blood pressure of 190/98 mm Hg, elevated jugular venous pressure (JVP), mild hepatomegaly, bilateral pedal edema, and rales at the lung bases. Diagnostic studies reveal concentric left ventricular hypertrophy without significant valvular abnormalities on echocardiogram. What drug is beneficial in the treatment of the patient's condition by virtue of both afterload and preload reduction? 1. Loop diuretics (e.g., furosemide) 2. Angiotensin-converting enzyme inhibitor (e.g., enalapril) 3 Positive inotropic agents (e.g., digoxin) 4 Sodium channel blocker (e.g., procainamide) 5 Arterial vasodilators (e.g., hydralazine)
Angiotensin-converting enzyme (ACE) inhibitors reduce both preload and afterload. The above signs and symptoms suggest a diagnosis of congestive heart failure (CHF), wherein the heart fails to adequately maintain the circulation of blood. The manifestations of CHF are cardiomegaly, elevated JVP, hepatomegaly, pedal edema, and pulmonary edema. The increased JVP, pedal edema, and hepatomegaly are due to the backflow pressure. The pulmonary edema is due to an imbalance between the mechanisms that keep the interstitium and alveoli dry and the opposing forces that are responsible for fluid transfer into the interstitium. Since the patient is having poorly controlled hypertension, 1 of the modalities of treatment in this patient is the administration of ACE inhibitors (e.g., enalapril). ACE inhibitors inhibit the conversion of angiotensin I to angiotensin II (AII) through ACE; this results in the favorable modification of the neurohormonal activation in heart failure. They cause favorable hemodynamic effects by causing peripheral vasodilatation, afterload, and blood pressure reduction. They also bring about reduction in the preload through the reduction of aldosterone, which in turn decreases sodium and fluid retention.
A 72-year-old man with a past medical history of hyperlipidemia, inferior wall myocardial infarction 6 months ago, and congestive heart failure has presented to his local medical office with complaints of increased tiredness and fatigue upon ambulation over the past several weeks. He denies edema, syncope, lightheadedness, dizziness, chest pain, palpitations, cough, shortness of breath, abdominal pain, fever, or chills. An EKG was performed, which revealed a first degree AV block. What is the most likely physical exam finding expected in this patient? 1 Increased intensity of S1 heart sound 2 An irregular cardiac rhythm 3 Systolic murmur decreased with squatting 4 Delayed carotid upstrokes 5 Blowing apical diastolic murmur
Blowing apical diastolic murmur This patient's EKG finding suggests a primary atrioventrucular heart block. First-degree AV block is the most common conduction disturbance and is characterized by a PR interval that is prolonged for greater than 0.2 seconds. In general, the PR interval is constant, and each atrial impulse is conducted to the ventricles; a regular cardiac rhythm is expected. First-degree AV block can be a normal variant in young or athletic individuals due to excessive vagal tone. It also occurs in elderly patients without underlying heart disease. It is also associated with myocarditis, mild digoxin toxicity, and inferior wall myocardial infarction secondary to AV nodal ischemia. The intensity of the first heart sound (S1) is decreased in patients with first-degree AV block. Patients with first-degree AV block may have a short, soft, blowing, diastolic murmur heard at the cardiac apex. This diastolic murmur is not caused by diastolic mitral regurgitation, because it reaches its peak before the onset of regurgitation. The diastolic murmur is thought to be related to antegrade flow through closing mitral valve leaflets that are stiffer than normal. A harsh, medium-pitched systolic murmur at the 3rd and 4th interspaces that is reduced with squatting and increased with strain from Valsalva or standing is consistent with a diagnosis of hypertrophic cardiomyopathy. Delayed carotid upstrokes characterized by a small and slow rise of amplitude suggests underlying aortic stenosis.
Which of the following is true regarding Mallory-Weiss syndrome? 1. Operative repair usually is necessary 2. Is the most common cause of gastrointestinal bleeding during pregnancy 3. Endoscopy is contraindicated 4. Bleeding stops spontaneously in the majority of cases 5. Accounts for 1/3 of deaths from upper gastrointestinal bleeding
Bleeding stops spontaneously in the majority of cases Mallory-Weiss syndrome involves a tear of the mucosa and submucosa of the lower esophagus or gastric cardia and accounts for up to 9% of cases of significant upper gastrointestinal bleeding. This syndrome results from forceful events that increase intra-abdominal pressure, including trauma, seizures, coughing, and, most commonly, vomiting, with retching preceding hematemesis in approximately 50% of patients. Mallory-Weiss syndrome is associated with alcoholism, hiatal hernias, esophagitis, or gastritis. The majority of episodes involve only mild to moderate bleeding that stops spontaneously, and surgery is rarely required. Only 3% of deaths from upper GI bleeding are due to Mallory-Weiss tears. Endoscopy is the diagnostic procedure of choice if performed within the first 12 to 24 hours. Esophagitis secondary to reflux and repeated vomiting is the most common cause of upper GI bleeding during the pregnancy.
A 70-year-old male presents with acute onset back pain for one hour. His past medical history includes coronary artery disease, for which he has undergone a coronary artery bypass grafting , 3 times, two years ago. He also has hypertension, and has smoked a pack of cigarettes per day for fifty years. His physical exam includes a BP of 80/76 mm Hg, pulse of 116/min, and has a palpable infraumbilical pulsatile abdominal mass with left lower quadrant fullness. What is the best diagnostic test to use in the diagnosis of this patient's condition? 1. Computed Tomography(CT) 2. Aortogram 3. Magnetic Resonance Imaging(MRI) 4. Venogram 5. Abdominal x-ray
Computed Tomography(CT) Explanation This patient has a ruptured abdominal aortic aneurysm, which is defined as a localized dilation of the abdominal aorta greater than or equal to twice the normal diameter, with a disruption which allows blood outside the aortic wall. Abdominal aortic aneurysm is seen in 5-7% of people above age 60 years in the US. The incidence rises sharply after 55 years of age in men and 70 years of age in women. Men outnumber women by a ratio of approximately 4 to 1. Most abdominal aortic aneurysms encountered by primary care physicians are intact, asymptomatic, and found incidentally on routine physical examination, or in radiographic studies performed for other indications. Cigarette smoking, family history and hypertension are all risk factors for abdominal aortic aneurysms. Due to the high death rate from rupture (35-80%), elective surgical repair or implantation of an endovascular stent graft is advisable in appropriate patients. In general, patients with aneurysms 5cm or larger in diameter, symptomatic aneurysms, or rapidly enlarging aneurysms should be considered for repair.
A 57-year-old woman is undergoing a workup by her primary care provider for abdominal pain. The pain is in her left lower quadrant, intermittent, "crampy," and has been present for about 2 months. As an initial imaging study, she underwent an abdominal ultrasound. The ultrasound was unremarkable except for the presence of a few small (<1cm) stones in her gallbladder. She is currently following up to discuss the results of the ultrasound. What treatment is likely to be suggested for the gallstones? 1. No treatment 2. Prophylactic elective cholecystectomy 3. Oral analgesics 4. Urgent cholecystectomy 5. Oral dissolution therapy
Correct answer: No treatment No treatment is the correct answer. The patient's left lower quadrant abdominal pain is not caused by the gallstones (cholelithiasis), which is an incidental finding on the ultrasound. If the gallstones were causing symptoms, this would be known as cholecystitis and the patient would have different abdominal complaints. Only 5-10% of patients with cholelithiasis actually have cholecystitis, and the symptoms could include right upper quadrant abdominal pain, epigastric abdominal pain, nausea, vomiting, indigestion, or fatty food intolerance. Since this patient is not experiencing symptoms related to the stones, the stones are under 2 - 3 cm, and the gallbladder is not reported as "calcified" in general, she does not need to undergo treatment for the incidental finding of cholelithiasis. Prophylactic elective cholecystectomy is not the correct answer. Treatment is not needed for asymptomatic stones. If her stones were greater than 2 - 3 cm, her gallbladder were calcified (risk for gallbladder cancer), or she were suffering from recurrent pancreatitis secondary to microlithiasis, then a prophylactic cholecystectomy would be indicated. Oral analgesics is not the correct answer. The patient has noted pain, but the source has not yet been determined. Whether the patient receives oral analgesics for the left lower quadrant pain while undergoing further workup would be up to the provider. However, analgesics would not be warranted for the cholelithiasis, as the stones are asymptomatic. Urgent cholecystectomy is not the correct answer. Urgent cholecystectomy is typically performed for patients who have severe symptoms, particularly pain that is intolerable or severe vomiting. Cholecystectomy is usually performed electively (for reasons discussed in "prophylactic elective cholecystectomy"), and very rarely needs to be done on an urgent basis. Oral dissolution therapy is not the correct answer. Oral dissolution therapy is very rarely used today and was much more popular 10 - 20 years ago. Oral dissolution therapy consists of taking bile acid in pill form to try to dissolve stones. It can work on a small percentage of patients with small, cholesterol gallstones, but has significant negative aspects that have taken the therapy out of popular use. It can be very expensive, take several years, and has a high recurrence rate. As this patient's stones are asymptomatic, the best choice is to watch the stones conservatively.
A 48-year-old Caucasian man presents with severe epigastric pain radiating to the back after a bout of drinking. 2 weeks later, a repeat ultrasound showed a round, thin-walled hypoechoic lesion near the pancreas tail measuring 4 cm in its largest diameter and with some calcifications in its walls. MRCP visualized a communication between this cavity and the pancreatic duct. Endoscopic ultrasound detected an area of high-speed flow close to the gastric wall. What is the most appropriate next step in management? 1. Perform angio-CT scan 2. Perform angiography with coil embolization 3 Refer for a Puestow procedure 4. Refer for ERCP and transpancreatic drainage of the cyst 5 Refer for endoscopic ultrasound (EUS)-guided transgastric drainage 6 Observation only
Correct answer: Observation only Explanation Pancreatic pseudocysts complicate about 10% of attacks of acute pancreatitis. A pseudocyst is formed when pancreatic fluid leaks and is confined by organs adjacent to the pancreas. Eventually, a fibrous wall forms around the collection. Most cysts regress spontaneously over a period of several weeks, but in some cases complications such as bleeding, abscess formation, and intractable pain may occur. Several interventional approaches — surgical, radiologic and endoscopic — have been tried. Endoscopic ultrasound has gained popularity in the management of pseudocysts, as it can identify complex cyst wall structures, in particular pseudoaneurysms, do fine-needle aspiration to rule out a neoplasm, and find a favorable spot for drainage. Ruling out a cystic pancreas neoplasm before endoscopic drainage is essential, as attempts at transgastric emptying of a neoplasm can have disastrous effects and compromise further surgical management of the neoplasia. Indications for intervention are intractable pain, expanding lesions, and infection. A frequently quoted study found that complications such as bleeding, abscess formation, or perforation rose sharply after 6 weeks of observation, and that intervention is warranted if a pseudocyst does not resolve in this period. However, more recent studies have challenged this assumption and proposed that observing asymptomatic pseudocysts for longer periods is safe. Pseudoaneurysms are found in 10% of pseudocysts. Signs suggesting a pseudoaneurysm include overt bleeding, sudden pseudocyst enlargement, and an abrupt fall in hematocrit. Their presence is a concern when drainage is considered and does not lead to a higher overall bleeding rate or constitute indication for intervention. Angiography has higher sensitivity and can perform therapeutics, but an angio-CT scan has adequate sensitivity for ruling out a pseudoaneurysm before proceeding with therapeutics. When intervention is warranted, angiographic coil embolization and operative resection can be performed.
A 65-year-old man presents with dry mouth and difficulties rising from a chair, climbing stairs, and walking. His symptoms started about 1 year ago, are worse in the morning, and improve with exercise. He is a heavy smoker and was recently diagnosed with lung cancer. What is the most likely underlying mechanism of his weakness? 1. Infiltration of the nerves 2. Spinal cord compression 3. Spinal cord metastasis 4. Paraneoplastic 5. Nicotine poisoning
Correct answer: Paraneoplastic Explanation Paraneoplastic syndrome (Lambert-Eaton syndrome), resembling myasthenia gravis, occurs in some people with small cell carcinoma of the lungs. It usually manifests as progressive weakness in the large muscles. Lambert-Eaton syndrome is caused by the inhibition of voltage-gated calcium channels on the presynaptic membrane of the neuromuscular junction; this prevents the release of acetylcholine. As the muscles continue to contract, acetylcholine can build up in sufficient quantities for the strength to get better; weakness improves after repetitive muscle contraction. Although the underlying mechanism is autoimmune, Lambert-Eaton syndrome in this patient is regarded as paraneoplastic because it is a consequence of a cancer, not due to a local presence of cancer cells. Tumor infiltration is a local manifestation of a tumor associated with the production of various types of extracellular matrix-degrading enzymes. Local infiltration of the nerves excludes variegated symptoms of autonomic nervous system disturbance and fluctuating weakness in several muscle groups. Secondary spinal cord tumors usually follow hematogenous spread to the vertebral bodies, epidural expansion, and/or intramedullary metastasis. Subsequent symptoms of compression (i.e., pain, radicular, or medullar symptoms) gradually worsen, not improve with exercise. Nicotine poisoning is not likely; it is impossible to overdose on nicotine through smoking alone. Smoking causes vascular disease, cancer, lung disease, peptic ulcer, and reproductive disturbances (e.g., premature birth). Nicotine may contribute to tobacco-related disease, but direct causation has not been determined because nicotine is consumed simultaneously with a multitude of other potentially harmful substances that occur in tobacco smoke. The effects of nicotine on nerves and muscles are generally dose-dependent; they also occur in nicotine-tolerant individuals. Initially, nicotine has a short-lived stimulatory phase followed by a longer inhibitory phase which leads to a neuromuscular blockade. Neuromuscular symptoms include hypotonia, decreased deep tendon reflexes, weakness, fasciculations, and paralysis of muscles (including respiratory muscles). Cholinergic effects on the autonomic nervous system, often observed initially, include diaphoresis, salivation, lacrimation, increased bronchial secretions, miosis, and later mydriasis. Nicotine acts on the sympathetic ganglia, chemoreceptors of the aorta, and carotid bodies; it affects the adrenal medulla, releasing catecholamines.
A 26-year-old man who has AIDS presents with rapidly progressing dyspnea. The symptoms became apparent following a diagnostic thoracentesis for a left-sided pleural effusion; you suspect the effusion is caused by mycobacterium. The patient reports left-side chest pain and difficulty in breathing. On examination, he is in respiratory distress; temperature is 38, RR is 39, and BP is 70/40 mm Hg. He has decreased breath sounds in his left lung field, and the percussion note is more resonant. What is the most appropriate course of action? 1. Administer 100% oxygen by mask 2. Sedate and endotracheally intubate 3. Perform needle decompression 4. Insert an underwater sealed chest tube 5. Order an urgent chest X-ray
Correct answer: Perform needle decompression Explanation This patient has the signs and symptoms of a tension pneumothorax; in this patient's case, it could have resulted from the diagnostic thoracentesis, or it could have resulted from his underlying pulmonary disease. The most appropriate measure would be to perform needle decompression as you prepare for insertion of an underwater sealed chest tube. Symptoms of tension pneumothorax include unilateral chest pain, dyspnea, tachypnea, and hypotension. On examination, there is diminished chest wall excursion on the affected side, increased tympany on percussion, and diminished or absent tactile fremitus. Mediastinal shift to the contralateral side may be detectable as displacement of the trachea, cardiac dullness, and apex beat away from the affected side. Breath sounds are markedly depressed or absent. The chest X-ray usually shows air without peripheral lung markings, limited by a sharp visceral pleural margin with lung markings medially, indicating the position of the collapsed lung. When the clinical situation suggests a tension pneumothorax, a needle decompression of the affected side (side with decreased breath sounds and hyperresonance on percussion) should be performed. Air is removed by inserting into the chest wall a large bore needle attached through a 3-way stopcock to a large syringe. The needle may be inserted anteriorly in the 2nd intercostal space in the midclavicular line. Giving the patient oxygen, either by mask or endotracheally, would not relieve him of his symptoms, which are due the pleural accumulation of the air causing a shift of the mediastinum. Due to the rapid deterioration of the patient's clinical condition, it would not be advisable to send him for a chest X-ray before relieving the tension pneumothorax. In tension pneumothorax, a bronchopleural fistula acts as a valve and allows air to enter but not leave the pleural space, causing pressure in the space to rise above the atmospheric pressure. The continued accumulation of air in the chest results in lung collapse and a shift of the mediastinal structures to the opposite side, which impedes venous return to the heart. In turn, this results in hypotension, abnormal gas exchange, and ultimately cardiovascular collapse. When the presentation is less urgent and there is time for a chest X-ray, sites where the lung is held to the chest wall by adhesions should be avoided. Air is alternately withdrawn from the pleural space and expelled from the syringe into the room until an underwater sealed tube thoracostomy has been inserted.
A 25-year-old man presents due to heartburn. It only occurs 1 or 2 times a month, but it is extremely noticeable when these episodes do occur. He states that eating large meals, eating late at night right before going to sleep, and excessive alcohol consumption seem to exacerbate and even initiate episodes. He has noticed that limiting the aforementioned behaviors has decreased the severity of the symptoms in recent months, but it is still occurring despite this modification. He denies any difficulty swallowing, weight loss, night sweats, chest pain, use of tobacco, or coughing up blood. Physical examination is unremarkable for any abnormalities. Since this patient's symptoms would be classified as mild, intermittent symptoms and modifications have been attempted and failed, what would be the next best option for this patient? 1 Antacids 2 H2-receptor antagonists 3 Proton-pump inhibitor 4 Antibiotics 5 Prokinetic agents
Correct answer: Antacids Mild or intermittent symptoms of gastroesophageal reflux disease (GERD) are not typically seen as adversely affecting patients' quality of life. Initially, action is taken in terms of modification of behaviors, such as eating smaller meals as well as eliminating acidic foods and foods known to precipitate the reflux (fatty foods, alcohol, chocolate, or peppermint). Weight loss has also been shown to help decrease issues, and so has advising patients to avoid lying down at least 3 hours after eating meals. Elevation of the head of the bed is also suggested to initiate relief. From a pharmacological standpoint, antacids are considered the mainstay for rapid relief of occasional heartburn. Antacids work by neutralizing stomach acid only; alone, they will not heal any inflammation that may be caused by stomach acid. Patients should be told that the duration of action of these medications is typically less than 2 hours. Antacids are also contraindicated in any patients with renal failure. Many types of antacids are available over the counter. The next step up in terms of treatment would be considered to be H2-receptor antagonists. These medications work by reducing stomach acid production. Again, these agents are available over the counter as 50% the prescription dose. Examples include cimetidine 200 mg, ranitidine 75 mg, and famotidine 10 mg. When these agents are taken for active reflux symptoms, they have a delay of onset of at least 30 minutes; once they do take effect, relief is supplied for up to 8 hours per dose. Proton pump inhibitors are initial pharmacological treatment reserved for patients with moderate to severe symptoms of reflux as well as known complications from GERD. Proton pump inhibitors work by actually blocking the production of acid. Examples of these include omeprazole 20 mg or lansoprazole 30 mg. There are proton pump inhibitors available over the counter, but some are only available with a prescription. Proton pump inhibitors help patients achieve adequate control of their heartburn symptoms, and they can potentially offer resolution of these symptoms; they even aid in healing of erosive esophagitis if it is present. Antibiotics in relation to reflux or heartburn would not be indicated directly for the symptoms of this pathology; antibiotics could potentially be used as a component of the treatment of a H. pylori infection. Prokinetic agents are not indicated as treatment of mild, intermittent reflux symptoms.
Case A 22-year-old man presents with a 1-week history of fever, chills, dry or mildly productive cough, and chest wall discomfort. He is on no medications, but he has a history of mild asthma for which he does not need chronic therapy. He states that he has been under a lot of pressure at work and has not been sleeping very well. He decided to seek medical attention today because he has not been improving and is concerned he might have pneumonia. On examination, vitals include a temperature of 102.6ºF, BP: 126/76 mm Hg, P: 82 beats/min, R: 20/min. Lung exam reveals coarse rhonchi throughout lung fields with rales in the right lower lobe with egophony demonstrated over the right lower lobe. The patient is sent for PA and lateral chest X-ray; he is found to have a right lower lobe infiltration. What would be the best empiric medication for this patient's illness? 1. Amoxicillin 875 mg twice daily for 10 days 2. Azithromycin 500 mg first dose, then 250 mg daily for 4 days 3. Ciprofloxacin 500 mg twice daily for 10 days 4. Cephalexin 500 mg 3 times daily for 10 days 5. Trimethoprim/sulfamethoxazole DS twice daily for 10 days
Correct answer: Azithromycin 500 mg first dose, then 250 mg daily for 4 days Explanation The correct answer is azithromycin. Azithromycin is a macrolide that is effective against almost all causes of community-acquired pneumonia (CAP), especially Mycoplasma pneumoniae, which may be suspected in this young patient. For empiric treatment of CAP, doxycycline and macrolides are preferable in patients under 50 and those without comorbidities. Fluoroquinolones, such as levofloxacin, are typically indicated in patients over 50 and those with significant comorbidities. Ciprofloxacin is not indicated in the empiric treatment of CAP because it does not have broad enough coverage. Amoxicillin and cephalexin are not appropriate alternatives; they are not effective against Mycoplasma sp. Trimethoprim/sulfamethoxazole is not indicated for empiric treatment.
A 57-year-old man presents with progressive dyspnea on exertion and left lumbar colic. He has a history of hypertension as well as a 40-pack/year history of smoking. He denies cough, orthopnea, and paroxysmal nocturnal dyspnea. He has some mild ankle swelling; however, he has no history of congestive heart failure. The only medication he is on is amlodipine. His vital signs are as follows: temperature 99.8° F, pulse 92/min, respiration 22/min and BP 128/88 mm Hg. Of significance on physical examination is the absence of breath sounds in the left lower lung zone. Laboratory data reveals WBC 1000/μL with 70% segmented neutrophils, serum glucose 106 mg/dl, sodium 138mmol/L, chloride 102mmol/L, potassium 4.2mmol/L, bicarbonate 22 mmol/L, BUN 32 mmol/L, creatinine 1.2 mmol/L, protein 8.2g/dL, amylase 56U/dL, and LDH 250 U/mL. Thoracentesis is done and pleural fluid analysis shows: WBC 910/μL, RBC 14/μL, LDH 108U/mL, protein 2.6g/dL, glucose 82mg/dL, and creatinine 1.2 mmol/L. What test will you do to find the cause of the effusion? 1. Pleural fluid culture 2. Chest Computerized tomogram 3. Sputum AFB 4. Pleural biopsy 5. Abdominal sonogram
Correct answer: Abdominal sonogram Explanation The correct response is abdominal sonogram. Cancer, tuberculosis, and uremia are causes of exudative effusions; since his is a transudate, they are unlikely causes of this patient's effusion. Cirrhosis typically causes bilateral effusions. Urinothorax, on the other hand, causes a unilateral effusion and is associated with obstructive uropathy. In addition to the transudative picture, a fluid to serum creatinine ratio of 1 also supports the diagnosis. An abdominal sonogram will help in identifying this as the cause. Pleural effusion occurs because of accumulation of excess amount of fluid in the pleural space. It could be transudative or exudative. Transudates have fluid protein less than 50% of the serum protein, and LDH less than 60% of serum LDH. Exudates, on the other hand, have protein more than 3g/dL and LDH greater than 200, or 50 and 60% of the serum respectively. Causes of transudative effusions include congestive heart failure, cirrhosis, nephritic syndrome, superior vena cava obstruction, pulmonary embolism, and urinothorax. Causes of exudative include cancer, tuberculosis, esophageal rupture, pulmonary embolism, connective tissue disorders, pancreatitis, and pneumonias. Pleural fluid culture and biopsy are unnecessary in transudative effusion. Chest CT will show the effusion and underlying causes such as cancer and pneumonia, which is not the case here. Therefore, it will not provide additional diagnostic benefit. Sputum AFB (acid-fast bacilli) will identify tuberculosis if the patient has active pulmonary TB. However, there is no reason to suspect tuberculosis based on this patient's symptoms.
A man presents with cough, chest constriction, fever, chills, night sweats, muscle aches, and joint stiffness. Physical exam shows multiple red, non-raised lesions on the anterior aspect of the tibia. The lesions are tender to palpation. The Spherulin test is positive. What is the most likely diagnosis? 1. eosinophilia 2. Histoplasmosis 3. Pulmonary actinomycosis 4. Acute coccidioidomycosis 5. Pulmonary asbestosis
Correct answer: Acute coccidioidomycosis Explanation Acute coccidioidomycosis is a disease caused by breathing in a fungus found in the soil in certain parts of the southwestern United States, Mexico, and Central and South America. Dark-skinned people and people with a weak immune system will have more serious infections. Infection is caused by breathing in spores of a fungus (Coccidioides immitis) found in desert regions. About 60% of infections cause no symptoms and are only recognized by a positive coccidioidin skin test. Symptoms include cough, chest pain (mild pain to severe constriction), fever, chills, night sweats, headache, muscle aches and stiffness, joint stiffness, and rash on the lower legs (erythema nodosum). Acute pulmonary eosinophilia is a self-limiting inflammation of the lungs associated with infiltration of eosinophils in the lungs and blood. The etiology includes exposure to various drugs, parasitic infestation (especially ascariasis in children), nickel exposure, recent blood transfusion, or lymphangiogram. Symptoms may include general malaise, loss of appetite, fever (greater than 2 to 3 days), productive cough (mucoid sputum), chest pain, shortness of breath, wheezing, rapid respiratory rate, headache, and muscle pain. Histoplasmosis is a chronic respiratory infection caused by inhaling the spores of the fungus Histoplasma capsulatum, which is found in bird and bat droppings common along river valleys. Most cases are mild or asymptomatic. Risk factors include travel or residence in central/eastern United States or South America, environmental or occupational exposure to droppings of chickens, bats, or blackbirds, and pre-existing COPD (chronic obstructive pulmonary disease). Immunocompromised people are also at risk. Symptoms include fever, chills, cough (with mucus or pus), skin lesions, and joint stiffness. The associated skin lesion usually presents as a lesion on the mouth or inner cheek as a papule; it may ulcerate. Pulmonary actinomycosis is an infection caused by Actinomyces israelii or actinomycete bacteria that causes disease of the chest, mouth, jaw, and pelvis. The bacterium is found in the normal flora of the mouth and gastrointestinal tract of humans. Symptoms include lethargy, weight loss, fever, productive cough, draining sinuses, night sweats, shortness of breath, and chest pain. In the chest, it results in cavities in the lung and pleural effusion, which may spread through the chest wall and produce sinuses. Pulmonary asbestosis is a respiratory disease caused by inhaling asbestos fibers, which results in pulmonary fibrosis. Asbestos-related disease includes pleural plaques (calcification), malignant (cancerous) tumors called mesotheliomas, and pleural effusion. Cigarette smoking increases the risk of developing the disease. Symptoms include shortness of breath on exertion, cough, tightness in the chest, chest pain, nail abnormalities, or clubbing of fingers.
A 45-year-old woman presents with a 3-day history of progressively worsening right upper quadrant abdominal pain. She also has intermittent nausea, but denies vomiting. She does not have a thermometer at home, but she has had chills and thinks she may have had intermittent low grade fevers. An abdominal ultrasound is performed and reveals cholelithiasis. What is the most appropriate first-line treatment course? 1. Intravenous hydration and analgesia followed by discharge 2. Intravenous hydration and analgesia followed by laparoscopic cholecystectomy within 72 hours 3 Emergency laparoscopic cholecystectomy within 8 hours 4. Intravenous hydration and analgesia followed by laparoscopic cholecystectomy within the following 2 weeks 5. Oral analgesia followed by elective laparoscopic cholecystectomy
Correct answer: Intravenous hydration and analgesia followed by laparoscopic cholecystectomy within 72 hours Explanation The correct answer is intravenous hydration and analgesia followed by laparoscopic cholecystectomy within 48 hours. Once a diagnosis of cholecystitis has been made, it is important to stabilize the patient with hydration and pain medication prior to surgical correction. Laparoscopic cholecystectomy is a preferred surgical treatment compared with open cholecystectomy due to decreased post-operative recovery period and decreased complication rate. Studies have investigated the optimal timing for surgical treatment and suggest that surgery within 72 hours results in a shorter hospital stay and lower readmission rate. Intravenous hydration and analgesia followed by discharge is not the correct answer. Once a diagnosis of cholecystitis has been made, it is important to stabilize the patient with hydration and pain medication. However, definitive management of the cholecystitis requires removal of the gallbladder (as the stones themselves cannot be removed) in order to decrease recurrent symptomatic episodes. Emergency laparoscopic cholecystectomy within 8 hours is not the correct answer. While early surgical intervention has been shown to be beneficial to the patient, this is defined as within the first 72 hours. Emergent surgical treatment should be reserved for patients suffering from a complication from acute cholecystitis, such as empyema, emphysematous cholecystitis, or a perforation. Intravenous hydration and analgesia followed by laparoscopic cholecystectomy within the following 2 weeks is not the correct answer. While hydration and pain medication are appropriate, studies have investigated the optimal timing for surgical treatment and suggest that surgery within 72 hours results in a shorter hospital stay and lower readmission rate. Oral analgesia followed by elective laparoscopic cholecystectomy is not the correct answer. Studies have investigated the optimal timing for surgical treatment and suggest that surgery within 72 hours results in a shorter hospital stay and lower readmission rate. Delayed surgical intervention is best reserved for patients with overall medical conditions that need to be stabilized prior to surgery or patients in whom the diagnosis of acute cholecystitis is in doubt.
A 68-year-old woman presents with a 4-hour history of severe left-sided abdominal pain. The pain was initially associated with several episodes of diarrhea with some hematochezia. Since then, she has had a few more stools, but she has not seen any more frank blood. Her pain is somewhat improved from what it was at the onset. Past medical history is positive for renal lithiasis and atrial fibrillation. Current medications are digoxin and aspirin. She is a recovering alcoholic who last had a drink 15 years prior to presentation. She recently returned home from a trip to Mexico; she was visiting relatives for 2 months. On exam, you see a well-nourished woman in extreme discomfort. Auscultation of her lungs reveals good breath sounds bilaterally; her heart has an irregularly irregular rhythm with a rate of 92 beats per minute; there is a soft II/VI systolic murmur. Her abdomen is mildly obese and tender on the left side; there is no appreciable mass or rebound. There is no flank tenderness. Rectal exam reveals guaiac positive mucus mixed with flecks of bright red blood. 12 hours after presentation, her left upper quadrant pain is still present; however, it is much improved, and she had only 2 more stools with small flecks of blood in them. What is the most likely diagnosis? 1 Left ureteral lithiasis 2 Diverticulitis 3 Infectious colitis 4 Small bowel ischemia 5 Ischemic colitis
Correct answer: Ischemic colitis Explanation This is a classic presentation of ischemic colitis. Ischemic colitis occurs when there is obstruction of the colonic blood supply, causing ischemia with inflammation and ulceration of the colonic mucosa. The area of the splenic flexure is most at risk for ischemia because it is the watershed area of the arterial supply to the colon. Ischemic colitis is most commonly seen in elderly individuals. These patients typically present with an acute onset of pain associated with bloody diarrhea, and they have an area of tenderness corresponding to the ischemic segment of colon. Patients may experience recurrent bouts of ischemic colitis. Sigmoidoscopy makes the diagnosis in 85% of patients; 15% will have ischemia in areas proximal to the reach of a sigmoidoscopy. The majority of these patients will improve with supportivecare (hydration and prophylactic antibiotics, in case of bacterial transmigration). However, some patients have much more severe disease, which may even require emergency colectomy. The most important condition to differentiate from ischemic colitis is that of small bowel ischemia. Small bowel ischemia can lead to gangrene of the intestine. Most commonly, these patients have developed occlusion of some of the distribution of the superior mesenteric artery, either from acute thrombosis in an area of atherosclerosis, or from embolic disease. These patients classically have pain out of proportion to their exam, at least early in the process, and they do not usually have an area of localized tenderness on palpation. These patients can have leukocytosis on the level of 20,000 to 30,000/μL, and they may exhibit a lactic acidosis and elevated amylase levels. Their stool is usually positive for occult blood, but frank blood is only rarely seen. Early diagnosis is very important, although often difficult, and urgent surgery is the only treatment. Renal or ureteral lithiasis can cause severe abdominal pain, and these patients can have diarrhea as well, but it should not cause hematochezia or localized anterior abdominal tenderness. Diverticulitis is more common on the left, which is the case with this patient's pain, but it typically does not cause hematochezia. Additionally, the sudden onset of very severe pain would not be typical of diverticulitis. Infectious colitis can cause abdominal pain and hematochezia, but the improvement in her pain over such a short time period would be atypical for an enterotoxic infection. Nevertheless, infection should be considered in this patient because of her recent travel history.
A 58-year-old man presents with a 1-day history of severe abdominal pain, nausea, and vomiting. He initially thought he had some indigestion, with pain located in the epigastric region, and tried some calcium carbonate (Tums) with no relief. The pain and vomiting progressed through the night and kept him from sleeping and going to work. He feels the pain boring through to his back. He denies hematemesis, fever, diarrhea, out-of-the-country travel, and contact with sick people. Prior to onset of pain, he reports good health. He has no known medical conditions and takes no medications. He has had no surgeries. He smokes cigarettes (1ppd x 40 years), admits "moderate" alcohol use, and denies drug use. He is married and works as a welder. Vitals are: BP: 102/56 mmHg; HR: 116bpm; RR: 15; Temp: 98.9F; O2Sat: 95% on room air. On physical exam, the patient appears uncomfortable on the exam table and grimaces when changing position for exam. He is cooperative, alert, and oriented. Abnormal physical exam findings include: abdomen distended, decreased bowel sounds, and tender epigastric region, with guarding. He is tachycardic. No jaundice is noted. The remainder of the exam is normal. Complete blood count (CBC) WBC: 15.1 RBC: 4.63 Hemoglobin: 15.4 Hct: 49.9 MCV 87.2 MCH: 29.4 MCHC: 32.1 RDW: 11.7 Platelets: 335 MPV: 7.3 WBC differential: Normal Complete Metabolic Panel (CMP) Sodium: 144 K: 3.7 Cl: 102 HCO3: 25 Glu: 82 BUN: 21 Creatinine: 1.0 Ca: 8.9 Albumin: 3.5 Total Protein: 6.4 AST (SGOT): 72 ALT (SGPT): 85 Alk Phosphatase: 155 Total Bilirubin: 0.9 eGFR: >60 Amylase: 272 Lipase: 290 Hepatitis panel: Negative Abdominal CT: Diffuse pancreatic enlargement and inflammation, with single fluid collection Urinalysis: Normal What pharmacologic treatment is the most important intervention for this patient's likely condition? 1. Ertapenem 2. Hyoscyamine 3 Lactated Ringer's 4 Pancrelipase 5 Promethazine
Correct answer: Lactated Ringer's Explanation This patient is presenting with an episode of acute pancreatitis. Pancreatitis is characterized by epigastric pain, nausea, and vomiting. Many other acute abdomen conditions may present similarly. Significant elevations of lipase and amylase, as well as CT evidence of pancreatic inflammation, fluid collections, and/or necrosis, will establish the diagnosis. Pancreatitis can be acute or chronic, and those with chronic disease may have periodic acute flares. In acute pancreatitis, early fluid resuscitation is one of the "few medical interventions that appears to affect outcome," so (of the choices listed) administering lactated Ringer's (or saline) would be the best answer for this patient. Also, ensuring the patient has "nothing by mouth" (NPO) until symptoms decrease is standard treatment. Ertapenem is a carbapenem antibiotic, indicated for complicated intra-abdominal, skin, and urinary tract infections. Although this patient's white blood cell count (WBC) is elevated, the primary process is not infectious. The WBC increases in inflammatory states. Rarely are antibiotics needed in acute pancreatitis. Hyoscyamine is an anticholinergic medication, with a wide range of uses, including many gastrointestinal conditions. It may be used for reducing spasm in irritable bowel syndrome and for biliary colic. Before the test results pointed to pancreatitis, as in this patient's diagnosis, biliary colic and obstruction would be on the differential. However, hyoscyamine would not be a primary treatment for pancreatitis. Pancrelipase is a digestive enzyme, taken with meals, and indicated for pancreatic insufficiency, a common complication of many years of chronic pancreatitis. This patient may be at risk for future pancreatitic disease, but the pancrelipase would not benefit him at this time. Promethazine is a common antiemetic with antihistamine and anticholinergic properties. Although it may be reasonable to consider antiemetic treatment for a patient with acute pancreatitis, it is an adjunct treatment and does not have a vital impact on the course of the disorder.
A 52-year-old Caucasian woman presents with a 2-week history of rectal bleeding. Over the last 3 months, she has become progressively more tired, and could no longer play tennis on weekends. She denies diarrhea, abdominal pain or tenesmus. Her past medical history includes peptic ulcer disease 5 years ago, treated with an anti-H.pylori scheme and which has not recurred. Her vital signs are stable, and the physical examination shows no abnormalities. A CBC shows the following: Ht: 28% Hb: 8.8 mg/dl MCV: 82 fl Leukocytes: 8.100/mm3 w/ normal differential An upper endoscopy is normal except for a duodenal ulcer scar without signs of active ulceration, and a repeat colonoscopy is normal. What is most likely to identify the source of bleeding? Celiac angiography Highlight Strikethrough 2 Computed tomography of the abdomen Highlight Strikethrough 3 Wireless capsule endoscopy Highlight Strikethrough 4 Push enteroscopy Highlight Strikethrough 5 Biopsy of the healed ulcer bed Highlight Strikethrough 6 99mTc scan Highlight Strikethrough 7 Small bowel follow-through99mTc scan
Correct answer: Wireless capsule endoscopy Explanation Wireless capsule endoscopy is performed by ingestion of a battery-powered capsule containing a camera and a transmitter. The capsule transmits images to sensors outside the patient during its transit in the gastrointestinal tract. Indications for capsule endoscopy are still evolving, but several series have shown that capsule endoscopy is more sensitive than small bowel radiography or push enteroscopy for locating the source of small-bowel bleeding. The most common etiologies of bleeding in this region are angioectasia and Crohn's disease. Several other small bowel lesions, such as tumors (leiomyoma, lymphomas, carcinoids, or carcinomas), varices, and polyps, have also been described. An important concern is the possibility of capsule impaction and small bowel obstruction. Some centers are reluctant to perform capsule endoscopy in patients with partial or intermittent obstruction of the small bowel. However, capsule impaction may reveal the site of a lesion and allow its surgical removal, obviating the need for intraoperative enteroscopy. Other relative contraindications are incapacity of cooperating with the examination (e.g. dementia), swallowing disorders, esophageal stricture, gastroparesis, and poor surgical risk. In patients with obstruction or gastroparesis, the examiner may place the capsule endoscopically. This patient has no signs of ulcer activity. Therefore, biopsy of the duodenal scar is not indicated. A 99mTc scan is the test of choice for Meckel's diverticulum, a remnant of the embryonic omphalomesenteric duct found in 1.5% of the general population. Its most common location is the antimesenteric border of the mid-to-distal small bowel. Some diverticula contain ectopic, acid-producing gastric mucosa. Therefore, they can ulcerate and bleed. Rectal bleeding caused by Meckel's diverticulum is often maroon or brick-red in color, and its incidence decreases with age. Therefore, this diagnosis is unlikely in this case. Angiography can be used if other less invasive procedures (i.e. small bowel radiography, push enteroscopy, or small bowel radiography) are non-diagnostic. Computed tomography is not indicated in the absence of other symptoms indicating a specific etiology (e.g. a palpable mass or suspicion of pancreatic disease). Of note, colonoscopy after rapid oral purging has emerged as the procedure of choice for the evaluation of acute lower GI bleeding, and it also provides a means for therapy. Scintigraphy and/or angiography also play important roles in diagnosis and embolization when colonoscopy reveals negative findings or when it is not feasible.
A 55-year-old man presents with a 12-hour history of severe epigastric abdominal pain that radiates into his back, nausea, vomiting, and chills. His pertinent physical examination findings include oral temperature of 102°F and epigastric tenderness upon palpation. He is admitted to the hospital for management of his condition. What criteria would increase the likelihood of a severe attack associated with a high incidence of short and long term morbidity and mortality? 1. White blood cell (WBC) count of 10,000cells/mm3 upon admission 2. A 14% decrease of his hematocrit at 48 hours after admission 3. Serum glucose of 130 mg/dL upon admission 4. Aspartate transaminase of 240 U/dL upon admission 5. Serum calcium of 10 mg/dL 48 hours after admission
Correct answer: A 14% decrease of his hematocrit at 48 hours after admission A patient who is admitted to the hospital for treatment of acute pancreatitis is assessed using the Ranson criteria to determine the severity of their disease, which in turn helps determine prognosis. About 70 - 80% of cases of acute pancreatitis are considered mild and result in virtually no morbidity or mortality. The remainder are severe attacks and have a 10 - 30% mortality rate. Fast identification of the severe cases is helpful to reduce the morbidity and mortality for each patient. There are 11 Ranson criteria, 5 of which are determined upon admission, and 6 at 48 hours after admission. Patients who have 2 or fewer of the criteria have minimal mortality. Patients with 3 - 5 of the criteria have about a 10 - 20% chance of mortality. Patients with 5 or more of the criteria have at least a 50% mortality rate. A patient with at least a 10% decrease in their hematocrit at 48 hours after admission meets one of the Ranson criteria, so the correct answer is a 14% decrease in hematocrit 48 hours after admission. WBC count of 10,000 cells/mm3 is not one of the Ranson criteria. A WBC count of >16,000 cells/mm3 would meet one of the Ranson criteria. Serum glucose of 130 mg/dL upon admission is also not one of the Ranson criteria. The patient's serum glucose would have to be at least 200 mg/dL to count as one of the Ranson criteria and increase the severity of their disease. Aspartate transaminase of 240 U/dL upon admission is not one of the Ranson criteria either. The patient's aspartate transaminase would have to be at least 250 U/dL to count as a Ranson criteria and increase the severity of their disease. A serum calcium of 10 mg/dL 48 hours after admission is not one of the Ranson criteria. If the patient's serum calcium is under 8 mg/dL 58 hours after admission, this would count as one of the Ranson criteria.
A 21-year-old man is brought to the emergency room by his friends. One hour earlier he started having excessive retching and he vomited violently several times. He has been drinking for the last 3 hours. The vomitus was mainly bright red blood (about 2 quarts) and dizziness followed. What is the likely causer of this clinical picture? 1. Acute gastritis 2. Carcinoma of stomach 3 Acute esophagitis 4 A gastroesophageal mucosal tear 5 Rupture of the lower esophagus
Correct answer: A gastroesophageal mucosal tear This is a classic presentation of Mallory-Weiss syndrome. Vomiting and retching may cause a tear that involves only the mucosa and is not transmural. The tear usually involves the gastric mucosa near the squamous to columnar mucosal junction, but it may also involve the esophageal mucosa. A wide variety of causes such as gastroenteritis, alcohol binging, ulcers, and hiatus hernia have been cited. In fact, any condition which causes retching and vomiting can cause the tear. Patients present with upper gastrointestinal bleeding that may be severe. Most patients recover with only conservative management, but those with severe arterial bleeding require surgery.
You are performing an annual physical examination on a 14-year-old girl. In the last couple years, she started having constipation "every other day or so," followed by loose stools; she still feels depressed because of her parents' recent divorce. She has no problems at school; her grades are good; and she participates in sport activities. Her mother is worried because she is so slim (despite excellent appetite), has not grown enough, and still has not gotten her period (her mother had her first period when she was 13-year-old). The rest of personal and family history is not contributing. Your patient's height is 5 ft, weight 79 lbs (BMI 15,46; percentile 3%); she is in Tanner stage 2 (the same as the last year according to her records); and her bone age is 12.5 years. The rest of physical examination is normal. Laboratory shows Hct of 31% and MCV of 73, low insulin-like growth factor (IGF), low FSH, positive anti-tissue transglutaminase antibodies, and the rest of laboratory results are within normal limits, including TSH and prolactin. What will restore normal puberty, growth, and weight in your patient? 1. A gluten-free diet 2. Ferrous sulfate 3. Fluoxetine 4. Estrogen 5. Growth hormone
Correct answer: A gluten-free diet In a patient with gastrointestinal symptoms, anemia, pubertal and delayed bone maturation, weight gain and linear growth, with positive anti-tissue transglutaminase antibodies, you should consider celiac disease. A gluten-free diet will restore both her physical and sexual development. Sideropenic anemia is a consequence of celiac disease. Iron supplements will not restore her pubertal, growth, and height development. She may feel depressed, but psychogenic delay is a diagnosis of exclusion, whereas here you have a probable organic cause of the delay that shall be addressed. Sex steroid replacement is probably not needed because her hypothalamic-pituitary-gonadal axis and LH&FSH secretion will resume after her nutritional status improves. IGF levels may be low in cases of nutritional deficiencies and are not diagnostic of growth hormone deficiency. Growth hormone therapy is not needed because her normal growth will probably resume after her nutritional status is corrected.
A 52-year-old man presents after coughing up blood. He has a 60-pack/year history of smoking, and he recently developed a persistent cough. An extensive work-up is done, and he is found to have small cell carcinoma of the lung. What is most likely to be secreted by the tumor? 1. ADH 2. HCG 3. CA 125 4. Alpha-fetoprotein 5. Calcitonin
Correct answer: ADH ADH is antidiuretic hormone. It is normally secreted by the posterior pituitary. It is involved in water homeostasis. Ectopic ADH production can sometimes be seen with lung cancer, intracranial neoplasms, and a few other tumor types. This is the syndrome of inappropriate secretion of ADH (SIADH). HCG is human chorionic gonadotropin. It is normally produced during pregnancy. It can be seen in gestational trophoblastic disease, choriocarcinoma, and germ cell tumors. CA 125 can be found with some ovarian cancer, uterine cancer, pancreatic cancer, lung cancer, breast cancer, and colon cancer. CA125 can also be seen in some normal conditions such as menstruation, pregnancy, and other medical conditions. Alpha-fetoprotein is a tumor marker that can be seen in hepatocellular carcinoma, some germ cell tumors, and some other malignancies and conditions. Calcitonin is normally secreted by the parafollicular cells (C cells) of the thyroid gland. It lowers serum calcium. Calcitonin can be seen with medullary cancer of the thyroid.
A 60-year-old man presents with severe abdominal pain that started 10 hours ago. It is increasing in severity and is colicky in nature. The patient has not had a bowel movement for 3 days. In the last 2 days, he vomited up what he ate 4 times. He looks tired and dehydrated. On examination, increased bowel sounds were noticed. There is also a mass in the right inguinal area. The patient said he the mass has been present for 10 years but disappears when he lies on his back. What is the investigation of choice to confirm the diagnosis? 1 Abdominal X-Ray Erect film 2 Abdominal Ultrasound 3 CBC 4 Colonoscopy 5 Upper GIT endoscopy
Correct answer: Abdominal X-Ray Erect film Intestinal obstruction occurs when bowel movement is encountering an obstacle in the passage through the bowel. Actually this can be caused by any mass, adhesion, or any other means of mechanical obstruction. The symptoms of obstruction vary according to the site of obstruction, but generally there is increased bowel movement proximal to obstruction with the reflected increase in bowel sounds. There may be vomiting. There may be no bowel movement, from absolute constipation to feces and flatus. Here the patient has inguinal hernia which is recently irreducible and obstructed. The best diagnostic procedure is erect X-ray film which shows multiple fluid levels and gives confirmation of the diagnosis. The treatment is exploratory surgery, and there is no place for any conservative treatment without surgery. Paralytic ileus is a type of intestinal obstruction that usually follows general anesthesia and is characterized by silent abdomen with no bowel sounds, usually treated by bowel rest and proper hydration. Colonoscopy here is contraindicated. Upper GIT endoscopy is generally used to detect pathology in upper GIT, not in the case of intestinal obstruction. Abdominal ultrasound is of value in diagnosis of gall bladder disease and also urinary bladder pathology. CBC is generally used to detect leukocytosis in acute appendicitis or inflammatory reactions.
A 35-year-old man presents after several episodes of vomiting in the last 24 hours; there is loose stool and strong pain that is localized in the upper-middle region of the abdomen. Physical examination indicates a temperature of 101°F and a tender epigastrium. Lab tests reveal an initial WBC count of 18x109/L. C-reactive protein level is 325 mg/L, and amylase is 130 U/L. There is a lactate dehydrogenase level of 816 U/L. The patient has no history of pancreatic disease, and declares himself a non-drinker. He is overweight. He has a history of diabetes type-2 and hypertension. He takes medicine to control his high blood pressure and obesity. What is the most appropriate next step in establishing diagnosis for this patient? 1. Abdominal radiography 2. Abdominal ultrasonography 3. Abdominal computed tomography scanning 4. Endoscopic retrograde cholangiopancreatography 5. Endoscopic ultrasonography
Correct answer: Abdominal computed tomography scanning The correct answer is abdominal computer tomography scanning. The symptoms and lab results are consistent with an acute episode of pancreatitis, but extra tests are necessary to confirm the diagnosis. The CT scan test is most appropriate because it provides the most accurate way to see the pancreas. A CT study is useful when other diagnostic studies are inconclusive, when the patient has severe symptoms, or if fever or secondary infection is present. The answer abdominal radiography is not correct. This test is mainlyperformed to detect free air in the abdomen, and it does not provide a good image of the pancreas. The answer abdominal ultrasonography is incorrect. Ultrasound imaging is often used as an initial diagnostic test for people suffering from mild pancreatitis, but when severe, acute pancreatitis is suspected a CT scan test is recommended. The answer endoscopic retrograde cholangiopancreatography is incorrect. This method combines the use of the endoscope and X-ray to determine the cause of pancreatitis. It requires anesthesia of the esophagus and stomach and sedation of the patient. It is considered too invasive to be recommended for patients with severe acute pancreatitis. Endoscopic ultrasonography is not the correct response. This test is usually performed to detect microlithiasis and periampullary lesions that are not easily revealed by other methods.
A 20-year-old female college student presents due to a 7-day history of daily heartburn. She has never had this as bad as she does currently. The patient denies any other significant past medical history and is currently taking only a multivitamin daily. She recently admits to having increased episodes of headaches that she believes are due to stress. For this reason, she has been taking ibuprofen 600 mg every 8 hours. She states that she has been taking this consistently every 8 hours for the last 10 days; she hopes that after finals are over her headaches will subside. She also states she was given a 10 day course of amoxicillin 2 weeks ago for a middle ear infection, which resolved without any further intervention. She denies any difficulty swallowing, weight loss, night sweats, chest pain, black tarry stools, use of tobacco/alcohol, or coughing up blood. Physical examination is unremarkable for any abnormalities. Which of the following medications would you suggest for the patient to begin for her headaches at this time? 1. Antibiotic 2. Anticholinergic 3. Proton pump inhibitor 4. H1-receptor antagonist 5. Acetaminophen
Correct answer: Acetaminophen Patients classified as having mild or intermittent symptoms of esophagitis and/or gastroesophageal reflux disease (GERD) typically are seen as not adversely affecting the patient's quality of life. Initially action is taken in terms of modification of behaviors, such as eating smaller meals and eliminating acidic foods or known foods that precipitate the reflux (fatty foods, alcohol, chocolate, or peppermint). The next step in terms of treatment would be to discontinue any medications that may be increasing the symptoms of GERD. Medications that potentially irritate the esophagus and have a high incidence of causing pill-induced esophagitis include: tetracycline, bisphosphonates, iron supplements, Non-Steroidal Anti-inflammatories (NSAIDS), and potassium supplements. Medications that can increase acid reflux and worsen the condition of GERD include: anticholinergics, calcium channel blockers, narcotics, progesterone, quinidine, benzodiazepines, or even theophylline. A proton pump inhibitor would be initiated in a patient who has moderate to severe esophagitis or gastroesophageal reflux symptoms; this patient does not fit this picture. Since this patient is currently choosing ibuprofen chronically for a recent onset of headaches, the appropriate instruction would be to discontinue the ibuprofen regimen and begin an acetaminophen regimen in its place. Antibiotics would be an incorrect answer. H1 receptor antagonists, or H1-antihistamines, are an inappropriate choice because these are used to treat the symptoms of allergies (seasonal and perennial). The mechanism of action of anticholinergics does not make it an indication for this type of medical complaint.
A 38-year-old woman is going through a divorce and simultaneously filing bankruptcy. She is very stressed about her financial situation and failed marriage. One day, after a particularly long crying spell, she notices a sudden onset of extreme difficulty swallowing at dinnertime. She has difficulty swallowing both solids and liquids. She feels that the food is sticking in her throat. She ignores it. She has numerous bouts of these episodes of difficulty with swallowing. She notices that when she lies down, undigested food comes up. Finally, she sees her doctor. On questioning by her doctor, she insists she has no heartburn. She has not seen any blood when she regurgitates nor has she vomited blood. Manometry is performed which shows an absence of normal peristalsis and an elevated LES pressure. What is the most likely diagnosis? 1 Achalasia 2 Scleroderma 3 Esophageal varices 4 Gastroesophageal reflux disease 5 Mallory-Weiss syndrome
Correct answer: Achalasia Achalasia is characterized by dysphagia. There is poor peristalsis in achalasia. This patient has a characteristic history for achalasia. The onset is during a period of stress. The symptoms consist of dysphagia, complaints of food sticking in the throat, and regurgitation. Scleroderma can present with dysphagia. However, manometry would show a decreased LES pressure, not an elevated LES pressure as the case here. In addition, scleroderma can be associated with heartburn. Esophageal varices usually present with hematemesis. Gastroesophageal reflux disease would present with heartburn. If anything, the LES pressure would be lower than normal, rather than elevated. Mallory-Weiss syndrome is an esophageal tear. This would present with hematemesis.
A 54-year-old male has had long term GERD symptoms. He has been on proton pump inhibitors and has had fair control of his symptoms. Other past history is unremarkable. He is a nonsmoker and drinks socially. Family history is significant for hypercholesterolemia in his father. Physical examination is unexceptional. An endoscopy a few years ago, revealed Barrett's esophagus by biopsy of the esophageal mucosa. He was recommended to have follow-up endoscopy every 2-3 years with mucosal biopsy. This screening was recommended to him because he is in danger of developing 1. Achalasia of the cardia 2 Esophageal stricture 3. Adenocarcinoma of esophagus 4. Squamous cell carcinoma of esophagus 5. Dysphagia lusoria
Correct answer: Adenocarcinoma of esophagus Barrett's esophagus is an abnormality of the distal esophagus characterized by the replacement of normal squamous epithelium by metaplastic columnar epithelium with goblet cells, as a result of continuous inflammation most commonly from acid reflux. This is a premalignant condition with potential to develop into adenocarcinoma over time. Surveillance with screening endoscopy and mucosal biopsy is indicated to detect early onset of high grade dysplasia, which is then treated with esophagectomy. Failure to be compliant with the screening regimen may lead to adenocarcinoma of the esophagus, which has a poor prognosis as it presents late. Achalasia of the cardia is a motility disorder resulting from degeneration of ganglion cells in the esophagus and lower esophageal sphincter. Barrett's esophagus does not predispose to this condition. Esophageal stricture is a distinct complication of long-standing GERD and may co-exist with Barrett's esophagus, since both have the same etiological factor, namely, reflux esophagitis. However, one does not necessarily predispose to the other, and in this patient absence of dysphagia makes the possibility of a stricture unlikely. Squamous cell carcinoma of esophagus is as frequent as adenocarcinoma, but risk factors are smoking and drinking alcohol, not Barrett's esophagus. Dysphagia lusoria occurs when an anomalous blood vessel crosses behind the esophagus causing extrinsic obstruction. The most common vessel is the right subclavian artery.
A 69-year-old edentulous, alcoholic man who lives alone presents for evaluation of a shoulder wound that is not healing well. On physical examination, numerous ecchymoses are noted on the posterior aspect of his legs and thighs. Careful examination of the man's skin reveals minute hemorrhages around hair follicles, and splinter hemorrhages in the nail beds. Laboratory analysis is remarkable for a hemoglobin of 10 (normal 14 - 18 g/dL); no other hematologic abnormalities are noted. What treatment should be included in therapy? 1. Administration of factor VIII 2. Administration of iron 3. Administration of vitamin B12 4. Administration of vitamin C 5 Administration of vitamin K
Correct answer: Administration of vitamin C The patient suffers from scurvy due to a deficiency of dietary vitamin C. Absence of vitamin C leads to impaired hydroxylation of proline residues in the nascent procollagen chains leading to weakness of blood vessel walls. Clinically, the deficiency syndrome is characterized by perifollicular hemorrhages, fragmentation of hairs, purpura, ecchymoses, splinter hemorrhages, and hemorrhages into muscle. In patients with normal dentition, gum changes (e.g., swelling, bleeding, and loosening of teeth) are also noted. Without supplementation with vitamin C, death may eventually occur. Administration of factor VIII would be indicated for factor VIII deficiency. This deficiency would lead to a prolonged PTT (partial thromboplastin time), which was not noted in this patient. Administration of iron would be of benefit in iron-deficiency anemia, but there is no indication of a hypochromic, microcytic anemia in this patient. The anemia of scurvy is typically normochromic and normocytic due to bleeding. Administration of vitamin B12 would be indicated for a megaloblastic anemia. Although a macrocytic anemia may be observed in scurvy (due to concomitant dietary folate deficiency or perturbations in the folate pool), this patient did not show macrocytosis. Administration of vitamin K would be appropriate for vitamin K deficiency. This deficiency would produce prolongations of the prothrombin time (PT), followed by prolongation of the PTT as the vitamin K-dependent factors (II, VII, IX, X, protein C, and protein S) are depleted.
A 55-year-old man from Southeast Asia is visiting his son in the U.S. for several months. While in the United States, he loses weight and starts to develop abdominal pain, anorexia, and ascites. His son takes him to the family doctor. On physical examination, ascites is present and his liver is enlarged. Interestingly, he is not jaundiced. Additional laboratory results are as follows: TEST RESULTS REFERENCE RANGE SGOT 134 IU/L 5-40 IU/L SGPT 121 IU/L 5-35 IU/L bilirubin (total) 2.8 mg/dL 0.2-1.5 mg/dL alkaline phosphatase 113 U/L 20-70 U/L A liver biopsy is done, and he is found to have hepatocellular carcinoma. Which of the following is most often associated with the development of hepatocellular carcinoma? 1. Clonorchiasis 2. Macadamia nuts 3. Aflatoxins 4. Arsenic 5. Opisthorchis viverrini
Correct answer: Aflatoxins Consistent with hepatocellular carcinoma are the weight loss, abdominal pain, and anorexia, which this patient experienced. The physical findings of ascites and hepatomegaly are consistent as well. The lab results reflecting elevated transaminases, elevated bilirubin, and elevated alkaline phosphatase are consistent was well. There is an association of aflatoxins and the development of hepatocellular carcinoma. Aflatoxins are produced by Aspergillus flavus. This is a mold that can be seen on peanuts and stored grains. Clonorchiasis and Opisthorchis viverrini infection are associated with cholangiocarcinoma. Cholangiocarcinoma is a tumor that arises from the bile ducts. It is an adenocarcinoma. Macadamia nuts are not associated with the development of hepatocellular carcinoma. Arsenic is associated with the development of angiosarcomas.
A 66-year-old man presents to your clinic with right upper quadrant pain. The patient immigrated to the United States from Japan 10 years ago, and he denies alcohol or illicit drug use. A CT exam of the abdomen is ordered for further evaluation and demonstrates an enhancing, 5 cm mass in the right posterior hepatic lobe that invades into the portal vein. What tumor marker would be expected to be elevated in this patient? 1 Alpha-fetoprotein (AFP) 2 Carcinoembryonic antigen (CEA) 3 CA-125 4 CA 19-9 5 Galactosyl Transferase II
Correct answer: Alpha-fetoprotein (AFP) Alpha-Fetoprotein is a normal fetal serum protein synthesized by the liver, yolk sac, and gastrointestinal tract that is similar to albumin. It is a major component of fetal plasma, but its primary clinical utility is in diagnosing hepatocellular carcinoma (HCC). AFP is also elevated in testicular germ cell tumors containing embryonal or endodermal sinus elements. A positive marker value is highly sensitive in indicating HCC relapse or response of existing HCC to treatment. The AFP is less frequently elevated in other malignancies such as pancreatic cancers, gastric cancers, colonic cancers, and bronchogenic cancers. AFP is rarely elevated in healthy persons, but mild AFP elevation can occur in certain liver diseases, especially acute viral or drug induced hepatitis and conditions associated with hepatic regeneration. Carcinoembryonic antigen (CEA) is a protein found in many types of cells but associated with tumors and the developing fetus. Although CEA was first indentified in colon cancer, an abnormal CEA blood level is specific neither for colon cancer nor for malignancy in general. Elevated CEA levels are found in a variety of cancers other than colonic, including pancreatic, gastric, lung, and breast. Elevated CEA levels can also be detected in benign conditions, though, including cirrhosis, inflammatory bowel disease, chronic lung disease, and pancreatitis. CA-125 is an antigen present in over 80% of nonmucinous ovarian carcinomas. In the setting of ovarian carcinoma, CA-125 often follows the patient's clinical course. With surgical resection or chemotherapy, the level correlates with patient response. It should be noted that the CA-125 level may be elevated in other cancers, including endometrial, pancreatic, lung, breast, and colon cancer, and in menstruation, pregnancy, endometriosis, and other gynecologic and non-gynecologic conditions. CA19-9 is a monoclonal antibody generated against a colon carcinoma cell line. CA19-9 is found it to be elevated in 20-40% of cases of gastric cancer, 20-40% of colon cancer, and 70-90% of pancreatic cancer. It may be able to differentiate benign from malignant pancreatic disease, but this utility has not yet been established. Galactosyl Transferase II, an isozyme of galactosyl transferase, has been shown to be elevated in a variety of malignancies, predominantly gastrointestinal. In colon cancer, its level correlated with the extent of disease and disease progression. In pancreatic cancer, Galactosyl Transferase II proved to be more sensitive and specific in distinguishing benign from malignant disease than CEA.
A 30-year-old woman presents with a 2-day history of severe pain on defecation; the patient has not had a problem with bowel movements in the past. Recently, she has noticed some increasing constipation; this may be due to starting vitamins that contain calcium. She has had 3 bowel movements in the past 2 days, which she describes as feeling like she was being cut with glass each time she passed them. The pain is excruciating and usually lets up in intensity when the bowel movement ends, but the anal area throbs for 1 - 2 hours afterwards. There is a small amount of blood noted when wiping, but none is noted in the toilet. The patient's past medical history is negative. She takes no prescription medications. There is no family history of colon cancer. On exam, what would you would expect to find? 1. An anal fistula 2. An anal fissure in the anterior anal verge 3. An anal fissure in the lateral anal verge 4. An anal fissure in the posterior anal verge 5 A thrombosed external hemorrhoid
Correct answer: An anal fissure in the posterior anal verge This patient presents with a common anorectal problem, which is an anal fissure. Most anal fissures are the result of constipation and hard stools; occasionally they are due to diarrhea. The history presented is typical, and the vast majority of fissures lie posteriorly in the anoderm; occasionally they are found anteriorly. Laterally positioned fissures are highly unusual and should raise a red flag. Conditions that may be associated with abnormally located fissures include Crohn's disease, HIV infection, anal sepsis, and trauma. An anal fistula (fistula in ano) is a communicating tract between the anal canal and the perineal skin. Patients with this condition would complain of a perianal discharge which may be persistent or intermittent; there may also be soreness. Most fistulas are the result of a persistent internal opening in the anal canal from a previous anorectal abscess. A thrombosed external hemorrhoid also presents as acute anal pain; however, in this instance the pain is continuous for 2 - 3 days. In addition, it is usually uncomfortable for the patient to sit down. The patient may report the presence of a highly tender lump in the anal area.
A 20-year-old woman presents with a 2-week history of anorectal pain and streaks of blood on her stool and toilet paper. She tells you that "she has a tearing pain during each bowel movement". She dreads having a bowel movement and attempts to hold it as long as she can. Her history is also significant for breaking her leg in a skiing accident 4 weeks ago, for which she was prescribed acetaminophen and oxycodone (Percocet) for the first few days due to her pain. Before you examine the patient, what is your top differential diagnosis for this patient? 1. Anal fissure 2. Anorectal fistula 3. Ulcer 4. Hemorrhoids 5. Anal cancer
Correct answer: Anal fissure The patient will complain of significant burning, tearing pain with defecation with bright red blood. Most anal fissures are caused by local trauma to this area. This patient was taking acetaminophen and oxycodone, which often has the common side effect of constipation. She most likely passed hard stool that caused a tear in the anodermis. Hemorrhoids are typically not painful unless they are thrombosed. They are usually very itchy; however, hemorrhoids and anal fissure typically present together. Anorectal fistula usually has purulent drainage around the anal area. This can be painful initially when the tract is forming until it bursts. Rectal ulcers are very uncommon and should not be on the top of your list. Anal cancer would be low on this differential due to absence of sexual history mentioned in this question. However, rectal bleeding is the most common initial symptom of anal cancer. There also is a sensation of a rectal mass in a many individuals.
A 42-year-old woman IV drug user presents with vague symptoms of fatigue, aches, pains, and nausea. Recently, she appeared jaundiced for a few days; however, she no longer does. Preliminary blood tests indicate the patient does not have hepatitis B. However, the tests could be wrong, as she may be in the window period. If she were in the window period, what would be the only evidence of hepatitis B infection? 1 Anti-HBc 2 HBsAg 3 HBcAg 4 HBeAg 5 Anti-HBs
Correct answer: Anti-HBc Hepatitis B is transmitted by exposure to blood or blood products. If she shared needles as an IV drug user, she has put herself at risk of infection with hepatitis B. The window is the period in which HBsAg (HBV surface antigen) is not detectable and anti-HBs (antibody to HBV surface antigen) has yet to appear. During the window period of a hepatitis B infection, the only marker present would be IgM anti-HBc (antibody to HBV core antigen). Because of this, the window period is also referred to as the core window. This IgM antibody is present in the early convalescence period (2 - 16 weeks after infection). It is an expensive test and is ordered only when there is very high suspicion. Positivity of this antibody, along with HbsAg, indicates acute infection. PCR test for genome identification is the other test useful during the window period. HBsAg (HBV surface antigen) is the 1st serological marker to appear in hepatitis B infection. After HBsAg is cleared from the serum, the patient is in the window period. HBeAg (HBV 'e' antigen) can be seen with acute hepatitis. It is a marker for infectivity. HBcAg is the HBV core antigen. Anti-HBs (antibody to HBV surface antigen) is seen at the end of an infection. The appearance of anti-HBs marks the end of the window period. The presence of anti-HBs gives a patient immunity.
A 42-year-old female IV drug user experiences vague symptoms consisting of fatigue, aches and pains, and nausea. She has developed distaste for her cigarettes. She appeared jaundiced for a few days, but the condition spontaneously resolved. She goes to the free clinic and they run preliminary blood tests. The staff at the free clinic tells the patient she does not have hepatitis B; however, they are wrong. In reality, she is in the window period. What would be evidence of a hepatitis B infection during the window period? 1 Anti-HBc 2 HBsAg 3 Anti-HBe 4 HBeAg 5 Anti-HBs
Correct answer: Anti-HBc Hepatitis B is transmitted by exposure to blood or blood products. Since this patient has used IV drugs, if she shared needles, she has put herself at risk to infection with hepatitis B. The window is the period in which HBsAg (HBV surface antigen) is not detectable. Although anti-HBs (antibody to HBV surface antigen) are present, they are actively bound to HBSAg, and therefore not detectable. During the window period of a hepatitis B infection, the only marker present would be anti-HBc (antibody to HBV core antigen). Because of this, the window period is also referred to as the core window. HBsAg (HBV surface antigen) is the first serological marker to appear in hepatitis B infection. After HBsAg is cleared from the serum, the patient is in the window period. Anti-HBs (antibody to HBV surface antigen) is seen at the end of an infection. The appearance of anti-HBs marks the end of the window period. The presence of anti-HBs gives a patient immunity. HBeAg (HBV "e" antigen) can be seen with acute hepatitis. It is a marker for infectivity.
A 28-year-old man presents with recurrent anal discomfort. He has pain with every bowel movement, and the pain will often last for several hours after defecation. He notes blood on the toilet tissue after wiping his anal area. He said he had similar symptoms about 1 year ago, but they resolved without treatment; however, this time he has experienced the pain for several weeks without resolution. He has tried warm sitz baths (which help the pain), but the episodes continue. He denies any anal intercourse. Past medical history is negative. Physical exam reveals a 1 cm narrow ulcer in the posterior midline segment of the anal canal. No fistulae are seen. What would you advise? 1. Continue sitz baths, but use very hot water 2. Apply topical nitroglycerin 0.2% to the perianal area 3. Use hydrocortisone 25 mg rectal suppositories 2 times a day until healed 4. Use sublingual nitroglycerin 0.4 mg as needed for pain 5. Use loperamide 2 mg 3 times a day until healed
Correct answer: Apply topical nitroglycerin 0.2% to the perianal area The patient should apply topical nitroglycerin 0.2% to the perianal area. This patient has an anal fissure. Anal fissures are thought to be caused by anal trauma from hard or large stools; often the fissures become secondarily infected. Fissures often heal with conservative therapy (e.g., warm sitz baths and stool softeners) to improve constipation and lessen trauma to the area during defecation. Glycerin suppositories can help with lubrication of the anal canal while the fissure is healing. For patients who fail conservative therapy, topical nitroglycerin has been shown to be a highly effective and well-tolerated treatment for anal fissure. Nitroglycerin is a smooth muscle relaxant, thereby decreasing anal sphincter pressure. Some patients complain of headaches with nitrate therapy. For those patients who fail a trial of topical nitroglycerin, surgery may be indicated. The other answers are not good choices. Sitz baths should always be warm, hot. Hydrocortisone suppositories can be useful for hemorrhoids, but they may interfere with the healing of a fissure and have not been shown to be beneficial in this condition. Sublingual nitroglycerin is not indicated, nor is loperamide; it would worsen this patient's constipation and anal trauma.
A 72-year-old man presents with progressive shortness of breath over the years. He denies chest pain or a history of smoking. The patient was in the construction business for many years, and before that he worked as a ship builder. Chest X-ray reveals marked interstitial fibrosis and calcified pleural plaques on the lateral chest wall. What is the most likely diagnosis? 1. Silicosis 2. Asbestosis 3. Siderosis 4. Byssinosis 5. Tuberculosis
Correct answer: Asbestosis Asbestosis is an interstitial fibrosis that occurs from lung irritation caused by exposure to asbestos; it is often seen in patients who have an extensive work history of shipbuilding and construction. Pleural calcifications or plaques are typically seen with this disease. Treatment for this disease is supportive. Silicosis, siderosis, byssinosis, and tuberculosis do not cause the calcified pleural plaques that are a typical presentation for asbestosis. Silicosis is often found in patients with occupations such as rock mining or sandblasting. Siderosis is often found in patients with occupations in mining and welding. Byssinosis is found in patients who work in textiles.
A 54-year-old woman presents to the emergency department with nausea, vomiting, right-upper quadrant abdominal pain, fever, and jaundice starting 4 hours ago. During the last 6 months, she suffered several bouts of upper-abdominal pain accompanied by nausea, vomiting, and occasional jaundice, for which medical attention was sought. Her past medical history includes hyperlipidemia, for which she first took simvastatin; she switched to cholestyramine because of side effects. Vital signs on admission are as follows: BP 110 / 80 mm Hg, HR 90 bpm, RR 20 rpm, temperature 38.1°C (100.6 F). She is alert and oriented, and mildly jaundiced. Her right-upper abdomen is diffusely tender to palpation. An upper abdominal ultrasound is performed and reveals a thickening and calcifications of the gallbladder wall, but there are no signs of air within the peritoneal cavity or the bile ducts. What is the most appropriate next step in management? 1 Intravenous hydration and antibiotics 2 Endoscopic papillotomy 3 Give analgesics and refer for elective cholecystectomy 4 Magnetic resonance cholangiography to look for choledocholithiasis 5 Emergency cholecystectomy
Correct answer: Intravenous hydration and antibiotics The presence of fever, jaundice, and right-upper quadrant pain defines Charcot's triad, which is the classical presentation of acute cholangitis. Reynolds' pentad consists of Charcot's triad plus sepsis/shock and mental status changes. 95% of patients presenting with these syndromes have common duct stones. In most cases, there is a favorable response to conservative treatment, which consists of interrupting oral feeding, analgesia, intravenous hydration, and antibiotics. The following antibiotic schemes can be used: A third-generation cephalosporin + an aminoglycoside Piperacillin or ampicillin + metronidazole + an aminoglycoside Monotherapy with imipenem, meropenem, mezlocillin, ampicillin-sulbactam, ticarcillin-clavulanate, or piperacillin-tazobactam After clinical improvement, cholecystectomy is undertaken in the first few days after the initial hospitalization in order to avoid recurrence. Surgery is warranted because the recurrence rate is as high as 10% per year in patients whose gallbladders are not removed. The use of cholestyramine is another risk factor for recurrence in this patient, and gallbladder calcifications (porcelain gallbladder) are thought to pose a risk of malignization and are considered an indication for cholecystectomy. When the disease progresses despite the initial conservative treatment (i.e., there is worsening fever, leukocytosis, abdominal pain, and guarding), emergency biliary drainage is warranted because of the possibility of gallbladder perforation or gangrene. Some possible approaches are cholecystectomy (conventional or laparoscopic), cholecystostomy, or percutaneous drainage. The latter 2 are usually reserved for sicker patients, who are less likely to tolerate surgery well.
A 48-year-old Caucasian woman with a chronic history of inability to tolerate oral intake is admitted to the hospital for J-tube placement. The patient also complains of a rash that she has developed on her upper extremities. Examination reveals a ring of hemorrhage around hair follicles in the upper extremities. What is the most likely etiology of the rash? 1. Niacin deficiency 2. Cobalamin deficiency 3. Biotin deficiency 4. Ascorbic acid deficiency 5. Riboflavin deficiency
Correct answer: Ascorbic acid deficiency The correct answer choice is ascorbic acid deficiency. This patient has scurvy due to ascorbic acid or Vitamin C deficiency. Patients with scurvy typically report a history of a poor diet (tea and toast), or in this case, malnutrition secondary to a chronic inability to tolerate oral intake of foods. Patients with scurvy also present with skin ecchymosis and the characteristic perifollicular hemmorhage. Note that lysyl oxidase, an important enzyme involved in cross-linking collagen, utilizes Vitamin C. Niacin deficiency usually presents as the 3 D's: diarrhea, dermatitis, and dementia. Patients on corn diets or those with an inability to reabsorb tryptophan (Hartnup disease) present with this condition. Cobalamin deficiency presents with megaloblastic anemia. Patients may also present with neurologic disease (subacute combined degeneration). Biotin deficiency presents with dermatitis, alopecia, and lactic acidosis. Riboflavin deficiency presents with cheilosis, angular stomatitis, and glossitis.
A 2-day-old male neonate is noticed to have some abdominal distension. The parents are concerned, since he has not yet passed his first meconium. A few hours later, he starts vomiting and the abdominal distension increases. He continues to throw up all day and his vomitus is noted to be greenish yellow. The baby is restless, irritable, and constantly crying. On exam, he has a temperature of 100.4 F and pulse 112/ minute. His abdomen is distended and tympanic; rectal exam reveals no stool. The physician considers Hirschsprung's disease. What should be ordered first? 1. Ultrasound of the abdomen 2. Anal manometry 3. Barium enema 4. Endoscopy 5. Blood cultures
Correct answer: Barium enema The baby probably has Hirschsprung's disease. It is a congenital motility disorder of the colon caused by lack of innervation in a short segment of the colon, usually the rectosigmoid. This results in failure of relaxation of that part and functional obstruction. Failure of passage of meconium in the first 48 hours is suggestive. If not treated quickly, sometimes eneterocolitis can occur with fever, sepsis, and life threatening toxic megacolon, as is the case in this patient. Barium enema shows a narrowed segment with a dilated proximal colon. Rectal biopsy is the gold standard and will reveal absence of ganglion cells in the affected segment. Treatment is surgical. The goal is to resect the affected segment, bring down normal ganglionic bowel up to the anus, and preserve sphincter function. It may be done as 2-step procedure with an initial diverting colostomy followed by definitive repair or a single stage repair. It should be done soon after diagnosis, often within the first few days or months of diagnosis. Emergency surgery is indicated in Hirschsprung-associated eneterocolitis (HAEC), which is the most lethal complication. Volume resuscitation, intravenous antibiotics, and rectal irrigation are the supportive measures. Ultrasound of the abdomen will be non-diagnostic and is not indicated. Anal manometry is a good screening test for this condition but is less accurate in infants less than 1 month old. It demonstrates lack of relaxation of the internal anal sphincter on rectal distension. Endoscopy is not indicated as visualization is adequate with barium enema and rectal biopsy is done by a mucosal suction technique. Blood cultures may be done in a septic baby for antibiotics, but it is non diagnostic for the cause of the obstruction.
A 38-year-old man presents with a 24-hour history of fever and severe abdominal pain. He has had 10 loose stools that were bloody and contained mucus; they were associated with abdominal distension. He denies any contact with sick individuals, or eating any unusual meals. He denies any recent use of antibiotics. His past medical history is significant for ulcerative colitis since the age of 30. His medication includes mesalamine and acetaminophen. He smokes about 5 - 10 cigarettes each day, and he drinks small amounts of alcohol on occasion. Upon examination, he looks ill and dehydrated. His BP is 130/80 mm Hg, pulse is 120 BPM, temperature is 38.3°C, and RR is 20/min. His abdomen is distended and tender, but without guarding or rigidity; his bowel sounds are decreased. Chest auscultation is clear, with good bilateral air entry. Cardiac sounds are normal, but his pulse is tachycardic. His lab results are: Serum glucose 66 mg/dL Sodium 130 mmol/L Potassium 3.1 mmol/L Chloride 99 mmol/L Bicarbonate 19 mmol/L BUN 25 mg/dL Creatinine 0.7 mg/dL Calcium 11.0 mg/dL Leukocytes count 13,800/μL Segmented neutrophils 95% Hemoglobin 8.1 g/dL Hematocrit 24.2% Platelet count 245,000/μL Supine plain radiographs of the abdomen show moderate dilation of the colon with loss of haustration in the descending colon, along with thumbprinting in the region of the splenic flexure of the colon. Because of the patient's condition and test results, what action is contraindicated? 1 Barium enema 2. CT abdomen 3. Steroids 4. Metronidazole 5. Cyclosporine
Correct answer: Barium enema Toxic megacolon is a complication in both types of inflammatory bowel disease. Common presenting symptoms include diarrhea, abdominal pain, rectal bleeding, tenesmus, vomiting, and fever. In cases of severe colitis, both a barium enema and a colonoscopy are contraindicated because of an increased risk of perforation. A computed tomography (CT) scan should probably be obtained in patients for whom the diagnosis of toxic megacolon (toxic colitis) is considered. A CT scan may identify a local, or contained, perforation. For moderate to severe ulcerative colitis, the initial therapy is oral prednisone. A patient with fulminant ulcerative colitis needs to be hospitalized and treated with IV steroids, infliximab, or cyclosporine; a patient may need a colectomy if the fulminant symptoms persist for more than 48 hours. Broad-spectrum intravenous antibiotics with coverage equivalent to ampicillin, gentamicin, and metronidazole should be initiated.
A 50-year-old man with a past medical history of hyperlipidemia presents with a recurrent "bitterness and sour sensation in his mouth" and a nonproductive cough; both have been occurring over the last year. There is associated progressive dysphagia and odynophagia to solid foods, but not to liquids; as a result, he has lost 15 pounds over the past 5 months. He admits to drinking 1 to 2 beers per week, but he denies additional alcohol use. He also denies cigarette smoking and illicit drug use. He denies fever, chills, sputum production, chest pain, palpitations, dyspnea, shortness of breath, wheezing, peripheral edema, abdominal pain, nausea, vomiting, diarrhea, melena, hematochezia, and changes in bowel habits. The physical exam reveals normal vital signs. Oropharyngeal, neck, cardiopulmonary, and abdominal exams are normal. What is the most appropriate initial intervention for this patient at this time? 1 Perform a chest X-ray 2 Upper endoscopic evaluation 3 Prescribe a trial of esomeprazole 4 Barium esophagography 5 Ambulatory esophageal pH monitoring
Correct answer: Barium esophagography This patient's most concerning diagnosis is a malignant esophageal lesion. However, nonmalignant etiologies may also be responsible for this presentation. A barium esophagography should be obtained as the first study to evaluate dysphagia. In patients with progressive esophageal dysphagia, a radiographic barium study is used to differentiate between mechanical lesions (such as peptic stricture and esophageal cancer) and motility disorders (such as achalasia). The sensitivity of barium radiography for detecting esophageal strictures is greater than that of endoscopy. Barium studies also provide an assessment of esophageal function and morphology that may be undetected on endoscopy. Hypopharyngeal pathology and disorders of the cricopharyngeal muscle are better appreciated on radiographic examination, particularly with videofluoroscopic recording. Either a positive or a negative study is usually followed by an endoscopic evaluation; it is done in order to clarify findings in the case of a positive examination or to add a level of certainty in the case of a negative one. A chest X-ray may reveal adenopathy, a widened mediastinum, metastatic lesions to the lungs or bone, or signs of tracheoesophageal fistula, such as pneumonia. It will not, however, reliably identify esophageal lesions. Endoscopy with biopsy establishes the diagnosis of esophageal carcinoma with high reliability. It is the study of choice for evaluating persistent heartburn, odynophagia, and structural abnormalities detected on barium esophagography. Ambulatory esophageal pH monitoring is the preferred study for documenting acid reflux. It is indicated in the following clinical scenarios: for patients being considered for anti-reflux surgery with a normal endoscopy; in patients with a normal endoscopy but with continued reflux symptoms following therapy with a proton pump inhibitor; and in the evaluation of atypical reflux symptoms that continues despite anti-reflux therapy. Patients with alarm symptoms, such as unexplained weight loss, odynophagia, jaundice, recurrent vomiting, blood loss, a palpable mass or lymph node, or a family history of GI malignancy, should have diagnostic testing performed promptly.
A 4-day-old male infant is seen for a routine newborn check; he has yellow skin from his face down to his chest. He is a full-term infant; vaginal delivery was normal. He weighed 3.180 kg at birth, and 3.100 kg at discharge. He is breastfeeding well; he is also stooling and voiding regularly. He is well-hydrated and active. Laboratory results reveal a direct bilirubin of 3.4 mg/dL, and an indirect bilirubin of 12.3 mg/dL. What is a possible pathologic cause of the jaundice? 1 Breast milk 2 Birth trauma 3 Breastfeeding 4 TPN cholestasis 5 Biliary atresia
Correct answer: Biliary atresia Breastfeeding contributing to dehydration and intensified physiologic jaundice, breast milk jaundice, and birth trauma causing cephalohematoma with increased hemolysis are all possible causes of indirect hyperbilirubinemia. Since the infant is 4 days old, his indirect bilirubin level is not so high as to require treatment or concern at this time. Any direct bilirubin >2.0 mg/dL is abnormal. If it is persistent or increasing it is considered pathologic; it must be evaluated immediately. TPN cholestasis and biliary atresia can both contribute to direct hyperbilirubinemia. TPN cholestasis can occur when an infant has received hyperalimentation for an extended period of time, so it would not occur in a 4-day-old infant. Cholestasis is impaired excretion of the conjugated bilirubin in bile. The most common hepatic cause of jaundice of direct hyperbilirubinemia in infancy is biliary atresia. Ultrasound should be performed to identify blockages or anatomical anomalies.
A 55-year-old woman presents with a feeling of fullness in her head, dyspnea, and cough. Clinical examination shows facial and neck swelling, dilated venous channels over the upper part of her body, facial flushing, and cyanosis. She was recently diagnosed with adenocarcinoma of the lung. Her symptoms are most probably of what nature? 1. Immunological 2. Constitutional 3. Metastatic 4. Paraneoplastic 5. Compressive
Correct answer: Compressive This patient has a clinical picture consistent with superior vena cava compression, which is not uncommon in the presence of a right upper lobe tumor. Because the superior vena cava has a thin wall coupled with a low intravascular pressure and is surrounded by rigid structures (i.e., sternum, trachea, right bronchus, aorta, pulmonary artery, perihilar, and paratracheal lymph nodes), it is relatively easy to compress. The subsequent obstruction to flow causes an increased venous pressure which results in interstitial edema and retrograde collateral flow. An immunological reaction is not likely the cause of symptoms that are strongly suggestive of a well-defined compressive lesion. This patient has a well-defined syndrome pointing towards specific organ involvement. Constitutional symptoms (e.g., fatigue, malaise, weight loss, fever, chills, and night sweats) are nonspecific manifestations of a vast number of diseases and conditions. In the presence of a nearby primary tumor, it is unlikely that a metastatic tumor cell from a distant location would cause superior vena cava compression. A paraneoplastic syndrome is not likely the cause of symptoms that are strongly suggestive of a compressive mechanism.
A 66-year-old man presents with tingling sensations in his hands and feet; his movements are clumsy, and he has difficulty with walking. He notes that he feels generally weak, and that he sometimes has 'the feeling that he can not locate his limbs'. He often feels depressed or irritable. His vitals are normal, but neurological examination reveals decreased reflexes. What test should be performed next? 1. Blood tests to measure level of cobalamin 2. Blood tests to measure level of thiamine 3. Blood tests to measure the level of glucose 4. Thyroid-stimulating hormone (TSH) test 5. Neuroimaging studies
Correct answer: Blood tests to measure level of cobalamin Blood tests should be performed to measure the level of cobalamin. Based on the symptoms, the patient likely suffers from subacute combined degeneration of spinal cord due to degeneration of the posterior and lateral columns of the spinal cord. This is a result of a vitamin B12 (cobalamin) deficiency; a blood test to determine the level of vitamin B12 should be performed to confirm the diagnosis. Performing blood tests to measure the level of thiamine is an incorrect response. A deficit in vitamin B1 (thiamine) causes symptoms that are characteristic of patients with Wernicke-Korsakoff syndrome. These include confusion and movement disability; however, the patients with Wernicke-Korsakoff syndrome do not generally have tingling or numbness sensations. Performing blood tests to measure the level of glucose is an incorrect response. Symptoms such as pain, tingling, and numbness in the hands and feet, as well as depression, can be signs of developing diabetic neuropathy. This patient lacks other symptoms associated with diabetes, making this diagnosis unlikely; therefore, testing for glucose levels is not a necessary next step. A thyroid-stimulating hormone (TSH) test is an incorrect response. Muscle weakness, numbness, pain, as well as depression, are associated with the late stages of chronic hypothyroidism; however, these symptoms are not sufficient to suspect hypothyroidism. Hypothyroidism is characterized by weight gain, loss of concentration, and hearing loss in the elderly, which are not conditions that affect this patient; therefore, a TSH test would not be useful in this case. Neuroimaging studies are performed to provide information about brain tumors, skull fractures, and other diseases of the brain and blood vessels in the brain. Based on the symptoms seen in this patient, neuroimaging would not help in establishing a diagnosis.
An 87-year-old woman presents with progressive shortness of breath. She has been in a wheelchair for 15 years due to paralysis of her lower extremities from unknown causes. At this time, she is unable to transfer from the chair to her wheelchair without having dyspnea. She is extremely tired, but denies chest pain, palpitations, cough, hemoptysis, dysphagia, hoarseness, or sick exposures. She has never smoked. Her past medical history is positive for hypertension (treated with enalapril), heart failure, chronic kidney disease, hepatitis C, breast cancer, s/p lumpectomy, and radiation treatment 10 years ago. You order a chest X-ray. What in her medical history would lead you to suspect an exudative pleural effusion? 1. Breast cancer 2. Chronic kidney disease 3. Cirrhosis 4. Heart failure 5. Hypertension
Correct answer: Breast cancer A pleural effusion results from an abnormal accumulation of fluid in the pleural space. Transudative pleural effusions suggest the absence of localized pleural disease and are caused by increased hydrostatic pressure, as seen in heart failure, or decreased oncotic pressure, as seen in cirrhosis. Because there is no underlying pleural disease process, transudative pleural effusions are often bilateral. Exudative pleural effusions are indicative of an underlying pleural disease process, such as pneumonia or a malignancy. A diagnostic thoracentesis can help distinguish transudative from exudative pleural effusions. The fluid should be sent for a cell count, chemistry, and, if indicated, cytology or culture. A pleural effusion is diagnosed as exudative based on the chemistry when the pleural fluid meets 1 or more of the following criteria: "1) ratio of pleural fluid protein to serum protein greater than 0.5; 2) ratio of pleural fluid LDH to serum LDH greater than 0.6; and 3) pleural fluid LDH greater than 2/3 the upper limit of normal serum LDH." The patient presented in this case was treated for left-sided breast cancer with a lumpectomy and radiation therapy 10 years ago. She underwent a thoracoscopy and pleurodesis. During the procedure, 1500 cc of pleural fluid was removed. Cytology was positive for breast cancer. A patient with chronic kidney disease, heart failure, or cirrhosis is more likely to have bilateral transudative pleural effusions. Hypertension by itself is not a risk factor for a pleural effusion.
A 72-year-old man is evaluated at the bedside following hospital admission for a 1-year history of progressive dyspnea, weight loss, low-grade fevers, fatigue, and myalgias. His past medical history is remarkable for atrial fibrillation, for which he takes amiodarone, hypercholesterolemia, and recurrent urinary tract infections, for which his urologist prescribed nitrofurantoin on a chronic, prophylactic basis. He denies any cigarette use, occupational exposure, history of murmurs or coronary artery disease, chest pain, wheezing, hemoptysis, syncope, abdominal pain, rashes, peripheral edema, diaphoresis, or vomiting. His physical exam reveals bilateral basilar crackles and digital clubbing, but is otherwise normal. A chest x-ray revealed peripheral reticular opacities at the lung bases and a generalized honeycombing pattern (refer to image). Which of these is correct regarding additional diagnostic findings in this patient? 1. A high C-reactive protein, ESR, and titers of antinuclear antibodies are confirmatory. 2. Pulmonary function testing is expected to demonstrate an obstructive pattern. 3. Transbronchial biopsy is required for the diagnosis of this illness. 4. Bronchoalveolar lavage typically reveals neutrophilia and eosinophilia. 5. Computed tomography of the chest is an unnecessary diagnostic test.
Correct answer: Bronchoalveolar lavage typically reveals neutrophilia and eosinophilia. This patient's most likely diagnosis is idiopathic pulmonary fibrosis. Reportedly, up to 30% of patients with idiopathic pulmonary fibrosis (IPF) have positive tests for antinuclear antibodies or rheumatoid factor; however, these titers are generally not high. The presence of high titers of antinuclear antibodies or rheumatoid factor may suggest the presence of a connective-tissue disease. The C-reactive protein value and erythrocyte sedimentation rate can be elevated in patients with idiopathic pulmonary fibrosis; however, this finding is nondiagnostic. The typical findings on pulmonary function tests in patients with idiopathic pulmonary fibrosis are a restrictive ventilatory defect and a reduced diffusion capacity for carbon monoxide. Bronchoscopy with BAL and/or transbronchial biopsy is not required for the diagnosis of idiopathic pulmonary fibrosis. However, it can be used to ensure that alternative diagnoses are excluded. Given the high-quality evidence regarding high resolution CT-scanning (HRCT) specificity for the recognition of histopathologic idiopathic pulmonary fibrosis pattern, transbronchial or surgical lung biopsy is not essential in making the diagnosis. Increased numbers of neutrophils in BAL fluid are found in 70-90% of all patients with idiopathic pulmonary fibrosis, and increased numbers of eosinophils in BAL fluid are found in 40-60%. Virtually all patients with idiopathic pulmonary fibrosis (IPF) have an abnormal chest radiograph at the time of diagnosis, with typical findings of peripheral reticular opacities (netlike linear and curvilinear densities) predominantly at the lung bases, honeycombing (coarse reticular pattern), and lower lobe volume loss. The chest radiograph lacks diagnostic specificity for idiopathic pulmonary fibrosis. The typical findings are peripheral reticular opacities predominantly at the lung bases, honeycombing, and lower lobe volume loss. High-resolution computed tomography (HRCT) findings are significantly more sensitive and specific for the diagnosis of idiopathic pulmonary fibrosis and are an essential component of the diagnostic pathway of idiopathic pulmonary fibrosis. On HRCT images, idiopathic pulmonary fibrosis is characterized by patchy, predominantly peripheral, predominantly subpleural, and bibasilar reticular opacities.
A 27-year-old man has been experiencing heartburn on and off for approximately 6 years. At this current visit, the patient is also describing symptoms of chest pain, dysphagia, and even early satiety. Because of the persistence of his symptoms despite pharmacologic therapy with both proton pump inhibitors and various histamine blockers, you decide to consult with a local gastroenterologist group. The patient undergoes an endoscopy, with biopsies of tissue. Results of the biopsy show no ulcers and are H. pylori negative; the tissue samples themselves are prominently infused with an abundant amount of eosinophils. Based on the history and physical exam findings above, what is the most effective pharmaceutical intervention at this time? 1. Oral omeprazole 2. Oral ampicillin 3. Oral cetirizine 4. Fluticasone metered dose inhaler 5. Topical fluconazole
Correct answer: Fluticasone metered dose inhaler The correct response is fluticasone metered dose inhaler. The patient described in the above scenario has the diagnosis of eosinophilic esophagitis (EE). Because this is a relatively new diagnosis, the prevalence of EE is unknown, although the incidence is increasing in both children as well as adults. These patients will have symptoms typically 4 - 5 years before they have a confirmed diagnosis. EE is more commonly seen in male patients who are Caucasian, and up to 70% of patients with EE have personal history of atopy to environmental or foods. The most common symptoms found in adults with EE are dysphasia, refractory heartburn, chest pain, and food impaction. Usually, no physical exam abnormalities will be detected. An endoscopy with esophageal biopsy is the only definitive method to diagnose EE. The presence of eosinophils of 15 or more per high power field is required for a diagnosis. Other physical findings that may be visualized during an endoscopy that are highly suggestive of EE include stacked circular rings ('feline esophagus'), proximal strictures, linear furrowing, as well as white exudates or even papules that signify eosinophilic purulent material. Currently there are no FDA approved medications for the treatment of EE. The most efficient treatment for symptoms is topical corticosteroids, either via fluticasone metered dose inhaler or viscous budesonide. These medications are swallowed and therefore coat the esophagus; the symptoms as well as the overall eosinophilia improve fairly quickly. Patient response is usually in the first few days of treatment; however, symptoms will reoccur once the medication regimen is discontinued. Topical fluconazole is not the correct option; EE is not a fungal infection. Oral ampicillin is not correct because EE's underlying pathology is not a bacterial infection. Oral omeprazole is not specifically recommended for treatment of EE; however, it may treat the co-existent GERD. Oral cetirizine is also not an appropriate choice; if there is evidence of severe atopy, therapy should be initiated, but no evidence presents in the case scenario showing that the treatment of airborne allergic disorders improves EE.
A 62-year-old woman has an unexplained weight loss. She has a vague, non-descriptive pain in her stomach accompanied by diarrhea. When her son sees that she has become yellow, he insists that she seek medical attention. Numerous tests are done, including endoscopic retrograde pancreatography, and she is told that she has pancreatic cancer. The tests reveal that the pancreatic cancer is located at the head of the pancreas. What tumor marker is most likely to be found? 1. CA 19-9 (carbohydrate antigen 19-9) 2. AFP (alpha-fetoprotein) 3. CALLA (common acute lymphoblastic leukemia associated antigen) 4. PSA (prostate specific antigen) 5. Gastrin
Correct answer: CA 19-9 (carbohydrate antigen 19-9) Pancreatic carcinoma can present with abdominal pain, jaundice, and weight loss. Jaundice would be seen with pancreatic carcinoma located at the head of the pancreas because it obstructs the common bile duct. CA 19-9 is carbohydrate antigen (CA) 19-9. This marker is a sensitive marker in patients with pancreatic carcinoma. This marker can be elevated in other GI cancers as well as some other medical conditions. AFP is alpha-fetoprotein. Alpha fetoprotein is a protein. It is produced by the human fetus. Alpha-fetoprotein is found with an assortment of neoplasms. Elevated levels of alpha fetoprotein are seen with hepatocellular carcinoma. It is also seen with pancreatic carcinoma, teratocarcinomas, and also some gonadal tumors. CALLA is common acute lymphoblastic leukemia associated antigen. CALLA can be seen on malignant lymphocytes. PSA is prostate specific antigen. It is a protein that is made in the prostate gland. PSA is elevated with prostatic tumors. An elevated PSA may be the 1st sign of prostate cancer. Gastrin is a hormone. Gastrin is also produced by gastrinomas. Gastrinomas are a type of islet cell tumor. Because of the excess gastrin production, peptic ulcers are seen with gastrinomas.
A 35-year-old female comes to your office complaining of frequent flushing and chronic diarrhea that started a few months ago. She also notices progressive edema of her lower limbs and enlargement of her abdomen. On examination, she looks cachexic with congested neck veins, lower limb edema, and a palpable tender liver with irregular surface. There is a pansystolic murmur heard inside the apex that increases in intensity with inspiration. Chest X-ray shows cardiomegaly with dilated right atrium and clear lung fields. ECG shows right atrial abnormality (P pulmonale). Echocardiography shows thickened, fibrotic and fixed tricuspid valve in a semi opened position along with right atrial and ventricular enlargement. 24-hour urinary excretion of 5-hydroxy-indole-acetic-acid (5HIAA) is grossly elevated. Liver ultrasonography shows multiple small nodules. What is the most probable diagnosis? 1. Endocardial fibroelastosis 2. Hypereosinophilic syndrome 3. Carcinoid syndrome 4. Dilated cardiomyopathy 5. Churg-Strauss syndrome
Correct answer: Carcinoid syndrome Explanation In Carcinoid Syndrome, 60-90% of tumors arise from the small bowel. It is caused by a metastasizing carcinoid tumor and is characterized by cutaneous flushing, diarrhea, bronchoconstriction, and endocardial plaques composed of a unique type of fibrous tissue. These effects are due to the circulating humoral substances as serotonin and bradykinin. Cardiac affection happens only when there is metastasis to the liver. The right side of the heart is mainly affected when the humoral substances are not inactivated by the liver due to hepatic metastasis. The tricuspid and pulmonary valves are affected with fibrous plaques (composed of smooth muscle cells in stroma rich in mucopolysaccharide) and as a result, they are fibrosed and fixed in a semi-opened position with both incompetence and stenosis. Fibrous plaques can invade the endocardium of the cardiac chambers, the intima of the inferior vena cava, pulmonary artery, and coronary sinus. Left side valvular affection occurs less commonly and is usually associated with right to left shunts, or when the tumor originates from the lung. The patients mainly complain of flushing, diarrhea with manifestations of systemic congestion as lower limbs edema, congested liver, congested neck veins, and ascites. Physical examination usually reveals a systolic murmur along the left sternal border produced by tricuspid regurgitation; there may be a mid-systolic murmur or early diastolic murmur due to pulmonary valve affection. There are 2 key diagnostic modalities for the diagnosis of carcinoid heart disease, which includes 24-hour urinary excretion of 5-hydroxy-indole-acetic-acid (5HIAA) and transthoracic echocardiography. ECG and chest X-ray are of limited value. The chest X-ray is normal in half the patients, but may reveal enlargement of the heart and pleural effusions or nodules. The pulmonary artery trunk is typically of normal size without evidence of post-stenotic dilatation as occurs in congenital pulmonic stenosis. ECG shows ST, T wave changes, right atrial abnormalities, and possibly sinus tachycardia. Echocardiography may reveal evidence of tricuspid and/or pulmonary valve thickening, along with right atrial and right ventricular dilatation; small pericardial effusions are present in a minority. The principles of management of patient with carcinoid heart disease involve treatment of heart failure, pharmacotherapy for the reduction of tumor products, and surgical/interventional management of valvular pathology. Treatment includes symptomatic digitalis and diuretics for treatment of heart failure, and somatostatin analogs such as (Octreotide). Octreotide has been found to improve symptoms as well as survival. GIT main side effects are loose stool, abdominal discomfort, and cholelithiasis. Balloon valvuloplasty of the tricuspid and pulmonic valves may improve survival and symptoms. Surgical treatment, like tricuspid valvotomy and valvectomy, may result in symptomatic improvement; mortality is high especially in the elderly. The long-term mortality remains high regardless of treatment modality, with half the patients dead within 1 to 2 years. Churg-Strauss syndrome is a granulomatous medium and small vessel vasculitis. The American College of Rheumatology (ACR) has proposed 6 criteria for the diagnosis of Churg-Strauss syndrome, which include the following: 1. Asthma 2. Eosinophilia of more than 10% in the peripheral blood 3. Pulmonary involvement such as infiltrates 4. Paranasal sinusitis 5. Mononeuritis multiplex or polyneuropathy 6. Histological proof of vasculitis with extravascular eosinophils. Cardiac manifestations include heart failure, myocarditis, pericarditis, constrictive pericarditis, and myocardial infarction. Chest X-ray may show mild infiltrates or nodules. Treatment is symptomatic and systemic anti-inflammatory may improve symptoms. Dilated cardiomyopathy is characterized by dilated hypokinetic left ventricle with impaired both systolic and diastolic functions and mitral incompetence. Patients complain of left sided heart failure manifestations such as dyspnea and orthopnea. The right ventricle may be affected as well with manifestations of congestive heart failure. Treatment is usually symptomatic with diuretics, vasodilators, and inotropic, cardiac transplantation may be the treatment of choice especially in end stage patients. Hypereosinophilic Syndrome typically presents in fourth decade males living in a temperate climate. The cause of hypereosinophilia (persistent eosinophilia with 1500 eosinophils/μL for at least 6 months) is unknown and is not associated with leukemia or reaction to parasitic infestations, allergic, hypersensitivity, or neoplastic disorders. There is organ involvement other than the heart, such as the lungs, skin, and bone marrow. Early in the disease (before fibrosis), medical therapy is the treatment of choice. It includes corticosteroids and cytotoxic drugs such as hydroxyurea. Routine cardiac therapy with digitalis, diuretics, afterload reduction, and anticoagulation as indicated are adjuncts in the management of these patients. Surgical therapy is indicated with advanced stage of the disease (fibrosis). Endocardial fibroelastosis is a condition commonly seen in infants and young children, which is characterized by endocardial thickening and myocardial dysfunction. This condition has a poor prognosis and terminates in death within weeks, especially in progressive heart failure. Viral etiology has been implicated in most cases. Infants present with feeding difficulty, failure to thrive, breathlessness, excessive sweating, and wheezing. Treatment is usually symptomatic with diuretics, vasodilators, and inotropic drugs. Cardiac transplantation may be the treatment of choice, especially in end stage patients.
A 65-year-old man presents with abdominal pain, vomiting, and weakness. On examination, he is cachectic, pale, and jaundiced. A walnut-sized mass is palpable in the right hypochondrium. What is the most likely diagnosis? 1 Carcinoma of head of pancreas 2 Acute or chronic cholecystitis 3 Fibrolipoma 4 Hydatid cyst of the liver 5 Primary biliary cirrhosis
Correct answer: Carcinoma of head of pancreas Carcinoma of the head of pancreas is a common 'duodenal C-loop cancer'. Seen mostly in men, it is usually a ductal adenocarcinoma. It causes obstructive jaundice, weight loss, itching, and upper abdominal pain. There may be diabetes, bleeding problems, malabsorption syndromes, and LFT showing obstructive jaundice. This carcinoma has a better prognosis than cancer of the body and tail of the pancreas when resection is possible. Courvoisier's law states that when the gall bladder is palpable (but not tender) in the presence of jaundice, the cause is usually cancer. Cholecystitis is a diagnosis of 'female, fat, fertile, forty, and flatulent'. It does occur in men. Jaundice may or may not be present. Vomiting usually brings relief to the upper abdominal pain. Muscle guarding and rebound tenderness is present. A fibrolipoma over the right hypochondrium may be a coincidental finding, but would not be the cause of the overall clinical features of a malignancy. A hydatid cyst results from an E. Granulosus infection, and cysts may form in the liver, lung and other organs. It may become palpable in the right hypochondrium and cause pressure symptoms. Migrant workers and immigrants from endemic areas may carry this infection. Primary biliary cirrhosis is an autoimmune disease that destroys the liver structure and system; it is mostly seen in women over the age of 40. There is itching, xanthomata, and hepatosplenomegaly.
A 42-year-old man presents with a 3-day a history of fever with chills, cough with greenish-yellow sputum, and chest pain. He finds himself short of breath after walking 1 block. He has body aches, headache, and lack of appetite. Since this morning, he has been nauseated, and he has thrown up 3 times. His past medical history is significant for an appendectomy at age 16 and right knee arthroscopy 3 years prior to presentation. He has no drug allergies. He is not presently on any medications. Family history is significant for coronary artery disease in his father. On exam, he has a temperature of 101.1°F, and pulse rate is 104/min; BP is 110/67 mm Hg, and SPO2 is 91%. Chest X-ray shows consolidation in the right lower lobe of the lung. EKG has sinus tachycardia; troponin is 0.1, BUN is 17 mg/dL, creatinine is 1.2 mg/dL, electrolytes are normal, and urinalysis is normal. What regimen should you start pending blood and sputum cultures? 1. Levofloxacin 500mg IV Q24H 2. Levofloxacin 500mg PO daily 3. Levofloxacin 250mg IV Q24H and azithromycin 250mg IV Q24H 4. Ceftriaxone 1gm IV Q24H and azithromycin 500mg IV Q24H 5. Ceftazidime 1gm IV Q24H and vancomycin 1gm IV Q12H
Correct answer: Ceftriaxone 1gm IV Q24H and azithromycin 500mg IV Q24H The patient is possibly suffering from community-acquired pneumonia (CAP). He was admitted to the hospital because of vomiting; as a result, he would be unable to keep oral antibiotics down, so IV antibiotics are required. CAP is defined as a pneumonia that starts outside the hospital or nursing home or occurs within 48 hours of being admitted to a hospital, without any history of residence in a long-term facility for 2 weeks or more prior to the onset of symptoms. The most common causative organism is streptococcus pneumoniae. Other organisms include Haemophilus influenzae (especially in smokers), atypical organisms (e.g., Mycoplasma pneumoniae, chlamydia, and legionella), and viruses, most commonly influenza, but occasionally parainfluenza, respiratory syncytial virus, adenovirus, etc. Staphylococcus aureus is found in patients with influenza, cystic fibrosis, bronchiectasis, nursing home residents, and drug abusers. Klebsiella pneumoniae may be isolated in alcoholics and diabetics. Anaerobic bacteria can cause pneumonia in patients with poor dental hygiene or aspiration. Pseudomonas is another bacteria found in cystic fibrosis, bronchiectasis, and institutionalized patients. A complete blood count with differential, basic metabolic panel and pre-antibiotic sputum and blood cultures should be done in all hospitalized patients. Sputum culture is recommended in non-hospitalized patients. When sputum is not provided by a patient and culture is necessary, then transtracheal aspiration, fiberoptic bronchoscopy, and transthoracic needle aspiration may be done to obtain samples. Chest X-ray is helpful in many ways. Usual findings include lobar consolidation, patchy infiltrates, pleural effusion, cavitations, and other pulmonary abnormalities. Radiography helps in diagnosis and is also useful in assessing severity and response to treatment. Lung infiltrates may take up to 6 weeks to clear on X-ray. Pleural tap may be done in patients to rule out empyema; it may also be done for diagnostic reasons. The causative organism directs treatment. A combination of ceftriaxoneandazithromycincoversGram-positive and atypical organisms. Levofloxacin IV is also a good monotherapy in CAP, but it is not recommended in order to avoid drug resistance to quinolones. It should be used only in cases where the other 2 medications are contraindicated, such as in the case of a drug allergy; however, it is an invaluable antibiotic in cases of severe healthcare-acquired pneumonia (HCAP). Treatment should be continued for 7 - 14 days. Levofloxacin PO is not recommended in this case due to the patient vomiting. If this patient had not been vomiting and was overall stable, outpatient treatment with oral levofloxacin for 7 - 14 days would have been an option; however, it would not have been highly recommended due to the danger of resistant organisms. In such a case, oral azithromycin for 7 - 10 days would also be a good option. Clarithromycin and doxycycline for 7 - 10 days may also be used. Ceftazidime and vancomycin combination is used in HCAP; its coverage includes Gram-negative organisms and methicillin-resistant staphylococcus aureus.
An 11-year-old boy presents with a chronic history of mild hemolytic anemia, intermittent jaundice, and right upper quadrant pain. He denies any shortness of breath, muscle aches, or joint pain. Supplementation with which of the following should be given to the patient as part of the treatment plan? 1 Thiamine 2 Cyanocobalamin 3 Folic Acid 4 Niacin 5 Riboflavin
Correct answer: Folic Acid Folic acid should be given to patients who present with hemolytic anemias such as those with hereditary spherocytosis as suggested in the above case. Hemolytic anemias cause reticulocytosis, which leads to an increased demand for folic acid in the formation of red blood cells. The treatment of choice for hereditary spherocytosis is splenectomy, as the spleen is responsible for sequestering and destroying the spherocytes, leading to the hemolytic anemia. Thiamine (or B1), Cyanocobalamin (or B12), Niacin (or B3), and Riboflavin (or B2) are all B vitamins; increased amounts are not required in the setting of a hemolytic anemia with reticulocytosis.
A 25-year-old man presents with fever, cough, and shortness of breath. He is experiencing chest pain and had a sore throat 2 weeks ago. He is working as a sales officer. He is married and denies any extramarital affairs. He has not traveled abroad in the recent past and denies using any illicit drugs. What is the most likely cause of his symptoms? 1. Chlamydophila pneumoniae 2. Legionella pneumophila 3. Streptococcus pyogenes 4. Klebsiella pneumoniae 5. Pneumocystis jiroveci
Correct answer: Chlamydophila pneumoniae Chlamydophila pneumoniae is a common cause of pneumonia around the world. C. pneumoniae causes mild pneumonia or bronchitis in adolescents and young adults. Older adults may experience more severe disease and repeated infections. These patients present with fever, malaise, headache, and nonproductive cough. Often there is biphasic illness. The patient presents with a sore throat that is negative for group A strep. Then, 2-3 weeks later, hoarseness and pneumonia develop. Effective antibiotic therapies include doxycycline or macrolides for 3 weeks. With Legionella, patients typically experience gastrointestinal symptoms, such as diarrhea, vomiting, and abdominal pain, as well as neurological manifestations and high fever. Labs can show hyponatremia and hypophosphatemia. Treatments include macrolides or quinolones. Streptococcus pyogenes presents with acute onset of fever, chills, chest pain, and dyspnea similar to pneumococcus. Klebsiella typically occurs in immuno-compromised patients and/or those with uncontrolled lung disease or diabetes. Pneumocystis jiroveci is the most common opportunistic infection in AIDS patients. P. jiroveci pneumonia should be considered in any HIV+ patients with pulmonary symptoms. Treatment includes IV or oral Trimethoprim-sulfamethoxazole or IV Pentamidine.
A 45-year-old man presents with a 1-week history of high-grade fever, abdominal pain, occasional rigors, loss of appetite, malaise, nausea, and vomiting. His symptoms began about a week ago when he developed a low-grade fever for 2 days. Fever was high-grade during the last 3 - 4 days, with occasional rigors accompanied by abdominal pain predominantly in the right upper abdomen. Pain is moderate in intensity, continuous without any radiation, and is not relieved by acetaminophen (Tylenol), which he is taking on his own. Examination reveals tender hepatomegaly. No other organs are palpable. Abdomen is slightly distended, but bowel sounds are normal. Vital signs are as follows: temperature 40° C, pulse 108 bpm, blood pressure 136/90 mm Hg, and respiratory rate 20 per minute. The patient appears toxic. The rest of the physical examination is as follows: Neck: supple, no stridor Lymphatics: no lymph nodes are palpable Lungs: decreased breath sounds at the base of the right lower lung Cardiac: normal S1 and S2, no extra heart sounds Back: negative Groin: unremarkable Genitalia: unremarkable Neurological: normal cranial nerves, normal motor and sensory examination Ultrasonography: multiple liver abscesses in both lobes of the liver, confirmed by similar findings from CT scan What is the most common cause of the patient's liver abscess? Pylephlebitis 2 Cholangitis 3 Endocarditis 4. Subphrenic abscess 5 Pyelonephritis
Correct answer: Cholangitis Cholangitis is the most common cause of liver abscesses. Biliary pathologies are the most common predisposing cause of liver abscesses and account for approximately 60% of the cases. The etiological classification of a bacterial abscess of the liver (pyogenic liver abscess) is made on the basis of the route by which the infection reaches the liver. There are 5 mechanisms: Portal bacteremia (via the portal vein) from an intra-abdominal site (e.g., diverticulitis, suppurative appendicitis, infected carcinoma of the colon especially after resection, ulcerative colitis, inflamed hemorrhoids, typhoid, and paratyphoid fever) Along the bile ducts, causing ascending cholangitis in a biliary tract partially or completely obstructed by stone, tumor, or stricture Systemic bacteremia originating in a distant location with organisms reaching the liver via the hepatic artery; causes include septicemia and pyemia, endocarditis, pyelonephritis, and infection of the hydatid cyst Direct extension from an adjacent infection outside the biliary tract (e.g., from a sub-diaphragmatic abscess, from an empyema thoracis, and from trauma, either penetrating with direct implantation of bacteria into the liver, or blunt causing a hematoma that becomes secondarily infected From the umbilicus along the umbilical vein of the newborn and along the paraumbilical veins A cause is usually obvious, but sometimes the abscess is unexplained. Although most abscesses are single, multiple (usually microscopic) abscesses are common with systemic bacteremia or complete biliary tract obstruction. Prompt antibiotic treatment, with agents such as meropenem, imipenem, and cefuroxime, followed by percutaneous drainage, form the essential components of treatment. Biliary tract diseases account for about 60% of pyogenic liver abscesses. Systemic bacteremia accounts for 15% of the cases. About 1/4 can be attributed to pylephlebitis, an inflammation of the portal vein or any of its branches. Direct extension from an adjacent infection, such as a subphrenic abscess, account for <5% of cases.
A 40-year-old man presents with diarrhea. The diarrhea is not blood stained. It began 12 hours after eating some meat at a cookout. Examination of his stool reveals no leukocytes or trophozoites. What is the causative organism? 1. Staphylococcus aureus 2. Clostridium perfringens 3. Escherichia coli serotype O157:H7 4. Giardia lamblia 5. Salmonella typhi
Correct answer: Clostridium perfringens In Clostridium perfringens food poisoning, patients usually present with abdominal cramps and watery diarrhea 8-24 hours after ingestion of the contaminated food. Common culprits include inadequately cooked meat or legumes. Diagnosis is made by isolating the organisms from the patient's stool. No fecal leukocytes or trophozoites are present on stool examination. Treatment is supportive with rehydration, as it is usually a mild self-limiting illness. InEscherichia coli serotype O157:H7 food poisoning, patients present with watery diarrhea, which can become bloodstained. Sources of infection include undercooked beef. A stool examination reveals polymorphonuclear leukocytes. Giardia lamblia can be acquired by drinking contaminated water. Patients present with watery diarrhea. The incubation period is around 1 week. There are trophozoites in the stool but no leukocytes. Patients with Salmonella typhi food poisoning usually present with bloody diarrhea. Sources of infection include beef, poultry, eggs, and dairy products. Examination of the stool reveals mononuclear leukocytes. Patients with Staphylococcus aureus food poisoning may present with vomiting and watery diarrhea 1 to 6 hours after ingesting the contaminated food like processed meat. There are no fecal leukocytes or trophozoites.
A 16-year-old girl with a 2-year history of ulcerative colitis presents with signs of an acute exacerbation (i.e., abdominal pain and passing large, frequent quantities of blood and mucus from the rectum). It is treated with sulfasalazine, glucocorticoids, and intravenous alimentation. The diarrhea decreases markedly, but her status continues to deteriorate. Tachycardia, volume depletion, and electrolyte imbalance develops; her current temperature is 101.8° F. Physical examination finds abdominal tenderness, but no mass. Plain radiography shows extensive ulceration and the transverse colon is dilated up to 7 cm. What is the most appropriate next step in the management? 1 Proctocolectomy, distal rectal mucosectomy, and ileal pouch-anal anastomosis 2 Colectomy 3 Barium enema 4 Colonoscopy 5 Tapering glucocorticoids
Correct answer: Colectomy This patient's symptoms and signs are characteristic of toxic megacolon, a dangerous complication of ulcerative colitis. It carries the life-threatening risk of perforation and septicemic shock. A barium enema and colonoscopy have to be avoided due to the danger of further precipitating toxic megacolon. The failure of medical treatment also indicates the need for surgery. Proctocolectomy, distal rectal mucosectomy, and ileal pouch-anal anastomosis is an increasingly popular surgical treatment of colitis ulcerosa; however, it cannot be performed in the midst of an acute exacerbation. Emergency colectomy remains the only viable option. Tapering of glucocorticoids is appropriate when healing progresses, but it does not represent an effective treatment of toxic megacolon.
A 44-year-old woman is referred to a hospital because of occasional headache accompanied by irritability, confusion, sweating, and hunger. She states that symptoms appeared approximately 3 months ago; they frequently tend to appear early in the morning before breakfast or between meals. On one occasion, during the crisis period, a decreased blood glucose level (55 mg/dL) was detected despite the fact that she consumes a diet rich in glucose. She admits to excessive abuse of alcohol, and she occasionally used aspirin for headaches up to 6 days before admission. Her medical records suggest she was previously treated as a psychopath who was prone to malingering. The patient herself confirms that she was haloperidol-treated for more than 1 year, but she discontinued haloperidol use 2 weeks ago. Her general physical findings are unremarkable. Routine laboratory tests taken on admission reveal no abnormalities. During echosonographic examination of abdomen, no abnormalities were noted. Fasting test was attempted, but it had to be discontinued due to a fall in blood glucose level from baseline value of 75 to 33 mg/dL 8 hours later. It required intravenous glucose administration and termination of the test. Laboratory analyses taken at the time of test termination also revealed elevated serum insulin (9 uU/ml), elevated serum proinsulin (6.3 pmol/l), and elevated C peptide (0.3 mmol/ml) levels. Insulin receptor antibodies are not present in the patient's serum, and sulfonylurea is absent in the patient's sera and urine. What disorder is the most likely cause of patient's hypoglycemia? Intake of aspirin 2. Chronic alcoholism 3. Insulin-producing tumor 4. Deliberate insulin intake 5. Haloperidol therapy
Correct answer: Insulin-producing tumor An insulin producing tumor (insulinoma) is characterized by an increased endogenous production of insulin by insulin-secreting tumor. Since a proportion of insulin is released into the blood in the form of proinsulin, proinsulin production is also increased in patients with insulinoma. Also, proinsulin production is increased in patients with liver cirrhosis or diabetes mellitus type II. Therefore, an increased proinsulin level in a hypoglycemic patient who has no liver cirrhosis indicates hypoglycemia is caused by increased production of endogenous insulin. Deliberate insulin intake causes hypoglycemia. However, if hypoglycemia is caused by intake of exogenous insulin, serum proinsulin level tends to be normal or low. Also, serum level of C peptide that is produced by insulin-secreting cells together with insulin is increased in patients with insulinoma. C peptide level is, however, decreased if hypoglycemia is caused by exogenous insulin administration. Since the patient has both elevated proinsulin and C peptide levels, it is unlikely that hypoglycemia is caused by deliberate patient's administration of insulin. Chronic alcohol intake may cause hypoglycemia by decreasing glucose release from the liver. However, it is not likely that patient's hypoglycemia can be attributed to alcohol intake because her diet was rich in glucose. Also, elevated serum levels of insulin, proinsulin, and C peptide, as well as high serum insulin/glucose ratio (9/33=0.33), point to the existence of insulinoma. Intake of aspirin may cause hypoglycemia. However, after discontinuation of aspirin intake, hypoglycemia is readily corrected. Hypoglycemia that persists 6 days after discontinuation of aspirin therapy cannot be attributed to aspirin intake. In addition to aspirin, haloperidol therapy may cause hypoglycemia that disappears after haloperidol withdrawal. The hypoglycemia in the presented patient appeared after almost 1 year of haloperidol treatment and persisted 2 weeks after haloperidol withdrawal, so it is unlikely that it is haloperidol-related.
A 55-year-old man presents with a COPD exacerbation that is being managed with a ventilator. The patient's blood pressure drops and the ventilator alarm goes off. The only medication administered is amlodipine via nasogastric tube. This patient is afebrile, even though the hospital has been having problems with pseudomonas infection in ventilated patients. On examination, there is a middle-aged orally-intubated man with temperature 99.4° F, pulse 145/min, and BP 62/34 mm Hg; he breathes above the ventilator at a rate of 36 cycles/min even. His breath is shallow and he has diminished breath sounds in his right hemithorax. What is the most appropriate next step? 1. Add positive end-expiratory pressure 2. Do thoracotomy/chest tube placement 3. Start antibiotics 4. Give IV boluses 5. Start pressor agents
Correct answer: Do thoracotomy/chest tube placement The patient's diagnosis is right tension pneumothorax in a COPD patient on a ventilator. Features supportive of tension pneumothorax are hypotension, tachycardia, tachypnea, shallow breathing, and decreased breath sounds in the right hemithorax. Management of this condition is emergent thoracentesis through insertion of a large bore needle into the second intercostal space. Chest tube placement is indicated for definitive treatment. Positive end-expiratory pressure is the maintenance of positive pressure in the alveolar at the end of expiration. Its application will cause an increase in the mean and peak airway pressure and worsen the pneumothorax by forcing more air into the lungs. Antibiotics will be used if there is an infective process going on, but this is not evident. Since the hypotension is related to decreased venous return because of increased intrathoracic pressure leading to decreased cardiac output, IV fluids or pressors are unlikely to increase the blood pressure.
An 80-year-old man has a past medical history of chronic obstructive pulmonary disease, persistent asthma, and hypertension; he presents due to acute chest pain, dyspnea, and pleurisy that began suddenly 30 minutes ago. He denies fever, chills, hemoptysis, wheezing, diaphoresis, cough, or abdominal pain. His vital signs are notable for tachycardia and tachypnea. His thorax demonstrates unilateral decreased tactile fremitus, hyperresonance to percussion, and decreased to absent breath sounds. There is no jugular venous distension, cyanosis or accessory muscle usage. He is identified as a poor surgical candidate, and has had previous episodes of similar manifestations in the past. The following chest X-ray image was obtained. What is the most beneficial medication for this patient? Shows pneumothorax. 1. Atorvastatin 2. Doxycycline 3. Furosemide 5. Prednisone
Correct answer: Doxycycline Explanation This patient's most likely diagnosis is a secondary pneumothorax. In patients with repeated pneumothoraces who are not good candidates for surgery, sclerotherapy with talc or doxycycline may be necessary. Nifedipine, furosemide, atorvastatin, and prednisone are pharmacological agents with no established role in the treatment of a pneumothorax.
A 70-year-old Caucasian man presents with a 2-month history of rapidly progressive jaundice, severe gnawing epigastric pain radiating to the back, loss of 15% of body weight, nausea, vomiting, loss of appetite, and asthenia. On physical examination, he is alert and oriented, emaciated, and is markedly jaundiced. Cardiovascular and respiratory systems are normal. The epigastrium is painful to palpation, but there is no well-defined palpable mass. The liver is palpable 2 cm below the right costal border, with a hardened consistency and irregular contours. The remainder of the examination shows no other abnormalities. An abdominopelvic CT scan shows an irregular mass originating in the pancreatic head, enlarged gallbladder, and small nodules throughout an enlarged liver. What is the most appropriate next step in management? 1. Duodenopancreatectomy (Whipple procedure) 2. Endoscopically insert a biliary stent 3. Cholecystectomy and T-tube placement 4. Gemcitabine-based chemotherapy 5. Radiotherapy
Correct answer: Endoscopically insert a biliary stent Pancreatic cancer is more common in men (1.2 to 1.5 male-to-female ratio). The disease is rare before the age of 50 years, and the median age of diagnosis is 69 in whites and 65 in blacks. Risk factors include smoking (heavy smokers have a two to threefold higher incidence), chronic pancreatitis, long-standing diabetes, family history, and obesity, The most common signs and symptoms are jaundice (in obstructive lesions), weight loss, and abdominal pain. More rarely, diabetes mellitus, splenomegaly, a palpable gallbladder (Courvoisier's sign), and migratory thrombophlebitis (Trousseau's sign) are seen. The delayed diagnosis explains the disease's 98% lethality rate. Several imaging modalities (e.g., transabdominal ultrasound, computed tomography, magnetic resonance imaging, endoscopic ultrasound, and endoscopic retrograde cholangiopancreatography [ERCP]) can visualize these tumors, but the vast majority of them are diagnosed late in their course because symptoms only occur in advanced disease stages. This patient has unresectable, stage IV pancreatic cancer (see pancreatic cancer staging) and biliary obstruction. Treatment is palliative, and the initial goal is to re-establish patency of the biliary tract. Several approaches are possible: endoscopic stenting using endoscopic retrograde cholangiopancreatography, percutaneous drain placement, and operative redirection of the bile ducts. Endoscopy is usually the preferred method. Several techniques are available, including plastic and expandable metal stents, lasers, and photodynamic therapy. Duodenopancreatectomy consists of removal of the pancreatic head, duodenal arch, gallbladder, and gastric antrum. Reconstruction is performed by anastomosing the remaining jejunal loop end-to-end to the pancreatic remnant, a biliary-enteric anastomosis, and a Billroth-II gastric reconstruction. It's the standard procedure for pancreatic cancer resection with curative intent. However, this patient has unresectable disease. Gemcitabine affords symptomatic improvement but improves survival only slightly. While it might be useful in this patient, establishing biliary drainage takes precedence over chemotherapy. Radiotherapy is sometimes useful for palliation of pain. However, pancreatic cancer is relatively radioresistant, and survival is not improved.
You have just finished your evaluation of a 67-year-old man who presented with dyspnea and chest pain 3 days after flying home from Europe. Work-up reveals a right lower pulmonary embolism. Your patient has required 6L of O2 to maintain a saturation above 90% and has continued to remain tachycardic. The remainder of his past medical history is unremarkable. What therapy should you initiate at this time? 1. Aspirin 2. Warfarin (Coumadin) 3. Clopidogrel (Plavix) 4. Enoxaparin Sodium (Lovenox) 5. Enoxaparin Sodium (Lovenox) plus Warfarin (Coumadin)
Correct answer: Enoxaparin Sodium (Lovenox) plus Warfarin (Coumadin) The American College of Chest Physicians (ACCP) recommends that anticoagulation therapy be initiated in all patients with diagnosed acute DVT or PE unless the patient has a known contraindication.1 Anticoagulation should be initiated as soon as possible, which includes either a low molecular weight heparin (LMWH), unfractionated heparin (UH), or fondaparinux (Arixtra), in addition to coumadin, with discontinuation of the LMWH, UH, or fondaparinux when the patient's INR is greater than 2.0.1
A 53-year-old man presents with increased difficulty swallowing and occasional regurgitation of his meals. His symptoms have been occurring with greater frequency and severity over the last 4 months. He also gets some shortness of breath, but attributes that to his weight and lack of physical activity. His past medical history is remarkable for chronic heartburn, which he treats intermittently with over the counter antacids. He takes no regular medications, and he has no allergies. He has not had any surgeries. He is smoker, but he denies use of alcohol and drugs. He works as a building inspector, and he lives with his wife and children. The patient is obese, but the rest of his physical exam is normal. Blood tests, electrocardiogram, and chest X-ray are done in the clinic; they are normal. He is referred for endoscopy, and esophageal biopsy shows specialized intestinal metaplastic cells (of columnar epithelium). What is the medication of choice for this patient's condition? 1. Alendronate 2. Dicyclomine 3. Esomeprazole 4. Famotidine 5. Simethicone
Correct answer: Esomeprazole This patient is presenting with Barrett's esophagus, which is a type of chronic esophagitis in which the normal squamous epithelium is replaced with columnar epithelium. Barrett's esophagus is a complication of chronic gastroesophageal reflux disease (GERD) and can develop into esophageal adenocarcinoma. Use of proton pump inhibitors (PPIs), such as esomeprazole, reduces the risk of cancer. In order to monitor for the development of cancer, a routine endoscopy should be periodically performed in patients with known Barrett's esophagus. Alendronate is a bisphosphonate; it is used for both treatment and prevention of osteoporosis. Alendronate can cause multiple gastrointestinal adverse effects, including esophagitis and esophageal ulcerations and strictures. This medication could dramatically worsen this patient's current condition. Dicyclomine is an anticholinergic medication that relaxes smooth muscle. It is used for spasms associated with irritable bowel syndrome. It does not have a role in treating GERD or Barrett's esophagus. Famotidine antagonizes H2 histamine receptors (H2 blocker). It is used in treating duodenal and gastric ulcers as well as GERD. H2 blockers are usually considered less potent than PPIs. PPIs are the treatment of choice for Barrett's esophagus. Simethicone is sold in various over-the-counter products for flatulence. It has no role in treating GERD or Barrett's esophagus.
A 58-year-old man with a history of COPD, hyperlipidemia, 40-pack/year smoking history, and obesity is being evaluated at his primary care office for complaints of post-prandial regurgitation that is associated with an acidic sensation in his mouth and a chronic, nonproductive cough, all of which have been occurring over the past year. He has taken over the counter Famotidine (Pepcid) without any relief. He denies any fever, chills, changes in weight, diaphoresis, chest pain, shortness of breath, sputum, palpitations, abdominal pain, or changes in bowel habits. His physical exam reveals an obese BMI, but is otherwise unremarkable. His stool hemoccult is negative. Bloodwork reveals an iron-deficiency anemia; a chest radiogram was without pulmonary disease. An upper barium esophagram noted an outpouching of barium at the lower end of the esophagus and a wide hiatus through which gastric folds are visible above the diaphragm. H. pylori antibody testing and urea breath tests were negative. What is the most appropriate therapy for this patient at this time? 1. Esomeprazole (Nexium) 2. Ranitidine (Zantac) 3. Metoclopramide 4. Nitroglycerin 5. Metronidazole (Flagyl), tetracycline, and bismuth subcitrate
Correct answer: Esomeprazole (Nexium) This presentation is most consistent with a sliding hiatal hernia. Barium esophagograms or swallows are helpful for identifying structural abnormalities of the esophagus and esophageal hiatus, which include esophageal rings, strictures and ulcers, and hiatal hernias. Large hiatal hernias may cause iron deficiency anemia regardless of whether Cameron ulcers are present. This anemia responds well to PPI therapy, with surgery offering no clear advantage over medical therapy. PPIs are also superior to H2 receptor antagonists for the resolution of associated GERD symptoms at 4 weeks and the healing of esophagitis at 8 weeks. This patient remains symptomatic despite initial H2-receptor antagonist therapy; ranitidine would not be appropriate at this time. Nitroglycerin is not indicated, as this patient is not suffering from angina pectoris. Prokinetic agents are somewhat effective, but only in patients with mild symptoms; other patients usually require additional acid-suppressing medications, such as PPIs. The usual regimen in adults is metoclopramide. Long-term use of prokinetic agents may have serious, even potentially fatal, complications and should be discouraged. Antibiotics with a proton pump inhibitor for eradication of H. pylori would be suitable with evidence of H. pylori infection; however, there is none in this patient.
A 42-year-old homeless man is admitted to the hospital with fatigue and chest pain. He states that he has been having fever, night sweats, nausea, vomiting, diarrhea, weight loss, and a burning sensation in the mouth. He also notes a sensation of skin burning, increased skin pigmentation, and emotional instability over the past 3 - 4 months. Physical examination reveals a bronze skin tone, several missing teeth, and a low-grade fever. Laboratory analysis revealed a normal CBC, WBC, and urinalysis. A PPD was performed and was found to be negative. Serum transferring is normal. What statement concerning this patient's condition is true? 1. It is caused by a protein deficiency 2. Foods rich in tryptophan may help to compensate for this deficiency 3. It is a common condition in the United States 4. It affects men more than women 5. It is difficult to treat
Correct answer: Foods rich in tryptophan may help to compensate for this deficiency Niacin (nicotinamide, nicotinic acid) deficiency is uncommon in the United States. It is often found in people who live on a diet that consists mainly of corn (maize). This is due to the fact that the niacin in corn cannot be absorbed unless it is chemically treated with alkali first. If a person consumes a diet rich in tryptophan, but low in niacin, they are able to compensate since tryptophan can be converted into niacin. Deficiency may also result from alcoholism, cirrhosis, or diarrhea. Men and women are affected equally. Symptoms of niacin deficiency include nausea, vomiting, diarrhea, rash, glossitis, stomatitis, depression, and psychosis. The rash may be found on any part of the body, but it is commonly found on the arms and legs. Niacin deficiency, also known as pellagra, manifests as the '3 D's': diarrhea, dermatitis, and dementia. If this diagnosis is suspected, supplemental niacin should be given immediately. Diagnosis can be made by getting a good history and from the physical symptoms. Laboratory tests can confirm the diagnosis. Treatment consists of niacin supplementation. This should be started as soon as the deficiency is suspected. Some patients deficient in niacin may also be deficient in other vitamins, such as B vitamins, so they should be given as well.
A 65-year-old woman presents with a 3-week history of stomach pain. Her family history is remarkable for her father's death due to prostate cancer and the deaths of her mother and sister due to breast cancer. Her 2 brothers experienced myocardial infarctions; both of them have had coronary bypass operations. The patient suffers from hypertension, hyperlipidemia, and coronary heart disease. She underwent angioplasty and stent placement 1 year prior to presentation. She denies having any GI problems in the past. Her daily medications include aspirin (81 mg daily), simvastatin (3 x 5 mg daily), metoprolol (3 x 50 mg daily), vitamin E (400 I.U. daily), and a multivitamin supplement. In addition, for the past 12 weeks, she has been taking diclofenac (3 x 50 mg daily) for osteoarthritis of the right knee. She admits to sometimes taking up to 5 x 50 mg of diclofenac for management of intense pain. She states that she always used to be constipated; lately, her stool is occassionally black. Her temperature is 36.8° C, blood pressure is 135/90 mm Hg, pulse rate is 105/min, and respiratory rate is 18/min. Physical examination reveals an obese, pale woman in no apparent distress. ECG shows slight ST depression and atrial fibrillation. There is pain upon pressure in the epigastric area. The rest of the examination is unremarkable. Lab analysis is as follows: hemoglobin 10.8 g/dL (normal 14 to 18 g/dL); hematocrit 35.2% (normal 42% - 52%); mean corpuscular volume 92 μm3 (normal 80 - 94 μm3), white blood cell count 26,900/mm; and platelet count 233,000/mm3. What is the most appropriate next step? 1. Gastroscopy 2 Colonoscopy 3 Helicobacter pylori treatment 4. Barium X-ray 5. Rectosigmoidoscopy
Correct answer: Gastroscopy Considering the history, physical examination and lab results, the patient most likely suffers from a NSAID-induced GI bleed; therefore, gastroscopy is the next logical step. Minor bleeding can be treated during the procedure by an injection of epinephrine (1:10,000) or by using a bipolar probe. Since the patient has melena, the bleeding is most likely occurring in the upper GI tract. If no bleeding source in the upper GI tract is found, a colonoscopy or barium X-ray should be done. The examination choice depends on availability, the expertise of the medical staff, and the patient's wishes. Since there has not been a test done for Helicobacter pylori infection, treatment is not justified. Helicobacter pylorus specifically and selectively colonizes human gastric mucin-secreting cells; it is the most common cause of non-erosive gastritis. It can be diagnosed by a urease breath test or serum IgG test. Eradication is achieved by treatment with clarithromycin (500 mg/day) and omeprazole (2 x20 mg/day for the 1st 2 weeks, and 1 x 20 mg/day for the next 2 weeks). The next step would be rectosigmoidoscopy; however, it is very unlikely that melena originates in the end part of the bowel.
An 18-year-old woman presents with symptoms of mild jaundice, fatigue, nausea, and abdominal pain. Her past medical history includes chronic jaundice. Physical exam shows pain elicited with palpation in the upper right quadrant. What is the most likely diagnosis? 1 Crigler-Najjar syndrome 2. Breast milk jaundice 3. Newborn jaundice 4. Gilbert's syndrome 5. Dubin-Johnson syndrome
Correct answer: Gilbert's syndrome Gilbert's syndrome is a multifactorial inherited disorder that affects the way bilirubin is processed by the liver and causes jaundice. The symptom of jaundice appears under conditions of exertion, stress, fasting, and infections. The condition is usually benign. Symptoms include mild jaundice, fatigue, nausea, and abdominal pain in the upper right quadrant. A serial serum indirect bilirubin shows changes consistent with Gilbert's disease. Usually no treatment is necessary, but phenobarbital can reduce the bilirubin level and relieve the jaundice. Jaundice may fluctuate and persist throughout the patient's life. It usually causes no health problems. Crigler-Najjar syndrome is inherited as an autosomal recessive trait. Infants who inherit the trait from both parents (this is called homozygous inheritance) develop severe jaundice (hyperbilirubinemia) a few days after birth. If these infants are not treated, they may develop kernicterus (a condition in the newborn marked by severe neural symptoms, associated with high levels of bilirubin in the blood). The jaundice will persist into adult life and require daily treatment. The constantly elevated levels of bilirubin eventually will produce an adult form of kernicterus despite daily treatment. Symptoms include yellow skin and eyes (jaundice) that begins on the 2nd or 3rd day of life and progressively worsen, jaundice that persists beyond 2 weeks without an obvious cause, and confusion (resulting from brain damage). Liver transplantation may be considered for some people with this disorder. Breast milk jaundice is a persistent jaundice in the newborn caused by a hormone found in breast milk. Bilirubin is eliminated by chemically hooking a sugar molecule to the bilirubin molecule, which then makes the bilirubin water-soluble. In its water-soluble form, bilirubin can be excreted in the urine. Breast milk contains a hormone, pregnanediol, which interferes with the body's ability to hook the sugar onto the bilirubin. Symptoms include jaundice in a breast-fed newborn that lasts longer than a week. Cessation of nursing for 24 to 48 hours will result in a rapid drop of bilirubin. Newborn jaundice is a condition of yellowish skin color during the newborn period. This results from the immaturity of liver function combined with the destruction of redblood cells present in the newborn infant. The jaundice usually appears between the 2nd and 5th days of life and clears by 2 weeks. Symptoms include yellow color of the skin (jaundice), poor feeding (may be present), and lethargy (may be present). An elevated serum bilirubin level will be present. Usually treatment is unnecessary. Sometimes artificial lights (called bili lights) are used on infants whose levels are very high, or on premature infants. The jaundice resolves without treatment within 1 to 2 weeks. Dubin-Johnson syndrome is an inherited disorder characterized by mild jaundice throughout life. The transport of bilirubin from the liver into the biliary system is abnormal, and the bilirubin accumulates in the liver. Affected people have lifelong low-grade jaundice that may be aggravated by alcohol, pregnancy, infection, and other environmental factors. The symptoms include a mild jaundice, which may not appear until puberty or adulthood.
A 12-year-old boy enjoys himself at an Independence Day picnic. The menu includes the standard fare: hamburgers, hot-dogs, deviled eggs, macaroni salad, and potato salad. Approximately 3 hours after eating, he suddenly develops nausea, vomiting, and non-bloody diarrhea. He does not have a fever. What is the classification of the underlying causative agent? 1. Spirochete 2. Gram-negative rod 3. Gram-positive rod 4. Gram-negative cocci 5. Gram-positive cocci
Correct answer: Gram-positive cocci Some food poisoning is due to the ingestion of a preformed toxin. The short time lag from the time the boy ate to the onset of his symptoms suggests that a preformed toxin in the food is the culprit. Staphylococcus aureus (Gram-positive cocci) food poisoning is due to a preformed enterotoxin. Staphylococcus aureus food poisoning can be seen with desserts that are filled or topped with cream, salads made with mayonnaise, dairy products, meat salads, and some other foods. Contamination occurs during the preparation of the food. When the food is kept at room temperature, the bacteria, Gram-positive cocci, rapidly reproduces and produces a heat stable toxin. Symptoms usually occur within several hours of eating the contaminated food. Food poisoning by Staphylococcus aureus produces nausea, vomiting, and diarrhea.
A 60-year-old hypertensive man presents with constipation; he states that he has not had a single bowel movement in the past 2 days. 1 month ago, he had a myocardial infarction, but he is now stable and is on a low-fat, low-salt diet. He He refuses a docusate sodium enema and is prescribed docusate tablets. What precaution should the patient take while using this laxative? 1. He should not use it long-term 2. It should be consumed with fruit juice only 3. He will have to increase his salt intake for the docusate to be effective 4. He should also consume mineral oil for the docusate to be effective 5. He may get dehydrated while using docusate
Correct answer: He should not use it long-term Docusate should not be used long-term; generally it should not be used for more than 1 week. Docusate is a stool softener which acts in the small and large intestines by increasing the water and fat content in the stool; this facilitates its easier movement. It is ideally suited for short-term treatment of constipation because long-term use may increase the risk of a deficiency of fat-soluble vitamins due to decreased absorption. Other contraindications include appendicitis, acute surgical abdomen, intestinal obstruction, fecal impaction, and undiagnosed abdominal pain. Mineral oil should not be prescribed along with docusate as the former may be absorbed, leading to toxicity. Docusate tablets are taken with a glass of water. A high-salt diet is not required for docusate to be effective. Dehydration is a side effect of sodium picosulfate, which is a contact laxative used to treat constipation or to prepare the bowel prior to surgery.
A 45-year-old man presents with extreme asthenia and weight loss. He has been suffering from celiac disease for the past 12 years. He is at increased risk of developing what type of intestinal malignancy? 1 Intestinal leiomyosarcoma 2 Intestinal adenocarcinoma 3 Intestinal lymphoma 4 Carcinoid tumor of intestinal tract 5 Intestinal hemangiosarcoma
Correct answer: Intestinal lymphoma Patients with long standing celiac disease are at increased risk of developing intestinal lymphoma. The early stage of celiac disease is characterized by increased accumulation of lymphocytes within lamina propria. Immunologic reaction against gluten containing diet is the probable cause of intestinal mucosal damage in patients with celiac disease (immune mechanism). Direct toxic effect of gluten and related peptide (gliadin) is also a possible mechanism of intestinal injury in such patients.
A 50-year-old man presents with a 2-week history of not being able to see well. He is a married newspaper editor and is not on any medications. He has been smoking 2 packs of cigarettes a day for the past 30 years. On examination of his right eye, there is ptosis and miosis. A chest radiograph reveals a rounded opacity in the right lung field What is the most likely diagnosis? 1. Lambert-Eaton myasthenic syndrome 2. Hypertrophic pulmonary osteoarthropathy 3. Horner's syndrome 4. Ectopic adrenocorticotropic hormone (ACTH) secretion 5. Syndrome of inappropriate antidiuretic hormone secretion (SIADH)
Correct answer: Horner's syndrome Horner's syndrome is due to a bronchogenic carcinoma tumor mass extending to the sympathetic chain. Horner's syndrome comprises of miosis, ptosis, enophthalmos, and hemianhidrosis. In addition, the patients may also have atrophy of hand muscles. The syndrome of inappropriate antidiuretic hormone secretion (SIADH) is a paraneoplastic syndrome associated with bronchogenic carcinoma. There is excessive water reabsorption by the kidneys due to the elevated antidiuretic hormone (ADH). Patients may complain of feeling weak and lethargic. Other symptoms include confusion and coma. Laboratory tests reveal hyponatremia, high urine sodium, low serum osmolality, and high urine osmolality in a euvolemic patient. Cushing's syndrome may develop as a result of the small cell carcinomas secreting ectopic adrenocorticotropic hormone (ACTH). Patients may complain of bruising easily. On examination, they may have red-purplish striae, especially over their abdomen, in addition to the typical moon facies and truncal obesity. They may also have muscle wasting, especially of the proximal limb girdle muscles. Laboratory investigations reveal hypokalemia, high plasma ACTH, as well as increased serum and urine cortisol. In cases of hypertrophic pulmonary osteoarthropathy (HPO), there is formation of new subperiosteal cancellous bone at the distal ends of long bones. Patients present with periarticular pain, polyarthralgia, and even painful swellings at the wrists, knees, elbows, and ankles. On examination, there is clubbing of the fingers and toes. There may also be localized articular erythema, tenderness, swelling, and even effusions. X-rays may reveal subperiosteal bone formation, with periosteal elevation seen as thickening and detachment of the periosteum. Lambert-Eaton myasthenic syndrome is an immune-mediated disorder of neuromuscular transmission, resulting in impaired release of acetylcholine from nerve terminals. Patients complain of weakness of the scapular and pelvic girdles muscles,and the resultant difficulties in performing activities such as climbing stairs. Other symptoms such as dry mouth, paresthesias, and sexual impotence due to autonomic dysfunction may also be present. On examination, there is proximal muscle weakness. Deep tendon reflexes are decreased or absent.
A 54-year-old man presents with a 1-year history of pneumonia, visceral Kaposi's sarcoma unresponsive to therapy, and chronic persistent diarrhea. He says that he uses heroin on and off because he is tired of visiting doctors all the time. He has elected to forego antiretroviral medication, chemotherapy, and prophylactic drug therapy. His caregiver mentions that his functional status has declined; he is confined to his bed for most of the day. His mental status is still intact, but activities of daily living (ADL) are notably affected. Physical exam reveals the patient has congestive heart failure. He is very thin, with recent wasting. His lab results show a viral load of 150,000 copies/ml, BUN 19 mg/dl, serum creatinine 1.3 mg/dl, CD4+ count 50 cells/mcL, and his serum albumin is repeatedly 2 mg/dl. His viral load 2 months ago was 140,000 copies/ml. What is the most appropriate action in this case? 1. Hospice care with the diagnosis of end stage heart disease 2. Hospice care with the diagnosis of end stage renal disease 3. Hospice care with the diagnosis of end stage HIV/AIDS disease 4. Hospice care with the diagnosis of end stage pulmonary disease 5. Hospice is not appropriate
Correct answer: Hospice care with the diagnosis of end stage HIV/AIDS disease This patient has end-stage HIV/AIDS, and hospice care is appropriate for him. Patients with HIV/AIDS should meet the following criteria to be eligible for hospice services: CD4+ count < 25 cells/mcL or persistent viral load >100,000 copies/mlfrom 2 or more assays at least 1 month apart, and at least 1 of the following conditions: Central nervous system (CNS) lymphoma, Visceral Kaposi's sarcoma unresponsive to therapy, Progressive multifocal leukoencephalopathy (PML), Cryptosporidiosis, Mycobacterium avium complex (MAC) bacteremia, untreated, refractory, or treatment refused Untreated or refractory wasting (loss of >33% lean body mass), Renal failure in the absence of dialyses, and Refractory toxoplasmosis There should also be a palliative performance scale of < 50% (the patient requires considerable assistance and frequent medical care as activity is mostly limited to bed or chair). Other supporting documentations include: Chronic persistent diarrhea for 1 year, Persistent serum albumin < 2.5 gm/dL, Concomitant substance abuse, Age greater than 50, Congestive heart failure which is symptomatic at rest, Patient has elected to forego antiretroviral and prophylactic medication related specifically to HIV. There is no indication that the patient has end-stage heart disease, end-stage pulmonary disease, or end-stage renal disease; however, pneumonia and congestive heart failure are helpful factors for making decisions regarding patients with end-stage HIV disease.
A 5-month-old female infant presents with a 3-day history of vomiting. She is exclusively breastfed, and her mother states that today she has vomited within 15 minutes of each feeding. Her last wet diaper was 10 hours ago. On physical examination, she is afebrile, tachycardic, irritable, and does not express tears when crying. She was a full-term infant born via vaginal delivery. She has no significant past medical history. Her 3-year-old sister has had gastroenteritis for the past few days. What is the most likely laboratory finding? 1. Decreased blood urea nitrogen 2. Hyponatremia 3. Hypokalemia 4. Decreased urine specific gravity 5. Hypernatremia
Correct answer: Hypokalemia This infant is showing signs of moderate dehydration due to emesis and limited breast milk intake. Gastric and urinary potassium losses can both contribute to hypokalemia. Dehydration leads to hemoconcentration with an increased hemoglobin and increased urine specific gravity. BUN and creatinine could be increased with volume depletion, and they could be increased further with renal insufficiency. Not eating for an extended period of time could lead to hypoglycemia in a 5-month-old infant. Hypernatremia implies a deficit of total body water. Major symptoms are thirst, confsion, neuromuscular excitability, seizures, and coma.
A 50-year-old man presents with episodes of flushing. He is quite disturbed because it starts in his face and goes from red to deep violet. He also experiences diarrhea, cramps, and nausea. It is determined that he has a carcinoid tumor that has metastasized. Where is the most likely site of the primary tumor? 1. Prostate 2. Sympathetic chain ganglia 3. Adrenal medulla 4. Adrenal cortex 5. Ileum
Correct answer: Ileum This patient has signs of carcinoid syndrome. The ileum is a common site for a carcinoid tumor; however, they can be found in other areas of the body as well. Other common sites for carcinoid tumors are other regions of the gastrointestinal tract and the bronchi. Of the choices given, the ileum is the most likely primary site of his tumor, not the prostate, sympathetic chain ganglia, adrenal medulla, or adrenal cortex. Carcinoids produce and secrete 5-hydroxytryptamine, which is serotonin. Carcinoids can produce other things as well; however, 5-hydroxytryptamine (serotonin) is the major product seen with carcinoid tumors. Urinary 5-hydroxyindoleacetic acid (5-HIAA) will be elevated with carcinoid syndrome and can be used in diagnosis.
A 36-year-old woman presents with confusion, a lack of control over her body, and unusual eye movements. She appears to be malnourished, and she shows signs of poor hygiene. She has been vomiting recently. Her pulse is 80/min, blood pressure is 122/70 mm Hg, temperature is 98.4° Fahrenheit (36.9 °C), and respiration is normal. What is the next best step in the management of this patient? 1 Intravenous glucose 2 Intravenous thiamine 3 Intravenous vitamin B12 4 Diazepam 5 Toxicological screening
Correct answer: Intravenous thiamine Based on the symptoms, this patient has Wernicke encephalopathy, part of a syndrome caused by vitamin B1 deficiency; it is a common consequence of poor nutrition and alcoholism. To prevent chronic brain damage and the onset of Korsakoff psychosis, a condition that often follows a Wernicke encephalopathy episode, the patient needs to be administered vitamin B1 (thiamine) immediately. Thiamine supplementation can provide complete resolution of Wernicke symptoms, especially if alcohol misuse is not the underlying cause. Intravenous glucose is not advised for patients with Wernicke-Korsakoff syndrome; it can cause the exhaustion of their thiamine and accelerate the disorder. Lack of vitamin B12 causes a different nutritional disease which can also lead to damage within the brain. It results in the subacute combined degeneration of the spinal cord (SCDSC); however, symptoms of this disease do not include confusion and ophtalmoplegia. The patient's symptoms are associated with alcoholism, and participation in an alcohol recovery program should be included in the treatment. Diazepam is commonly used for relief of the withdrawal symptoms, but this is not the 1st step in the treatment of Wernicke encephalopathy. Toxicological screening is often performed in emergency situations, but given the symptoms in this case, the test will not provide more information or change the treatment steps.
What is the clinical presentation of erosive gastritis? 1. It is occult and usually not a problem 2. It occurs within 24 hours of injury 3. It typically occurs 7 days after the initial injury 4. Patients present with severe abdominal pain and peritonitis 5 The patients will have elevated WBC, fever, and pneumoperitoneum
Correct answer: It typically occurs 7 days after the initial injury Erosive gastritis typically will present 7 to 10 days after the initial injury. The initial presentation is bleeding, either as hematemesis or as blood in the nasogastric tube effluent. Diagnosis is usually confirmed by endoscopic evaluation of the gastric lumen. Since erosive gastritis is a condition affecting the mucosa and not the deeper parts of the gastric wall, patients will not have complaints or findings of perforation or peritoneal irritation such as is found in peptic ulcer disease.
A 44-year-old man with a history of ulcerative colitis presents with bloody diarrhea. The patient has had a long history of poorly controlled ulcerative colitis despite being on optimal medical management. The patient subsequently is surgically managed with a proctocolectomy with permanent ileostomy. However, complications in his procedure call for a long hospital stay. After a few months, the patient is found to have ascites, pitting edema, and a fatty liver. What is the most likely diagnosis? 1 Hypovitaminosis B1 2 Kwashiorkor 3 Total parenteral nutrition-induced steatosis 4. Marasmus 5 Primary sclerosing cholangitis
Correct answer: Kwashiorkor This patient is having protein deficiency, and therefore the most likely answer is kwashiorkor. Kwashiorkor is secondary to total protein deficiency. This finding is typically found in patients who are severely malnourished. The typical findings in these patients are ascites and pitting edema due to hypoalbuminemia. In this patient, his long hospital stay with a complicated proctocolectomy, has led to protein deficiency. Marasmus is a total caloric and protein deficiency. These patients typically present with broomstick extremities (muscle wasting). These patients do not present with ascites and pitting edema. Total parenteral nutrition (TPN) related microvesicular steatosis is a common finding in patients who are on TPN. However, there is no mention in the question stem that this patient was on TPN. Primary sclerosing cholangitis (PSC) is commonly associated with patients who have ulcerative colitis. PSC is a chronic liver disease, which presents as inflammation, destruction, and fibrosis of both the intrahepatic and extrahepatic bile ducts, with the resultant destruction leading to cirrhosis of liver. Hypovitaminosis B1 (thiamine) is not the correct answer choice. This deficiency is known as wet beri beri. Though it may cause edema, there will be dilated cardiomyopathy, which is not found in this case.
A 35-year-old Asian man presents with diarrhea; the diarrhea has worsened over the past year since getting married and changing his diet. It is associated with abdominal discomfort and the feeling of a lot of gas in his stomach. On examination, his abdomen is distended; there is a tympanic note on percussion. There is no area of tenderness. The hydrogen breath test reveals elevated hydrogen in his expired air. What is the most likely diagnosis? 1 Celiac disease 2 Whipple's disease 3 Short bowel syndrome 4 Tropical sprue 5 Lactose intolerance
Correct answer: Lactose intolerance Lactose intolerance results from a deficiency of lactose, a disaccharidase in the mucosal cells of the small intestine which splits the disaccharide lactose into glucose and galactose. Patients note borborygmi, flatulence, nausea, abdominal cramps, pain, and diarrhea after ingesting lactose-containing food (e.g., milk). Laboratory investigations reveal that the diarrheal stools are acidic. When the hydrogen breath test is administered, there is elevated hydrogen content in the expired air because the colonic flora digests the unabsorbed lactose. Treatment includes following a lactose-free diet. Celiac disease is a hereditary disorder caused by gluten intolerance; gluten is found in wheat, rye, barley, and oats. Patients may be asymptomatic or may present with diarrhea, abdominal discomfort, distention, and steatorrhea; their stools are pale, malodorous, and difficult to flush because they float on the toilet water. Patients may have anemia from iron and folate deficiencies, osteomalacia and bone pains from calcium deficiency, and edema from hypoproteinemia. Diagnosis is confirmed by a small intestine mucosal biopsy which shows a flat mucosa due to villous atrophy, and by subsequent improvement on a gluten-free diet. Specific treatment includes a gluten-free diet. Whipple's disease mainly affects men; it is caused by the bacterium Tropheryma whippelii. It is a multisystemic disease which affects the small intestines, joints, brain, heart, and eyes. Patients can present with diarrhea, steatorrhea, abdominal pain, weight loss, and joint pains. On examination, they may be pale and have lymphadenopathy. Histological examination of a small bowel mucosal biopsy reveals PAS-positive, foamy macrophages. Treatment is with trimethoprim-sulfamethoxazole or chloramphenicol. Short bowel syndrome is usually the result of surgical resection of the intestines or a jejunoileal bypass. The malabsorption is a result of inadequate absorptive surface. Malabsorption of vitamin B12 results in paresthesias; malabsorption of calcium results in bone pain and carpopedal spasms. Tropical sprue is an acquired disease that affects both visitors and natives of tropical areas (e.g., the Caribbean and South India). Its etiology is unknown. Patients usually present with diarrhea and weight loss; they report that their stools are soft and bulky (steatorrhea). They may also develop deficiencies of folate and cobalamin. Stool microscopy should be done to look for cysts and trophozoites. Histological examination of a small bowel mucosal biopsy aids in making the diagnosis. Treatment is with tetracycline or oxytetracycline.
A 22-year-old woman presents with a history of chronic diarrhea. She gives a history of large volumes of watery fecal output that is non-bloody and can be easily flushed; the diarrhea is painless and persists with fasting. The patient's mother complains that she eats less and has lost weight. She has no history of fever, flushing, wheezing, intolerance to heat/cold, or infection. Her family history and menstrual history are insignificant. On examination, her weight is 130lb; height is 5 feet 5 inches; pulse is 80/min; BP is 100/74mm Hg; and temperature is 98.8°F. The tongue is dry, but the rest of her physical examination is within normal limits. What is the most likely diagnosis? 1 Carcinoid tumor 2 Laxative abuse 3 Celiac disease 4 Hyperthyroidism 5 Inflammatory diarrhea
Correct answer: Laxative abuse The most likely diagnosis is laxative abuse. Large volumes of watery fecal output that is non-bloody, painless, and persists with fasting suggests secretory diarrhea that can result from use of stimulant laxatives like senna, bisacodyl, or castor oil1. The patient is young, dehydrated, and based on her symptoms, is likely taking laxatives to lose weight and get rid of unwanted calories. There are no signs and symptoms of other causes of secretory diarrhea, such as carcinoid syndrome, and the rest of the options do not have conditions that cause secretory diarrhea. Many patients might not admit the use of laxatives to lose weight. Carcinoid tumor can cause secretory diarrhea1, but it is unlikely in this patient; she has no associated symptoms, such as episodic wheezing or flushing. Hyperthyroidism results in diarrhea due to dysmotility1. However, the patient does not have any symptoms of hyperthyroidism, such as unexplained weight loss or heat intolerance. Inflammatory diarrhea can be ruled out in this patient; there is no fever or signs of inflammation or bloody stools. Celiac disease causes diarrhea due to mucosal malabsorption. Patients present with multiple nutritional deficiencies and fatty diarrhea, which are not present in this patient, making this diagnosis unlikely1.
A 45-year-old man presents with a 3-day history of fever (T max-103.5°F), chills, anorexia, diarrhea, and a non-productive cough. On general examination, vitals are as follows: pulse is 98/min, RR is 24/min, BP is 120/60 mm Hg, and temperature is 103.5 °F. There are coarse basal crepitations and scattered rhonchi on examination of the lungs. Other systems exam are normal. Chest X-ray (CXR) shows patchy alveolar infiltrates with consolidation in the lower lobe. Complete blood count reveals leukocytosis; sputum Gram stain reveals only a few polymorphonuclear (PMN) leukocytes, and no predominant pathogens. What is the most likely diagnosis? 1. Tuberculosis 2. Klebsiella pneumonia 3. Cryptococcal pneumonia 4. Legionnaire's disease 5. Haemophilus influenzae
Correct answer: Legionnaire's disease Legionnaire's disease is caused by Gram-negative, non-acid-fast bacilli that present as an atypical pneumonia with high fever, pleuritic chest pain, non-productive cough, and dyspnea. Associated systemic symptoms include diarrhea, confusion, and renal dysfunction. A mild leukocytosis is generally present. Sputum production is scant and fails to reveal pathogens. CXR often shows patchy alveolar infiltrates with consolidation in the lower lobe; pleural effusion is seen in 50% of the patients. Serology by urine antigen testing is highly sensitive and specific for the diagnosis of Legionnaire's disease. Treatment includes administration of oxygen, antipyretics, and antibiotic therapy with erythromycin with or without rifampin, co-trimoxazole, tetracycline, doxycycline, or ciprofloxacin used alone or in combination (depending on the severity of the disease). Tuberculosis may present either as a primary infection with fever, productive cough, and pleurisy, or as post-primary infection with constitutional symptoms of anorexia, weight loss, and night sweats along with productive cough and hemoptysis. Typical features of the chest X-ray are unilateral middle or lower lobe infiltrate and cavitation. Sputum staining shows acid-fast bacilli. Klebsiella pneumonia typically produces illness in debilitated patients, especially alcoholics, presenting as hemoptysis, dense lobar consolidations, and a high incidence of abscess formation. Sputum Gram stains show large Gram-negative bacilli and abundant PMN leucocytes. Cryptococcal pneumonia is often seen in the setting of immunodeficiency. Typical CXR is that of diffuse bilateral interstitial infiltrates. The diagnosis can be confirmed with sputum test for cryptococcal antigen (CRAG) and culture. Haemophilus influenza is the one of the most common causes of bacterial pneumonia, more common in children than adults. Haemophilus pneumonia may be multilobar; there may be areas of pathy bronchopneumonia or frank consolidation. Effusions may be present and occasionally progress to empyema. Gram stain of sputum may reveal coccobacilli, though they may stain poorly.
A patient with a history of alcohol abuse presents with a histopathological report; he is asking about the 'bodies found in his liver cells'. What type of bodies is he talking about? 1 Russell bodies (protein inclusions) 2 Lewy bodies (protein aggregates) 3 Negri bodies (eosinophilic inclusions) 4 Heinz bodies (hemoglobin precipitations) 5 Mallory bodies (hyaline inclusions) 6 Guarnieri bodies (intracytoplasmic inclusions)
Correct answer: Mallory bodies (hyaline inclusions) Mallory bodies (alcoholic hyaline) are cytoplasmic inclusions found most often in alcoholic hepatitis. They are abnormal aggregations of cytoskeletal proteins. Russell bodies are found in reactive or malignant plasma cells, not in liver cells. They represent excess immunoglobulin. Lewy bodies are abnormal aggregates of protein inside nerve cells in Parkinson's disease (PD) and some other degenerative brain disorders. They are not found in liver cells. Negri bodies are eosinophyllic, viral, intracytoplasmic inclusions found in hippocampal nerve cells. They are pathognomonic for rabies infection. Heinz bodies are inclusions within red blood cells that are composed of denatured hemoglobin. Although they can be found in liver disease, as well as alpha thalassemia and glucose-6-phosphate dehydrogenase, they are not found in liver cells. Guarnieri bodies, or B-type inclusions, are the sites of viral replication; they are found in all poxvirus-infected epithelial cells, but not in the hepatocytes.
A 23-month-old boy presents with a sudden onset of bright red blood in his bowel movements. He is afebrile and does not seem to be in any pain. A technetium-99m pertechnetate scan after enhancement with cimetidine shows a 'hot spot' in the area of his ileum. What is the most likely cause? 1. Bleeding arteriovenous ileal malformation > 1 cm 2. Ileal bleeding secondary to factor VIII deficiency (hemophilia) 3. An ileal polyp 4. Intussusception 5. Meckel diverticulum
Correct answer: Meckel diverticulum This patient is most likely suffering from a common congenital abnormality of the development of the ileum called a Meckel's diverticulum. In the embryo, the vitelline duct is a communication between the yolk sac and the lumen of the gastrointestinal tract at the midgut. Normally, it degenerates completely, but the persistence of a portion of the vitelline duct leads to the development of a cul-de-sac on the ileum (Meckel's diverticulum). This congenital birth defect follows a rule of 2s. It occurs in 2% of the population, but only 2% show symptoms. It is usually located about 2 feet from the ileocecal valve and is about 2 inches long. It can contain 2 types of ectopic tissue: gastric or pancreatic. It usually presents by 2 years old. The presence of ectopic gastric mucosa in the diverticulum can lead to secretion of stomach acid downstream from the duodenum, which has bicarbonate-secreting submucosal Brunner glands to neutralize gastric acid. Ectopic gastric tissue in the ileal diverticulum can lead to ulceration and bleeding of the adjacent ileal mucosa. Vascular malformations, intussusception, and coagulation disorders are more likely to manifest themselves in the first year of life. Cow's milk colitis is a problem of the first year of life and spontaneously resolves by the end of that year for most children. NEC is a problem of the stressed, usually premature, newborn. Rectal polyps are likely to present in older children, as is HSP. Babies and older children can have gastric bleeding from gastritis or gastric ulcers. Duodenal ulcers are much more common in older children.
A 1-year-old boy presents with a 1-week history of bloody diapers. The child has been crying almost constantly. On clinical exam, there is abdominal tenderness. Guaiac test is positive, hematocrit is 39.0, and hemoglobin is 13.0. What is the most likely diagnosis? 1. Diverticulitis 2. Meckel's diverticulum 3. Peutz-Jeghers syndrome 4. Ulcerative colitis 5. Crohn's disease
Correct answer: Meckel's diverticulum The Meckel diverticulum appears to be a remnant of developmental structures that were not fully reabsorbed. It is a common congenital abnormality that consists of a small pouch called a diverticulum located off the wall of the small bowel. Symptoms generally occur during the 1st few years of life. Symptoms include passing of blood either with or without stool and abdominal discomfort ranging from mild to severe. Tests should include stool smear for occult blood (stool guaiac), hematocrit, hemoglobin, and technetium scan to demonstrate diverticulum. Surgery to remove the diverticulum is recommended if bleeding develops. Iron replacement may be needed to correct anemia. If bleeding is significant, blood transfusion may be necessary. Diverticulitis is inflammation of an abnormal pouch (diverticulum) in the intestinal wall, usually found in the large intestine (colon). Small protruding sacs of the inner lining of the intestine (diverticulosis) may occur in any part of the intestine. They occur with increasing frequency after the age of 40. Diverticulitis is an inflammatory condition where gross or microscopic perforation (hole) of the diverticula has occurred. A low-fiber diet may be a contributing factor to the development of diverticula. Symptoms include left lower abdominal pain, constipation or diarrhea, chills, fever, swallowing difficulty, stools (clay colored or bloody), nausea and vomiting, heartburn, cough, and breath odor. Tests should include colonoscopy, sigmoidoscopy, barium enema, rectal examination (showing bleeding), abdominal palpation (showing left lower quadrant mass), and stool hemoccult test (revealing blood). Treatment should include increasing the bulk in the diet with high-fiber foods and bulk additives. Peutz-Jeghers syndrome is transmitted as an autosomal dominant trait. Pigmented spots, brownish or bluish gray, develop from infancy through childhood around the lips, gums, and mucus membranes in the mouth. Symptoms include crampy abdominal pain, vomiting, occasional gross blood in the stool, and intussusception (a telescoping of one portion of the intestine into another). Intestinal polyps also develop that can be detected with special studies. Diagnostic tests include X-ray of abdomen (shows polyposis), occult blood in stool, CBC, serum iron, serum, total iron binding capacity, and biopsy of polyps. Treatment includes surgery to remove polyps that cause chronic problems. Iron replacement therapy (iron supplements) helps counteract blood loss. Periodic studies are recommended to watch for malignant changes in polyps. Crohn's disease (also called ileitis or enteritis) causes inflammation in the small intestine. The most common symptoms of Crohn's disease are abdominal pain (often in the lower right area) and diarrhea. Rectal bleeding, weight loss, decreased appetite, and fever may also occur. Bleeding may be serious and persistent, leading to anemia. Tests should include CBC (increased white count), upper GI, and colonoscopy. The goals of treatment are to control inflammation, relieve symptoms, and correct nutritional deficiencies. Surgery is indicated to relieve chronic symptoms that do not respond to treatment. Ulcerative colitis is chronic, episodic, inflammatory disease of the large intestine and rectum characterized by bloody diarrhea. Ulcers form in the inner lining, or mucosa, of the colon or rectum, often resulting in diarrhea, blood, and pus. The inflammation is usually most severe in the sigmoid and rectum and usually diminishes higher in the colon. It may affect any age group, although there are peaks at ages 15 to 30 and then again at ages 50 to 70. Symptoms include diarrhea of between 10 and 25 times a day (in which blood and pus may be present), abdominal pain and cramping that usually subsides after a bowel movement, abdominal sounds (borborygmus, a gurgling or splashing sound heard over the intestine), fever, weight loss, stools (foul smelling), and tenesmus (pain while passing stool). Tests include colonoscopy with biopsy, and barium enema. The goals of treatment are to control the acute attacks and prevent recurrent attacks. Corticosteroids are prescribed to reduce inflammation. Sulfasalazine may decrease the frequency of attacks. Surgery may be indicated in refractory disease.
A 65-year-old female is brought to the ER after she complained of sudden onset of dizziness early this morning when she was in the bathroom and fell to the floor, hitting her head. She had 3 episodes of dark semi-solid stool since last night but no nausea or vomiting. She has a history of hypertension and chronic osteoarthritis, and she takes amlodipine 10 mg daily as well as ibuprofen 400 mg 3-4 times daily as needed for the last 4-5 years. She does not smoke or drink. Her family history is insignificant. On exam she is afebrile, BP is 90/52 mm Hg, pulse is 112/min, and SPO2 is 92%. She looks pale and tired. Her lungs are clear, and she has some epigastric tenderness. Chronic arthritic changes are noted in her knees and fingers. Neurologically she is awake but somewhat lethargic and fully oriented. No other deficit is found. Labs show Hb 8.2 g/dl, WBC 12,000/uL and platelets 350,000/uL. Chest X ray is clear, EKG shows sinus tachycardia, and cardiac enzymes are normal. Metabolic panel is within normal limits, except BUN being 32. The likely diagnosis is 1 NSAID-induced peptic ulcer 2 Septic shock 3 Amlodipine-induced hypotension 4 Arteriovenous malformation in the GI tract 5 Intracranial bleed from head injury
Correct answer: NSAID-induced peptic ulcer This patient is using NSAIDs chronically for arthritis without any protective therapy with proton pump inhibitors or H2 receptor blockers. She has developed a bleeding peptic ulcer, as evidenced by melena, which can be catastrophic in the elderly. The patient now has anemia due to acute GI bleed with hypotension and hypovolemia. Therapy includes vigorous fluid resuscitation, intravenous proton pump inhibitor, and life-saving blood transfusion followed by urgent upper endoscopy. Long term follow-up includes indefinite proton pump inhibitor therapy and acetaminophen for arthritis. Even though the patient has leukocytosis, there is no obvious focus of infection or fever. This can occur in response to the acute bleeding. Amlodipine will not cause melena and in supervised treatment of hypertension, it is unlikely to cause such a severe hypotension. AV malformation usually presents with lower GI bleed, not melena. It may be a chronic slow bleed causing iron deficiency in the elderly. Intracranial bleeding does not cause melena and is associated with focal neurological signs.
Pellagra, known by many medical students to produce the "4 D's" (diarrhea, dermatitis, dementia, and death), is caused by what deficiency? 1. Vitamin E 2 Vitamin B6 3 Vitamin B1 4 Vitamin B12 5 Niacin
Correct answer: Niacin Niacin deficiency has been linked to pellagra. This condition, now quite rare, consists of nausea with vomiting, diarrhea, dermatitis, and cognitive decline (such as irritability and memory loss). Vitamin B6 (pyridoxine) deficiency and excess has been associated with neurological disease. Both lead to a distal symmetrical polyneuropathy. A deficiency of B6 is commonly found in patients taking INH, an anti-tuberculosis drug. INH inhibits pyridoxine phosphorylation, thereby decreasing the concentration of the active compound-pyridoxal phosphate. Excessive use of B6 leads to an excess of pyridoxine, which competes with pyridoxal phosphate for binding sites on the enzyme. Vitamin B12 deficiency causes both a peripheral neuropathy and a degeneration in the spinal cord known as subacute combined degeneration. This affects the dorsal column sensory functions (position sense and vibration) and the lateral corticospinal tract. The resulting symptoms include paresthesias of the feet and hands with subsequent weakness and stiffness of the legs and a spastic gait. Vitamin B1 (Thiamine) deficiency has been linked to the neurological syndrome of Wernicke's encephalopathy. This condition, although often associated with alcoholism, is a true nutritional deficiency, causing neuropathological lesions in areas such as the thalamus, mammillary bodies, and the periaqueductal areas. A polyneuropathy has also been linked to Vitamin B1 deficiency, especially in alcoholism. This deficiency plays at least a role in the development of this disease, along with the possible neurotoxic role of alcohol. Vitamin E deficiency occurs usually with malabsorption of the fat-soluble vitamins A, D, E, and K. This can occur with conditions such as cholestatic liver disease or celiac disease. Neurological symptoms include a peripheral neuropathy and spinocerebellar degeneration. The spinocerebellar degeneration can occur as a defect in abetalipoproteins, as occurs in Bassen-Kornzweig syndrome. This is also associated with acanthocytes and retinitis pigmentosa.
A 37-year-old man presents with diarrhea, nausea, vomiting, and a rash. He also complains of memory problems and depression. He states that he and his family live on a farm that produces corn. Due to recent financial problems, his diet has become more limited. He eats mainly corn products and only small amounts of fish and vegetables. He takes no vitamin supplementation, and he states that his symptoms began about 1 month ago. He began having trouble with his memory about 1 week ago. He has no significant past medical history, but he does admit to daily alcohol consumption. His physical examination is significant for stomatitis, glossitis, and a rash over his arms and legs. His mini mental status exam score is 24. Question What deficiency does this patient most likely have? 1. Niacin 2. Zinc 3. Folate 4. Vitamin B12 5. Vitamin B1
Correct answer: Niacin This patient probably has niacin deficiency. Niacin (nicotinamide, nicotinic acid) deficiency is uncommon in the United States. It is often found in people who live on a diet that consists mainly of corn (maize). This is due to the fact that the niacin in corn cannot be absorbed unless it is chemically treated with alkali first. If a person consumes a diet rich in tryptophan, but low in niacin, they are able to compensate since tryptophan can be converted into niacin. Deficiency may also result from alcoholism, cirrhosis, or diarrhea. Men and women are affected equally. Symptoms of niacin deficiency include nausea, vomiting, diarrhea, rash, glossitis, stomatitis, depression, and psychosis. Niacin deficiency, also known as pellagra, manifests as the '3 Ds': diarrhea, dermatitis, and dementia. Treatment consists of niacin supplementation. The causes of zinc deficiency include malnutrition, chronic debilitating diseases, chronic renal disease, alcoholism, drugs such as penicillamine and diuretic, and genetic disorders, such as sickle cell disease. Clinical manifestations in severe cases include alopecia, diarrhea, weight loss, infections, dermatitis, hypogonadism in men, and intercurrent infections. Supplementation with zinc is the treatment of choice. Folate deficiency causes megaloblastic macrocytic anemia, as folate plays a key role in nucleic acid synthesis. The early manifestation of folate deficiency, especially in its suboptimal state, predisposes to occlusive vascular disease and thrombosis. These manifestations are linked to increased homocysteine levels found in folate deficiency. Neurological disturbances, such as mood disturbance and spinal cord syndromes, are also seen. It is also associated with predisposition to neoplasia and interferes with immunologic status. Folate replacement is the option to prevent and to treat the deficiency. Vitamin B1 (or thiamine deficiency) causes beri beri, occurring mostly in the malnourished and alcoholics. The deficiency manifests with acute heart failure, neurologic deficits, and epilepsy. Empiric use of thiamine and prophylactic use in high-risk population is strongly recommended even before blood reports are obtained, as the treatment is inexpensive and prevents major catastrophes. Vitamin B12 (or cobalamin deficiency) manifests as megaloblastic macrocytic anemia, pancytopenia, and a spectrum of neuropsychiatric disorders such as peripheral neuropathy, parasthesias, and demyelination of corticospinal tract. Nutritional deficiency, alcoholism, and malabsorption syndromes are some causes of B12 deficiency. It is also associated with homocysteinemia and atherosclerosis. Diagnosis is by serum estimation of B12, and oral supplementation is safe and effective. Intramuscular injections may also be used.
An 88-year-old male complaining of abdominal pain enters the emergency room with his wife. A mini-mental status exam reveals pronounced forgetfulness and confusion. The patient is discovered to have acute appendicitis requiring immediate surgery. He is unable to understand the situation and cannot provide informed consent. What further action should the physician take? 1 Do not perform surgery 2 Have another doctor confirm the necessity of surgery 3 Obtain a court order to perform surgery 4 Obtain consent from his wife to perform surgery 5 Try to persuade the patient to consent to surgery
Correct answer: Obtain consent from his wife to perform surgery In cases in which an emergency exists, the patient is incompetent to give consent, and the withholding of treatment would be potentially life-threatening, the physician must seek out close relatives of the patient to supply consent. The physician should proceed with treatment, assuming the patient would want the treatment had he or she understood the situation. Not performing surgery could cost the patient's life. Having another doctor confirm the necessity of surgery is favorable (if done immediately) but not mandatory and does not change the patient's consent status. Obtaining a court order is not necessary with the patient's wife immediately accessible. Trying to persuade the patient to consent to surgery would not only waste time and prove futile but might agitate the patient as well.
A 49-year-old Caucasian woman presents with a 2-month history of worsening abdominal pain. The patient describes the pain as crampy, unrelenting abdominal pain that begins in the right upper and middle quadrant of the abdomen; very occasionally, it will go into right shoulder area. The pain usually occurs shortly after especially heavy meals, usually within 30 minutes; it then eventually subsides, although this takes anywhere from 60 - 120 minutes. Physical examination reveals significant right upper quadrant tenderness and rigidity with a palpable, firm mass. Ultrasound performed in the office reveals complete calcification of the gallbladder wall; there is a hyperechoic semilunar appearance with posterior acoustic shadowing. A symptomatic patient as described above with such severe calcification as seen on ultrasound would most likely be treated via what method? 1 Reassurance only 2 Periodic ultrasound imaging study 3 Open cholecystectomy 4 Laparoscopic cholecystectomy 5 Percutaneous biliary stent placement
Correct answer: Open cholecystectomy Patients withsymptomatic porcelain gallbladdersare much more commonly female than male, with the typical age range being from 38 - 70 years old. In general, patients will describe a history of biliary type pain. Diagnostically, an ultrasound or CT scan will most efficiently give the visualization of the calcification necessary to confirm a porcelain gallbladder.Those found to have complete mural calcification (complete type) generally will need to have it treated with an open cholecystectomy. This is because the mucosal calcification that creates the porcelain characteristic makes the gallbladder very thick and fibrotic, which in turn makes it potentially difficult to grab by forceps and dissect out in a laparoscopic approach. There have been recent studies suggesting a laparoscopic cholecystectomy would be acceptable in patients who have evidence of a long cystic duct and biliary anatomy that is well-defined perioperatively, as well as in those who have evidence of a less advanced form of porcelain gallbladder, although there is no confirmed change of recommended treatment plan at this time. Anytime a porcelain gallbladder is found, there has to be a heightened alert for the suspicion of gallbladder carcinoma. The more advanced and severe the calcification is of the gallbladder, the heightened potential of it being malignant. Reassurance only and periodic ultrasounds would not be appropriate in this scenario. This approach may be followed in some part in a patient with less severe signs and symptoms. Percutaneous biliary stent placement is not the appropriate choice for this clinical scenario, as it is not treating the main pathology being presented.
A 68-year-old nonsmoking man presents to your pulmonology practice for long-standing dyspnea and non-productive cough. The patient has had the cough and progressively worsening dyspnea for about 1.5 years, with no current exacerbation. He denies fevers, chills, night sweats, and any other symptoms. He denies unusual travel, hobbies, or occupational exposures. The rest of his review of systems is negative. His past medical history is remarkable only for gastroesophageal reflux (GERD). He has had a workup through his family practice and was then sent to the cardiologist, who ruled out cardiovascular causes of his dyspnea. He has been given trials of various antibiotics, inhalers, and steroids, all without improvement in symptoms, despite good compliance. He currently takes no medications. Several tests have been performed, and results are shown in the table. Blood count, metabolic panel, HIV, and autoimmune markers: Normal High-resolution computed tomography (HSCT) scan: Few reticular opacities Pulmonary function test (PFT): Restrictive impairment and reduced perfusion of carbon monoxide Physical exam is significant for fine inspiratory bibasilar crackles and clubbing in the fingers. An occasional dry cough is noted. Which of the following medications is most likely to provide a long-term benefit for this patient's condition and is linked with increased survival? 1. Cisplatin-based chemotherapy 2. Inhaled albuterol 3. Inhaled budesonide 4. Oral omeprazole 5. Oral prednisone
Correct answer: Oral omeprazole Explanation This patient likely has idiopathic pulmonary fibrosis (IPF). IPF is a chronic, progressive restrictive pulmonary disease of the lung parenchyma. IPF presents with exertional dyspnea and non-productive cough, as well as the imaging characteristics described for this patient. Treatment is complex. Overall, medications play a minor role in this relatively rare disease. However, a prescription for a proton-pump inhibitor (PPI), such as oral omeprazole, is likely to provide the most benefit for this patient. There are various theories (such as microaspiration) regarding the association of GERD and IPF. Treatment with acid-lowering therapies is linked with disease stabilization and improved survival in patients with IPF. Cisplatin-based chemotherapy has a role in treatment of non-small cell lung cancer. It has not been proven to alter the course of IPF. Inhaled albuterol may appear helpful in a patient with dyspnea, but it does not treat IPF. The beta-agonistic effects cannot override the damaged, fibrotic tissue. Inhaled budesonide, an inhaled steroid, is commonly used for asthma and chronic obstructive pulmonary disease. However, this patient has a restrictive pulmonary disease. The pathophysiologic basis for IPF is much more a disorder of fibroblastic disease, in which there is an abnormal accumulation of extracellular matrix, and not inflammatory in nature. Anti-inflammatory medications (such as inhaled or oral steroids) do not alter the course of IPF. Oral prednisone, another steroid, is often used in IPF in acute exacerbations, but has little evidence for any benefit and does not seem to alter the disease course. This patient is not having an acute exacerbation, so the PPI would be the best choice of the listed medications.
A 72-year-old man is evaluated at the bedside following hospital admission for a 1-year history of progressive dyspnea, nonproductive cough, weight loss, low-grade fevers, fatigue, and myalgias. His past medical history is remarkable for atrial fibrillation, for which he takes amiodarone, hypercholesterolemia, and recurrent urinary tract infections, for which his urologist prescribed nitrofurantoin on a chronic, prophylactic basis. He denies any cigarette use, history of murmurs or coronary artery disease, chills, fatigue, rhinitis, otalgia, chest pain, wheezing, hemoptysis, syncope, abdominal pain, rashes, peripheral edema, diaphoresis, arthralgias, vomiting, or urinary complaints. Arterial blood gas analysis demonstrated a PaO2 of 50 mmHg and pulse oximetry of SpO2 of 87%; bedside echocardiogram and electrocardiograms are unremarkable for abnormalities; a chest x-ray revealed peripheral reticular opacities at the lung bases and a generalized honeycombing pattern. What treatment is the most beneficial in the management of this patient? 1. Prednisone 2. Colchicine 3. Oxygen 4. N-acetylcysteine 5. Bosentan
Correct answer: Oxygen This patient's most likely diagnosis is idiopathic pulmonary fibrosis (IPF). It is defined as a specific form of chronic, progressive fibrosing interstitial pneumonia of unknown cause, primarily occurring in older adults, limited to the lungs, and associated with the histopathologic and/or radiologic pattern of usual interstitial pneumonia (UIP). Patients with hypoxemia (PaO2 < 55 mmHg or oxygen saturation as measured using pulse oximetry [SpO2] < 88%) at rest or with exercise should be prescribed oxygen therapy to maintain a saturation of at least 90% at rest, with sleep, and with exertion. Corticosteroids have not been evaluated in a randomized, placebo-controlled trial to determine their benefit in treating patients with idiopathic pulmonary fibrosis. Retrospective uncontrolled studies have reported no survival benefits. Evidence-based guidelines recommend that patients with idiopathic pulmonary fibrosis should not be treated with corticosteroid monotherapy. Colchicine has been shown to inhibit fibroblast proliferation and collagen synthesis in vitro; however, evidence-based guidelines recommend that patients with idiopathic pulmonary fibrosis should not be treated with colchicine. Evidence-based guidelines recommend that the majority of patients with IPF should not be treated with N-acetylcysteine monotherapy. Bosentan is an endothelin receptor A and B antagonist that is approved for the treatment of pulmonary hypertension. While bosentan has been shown to have antifibrotic effects in an animal model of pulmonary fibrosis, current evidence-based guidelines recommend that patients with idiopathic pulmonary fibrosis should not be treated with bosentan.
A 45-year-old man presents with hematemesis. He has had 2 episodes of vomiting 'coffee-ground'-appearing material; the vomiting began 45 minutes prior to presentation. Additionally, he reports passing black, sticky stools for the past 3 or 4 days. Past medical history is positive for occasional headaches; they have been coming more frequenly lately. Social history reveals alcohol use (1 case of beer each weekend) and tobacco (1 pack per day). Medications include ibuprofen as needed for headaches; he has been taking 800 mg 3 times a day for the past week. You place a nasogastric tube and find bright red blood that fails to clear with saline irrigation. Hemoglobin is 8.9 g/dL. Evaluation of his blood pressure and pulse reveals orthostatic changes that resolve with an intravenous fluid bolus of 500 cc of Lactated Ringer's solution. What should you do next? 1. Transfuse 2 units of packed red blood cells and recheck CBC in 8 hours 2. Check serum for Helicobacter pylori antibody 3. Order a double contrast barium swallow to evaluate for a gastric ulcer 4. Refer for emergency upper endoscopy 5. Discontinue his ibuprofen use and encourage cessation of alcohol and tobacco
Correct answer: Refer for emergency upper endoscopy kk This patient is most likely bleeding from a gastric ulcer. His recent NSAID use, as well as his alcohol and tobacco habits, make him at risk for peptic ulcer disease. His symptoms of melena and hematemesis, along with his anemia, make the diagnosis quite straightforward. It appears that this patient is still actively bleeding based on the results of the nasogastric tube irrigation; therefore, the priority should be getting the ulcer to stop bleeding. Upper endoscopy should be performed so that the bleeding site can be identified and treated with electrocautery, coagulation, or injection of epinephrine or a sclerosing agent. If the bleeding cannot be stopped with endoscopic interventions, angiographic embolization should also be tried. If these interventions do not succeed, the patient has rapid deterioration, or if he requires more than 6 units of blood in a 24-hour period, then emergency surgery may be indicated. The other choices are not the best options for immediate management. This individual cannot be followed simply with transfusions and serial CBC's because he appears to still be actively bleeding. Helicobacter pylori infection may very well be playing a part in the etiology of this man's ulcer, but evaluation for H. pylori can be done with a biopsy at the time of his endoscopy; it will not help in his immediate management. A barium esophagram will not identify actively bleeding ulcers and cannot treat active bleeding. While NSAID, alcohol, and tobacco use may have precipitated this man's GI bleed, counseling about his use of these substances will not sufficiently treat his immediate bleed.
A 60-year-old man presents with a history of alcohol consumption; he has consumed at least 3 beers per day for the past 15 years. He has been unable to carry out his regular activities for the past 6 months. He just sits around and drinks all day. He has lost 10 pounds over the last 3 months. Vitals are as follows: Temp 99 degrees F, RR 24/min, HR 90/min, BP 130/90 mm Hg. Physical exam is significant for icterus, digital clubbing, and hepatomegaly. Lab and ultrasound reports are pending. What is the best treatment for this patient? 1 Erythromycin 2. Interferon 3. Pentoxyfylline 4. Ribavirin 5 Lamivudine
Correct answer: Pentoxyfylline A long history of alcohol consumption along with hepatomegaly on examination points to a diagnosis of alcoholic liver disease (ALD). Pentoxyfylline is used in the treatment of ALD, as it reduces the production of tumor necrosis factor and other proinflammatory cytokines. It has been found to decrease portal hypertension in experimental animals. It also appears to decrease short-term mortality by reducing the risk of hepatorenal syndrome. Erythromycin is a macrolide antibiotic that would be useful only if the patient had an infection. Ribavirin and lamivudine are antivirals. Interferons are mainly used in the treatment of hepatitis C. None of these drugs are useful in alcoholic cirrhosis.
What is the most common cause of upper gastrointestinal bleeding? 1. Gastritis 2. Esophageal varices 3. Peptic ulcer 4. Mallory-Weiss syndrome 5 Cancer
Correct answer: Peptic ulcer A peptic ulcer is the most commonly uncovered lesion in the diagnostic workup of a patient with upper gastrointestinal bleeding. The location of the ulcer responsible for the bleeding is most often the duodenum, and ulcers of the stomach and esophagus are less frequently found. Rarely, ulceration of the small intestine may be the origin of upper gastrointestinal bleeding. Hemorrhage from a peptic ulcer may be acute or chronic. Gastritis, esophageal varices, Mallory-Weiss syndrome (esophageal mucosal tear), and cancer (mostly stomach and esophagus, rarely small bowel) are less common causes of upper gastrointestinal bleeding.
A 35-year-old Hispanic man presents to your office but is too embarrassed to tell the nurse his chief complaint. You enter the room, and he admits to severe, intense itching around his anus that has been worsening the last several weeks. He further states that he has noticed increasingly severe and tearing pain in the anal area with each bowel movement. He would rank this pain as a 10/10 on a pain scale. This intense pain makes him not want to have any bowel movements. He admits to only 1 episode of a small amount of bright red blood on the toilet paper as well as on the stool itself. The patient denies fever, diarrhea, or ever being diagnosed with inflammatory bowel disease. Considering the findings in the patient above, what is best choice in diagnostic studies to assist in confirmation of diagnosis? 1 Physical examination 2 Digital rectal exam 3 Anoscopy 4 Sigmoidoscopy 5 Colonoscopy
Correct answer: Physical examination Most likely this patient is suffering from an anal fissure. Anal fissures most often affect infants, as well as middle-aged individuals. The majority of fissures are considered primary and caused by local trauma such as passage of hard stool, prolonged diarrhea, vaginal delivery, or anal sex. Presentation of anal fissures is a tearing pain accompanying bowel movements as well as bright red rectal bleeding that is limited to a small amount noted on the toilet paper or surface of the stool. The patient described all of these components. Patients will also complain of perianal pruritus or irritation, which he also admits to experiencing. The best diagnostic approach to confirming an anal fissure is usually based on the history of pain with defecation, as well as simply the physical examination finding of a superficial tear noted in the anoderm; this type of tear, no matter if located posteriorly or laterally, is considered a pathognomonic feature of an acute anal fissure. The examiner should carefully spread the buttocks apart and examine the area gently. Digital rectal examination, or anoscopy, is not recommended for diagnosis of this condition due to the fact that patients will not be able to tolerate them due to the severe pain they will be experiencing from the anal fissure. Sigmoidoscopy or colonoscopy are both inappropriate diagnostic tools in this patient case scenario. These studies should only be considered if there is presence of otherwise unexplainable rectal or gastrointestinal bleeding. Either a sigmoidoscopy or a colonoscopy can assist in evaluating the corresponding source of bleeding.
A 55-year-old woman was diagnosed with small cell cancer of the lung 2 months ago; she presents with increasing dysphagia, respiratory difficulties, and weakness of the upper limb. Her vital signs are: pulse 85/min, BP 120/90 mm Hg, resp. 12/min, and temp. 37.7 C. On examination, she has ptosis of both eyes, and she reports diplopia. Her pupillary responses are normal. The strength in the muscles of her arm on testing is 2/5; on repeated testing, the strength improves to 4/5. Sensation is intact in both upper limbs. What is the most appropriate treatment for her condition? 1. Neostigmine 2. Atropine 3. Plasmapheresis 4. Amikacin 5. Cisapride
Correct answer: Plasmapheresis The Lambert-Eaton syndrome is caused by antibodies to the presynaptic calcium channel, which decreases the release of acetylcholine. Repeated stimulation of the nerve ending increases the intracellular calcium concentration, which allows enough acetylcholine to be released to cause muscle contraction. Anticholinergics would not help in this condition. The treatment lies in removing the circulating antibodies using plasmapheresis or immunosuppressants, such as prednisone or azathioprine. The Lambert-Eaton (or myasthenic syndrome) may occur as a paraneoplastic syndrome of small cell lung cancer. The presentation is similar to myasthenia gravis; however, the weakness is made worse by sustained movement in myasthenia gravis. It is improved in the Lambert-Eaton syndrome. Myasthenia gravis is caused by antibodies to the acetylcholine receptor; consequently, the treatment lies in increasing the acetylcholine concentration in the synaptic cleft using an anti-cholinergic medication such as neostigmine. Amikacin is an aminoglycoside; it can cause weakness of skeletal muscles. Cisapride is a promotility agent use for dysphagia or gastro-esophageal reflux. It has been withdrawn from the US market, and it is available only to patients who meet specific criteria. Atropine is a cholinergic antagonist at muscarinic receptors and would not be useful in this case.
A 2-year-old girl presents with cough, fever, decreased oral intake, and irritability for a few days. Prior to presentation, she had a 1-week history of an upper respiratory infection. On exam, she is febrile to 102°, has a respiratory rate of 28, a pulse of 110, and blood pressure of 88/56. She appears tired, but she is responsive, interactive, and in no acute respiratory distress. On lung exam, she has decreased breath sounds, dullness to percussion, and decreased tactile fremitus on the right side. What is the most likely diagnosis? 1. Pleural effusion 2. Bronchiectasis 3. Viral pneumonia 4. Pulmonary embolism 5. Hypoalbuminemia
Correct answer: Pleural effusion A patient with a pleural effusion usually presents with decreased breath sounds, dullness to percussion, decreased tactile fremitus, and egophony. If pneumonia is present, which is the most common cause of effusion in children, a crackle may also be detected. Small effusions may be difficult to detect on exam. An X-ray will show a density obliterating the normal markings of the lung with an air-fluid level. Therapy should address the underlying disease. Thoracentesis can be done to make the patient more comfortable or to gain additional information. Appropriate antibiotics should be instituted.
A 65-year-old man presents with a bulge in his lower abdomen that has been present for 5-10 years. He can usually push this bulge back, but, about 4 hours earlier, it came out and could not be pushed back. There is an aching pain, but no nausea or vomiting. He admits to straining at the stool and getting up twice a night to urinate. He smokes 1 pack of cigarettes per day. On physical examination, his vital signs are within normal limits; his pulse oxy is 91%. There is no jugular vein distention. Chest shows an increased A-P diameter, but breath sounds are good. Heart sounds are distant, and the rate and rhythm indicate normal sinus. There is an obvious bulge in the right lower quadrant extending into the right scrotal sac. The abdomen itself is soft, flat, and non-tender. Bowel sounds are slightly hyperactive; there are no rushes or tingling sounds. The bulge cannot be reduced and is tender. Bowel sounds are present within the mass. A somewhat enlarged, non-tender prostate is detected on rectal examination. Brown stool is obtained and is negative for blood. There is no abnormality on EKG. Chest x-ray shows some hyperinflation, but no infiltrates. An x-ray, flat and upright, of the abdomen shows some dilated small bowel extending into the right scrotal sac, but signs of obstruction are absent. Routine lab and urine studies are within normal limits. You start an intravenous line of 0.9 N/S to run at keep-open rate. What would be the next logical course of action? 1 Order an immediate surgical consultation 2 Admit the patient to the observation unit for 4 hours and reassess 3 Sedate the patient well and attempt to manually reduce the mass 4 Pass an intestinal tube to reduce small bowel distention 5 Pass a Foley catheter to reduce bladder distention
Correct answer: Sedate the patient well and attempt to manually reduce the mass The most likely diagnosis in this patient is incarcerated indirect hernia. Direct hernias do not usually incarcerate and come straight through the abdominal wall. Indirect hernias, by definition, come down the inguinal canal into the scrotal sac. Intestinal contents accompany the sac in either one. This patient's lung signs and signs of prostate enlargement are both impediments to emergency surgery, and they are additional reasons to delay repair to a more elective time if possible. Every reasonable attempt should be made to reduce this hernia using manual compression after adequate sedation. A warm compress on the area may help reduce muscle spasm and relax the abdominal musculature. There are times when either a nasogastric tube and/or a long intestinal tube are warranted. This patient has no signs of intestinal obstruction (vomiting, constipation, etc.). The minimal dilatation of the small bowel is an accompanying sign of an incarcerated hernia. An obstructed segment of bowel may become incarcerated and/or strangulated; when reduced or replaced in the peritoneum, signs and symptoms of mechanical obstruction occur. Of course, should it not be possible to reduce the hernia, obstruction and strangulation both may develop. Strangulation occurs when blood flow (venous, arterial, or both) is closed off, and the bowel segment dies. Then an emergency exists, and repair must occur upon recognition. A Foley catheter has no value unless there is urinary outlet obstruction.
A 55-year-old man visits his family physician for an accidental cut to the forearm with an old rusty wrench. He works as a tank mechanic for the US army and has been an army recruit since the age of 18. The patient received his influenza vaccine a few months ago. He also received a tetanus and diphtheria toxoid approximately 2 months ago when he had an unrelated injury. The patient has been consuming a lot of alcohol over the past few years. He admits to wanting to cut down. He currently drinks a half bottle of red wine every day. He also admits to taking a shot of vodka in the morning to "tide" him through the day. The wound is cleaned and dressed appropriately and his liver function is tested, which reveals the following: Alanine transaminase (ALT) 35 units/L Aspartate transaminase (AST) 76 units/L What prophylactic treatment is indicated? 1 Pneumococcal vaccination and hepatitis A and B 2 Diphtheria and tetanus toxoid and hepatitis A and B 3 Diphtheria and tetanus toxoid only 4 Hepatitis A and hepatitis B only 5 Measles, mumps, and rubella only 6 Pneumococcal vaccination only
Correct answer: Pneumococcal vaccination and hepatitis A and B Alcoholism predisposes patients to pneumonia (streptococcus pneumoniae infection). Therefore, administering the annual influenza vaccination and the pneumococcal vaccination is recommended in patients with alcoholism. Hepatitis A or B can be particularly severe in persons with chronic liver disease; therefore, many authorities recommend vaccination against both conditions for persons with chronic liver disease of any cause (particularly those with hepatitis C). This patient is an alcoholic with elevated AST and ALT levels; therefore, he requires administration of pneumococcal vaccine along with Hepatitis A and B vaccines. The MMR vaccine is routinely given to all healthy children at age 12 to 15 months with a second dose at age 4 to 6 years. A second dose is especially important for military recruits, college students, and susceptible health-care personnel. The tetanus toxoid is not required for this patient because he received them 2 months ago. The tetanus toxoid is indicated in clean minor wounds if the person has not received the toxoid in the past 10 years. For other wounds, it is indicated if the person has not received the toxoid in the past 5 years.
A 47-year-old man presents with abdominal pain and difficulties breathing. He has a history of alcohol abuse and confirmed cirrhosis of the liver. On examination, you see a malnourished, jaundiced patient with a distended belly. Percussion of the abdomen reveals a huge amount of fluid and wave sign. What is the primary cause of the ascites? 1. Increased albumin production 2. Increased ammonia production 3 Portal hypertension 4 Decreased fluid intake 5 Blockage of the common bile duct
Correct answer: Portal hypertension In cirrhosis the fibrotic changes in the liver tissue increase the resistance to blood flow through the organ and this results in increased portal pressure. The lesion is intrahepatic and sinusoidal. Hepatic synthetic failure causes hypoalbuminemia and decreased albumin production, which leads to reduced portal oncotic pressure in cirrhotic patients. Ammonia levels may be elevated, but elevated levels do not cause ascites. Ammonia is a highly toxic metabolic product of the urea cycle. The urea cycle is the only major pathway to remove waste nitrogen, and it usually gets converted to non-toxic urea in the liver. Increased portal pressure, reduced portal oncotic pressure, and elevated aldosterone level with sodium retention cause ascites. Decrease in fluid intake is actually one of the therapeutic strategies for ascites. Blockage of the common bile duct results in jaundice, but it has nothing to do with the development of ascites.
A 77-year-old woman presents with a cough of several weeks duration. The patient has seen 2 physicians for the cough; the 1st time was about 6 weeks ago, and the 2nd was 4 weeks ago. She was given antibiotics and a steroid inhaler. The cough is still there throughout the day, and it is particularly bothersome at night. She states the cough started after a mild upper respiratory tract infection that has cleared. She denies wheezing, shortness of breath, or chest pain. She has no symptoms referable to the head and neck; in particular, she has no throat pain or postnasal drip. Her past medical history is significant for celiac disease, and she follows a strict gluten-free diet. She also has mild hypertension; it is controlled with thiazides. She denies a history of asthma, smoking, or exposure to second-hand smoke. On exam, her vital signs are normal. You observe an elderly woman in no acute distress; her HEENT, heart, abdominal, and lung exams are within normal limits. A chest X-ray is also within normal limits. What would be the next step in the management of this patient? 1. Prescribe a decongestant/antihistamine 2. Prescribe a trial of prednisone 3. Perform sinus X-rays 4. Prescribe a proton pump inhibitor 5. Prescribe a cough suppressant, like codeine
Correct answer: Prescribe a proton pump inhibitor The most common causes of a chronic cough in immunocompetent adults (explaining 95% of cases) are post-nasal drip syndrome, gastroesophageal reflux disease (GERD), asthma, chronic bronchitis from cigarette smoking, bronchiectasis, eosinophilic bronchitis, or the use of an angiotensin-converting enzyme inhibitor (ACEI). The rest of the patients (5%) have chronic cough caused by bronchogenic carcinoma, carcinomatosis, sarcoidosis, left ventricular failure, or aspiration. A psychogenic cough, or a cough persisting from habit, is rare and is a diagnosis of exclusion. Post-nasal drip syndrome could be due to vasomotor rhinitis, allergic or non-allergic rhinitis, or chronic bacterial sinusitis. It would be unusual for a patient to present with a chronic cough due to a post-nasal drip syndrome with no symptoms referable to the HEENT (e.g., the absence of post-nasal drip, sore throat, nasal symptoms, or a feeling of fullness/gagging in the throat); therefore, the prescription of a decongestant/antihistamine is unwarranted. Sinus X-rays are indeterminate in the diagnosis of chronic sinusitis. Asthma may present with chronic cough in adults, without wheezing or shortness of breath; however, even if used suboptimally, the steroid inhaler should have some effect. It should be noted that a steroid inhaler is also the treatment for eosinophilic bronchitis. Moreover, if a further trial of steroids were to be indicated, it would be prudent to try the steroid inhaler again (this time with a spacer) instead of subjecting the patient to possible complications of oral steroids. Of the patients who present with a chronic cough who are subsequently found to have GERD as a cause, about 75% do not have symptoms of heartburn or regurgitation. Given this patient's presentation, it seems reasonable to approach her as if she had GERD and prescribe a proton-pump inhibitor as a trial for 2 - 3 weeks. It is very likely that this will control her cough, and over time the proton-pump inhibitor (PPI) can be downgraded to less aggressive treatment with H2-blockers. This would avoid both the high cost of PPIs, as well as the chronic use of PPIs in patients. The use of codeine in an elderly patient is not a good idea unless there is a high degree of assurance that its use will substantially help the patient. Excessive sedation, falls, and constipation are common side effects of narcotics in the elderly.
A 42-year-old morbidly obese woman is referred back to her primary care provider to supervise her weight loss program. A surgeon evaluated her for a 'gallbladder attack' 1 month ago, which then resolved; she currently denies abdominal pain. When examined via ultrasound, she still has visible stones in the gallbladder. She declined to have surgery until she achieves her initial weight loss goal of 50 pounds. She is on a supervised healthy diet of about 1200 kilocalories daily; she exercises 1 hour a day 6 days per week. She is losing about 15 - 16 pounds a month. What intervention would be most effective at preventing the recurrence of her gallbladder "attack" until she achieves her weight loss goal? 1. Prescribe orlistat 2. Prescribe ursodiol 3. Recommend over-the-counter omeprazole 4. Reduce patient's exercise 5. Restrict caloric consumption further
Correct answer: Prescribe ursodiol Both morbid obesity and rapid weight loss are risk factors for development of cholecystitis. Gallstones may be present within the gallbladder and remain asymptomatic, or the gallbladder walls may become inflamed, resulting in cholecystitis. Ursodiol is approved for prevention of gallstones in obesity patients with rapid weight loss. It would be the best choice in health maintenance and delaying or preventing need for surgical cholecystectomy. Orlistat is a prescription medication approved for treatment of obesity. Its mechanism is to block fat absorption. It does not have a direct role in health maintenance or prevention of cholecystitis or cholelithiasis. In fact, it has pronounced gastrointestinal side effects. Over-the-counter omeprazole is a commonly used proton-pump-inhibitor (PPI); it is helpful in acid lowering in the stomach and treatment of gastroesophageal reflux disease. It has not been shown to help gallbladder disease, and there is some evidence that PPIs may worsen it. Reducing the patient's exercise may aggravate her condition and negatively impact her weight loss. Exercise has an inverse relationship with cholecystitis and rates of cholecystectomies, so this patient should be encouraged to continue frequent exercise. This patient should not be instructed to restrict caloric consumption further. She is already losing weight quite rapidly at around 4 pounds per week. Her daily caloric intake is low; recommending further restriction will likely discourage her, and there will not be any benefit in regard to her gallbladder disease.
A 35-year-old man presents with a 4-week history of difficulty swallowing food. He has developed chest pain over the last few days, and he has also noticed regurgitation of food. On detailed history, he says that the regurgitation contains undigested fragmented food material; it is unassociated with a foul smell. He has lost 3 kg over the past month. On examination, his pulse is 94 bpm and blood pressure is 130/86 mm Hg. Mild pallor is noted, but there is no icterus. Epigastric tenderness is present. CBC shows a hemoglobin of 9.5 g/dL (total count 7400 cells/mm3 and ESR 12 mm/hr). An X-ray of the abdomen shows an absence of a fundic air shadow. The patient undergoes barium studies which show a dilated esophagus; the lower end appears beak-shaped. What is the most common cause for the above condition? 1 Diabetes mellitus 2 Chagas disease 3 Eosinophilic gastroenteritis 4 Neurodegenerative disorder 5 Primary idiopathic
Correct answer: Primary idiopathic Gradual onset of difficulty in swallowing food, regurgitation of undigested food, absence of fundic air on plain abdominal X-ray, and dilated esophagus with beak shaped lower end are diagnostically indicative of Achalasia cardia. It is due to degeneration of inhibitory neurons in the lower end of esophagus. The most common cause of Achalasia is primary idiopathic. There is a deficiency of inhibitory neurons, vasoactive intestinal peptide, and nitric oxide synthase in this condition. Other rarer causes of achalasia include Chagas' disease, lymphoma, neurodegenerative disorders, esophageal gastroenteritis, and certain viral infections.
A 6-month-old infant is brought to your office. The mother indicates that the infant had several scratches that took a long time to stop bleeding. After ruling out various forms of hemophilia, you suspect that the infant has a vitamin deficiency that is causing this problem. What proteins would be affected by this vitamin deficiency? 1. Protransglutaminase 2. Urokinase 3. Prekallikrein 4. Prothrombin 5. Fibrinogen
Correct answer: Prothrombin Prothrombin and many of the other blood clotting factors contain the modified amino acid γ-carboxyglutamic acid (gla). This modification occurs via an enzymatic reaction requiring vitamin K and converts glutamic acid, a weak calcium chelator, to gla, a much stronger calcium chelator. Vitamin K is an isoprenoid compound, classified as a fat-soluble vitamin; it is found in green plant leaves and is also synthesized by intestinal bacteria. The formation of gla residues is required for the "active" form of prothrombin. These gla residues are found in the amino terminal region of the protein and function by chelating calcium ions. This forms part of the binding site to phospholipid membranes, where the conversion from the zymogen prothrombin to active thrombin takes place. A vitamin K deficiency results in an increased coagulation time. A 2,3-epoxide derivative of vitamin K is an intermediate in the formation of gla residues. Other proteins that contain gla residues include Factors VII, IX, and X, several other anticoagulation proteins, and the bone proteins (Osteocalcin and matrix-Gla protein). Dicumarol is a naturally occurring anticoagulant that is an antagonist of vitamin K. It inhibits the reductase that converts the epoxide form of vitamin K back to the active vitamin.
A 52-year-old man presents with rectal pain during bowel movements. Patient describes his pain as tearing. Physical examination reveals a tear in the anal mucosa. What is considered first-line treatment in this case? 1. Psyllium 2. Topical nitroglycerin 3. Oral ciprofloxacin 4. Botox injection 5. Surgery
Correct answer: Psyllium Psyllium is the correct response. The patient has an anal fissure. Anal fissure is exacerbated by constipation. First-line treatment is fiber supplementation, such as psyllium and sitz baths. Topical nitroglycerin is an incorrect response. Nitroglycerin is used in the treatment of anal fissure, but is not a first line treatment. Oral ciprofloxacin is an incorrect response. The patient has no signs of infection, so there is no role for antibiotics. Botox and surgery are incorrect responses. These therapies are reserved for refractory cases.
A mother has brought in her 2-year-old son with a 2-day history of bleeding of the rectum. Bleeding was very little to begin with, but today the child has soaked 3 diapers. There is no history of fever, vomiting, or loose bowel movements. Child is active and playful and has no obvious discomfort. Stool is brick red in color. Abdomen is soft with no distension or organomegaly. Perianal examination is normal, and there is no fissure. What is the investigation of choice to confirm the diagnosis? 1 Ultrasound abdomen 2 CT abdomen 3 Lower GI barium studies 4 Colonoscopy 5 Radionuclide scan
Correct answer: Radionuclide scan The clinical presentation of the child with intermittent painless bleeding per rectum is suggestive of Meckel diverticulum (MD), which accounts for 50% of lower GI bleeds in children below 2 years of age. Radionuclide scan is the most sensitive study that can confirm the presence of Meckel diverticulum. It is performed after intravenous infusion of technitium-99m pertechnetate. The mucous secreting cell of the ectopic gastric mucosa in the Meckel diverticulum takes up pertechnetate so that it can be visualized on the scan. The uptake can be enhanced by ranitidine, cimetidine, or glucagon. Sensitivity of the enhanced scan is about 85%, and specificity is approximately 95%. During active GI bleeding, the radioactive red cells leak into the intestines where bleeding is occurring and will appear as 'hot areas' on the scan. 'Hot areas' are not seen on the scan in the absence of active bleeding. Meckel divertculum is a remnant of the omphalomesenteric duct, which connects the yolk sac with the gut in the developing embryo and provides nutrition to the embryo until the placenta develops. Later this duct is obliterated and separates from the intestine. The lining of the yolk sac is similar to the lining of the stomach. Failure of involution of the omphalomesenteric duct results in various residual structures, of which Meckel diverticulum is the most common. It is a 3-6 cm long pouch of ileum, about 50-75 cm from the ileocecal valve. This distance depends on the age of the patient. Meckel divertulum is lined by the acid-secreting mucosa that can cause intermittent, painless rectal bleeding due to ulceration of the diverticular and the adjacent ileal mucosa. Meckel divertculum can act as a lead point for intussusception or may lead to a volvulus of itself and of the small intestines, thus presenting with bowel obstruction. Routine barium studies usually do not fill the diverticulum; hence, they are not helpful in the diagnosis. Double contrast barium enema can be performed for evaluation of unexplained lower GI bleeds. However, it should not be undertaken during acute hemorrhagic phase, as it makes subsequent diagnostic evaluation like colonoscopy very difficult. Small bowel barium examination has a low yield because the divertculum fills transiently and the surrounding loops of small bowel overlap and obscure the diverticulum. CT scan is rarely used as a primary imaging modality where Meckel diverticulum is suspected. If Meckel diverticulum is associated with intussusception, it may be revealed as an intraluminal mass, but it cannot be differentiated from intussusception due to other causes. Ultrasound has a limited role in evaluating GI hemorrhage. Sometimes intussusception due to Meckel diverticulum has been diagnosed by ultrasound, but the sensitivity and specificity is generally low. Colonoscopy cannot positively diagnose bleeding from a Meckel diverticulum because the colonoscope usually cannot reach the part of small intestines where the Meckel diverticulum is located. It can be helpful in its diagnosis if blood-filled colon is seen without another source of bleeding, particularly if it is accompanied by an abnormal Meckel radionuclide scan.
A 39-year-old CFO of a finance company is seen for upper abdominal discomfort. She experiences heartburn, belching, and epigastric burning pain on and off for the last several months. She does say that her food habits are irregular, due to the high stress at her job, and her caffeine intake is more than she thinks is healthy. She does not smoke or drink. She has no medical problems, and she is not taking any medications. On exam, she is afebrile, pulse is 78/min, BP 110/70, height is 5'6, and weight 135 pounds. Further exam reveals some epigastric tenderness but is otherwise normal. She is advised a trial of lansoprazole by her doctor and follow-up in 2 weeks. However, she says that she has been reading about her symptoms and is concerned about infection with helicobacter pylori. What is the most cost-effective initial measure to diagnose this? 1. Referral to gastroenterologist for endoscopy 2. Empiric treatment with triple therapy 3. Urea breath test 4. Rapid antibody test 5. Fecal antigen test
Correct answer: Rapid antibody test An office-based rapid antibody test is still considered the most cost effective mode of diagnosis of infection with H. pylori, especially in a patient not treated in the past for the same. This test is simple, inexpensive, and sensitive, as well as specific. However, it is not useful as a follow-up test to confirm eradication of infection. Other tests include stool antigen testing, which is also inexpensive, noninvasive, highly sensitive, and specific, and can be used as a follow-up test as well. This infection is common in the general population. It is the cause of 50 - 80% of peptic ulcers, and incidence of ulcer disease in those with the infection is 20%. Testing is indicated in active or documented past history of peptic ulcer disease and gastric mucosa associated lymphoid tissue (MALT) lymphoma. Many physicians test patients with functional dyspepsia, as this patient, though it is still controversial. However, if testing is done and it is positive, treatment with triple therapy must be done. Referral to a specialist for endoscopy is indicated in patients with known peptic ulcer disease, in suspected MALT lymphoma, and follow-up of ulcers. Endoscopic based diagnostic testing of H.pylori should only be done if endoscopy is indicated for other reasons. Empiric treatment for H.pylori should never be done without testing. Urea breath test is recommended at least 4 weeks after treatment and is the test of choice to confirm eradication of infection. Fecal antigen test is done in a lab and delays the diagnosis, so it is recommended to confirm eradication of the infection 4 weeks after treatment.
A 3-week old infant is admitted to the hospital with complaints of abdominal distention, poor feeding, and constipation since birth. The mother mentioned that her child had passed meconium a few days ago. Rectal examination of the child revealed an empty anal canal and rectum. Which of the following would have been used to absolutely confirm the diagnosis made for this child's condition? 1. Plain X-ray 2. Barium enema 3. Manometry 4. Rectal biopsy 5. Acetylcholine esterase stain
Correct answer: Rectal biopsy Explanation Hirschprung's disease is the possible diagnosis in this child. It is a genetic disorder named after Harold Hirschsprung, the Dutch physician who first described the disease in 1886. The diagnosis of Hirschsprung's disease is based on the absence of parasympathetic ganglion cells in the submucosal and myenteric plexuses in the affected segment of the large intestine. Diagnosis of Hirschsprung's disease was formerly established by means of a full thickness biopsy, a procedure requiring general anaesthesia. The development of rectal suction biopsy techniques has greatly reduced the patient risk but has given diagnostic difficulties to the pathologist because they contained few or no ganglion cells. Also, the immature ganglion cells of the neonates could be confused with other cells such as macrophages, fibroblasts, and Schwann cells. The acetylcholine esterase (AchE) enzyme stains have diagnostic value but require frozen section, and results are difficult to interpret. Recent studies have shown that immunoperoxidase technique with antibodies against various nervous system proteins could prove as a potential aid in the diagnosis of Hirschsprung's disease. They can be used to demonstrate the hyperplastic nerve fibres and the ganglion cells for diagnostic value in cases suspected of Hirschsprung's disease. To diagnosis Hirschsprung's disease, a barium enema x-ray test is used to identify the narrow collapsed segment of bowel in front of the affected regions. In an infant younger than 3 months of age, or in any child with a very short or very long collapsed portion of bowel, the x-ray may appear normal despite the presence of Hirschsprung's disease. A biopsy, or tissue sample, from the part of the bowel an inch above the anus is then necessary to confirm the absence of ganglia. In cooperative children, a painless test called anorectal manometry may complement the rectal biopsy result. If Hirschsprung's disease is not recognized in the neonatal period, the affected infant may present with symptoms such as abdominal distension, pencil-thin stools, failure to thrive, and bilious vomiting. If an infant has any of these symptoms, and the physical examination shows an empty rectum, Hirschsprung's disease should be suspected. A delay in diagnosing this disease places the infant at risk for enterocolitis, with fever, explosive bloody diarrhea, and abdominal distension, in the second or third month of life. Treating Hirschsprung's disease requires surgery to remove the affected bowel and then joining the healthy bowel segments. For most children with Hirschsprung's disease, there are no long-term complications after successful surgery. A significant minority of children, though, are troubled with persistent constipation, encopresis (stool incontinence), or persistent enterocolitis.
A 43-year-old man presents with a "sore" near his rectum. He reports rectal pain with bowel movements; there is some anal discomfort with sitting and physical activity. He occasionally notes some drainage from the lesion and rectal pruritus. He denies anal intercourse, unusual travel, and other skin lesions. He has tried over-the-counter hemorrhoid treatments without relief. His past medical history is significant for Crohn's disease; he is not being treated for it currently. He has no known allergies and takes no medications. He is married/monogamous; he works as a bartender and denies the use of tobacco. He reports occasional alcohol use, but no drug use. On physical exam, a single 4mm pustule-like lesion is visualized on the perianal skin; it is located about 2 cm away from the anus. No other skin lesions are found. Rectal exam is normal. What is the most likely diagnosis? 1 Hemorrhoids 2 Herpes simplex 3 Lymphogranuloma venereum 4 Rectal fistula 5 Syphilitic chancre
Correct answer: Rectal fistula This patient most likely has a rectal fistula. A fistula (or fistula-in-ano) is a communicating tract (usually from an anal crypt) to the skin surface, and it often develops from a rectal abscess. Rectal fistulas are more common in patients with Crohn's disease. Fistulas are associated with rectal pain, itching and tenderness. External hemorrhoids are a common condition and can be associated with anal pain, constipation and bright red rectal bleeding. Hemorrhoids are dilated blood vessels and appear as protrusions, either singly or several clustered together, such as a bunch of grapes. (Internal hemorrhoids would not be visualized on this patient without anoscopy or colonoscopy.) Genital herpes simplex lesions are caused by sexual transmission of herpes simplex virus types 1 or 2 (HSV-1 and HSV-2). The classic presentation is a tingling, burning, and/or painful sensation from 1 or more vesicular or ulcerated lesions on the genitals. This patient's single perianal pustule and history of Crohn's disease make fistula a much more likely diagnosis. Lymphogranuloma venereum (LGV) is a sexually transmitted disease; it presents with a primary painless lesion on the genitals/anus. LGV is rare is developed countries, with greater incidence in the tropics. This patient noted a painful lesion, making LGV unlikely. Similarly, a syphilitic chancre is the primary lesion of the sexually transmitted Treponema pallidum. It presents as a solitary, painless anogenital lesion.
A 47-year-old man presents with a 6-month history of intermittent abdominal pain. He notes that his episodes of pain developed after eating large amounts at holiday parties or work events. During episodes, his pain is located in the right upper quadrant without radiation, is described as sharp, and is accompanied by nausea. He is currently pain free. On examination, his abdomen is soft, nontender, and nondistended. He is afebrile. Labs are as follows: WBC: 6.24 Hemoglobin: 16.8 Hematocrit: 49.6 Platelets: 187 Bilirubin total: 1.4 Alkaline phosphatase: 48 AST: 14 Glucose: 95 BUN: 8 Creatinine: 1.03 Sodium: 143 Potassium: 3.7 Chloride: 105 CO2: 30 ALT: 11 An abdominal ultrasound shows the presence of multiple gallstones with some sludge. The common duct is normal. There is no pericholecystic fluid present. No sonographic Murphy's sign is detected. What is the proper course of action? 1. Oral antibiotics 2. Admission to the hospital for IV antibiotics 3. Admission to the hospital for immediate laparoscopic cholecystectomy 4. Referral to a surgeon for elective laparoscopic cholecystectomy 5. Referral to a surgeon for ERCP
Correct answer: Referral to a surgeon for elective laparoscopic cholecystectomy The patient should be given referral to a surgeon for laparoscopic cholecystectomy. The patient has cholelithiasis, given the presence of gallstones on abdominal ultrasound. His history of multiple bouts of right upper quadrant abdominal pain following large meals is suggestive of cholecystitis. The recommended treatment is laparoscopic cholecystectomy. Since the patient is currently pain-free, has normal labs, and there is no sign of duct obstruction on ultrasound, this procedure can be done electively. Oral antibiotics and admission to the hospital for IV antibiotics are both incorrect. The patient has a normal white blood cell count and is afebrile, suggesting that his gallbladder is not acutely inflamed and he does not presently require antibiotic therapy. Admission to the hospital for immediate laparoscopic cholecystectomy is incorrect. The patient is stable and can wait for the procedure to be done electively. Referral to a surgeon for ERCP is incorrect. The patient's ultrasound shows a normal common duct. An ERCP is not indicated at this time.
A 23-month-old boy is brought in by his distraught mother. Both are crying. It seems that while crawling on the floor, the child found a penny and put it in his mouth. Before the mother could get to him, he swallowed it. She put her finger in the baby's mouth and could not find anything. She immediately drove to the emergency department. It has been about an hour since the ingestion. When you see the child, he is just finishing a 4 ounce bottle. When the mother takes it away, he cries. His color is pink. Pulse ox is 98%. Chest sounds are clear and present bilaterally. Heart sounds are clear, and he has a sinus tachycardia. Abdomen is soft and flat. There is no tenderness. On rectal examination, brown stool is found, and there is no blood. A plain x-ray shows an object about the size of a penny present in a slightly distended stomach. The lungs are clear, and the heart shadow is in the midline and appropriate size and shape. What is your next step? 1. Get an immediate surgical consult 2. Place a nasogastric tube 3. Intubate the child to prevent aspiration 4. Repeat an abdominal x-ray in an hour 5. Discharge the child and tell the mother to check his stool
Correct answer: Repeat an abdominal x-ray in an hour The critical issue in treating and managing swallowed foreign bodies is their size and their ability to pass through the pylorus. There are 5 areas of natural constriction in the pediatric age group where foreign bodies can get trapped: (1) cricopharyngeal narrowing at the level of C6; (2) the thoracic inlet; (3) aortic arch; (4) tracheal bifurcation; and, (5) hiatal narrowing. When the foreign body has passed the pylorus, it will almost always work its way down and be passed in the stool. Of course, this is contingent on the object's presence prior to GI surgery or anomalies. Hence, the second x-ray of the abdomen to confirm its passage beyond the pylorus is warranted. Some may argue that the second x-ray is not necessary and the mother can take the child home, checking the stool for the penny and returning only if it is not passed in 24 hours, but the safest thing to do is to confirm passage beyond the pylorus.
A 4-year-old boy presents with colicky pain, a history of irritability, and a 2-day history of lethargy. There is also history of rectal bleeding and passage of "currant jelly" stool for the past 2 days. Plain abdominal film shows evidence of obstruction, and barium enema detects coiled-spring appearance to the bowel. Intussusception is diagnosed. Investigations have revealed that the lead point causing the intussusception is an ileal lipoma. What is the definitive management in this case? 1 Reduction by air enema 2 Reduction by contrast enema 3 Observation for spontaneous reduction 4 Supportive therapy by rehydration 5 Resection of the bowel
Correct answer: Resection of the bowel Intussusception is a common cause of childhood intestinal obstruction in which there is invagination of a segment of the bowel (usually small bowel) into the lumen of a more distant bowel segment. Peristalsis causes the invaginated segment, called the intussusceptum, to be carried distally and along with it the mesentery and vessels also become involved with the intraluminal loop. This segment is squeezed within the engulfing segment, which is called the intussuscipiens. Most often the cause is idiopathic. However, in neonates and in children older than 3 years, a mechanical lead point is usually the cause. Lead points are nonidiopathic causes of intussusception and include Meckel's diverticulum, lymphoma or leukemia of the bowel, Peutz-Jeghers syndrome, polyps, lipomas, post-abdominal trauma, and Henoch-Schönlein purpura. Bowel resection and primary anastomosis are required when pathological lead points are present. Also, surgery may be required when intussusception occurs outside the ileocolic region. Delay in presentation also necessitates surgery, as reduction by air or contrast would be ineffective or associated with complications. Air enema reductions are less likely to succeed in patients less than 3 months old and in those more than 3 years old. Supportive therapy by rehydration is not the definitive management. Adequate rehydration and stabilization are required for all cases of intussusception. Reduction by contrast enema and observation for spontaneous reduction would not be beneficial in a child with a lead point presenting late.
A 35-year-old woman presents with early pregnancy symptoms. She missed her period and experienced fatigue; she has also been experiencing morning episodes of nausea and vomiting. However, pregnancy tests performed at home and in the office are negative. Ultrasound imaging supports a partial molar pregnancy diagnosis. Additional physical examination reveals pallor, bleeding gums, conjunctive inflammation, and corkscrew hair. A blood test will most probably indicate low levels of what compound? 1 Iron 2 Protein 3 Folic acid 4 Retinol 5 Vitamin C
Correct answer: Retinol The answer 'retinol' is correct. All the symptoms described can be associated with vitamin A deficiency. Vitamin A deficiency is also known to be associated with molar pregnancies. The answers 'iron' and 'folic acid' are incorrect. Even though skin pallor may indicate lack of either of these 2 elements, and bleeding gums can be seen in patients lacking folic acid, the other symptoms are not typically associated with iron or folic acid deficiencies; furthermore, iron and folic acid deficiencies are not known to cause molar pregnancy. The answer ' Vitamin C' is incorrect; other symptoms in addition to bleeding gums and corkscrew hair, such as follicular hyperkeratosis, are usually associated with lack of vitamin C. Also, a lack of vitamin C is not a major factor in the onset of molar pregnancies. The answer 'protein' is incorrect. Having too little protein does not cause a molar pregnancy. Symptoms induced by lack of protein (e.g., thin and easy to pull out hair, transverse lines on nails) were not observed.
A 30-year-old woman presents with a 2-week history of small wounds on the sides of her mouth. She recently adopted a strict vegetarian diet. On examination, you note she has pale conjunctivae, a magenta tongue, and macerated lips (in addition to the angular stomatitis). What is the most likely diagnosis? 1 Thiamine deficiency 2 Riboflavin deficiency 3 Vitamin K deficiency 4 Vitamin D deficiency 5 Iron deficiency
Correct answer: Riboflavin deficiency In cases of riboflavin (or Vitamin B2) deficiency, patients present with angular stomatitis and cheilosis. On examination, they are pale, have atrophic glossitis, and the tongue may appear magenta. They may also have a sebaceous dermatosis with greasy material in their nasolabial folds, alae nasi, and genitals. Causes include inadequate dietary intake. Dietary sources of riboflavin include milk, cheese, meat, and enriched cereal products. Vitamin B1 (or thiamine) deficiency results in beriberi, which is characterized by: A bilateral symmetric peripheral neuropathy beginning in the legs. Wernicke-Korsakoff syndrome, which is comprised of nystagmus, ophthalmoplegia, ataxia, memory loss, and confabulation. Congestive heart failure with tachycardia, peripheral edema, and cardiomegaly. In cases of vitamin K deficiency, patients present with bleeding tendencies, which include epistaxis, menorrhagia, and hematuria. The prothrombin time (PT) and the activated partial thromboplastin time (aPTT) are usually prolonged. In cases of vitamin D deficiency, children can present with inability to walk unsupported due to muscle weakness and lower limb skeletal deformities, such as genu varum and genu valgum. They may also have prominent costochondral junctions. In cases of iron deficiency, patients can present with feeling weak, dizzy, and tired; they may experience syncope. On examination, they have pale conjunctivae and koilonychia (spooning of the nails).
A woman brings in her exclusively breast-fed, 7-month-old male infant for a routine check up. You see a well-fed, African-American boy with pigeon breast, depression along the line of diaphragmatic insertion into rib cage, and costochondral thickening that looks like a string of beads. On an X-ray the bones are translucent, and the skull bones are thinning. What is the most likely diagnosis? 1 Osteomalacia 2 Primary hyperparathyroidism 3 Osteitis deformans 4 Rickets 5 Silverman syndrome
Correct answer: Rickets These symptoms are classical for rickets. Pigeon breast = pectus carinatum (sternum protrusion), Harrison's groove = depression along the line of diaphragmatic insertion into rib cage, rachitic rosary = costochondral thickening, craniotabes = thinning of skull bones, and radiolucency of bones. This child has 2 risk factors for rickets. His dark skin pigmentation interferes with the penetration of ultraviolet light in the skin, and he is exclusively breast-fed. Breast milk of African American women contains only about 35 IU/l vitamin D, which is far below the 400 IU/d recommended for infants by the American Academy of Pediatrics. Osteomalacia would be the right diagnosis for an adult showing these symptoms. Primary hyperparathyroidism is caused by either tumor or hyperplasia of the parathyroid glands. In this case, it would be secondary hyperparathyroidism caused by hypocalcemia. Osteitis deformans or Paget's disease is an idiopathic increase of osteoblast and osteoclast activity, usually in elderly people. It presents with diffuse fractures and bone pain. Silverman syndrome is congenital pigeon breast often combined with heart abnormalities.
A 69-year-old man with a 50-pack/year smoking history, COPD for 12 years, and a myocardial infarction 2 years ago has been experiencing increased exertional dyspnea for 4 months. There is associated easy fatigability, exertional chest discomfort, and lightheadedness. He denied fever, chills, palpitations, cough, wheezing, abdominal pain, nausea, vomiting, and diarrhea. Physical exam findings were remarkable for a right ventricular heave, widely split S2 with an accentuated pulmonic component, a pulmonary ejection click, an S3 and 1+ pitting edema to the bilateral lower extremities. There was also evidence of a 5 cm jugular vein distention. What diagnostic test results would be expected in this patient? 1. Underdevelopment of central pulmonary arteries and hyperemic lung field on chest X-ray 2. Right axis deviation, R wave greater than S wave in V1, and peaked p-waves on EKG 3. A normal FEV1 to FVC ratio on pulmonary function testing 4. Increased thickness of the left atrium on echocardiography 5. Pulmonary arterial pressure of 15 mm Hg on pulmonary artery catherization
Correct answer: Right axis deviation, R wave greater than S wave in V1, and peaked p-waves on EKG This patient's presentation is consistent with secondary pulmonary arterial hypertension. Common ECG findings include right axis deviation, R > S in V1, S1Q3T3, and peaked P waves. Common chest X-ray findings include enlarged hilar vessels that rapidly prune into the periphery, enlargement of central pulmonary arteries, attenuation of peripheral vessels, oligemic lung fields, and a right ventricle that fills the anterior airspace on lateral view. Given this patient's prior history of COPD, an obstructive pulmonary function test pattern is expected, not normal FEV1 to FVC ratios. Echocardiography reveals signs of chronic right ventricular pressure, such as increased thickness with paradoxical bulging of the septum into the left ventricle during systole. In later stages, right ventricular dilatation occurs, leading to hypokinesis. Right atrial dilatation and tricuspid regurgitation are also present. A mean pulmonary arterial pressure of > 25 mm Hg in the absence of an underlying disorder identifies pulmonary hypertension (most patients with pulmonary arterial hypertension present with substantially higher pressures).
A 44-year-old woman has symptoms of alternating constipation and diarrhea for 18 months. These symptoms are followed by rectal pain for the next 6 months. She seeks treatment, but she is prescribed only topical medicine and pain killers. She ends up with metastatic rectal cancer and dies within 2 months. Her 41-year-old sister is furious with what she conceives as poor medical care for her sibling and decides that she will not go through the same. What advice should you give the sister regarding screening for colon cancer? 1. Fecal occult blood testing annually only 2. Fecal occult blood testing annually until age 50 then screening colonoscopy every 5 years 3. Fecal occult blood testing annually until age 50 then sigmoidoscopy every 3 years 4. Fecal occult blood testing and double contrast barium enema right away then repeat every 5 years 5. Screening colonoscopy right away and repeat every 3-5 years if normal
Correct answer: Screening colonoscopy right away and repeat every 3-5 years if normal The recommendations for screening someone who has a first degree relative who has had colorectal cancer diagnosed at less than 60 years of age include screening colonoscopy starting at age 40 years, or 10 years younger than the earliest diagnosis in their family, whichever comes first. It should be repeated every 5 years. The same is advised for anyone with 2 first degree relatives with colorectal cancer at any age. Fecal occult testing annually alone is not a sensitive screening test for colon cancer. It should be used with sigmoidoscopy every 3-5 years, or with colonoscopy every 10 years for screening in the general population, starting at age 50.
A 40-year-old woman presents with rectal pain. She reports that the pain is especially severe with bowel movements, and on exam she is noted to have an anal fissure. What is the best initial treatment? 1 Application of hydrocortisone cream to the area 3 times a day 2 Stool softeners 3 Internal anal sphincterotomy 4 Hemorrhoidectomy 5 Fissurectomy
Correct answer: Stool softeners Anal fissures are usually caused by trauma to the anal canal due to the passage of hard stool. Most are found in the posterior midline of the anus, but some are found in the anterior midline. The ones found laterally may be associated with TB, syphilis, or AIDS. Hydrocortisone creams are used to decrease the inflammatory process usually associated with inflamed hemorrhoids. Anal fissures are initially treated conservatively using stool softeners and bulking agents to ease the passage of a bowel movement. If there is no improvement after 6 weeks of conservative therapy, surgery is usually required. The operation for non-healing anal fissures is an internal anal sphincterotomy. Fissurectomy is not a procedure used for the treatment of anal fissures. Hemorrhoidectomy is the operative resection of hemorrhoids, and it is not associated with the treatment of anal fissures.
A 38-year-old man presents with a 2-day history of a mass and severe pain in his scrotum. Physical examination reveals that his right testicle appears much larger than his left. On palpation, you note a small hole in his inguinal canal, and you are unable to place the contents into the canal. The contents of the hernia appear ischemic. What is the best description of the hernia? 1 Reducible 2 Recurrent 3. Irreducible 4 Strangulated 5 Incarcerated
Correct answer: Strangulated The clinical picture is suggestive of an inguinal hernia. The hernia has become ischemic, which is better known as a strangulated hernia. In a reducible hernia, the contents would be able to be placed back into the abdominal cavity with simple manipulation. A recurrent hernia is a hernia that has been previously repaired surgically and has now returned. There is no history of a previous hernia in this patient. An irreducible (also known as incarcerated) hernia is when the hernial contents cannot be returned to their normal site with simple manipulation. This type of hernia would cause edema and entrapment due to impaired venous return. An irreducible (also known as incarcerated) hernia is when the hernial contents cannot be returned to their normal site with simple manipulation. This type of hernia would cause edema and entrapment due to impaired venous return.
A 1-month-old formerly premature male infant with BPD remains intubated and monitored in the NICU. He has been doing relatively well and is being gradually weaned from the respirator. Suddenly, his O2 saturations and heart rate plummet, and he becomes very dusky. On quick exam, there are decreased breath sounds on the right with an asymmetric chest rise. What is the most likely explanation for his sudden respiratory and clinical change? 1. Inadequate tidal volume 2. A large leak around the tracheal tube 3. Displacement of the tracheal tube 4. Tension pneumothorax 5. Disconnected oxygen supply Submit Answer
Correct answer: Tension pneumothorax Air trappings, which can occur in patients with lower airway diseases such as BPD, may result in tension pneumothorax. Since the tracheal tube tends to enter the straighter right main bronchus, a tension pneumothorax most frequently occurs on the right side. This presents as a marked, sudden deterioration in oxygenation, as a decreased portion of the lungs is now being adequately ventilated, marked bradycardia, as the increasing pressure on the affected side of the chest directly impedes venous return to the heart, and central cyanosis. Breath sounds and chest rise will be decreased on the affected side due to increased pressure impeding air entry. Hyperresonance to percussion on the affected side and tracheal deviation away from the affected side may also be present, but it is better appreciated on a larger patient. Treatment consists of immediate needle decompression. Waiting for x-ray confirmation may prove fatal. An over-the-needle catheter of appropriate size for the patient is inserted in the intercostal space over the 3rd rib to avoid nerve structures in the midclavicular line. A gush of air and sudden improvement in oxygenation, cardiac output, and clinical status will be evident. Inadequate tidal volume being delivered would cause a slow and gradual deterioration that would be most evident on blood gas measurements. A large leak around the tracheal tube would be most evident by a hissing sound around the tube during the inspiratory phase. A large leak would also be unlikely in a small former premie with small airways. Displacement of the tracheal tube, though a common occurrence, would most commonly cause decreased breath sounds on the left because tracheal tube tends to enter the straighter right main bronchus. Indeed, this probably does precede a tension pneumothorax in most cases. Though a disconnected oxygen supply might cause a sudden change in clinical status, it would probably not be as marked, and it would not involve the findings of decreased breath sounds on the right and asymmetric chest rise.
A 4-year-old boy presents with bloody diarrhea, anemia, thrombocytopenia, fever, and lethargy following the exposure to the homemade hamburgers. His urine output is decreasing. What is the acute mechanism of kidney failure in this child? 1. Consumptive coagulopathy 2. Autoimmune coaguloptahy 3. Thrombotic microangiopathy 4. Segmental glomerulosclerosis 5. Tubulointerstitial nephritis
Correct answer: Thrombotic microangiopathy Based on the symptoms in the case, the patient is diagnosed with hemolytic uremic syndrome (HUS). E coli serotype O157:H7, which causes more than 80% of infections leading to HUS, produces shiga-like toxin. Toxin enters the circulation and causes the endothelial injury with the formation of arteriolar and capillary thrombi and red cell fragmentation. Thrombi damage glomerular filtration system in the kidneys and cause kidney failure; therefore, the progressive renal failure in HUS is the result of microangiopathic non-immune hemolytic anemia, and thrombocytopenia and the main mechanism is endothelial injury. Consumptive coagulopathy,which is also knowndisseminated intravascular coagulation (DIC), is a result of a pathological activation of coagulation cascade that causes intravascular clot formation. Signs and symptoms depend on the organ involved, but the most common is bleeding due to the consumption of platelets and clot factors in the blood. Consumptive coagulopathy is rarely seen in HUS. Autoimmune mechanism can also cause coagulopathy with bleeding symptoms, but the formation of autoantibodies is not the characteristic of the hemolytic uremic syndrome. Focal segmental glomerulosclerosis is caused by a variety of factors (infections, inflammations, toxins, hemodynamic and genetics) that cause nephrotic syndrome. Proposed mechanisms are either primary alteration of epithelial cells (idiopathic, viral-associated, drug-induced, and genetic) or secondary to reduction in nephron mass or hemodynamic adaptation (obesity, a single kidney, renal dysplasia/agenesis, reflux nephropathy, sickle cell disease, hereditary nephropathies, and other primary glomerular diseases). It is not characteristic for the HUS. In acute interstitial nephritis, renal tubular cells dysfunction is caused primarily by a hypersensitivity reaction to drugs or by infection. In hemolytic uremic syndrome, the initial damage is in the vascular endothelium.
A 62-year-old man presents with shortness of breath, some chest discomfort, and palpitations. Upon examination, his pulse rate is 200/ min; blood pressure is 100/75 mm Hg, and he has an oral temperature of 98.7°F. A 12-lead ECG examination reveals the wave patterns in the figure below. He has a history of chronic bronchiolitis. He has been taking drugs to control his condition for the last 6 months, since he quit smoking. What is the most likely cause of these symptoms? 1. Treatment with inhaled corticosteroids 2. Treatment with oral steroids 3. Treatment with a phosphodiesterase-4-inhibitor 4. Treatment with theophylline 5. Treatment with antibiotics
Correct answer: Treatment with theophylline Treatment with theophylline is the correct response. Theophylline is a bronchodilator medicine frequently used in the treatment of asthma and other airway disease including chronic bronchiolitis. Some of the side effects associated with its use are nausea, fast heartbeat, and tremor. The ECG result depicted in the figure illustrates a fast heartbeat condition called ventricular tachycardia, in which the ventricles are contracting faster than normal. This can be a consequence of long-term administration of theophylline, especially in elderly people. Treatment with inhaled corticosteroids is an incorrect response. Inhaled corticosteroids are used to reduce airway inflammation, but their side effects are different; they include oral infection and bruising. Treatment with oral steroids is an incorrect response. Oral steroids are prescribed to prevent worsening of chronic obstructive pulmonary disease (COPD). Their side effects are weight gain, diabetes, osteoporosis, cataracts, and an increased risk of infection; these are different from the symptoms of the examined patient. Treatment with a phosphodiesterase-4-inhibitor is an incorrect response. Phosphodiesterase-4 inhibitors, such as roflumilast, are used to decrease the inflammation of the airways. The most common side effects associated with their use are weight loss and diarrhea. Treatment with antibiotics is an incorrect response. Respiratory infections are generally viral infections that cannot be cured with antibiotics. Sometimes, they are used for their anti-inflammatory properties and to prevent bacterial infections that might worsen the condition; however, they do not cause cardiac problems.
A 60-year-old man presents with a 2-week history of nausea and several episodes of epigastric pain. The pain is characteristically dull and lasts for several hours. His symptoms are not worsened by any particular food or activity. What is the most appropriate initial test? 1. Upper endoscopy 2. Gastrointestinal series 3. CT Scan of the abdomen 4. Ultrasonography of the upper abdomen 5. MRI of the abdomen
Correct answer: Ultrasonography of the upper abdomen Ultrasonography (USG) of the upper abdomen is the 1st choice in the investigation of acute or subacute abdominal pain. A common cause of acute abdominal pain in the elderly is cholecystitis, which can be investigated through USG. Changes occurring in the biliary system because of aging make older patients susceptible to cholecystitis, the most common indication for surgery in this population. Differential diagnosis would include diverticulitis (the most common cause of severe abdominal pain in patients over the age of 50), as well as bowel obstruction, pancreatitis, and peptic ulcer disease. Catastrophic conditions (e.g., abdominal aortic aneurysm rupture and mesenteric ischemia) must also be considered. This patient's symptoms are not very intense (his symptoms have been present for 2 weeks) and, therefore, do not necessitate urgent invasive investigations. If ultrasonography of the upper abdomen shows no significant findings, then a number of other tests can be performed (e.g., an upper endoscopy to rule out peptic ulcer disease, a gastrointestinal series to rule out bowel obstruction, and a CT scan or MRI of the abdomen to rule out pancreatitis or diverticulitis); any other undetermined pathology may be undertaken to exclude other possibilities.
A 53-year-old man presents with increased difficulty swallowing and occasional regurgitation of his meals. His symptoms have been occurring with greater frequency and severity over the last 4 months. He also gets some shortness of breath, but attributes that to his weight and lack of physical activity. His past medical history is remarkable for chronic heartburn, which he treats intermittently with over the counter antacids. He takes no regular medications, and he has no allergies. He has not had any surgeries. He is smoker, but he denies use of alcohol and drugs. He works as a building inspector, and he lives with his wife and children. The patient is obese, but the rest of his physical exam is normal. Blood tests, electrocardiogram, and chest X-ray are done in the clinic; they are normal. He is referred for endoscopy, and esophageal biopsy shows specialized intestinal metaplastic cells (of columnar epithelium). What recommendation would best prevent complications from this patient's current condition? 1. Avoidance of antacids 2. Referral for fundoplication 3. Regular exercise 4. Use of proton pump inhibitors 5. Weight loss
Correct answer: Use of proton pump inhibitors This patient is presenting with Barrett's esophagus, a type of chronic esophagitis in which the normal squamous epithelium is replaced with columnar epithelium. Barrett's esophagus is a complication of chronic gastroesophageal reflux disease (GERD) and can develop into esophageal adenocarcinoma. Use of proton pump inhibitors (PPIs) reduces the risk of cancer. In order to monitor for the development of cancer, a routine endoscopy should be periodically performed in patients with known Barrett's esophagus. While the antacids are not necessarily adequate to control the GERD and damage to this patient's esophagus, avoidance of antacids will have no role in prevent subsequent Barrett's esophagus complications, such as adenocarcinoma. While the acid reflux appears to be a major mechanism for damage to the esophageal tissue, anti-reflux surgeries do not appear to prevent complications from Barrett's esophagus and subsequent development of adenocarcinoma. This patient does not need referral for fundoplication (surgical 'wrapping' of the stomach around the esophagus). Regular exercise would be helpful for this patient in general terms for cardiovascular health and weight reduction. However, exercise does not play a significant role in reducing risks for Barrett's esophagus complications. Obesity is a risk factor for development of Barrett's esophagus. (Other risk factors include male gender, smoking, and especially chronic GERD.) Once Barrett's esophagus has developed, however, weight loss does not dramatically alter the risk of complications, such as adenocarcinoma. Weight loss should be recommended for any obese patient, but the PPIs are the best approach to reduce complications from his Barrett's esophagus.
A 1-year-old boy presents with colicky abdominal pain, vomiting, and a 1-day history of irritability and lethargy. The child has vomited once, and the vomitus contained only food particles, without blood or bile. Rectal bleeding and "currant jelly" stool are also noted. The child has been healthy, and this is the first such episode since birth. Vitals were as follows: PB 110/70 mm Hg, PR 100/min, RR 20/min, temperature 98.8° F. On examination, a soft, slightly tender abdomen is noted. A vague vertical mass is palpable in the right upper quadrant. Other systems are normal on examination. Plain abdominal film shows evidence of obstruction, and barium enema detects a coiled-spring appearance to the bowel. Intussusception is diagnosed. What disease would you suspect if the child is under the age of 2 years and has hypertrophy of Peyer's patches? 1 Viral infection 2 Meckel's diverticulum 3 Peutz-Jeghers polyps 4 Juvenile polyposis 5 Lymphoma
Correct answer: Viral infection Intussusception is the 'telescoping' of bowel into the adjacent segment of the bowel. It is most common in male children of the age group 3 months-1 year. The triad of colicky abdominal pain, vomiting, and rectal bleeding in this age group is a reliable hint to the diagnosis. Rare presentations, such as those with neurological symptoms, subacute, or chronic forms, can also occur. The most commonly involved sites are the terminal ileum and ileocecum. A mechanical lead point can be identified in children above 2 years, such as abdominal trauma, hemangioma, Meckels diverticulum, and foreign body; however, in younger children it is assumed to be a previous viral infection causing hypertrophy of Peyer's patches. This leads to intussusception, strangulates venous return, and results in the classic consequences of bowel swelling, ischemia, and perforation. A contrast enema is both diagnostic and therapeutic. Reduction can be achieved in most cases. There is a chance of recurrence in 10% of children, which can be treated again by contrast barium enema.
A 60-year-old patient is referred to your office for colon cancer screening. He has a history of cardiovascular disease and had bypass surgery four years ago. His medications include metoprolol, ramipril, and simvastatin. He also takes an aspirin each day, but stopped a week ago on your nurse's advice. He is not allergic to any medications. Family history is negative for gastrointestinal cancer. Social history reveals that he is not a smoker. He works in an office setting. Review of systems is negative for abdominal pain, rectal bleeding, loss of appetite, weight loss, nausea, or dyspepsia. He had three hemoccult cards done by his primary physician, all of which were negative for occult blood. You discuss the options regarding colon cancer screening, and the patient agrees to have a flexible sigmoidoscopy. As is standard practice, you review the risks and benefits with the patient so he can make an informed decision about signing the consent form. The patient agrees to proceed. As you look through the eyepiece and attempt to advance the fiberoptic instrument, the finding most indicative of a severe complication would be 1. Difficult advancement and visible "red out" 2. Visible bowel serosa and mesenteric fat 3. Advancement obstructed with "white out" 4. "Sliding by" an apparent bowel wall spasm 5. Completely black picture without any light
Correct answer: Visible bowel serosa and mesenteric fat A flexible sigmoidoscope is a fiberoptic instrument used to screen patients for colon cancer. Many authorities recommend routine screening starting at age 50-60. If there are no symptoms or family history, most experts recommend performing a combination of three hemoccult cards and flexible sigmoidoscopy every five years. If there are any danger symptoms (rectal bleeding, positive hemoccults, nausea, weight loss) then colonoscopy should be considered. Sigmoidoscopy is a fairly safe procedure, but there are some potential complications. The most feared complication is bowel wall perforation, where the instrument suddenly views bowel serosa (not intraluminal mucosa!) and peritoneal mesentery. This complication requires emergency surgery. A "red out" occurs when the scope is difficult to advance and the view becomes red. This indicates that the scope has been pressed up against the wall of the colon and red vasculature is all you can see. A "white out" occurs when the scope has been pressed up against the wall of the colon so hard that the vasculature blanches, turning the screen white. This is extremely dangerous, and signifies impending bowel perforation. A completely black view simply indicates that the fiberoptic lens (or the light source) at the tip of the instrument has been covered by debris, typically retained stool. Most instruments have the ability to clean the lens with pressurized water and air. Sliding by a bowel wall spasm is often performed by experienced clinicians, but requires a delicate touch to minimize the risk of perforation. The tip of the instrument is angled and advanced so that it bluntly pushes its way past an obstruction.
A 32-year-old woman presents with a pins and needles sensation on the sides and the bottom of her feet, as well as severe burning sensation in the feet. The burning sensation manifests primarily at night. Examination reveals a profoundly cachectic but alert patient with atrophy of muscles of the leg. Examination of the eye indicates that she also has nystagmus. Further questioning reveals that she has a history of homelessness and chronic alcoholism. What vitamin deficiency is most commonly associated with chronic alcoholism? 1. Vitamin C 2. Vitamin B1 3. Niacin 4. Vitamin E 5. Vitamin B6
Correct answer: Vitamin B1 Wernicke's encephalopathy is a result of the inadequate intake or absorption of thiamine (Vitamin B1) coupled with continued carbohydrate ingestion. It is most common among chronic alcoholics who suffer from poor nutrition. It is therefore imperative that all chronic alcoholics receive a dose of thiamine before they receive any glucose solution IV. It is also seen in patients on dialysis and during hyperemesis gravidarum of pregnancy. Clinical features appear acutely and include ataxia, confusion, nystagmus, partial ophthalmoplegia, apathy, confusion, severe memory loss, as well as autonomic dysfunction. Treatment of suspected or borderline cases consists of giving 5 - 30 mg of thiamine orally as a single dose for a period of 1 month. Parenteral thiamine of 50 - 100 mg is given to prevent Wernicke's encephalopathy, and it is also used in its treatment. Thiamine has to be given as IV, especially before administering fluids containing glucose. Rehydration of the patient along with supplementation of electrolytes and nutritional therapy is also required. Advanced cases require hospitalization and close supervision. Other vitamins deficiencies are not commonly associated with chronic alcoholism.
What deficiency causes Pellagra-diarrhea, dermatitis, and mental disturbances? 1. Vitamin A 2. Vitamin B1 (thiamine) 3. Vitamin B2 (riboflavin) 4. Vitamin B3 (niacin) 5. Pantothenic acid
Correct answer: Vitamin B3 (niacin) Vitamin Symptoms of Deficiency A (retinoids, carotenoids) Rhodopsin deficiency, night blindness, retarded growth, skin disorders, and increased risk of infections B1 (thiamine) Beriberi-muscle weakness (including cardiac muscle), neuritis, and paralysis B2 (riboflavin) Eye disorders and skin cracking, especially at corners of the mouth B3 (niacin) Pellagra-diarrhea, dermatitis, and mental disturbances Pantothenic acid Neuromuscular dysfunction and fatigue
A 32-year-old woman presents with a 1-month history of bleeding gums when brushing her teeth. She also reports that her wounds are taking longer than usual to heal. She is a stay-at-home mother and is breastfeeding her 6-month old twins. On examination, you note multiple splinter hemorrhages on her nails and ecchymoses over her lower limbs. What is the most likely diagnosis? 1. Vitamin A deficiency 2. Pyridoxine deficiency 3. Vitamin C deficiency 4. Niacin deficiency 5 Vitamin E deficiency
Correct answer: Vitamin C deficiency In cases of vitamin C (or ascorbic acid) deficiency, patients can present with bleeding tendencies (as a result of weakened capillaries) and impaired wound healing due to impaired formation of connective tissue. On examination, the gums may be swollen and friable; the teeth may be loose. There may also be multiple splinter hemorrhages on the nails and ecchymoses, especially over the lower limbs. Causes include inadequate dietary intake and certain conditions, such as pregnancy and lactation, which increase vitamin C requirements. Dietary sources of vitamin C include citrus fruits, such as oranges, lemons, and tangerines, as well as tomatoes and potatoes. Vitamin E deficiency may cause a hemolytic anemia in premature infants. Laboratory investigations reveal low plasma tocopherol levels, a low hemoglobin level, reticulocytosis, hyperbilirubinemia, and creatinuria. Causes of vitamin E deficiency in premature infants include limited placental transfer of vitamin E and the resultant low levels at birth combined with its relative deficiency in the infant diet. Dietary sources for older children and adults include wheat germ, vegetable oils, egg yolk, and leafy vegetables. In cases of vitamin A deficiency, patients can present with inability to see well in dim light or night blindness. There may also be conjunctival and corneal xerosis, as well as pericorneal and corneal opacities, and Bitot's spots. Bitot's spots are a collection of keratin appearing as triangular foamy spots on the conjunctiva. Patients may also have xeroderma, hyperkeratotic skin lesions, and increased susceptibility to infections. Causes include inadequate dietary intake and malabsorption. Dietary sources of vitamin A include fish, liver, egg yolk, butter, cream, dark green leafy vegetables, as well as yellow fruits and vegetables. Niacin deficiency causes pellagra, which is characterized by: A symmetrical dermatitis, usually on parts of the body exposed to sunlight Scarlet glossitis and stomatitis Diarrhea Mental aberrations, such as memory impairment, depression, and dementia. These may appear alone or in combination. Causes include inadequate dietary intake, especially in patients with corn-based diets or alcoholism. Dietary sources include legumes, yeast, meat, and enriched cereal products. In cases of pyridoxine or (vitamin B6 deficiency), patients can present with peripheral neuropathy, seborrheic dermatosis, glossitis, and cheilosis. Laboratory investigations reveal anemia with lymphopenia. Causes include malabsorption as well as medications, such as isoniazid and penicillamine. Dietary sources of vitamin B6 include liver, legumes, whole grain cereals, and meats.
A 38-year-old man presents with sudden onset of acute upper abdominal pain since the previous night associated with nausea, several episodes of vomiting, and weakness. The pain is mostly in the epigastric region with constant, severe, and steady radiation to the back. He also has a low grade fever since this morning without any chills. He denies diarrhea or dysuria. His past history is significant for hypertension, for which he takes amlodipine 10 mg daily and enalapril 5 mg daily. He has smoked half a pack of cigarettes daily for the past 12 years. He initially denies drinking alcohol except on the weekends occasionally, but on further and repeated questioning he says he drinks 3-4 beers daily and had been drinking continuously for the last 2 days with his friends while watching sports on TV. Family history is unremarkable. On examination he has a temperature of 100.6° F, pulse 106/minute, BP 150/92 mm Hg, and respiratory rate is 20/minute. There is no pallor, icterus, cyanosis, or lymphadenopathy. Mucus membranes are dry, and skin is somewhat clammy. Lungs are clear, and heart sounds normal except for sinus tachycardia. Abdominal exam reveals diffuse tenderness in the epigastric and right as well as left upper quadrants. There is some distension and mild guarding in the upper abdomen. Bowel sounds are hypoactive, but there is no ascites or hepatosplenomegaly. Rectal exam is normal. Labs reveal Hb 15g%, WBC 14,500/uL, platelets 400,000/uL, AST 42 U/L, ALT 36 U/L, AP 26 U/L, amylase 3600 U/L, lipase 546 U/L, BUN 25 mg/dL, creatinine, 1.5 mg/dL, bilirubin 1.2 mg/dL, and random blood sugar 110 mg/dL. Which of the following is one of the predictors of acute pancreatic necrosis if present at diagnosis along with 2 other factors? 1. Age over 50 years 2. WBC count more than 16,000/uL 3. Blood glucose over 180mg/dL 4. Serum LDH over 300 U/L 5. AST more than 200 U/L
Correct answer: WBC count more than 16,000/uL This patient is suffering from acute alcoholic pancreatitis. The 2 most common causes of acute pancreatitis are gallstones and alcohol. Other causes include hypertriglyceridemia, hypercalcemia, abdominal trauma, ERCP, and drugs like valproic acid, azathioprine, mercaptopurine, didanosine, thiazides, tetracyclines etc. Assessment of severity is done by either Ranson's criteria on admission and at 48 hours or by the Acute Physiology and Chronic Health (APACHE) II scoring system. Ranson's criteria include: (1) Age more that 55 years (2) WBC count more than 16,000/Ul (3) Blood glucose more than 200mg/dL (4) Serum LDH over 350 U/L (5) AST over 250 U/L. 3 or more criteria on admission predict a complicated course with possibility of pancreatic necrosis. At 48 hours development of any of the following indicates a worsening prognosis: (1) Hematocrit drop of more than 10% (2) BUN rise greater than 5 mg/dL (3) Arterial PO2 less than 60 mm hg (4) Serum calcium less than 8 mg/dL (5) Base deficit over 4 meq/L (6) Estimated fluid sequestration of more than 6 L. An elevated C reactive protein at 48 hours suggests the development of pancreatic necrosis. A high amylase and lipase are suggestive of acute pancreatitis. Leukocytosis is usually present. Other than the labs ordered above, imaging may also be done. Plain abdominal X-ray may show radio-opaque gallstones, the sentinel loop sign (localized ileus of a small segment of small intestine, usually in the left upper quadrant), and the colon cut-off sign (lack of air in the colon in the area of the inflamed pancreas immediately preceded by a gas filled segment of transverse colon). There may be a reactive pleural effusion with atelectasis in the lower lobes of the lungs. Ultrasound is non specific and may show gallstones. CT scan of the abdomen will show the inflamed pancreas and detect complications like necrosis or pseudocyst formation. Since Ranson's criteria and APACHE scoring systems are cumbersome and time consuming, a CT Severity Index (CTSI) has recently become popular. It uses a grading system based on unenhanced CT of the pancreas and a necrosis score based on contrast enhanced CT of the pancreas. CTSI is the unenhanced score plus the necrosis score, the maximum of which can be 10 and more than 6 indicates severe disease. MRI and MRCP are being increasingly used where available for diagnosis and management of pancreatitis. Treatment involves bowel rest, aggressive hydration, pain control, and bed rest in a hospital. Electrolyte imbalances should be corrected, especially calcium since saponification may lower the level. Broad spectrum antibiotics are not routinely recommended, but in patients with impending necrotizing pancreatitis, antibiotics have shown to decrease mortality. Nutritional support is of utmost importance. For mild to moderate cases, IV hydration and gentle advancement of oral feeding is recommended, where as in severe cases total parenteral nutrition may be needed. Alcohol abstinence will be needed to prevent further episodes. Complications include prerenal azotemia, acute tubular necrosis, shock, pancreatic necrosis, pseudocyst formation, ARDS, and pancreatic abscess. Intra-abdominal hemorrhage may cause ecchymoses around the umbilicus (Cullen's sign) or in the flanks (Grey-Turner sign), though these are rare and not specific for pancreatitis. Prognosis is good for mild cases who abstain from drinking. Recurrences are common in alcoholics. Prognosis is poor for severe necrotizing pancreatitis, especially with multi-organ involvement.
When is surgery necessary for gastric polyps? 1. For all polyps larger than 2 cm 2. When the question of cancer cannot be completely answered by endoscopic biopsy 3. When the polyp is pedunculated and greater than 2 cm in size 4 When there are multiple polyps 5 For ulcerated bleeding polyps only
Correct answer: When the question of cancer cannot be completely answered by endoscopic biopsy Gastric polyps are often found by barium x-ray studies or at the time of upper endoscopy. Polyps which are small can be easily biopsied, and pedunculated polyps, even those larger than 2 cm in diameter can be excised by use of a snare. Lesions which are sessile do not lend themselves to snare excision endoscopically, and large sessile polyps may harbor cancer not found in the forceps biopsy specimen. When patients have multiple large polyps or have polyposis, it is difficult to excise all the polyps to ensure that none of them are harboring cancer. Bleeding polyps are not an absolute indication for surgical excision. When there is a question as to the presence of cancer, surgery is indicated for resection and possible cure of early gastric carcinoma.
A 45-year-old man presents with fatigue and hair loss that began about 2 weeks ago. His past medical history is significant for alcoholism and type II diabetes mellitus. He states that over the past 2 weeks, he has begun feeling more and more tired, and he has noticed that he was losing much more hair than usual. He denies pain, nausea, vomiting, fever, and chills; he states that he had been in his usual state of health until 2 weeks ago. He also denies recent illness and travel. He states that he now gets tired after very minimal activity and has poor wound healing. He admits that his diet is poor and that he does not control his blood glucose very well. Physical examination reveals a thin, frail man in no acute distress. He has an ulcerous lesion on his left heel that is draining but does not appear to be infected. Blood work including urinalysis, CBC, and WBC were within normal limits. What deficiency does this patient most likely have? 1. Niacin 2. Zinc 3. Folate 4. Vitamin B-12 5. Vitamin B-1
Correct answer: Zinc This patient most probably has a zinc deficiency. The causes of zinc deficiency include malnutrition, chronic debilitating diseases, chronic renal disease, alcoholism, drugs such as penicillamine and diuretics, and genetic disorders, such as sickle cell disease. Clinical manifestations in severe cases include alopecia, diarrhea, weight loss, infections, dermatitis, hypogonadism in men, and intercurrent infections. Supplementation with zinc is the treatment of choice. Niacin (nicotinamide, nicotinic acid) deficiency is uncommon in the United States. It is often found in people who live on a diet that consists mainly of corn. Deficiency may also result from alcoholism, cirrhosis, or diarrhea. Men and women are affected equally. Symptoms of niacin deficiency include nausea, vomiting, diarrhea, rash, glossitis, stomatitis, depression, and psychosis. Niacin deficiency, also known as pellagra, manifests as the '3 Ds': diarrhea, dermatitis, and dementia. If this diagnosis is suspected, supplemental niacin should be given immediately. Some patients deficient in niacin may also be deficient in other vitamins, such as B vitamins, so they should be given that as well. Folate deficiency causes megaloblastic macrocytic anemia, as folate plays a key role in nucleic acid synthesis. The early manifestation of folate deficiency, especially in its suboptimal state, predisposes to occlusive vascular disease and thrombosis. These manifestations are linked to increased homocysteine levels found in folate deficiency. Neurological and immunological disturbances may also be seen. Folate replacement is the option to prevent and to treat the deficiency. Vitamin B1 (or thiamine deficiency) causes beri beri, occurring mostly in the malnourished and alcoholics. The deficiency manifests with acute heart failure, neurologic deficits, and epilepsy. Empiric use of thiamine and prophylactic use in high-risk population is strongly recommended even before blood reports are obtained, as the treatment is inexpensive and prevents major catastrophes. Vitamin B12 or cobalamin deficiency manifests as megaloblastic macrocytic anemia, pancytopenia, and a spectrum of neurological disorders such as peripheral neuropathy, parasthesias, and demyelination of corticospinal tract. Nutritional deficiency, alcoholism, and malabsorption syndromes are some causes of B12 deficiency. It also is associated with homocystenemia and atherosclerosis. Diagnosis is by serum estimation of B12, and oral supplementation is safe and effective, but intramuscular injections may also be used.
A 15-year-old girl presents with a 1-year history of intermittent abdominal pain with nausea and occasional bloody diarrhea. She denies fever or weight loss, as well as any travel history. Past medical history is significant only for migraines. She takes a multivitamin. Her vital signs are within normal limits. She has mild diffuse abdominal tenderness to palpation and guaiac-positive stool. Her exam is otherwise normal. Her hemoglobin is 9.7, hematocrit is 28%, and her WBC is 12,000/uL. What finding would indicate a possible cure if a colectomy is performed? 1 Skip lesions, transmural involvement on colonoscopy 2 Crypt abscesses, superficial mucosal involvement on colonoscopy 3 Ileal disease 4 Fistulas, fissures 5 Mouth ulcers
Crypt abscesses, superficial mucosal involvement on colonoscopy Crypt abscesses and superficial mucosal involvement are colonoscopy findings associated with ulcerative colitis. A colectomy is performed for severe and intractable disease. A colectomy can be a relative cure since the disease is limited to the colon. Skip lesions, transmural involvement, fistulas, and fissures are all associated with colonoscopy findings in Crohn's disease. Crohn's disease can also affect any structure of the alimentary tract from the mouth to anus, causing ileal disease and mouth ulcers.
A 40-year-old man presents with a 2-year history of severe, burning epigastric pain. A detailed history reveals that the pain is greatest in the early hours of the morning and wakes him up from sleep. The pain is also felt 2-3 hours after meals. He reports diarrhea for the past 2 years and black stools for the past 3 days. On examination, his pulse is 74/min and blood pressure 136/84 mm Hg. There is slight epigastric discomfort on palpation. Lab examination shows occult blood in the stool and hyperchlorhydria. What could be a complication in this patient? 1. Gallstones 2. Intestinal ulcers 3. Pernicious anemia 4. Intestinal alkalosis 5. Weight gain
Intestinal ulcers The diagnosis in this case is gastrinoma (Zollinger-Ellison disease). Gastrinomas are small tumors that continually release excessive amounts of gastrin. Hypergastrinemia has a trophic effect on the gastric mucosa and enhances the secretion of hydrochloric acid from gastric parietal cells in both basal and stimulated states. The continued delivery of acid to the upper small intestine exceeds the capacity of acid-neutralizing mechanisms and results in the erosion and ulceration of the intestinal mucosa. In addition to acid secretion, parietal cells release intrinsic factor in response to gastrin. Intrinsic factor is a glycoprotein required for the normal intestinal absorption of vitamin B12. Red blood cell maturation is retarded in the absence of intrinsic factor resulting in pernicious anemia. Intestinal acidosis in patients with hypergastrinemia also reduces the activity of pancreatic enzymes, particularly lipases. This inhibits digestion and reabsorption of fat resulting in steatorrhea (fat in the feces) and weight loss. Gastrinomas may be either sporadic or associated with autosomal dominantly inherited Multiple Endocrine Neoplasia (MEN) type I.
A 45-year-old woman presents for initial history and physical at her new primary care provider's office. She is currently asymptomatic, but based on her sexual history with multiple partners, you decide to obtain screening hepatitis lab tests in addition to other screening tests. The following results are obtained: Hep A Ab: negative HBcAb: positive HBeAg: negative HBsAg: positive HBsAb: positive Hep C Ab: positive Hep D Ab: negative What is the most likely diagnosis based on these results? 1 She has not been infected with or been vaccinated against Hepatitis B virus but has been exposed to Hepatitis C 2 She has been vaccinated against Hepatitis B in the past but is not currently infected with Hepatitis B nor exposed to Hepatitis C 3 She has had Hepatitis B infection in the past, is currently infected with Hepatitis B, and has been exposed to Hepatitis C 4 She has had Hepatitis B infection in the past but is not currently infected and has been exposed to Hepatitis C 5 She has been vaccinated against Hepatitis B but has not been infected with it and has been exposed to Hepatitis C
She has had Hepatitis B infection in the past, is currently infected with Hepatitis B, and has been exposed to Hepatitis C HBcAb, HBsAg, and HBsAb become positive with Hepatitis B virus infection, but over time HBsAg will clear from the serum only if th e patient is not chronically infected. These results demonstrate chronic infection, as all three are positive. She also shows exposure to Hepatitis C, as the surface antibody is positive. The remaining answers are incorrect because vaccination alone will cause only the HBsAb to become positive. All Hepatitis B results would be negative if the patient had not had Hepatitis B virus infection or been vaccinated for Hepatitis B virus. Also, only HBsAb would be positive if the patient was never infected but had received the vaccine.
A 64-year-old man presents with a 3-day history of insidious chest pain. He has a past medical history of hypertension, coronary artery disease, and poorly controlled left ventricular congestive heart failure due to medication noncompliance. Pain is made worse when he takes a deep breath in and when he coughs. He denies any relation of pain to position, activity, or food intake. He denies fever, chills, palpitations, sputum production, wheezing, shortness of breath, abdominal pain, nausea, vomiting, diarrhea, and peripheral edema. His physical exam reveals no respiratory distress, cyanosis, or accessory muscle usage. There are bibasilar thoracic friction rubs upon inspiration, an absence of lung fremitus, dullness to percussion, and reduced lung sounds. A chest X-ray is performed; it reveals the following image. What health maintenance approach should be recommended at this time? 1. Aggressive restriction of dietary fat intake 2. Explain the necessity of an emergent and repeated thoracentesis 3. Counseling regarding compliance with heart failure medications 4. Prophylactic use of broad-spectrum antibiotics 5. Emergent evaluation of shortness of breath is unnecessary
Correct answer: Counseling regarding compliance with heart failure medications Explanation The patient should be counseled regarding compliance with heart failure medications. This patient's underlying condition is a transudative pleural effusion, most likely due to poorly controlled congestive heart failure. Transudative effusions are usually managed by treating the underlying medical disorder. Observation of pleural effusion is reasonable when benign etiologies are likely, as in the setting of overt congestive heart failure, viral pleurisy, or recent thoracic or abdominal surgery. Restriction of fat intake may help in the treatment of chylous effusions, which is classified as an exudative effusion characterized by elevated triglyceride levels in pleural fluid. Thoracentesis should be performed for new and unexplained pleural effusions when sufficient fluid is present to allow a safe procedure. Patients with poor performance status (Karnofsky score < 70) and a life expectancy of less than 3 months can be treated with repeated outpatient thoracentesis as needed to palliate symptoms. Antibiotics should be administered early in patients with parapneumonic effusions, empyemas, and effusions associated with esophageal perforation and intra-abdominal abscesses. Patients should be instructed to seek early assessment and intervention for shortness of breath, especially if there is chronic underlying lung disease.
A 60-year-old man with history of heavy smoking and moderately severe chronic obstructive pulmonary disease (COPD) has been feeling weak recently. He notes a 3- to 4-day history of cough, chills, pleuritic chest pain, and low-grade fever. Chest X-ray shows a small dense infiltrate in the right lower lobe. Gram stain of the patient's sputum reveals numerous Gram-negative cocci, many of which occur in pairs. What is the most appropriate treatment in this case? 1. No antimicrobial treatment 2. Tetracycline 3. Ciprofloxacin 4. Ampicillin 5.Amoxicillin and clavulanic acid
Correct answer: Amoxicillin and clavulanic acid Explanation The correct response is amoxicillin and clavulanic acid. The history, along with the X-ray findings, suggests that the patient is suffering from lobar pneumonia. In addition to S. pneumoniae and H. influenzae, the Gram-negative coccus Moraxella (Branhamella) catarrhalis is a common cause of exacerbation of chronic bronchitis and pneumonia in patients with moderately severe COPD. Lower respiratory tract infections due to M. catarrhalis have been shown to be associated with smoking. The symptoms are typical with chills, pain, and malaise, but sometimes low-grade fever and lack of leukocytosis can be seen. The X-ray appearance can be variable, and sometimes the clinical parameters do not determine the organism causing the illness in a heavy smoker with COPD. The Gram stain in this case shows the abundant presence of Gram-negative cocci in pairs, which is typical of M. catarrhalis. M. catarrhalis is a Gram-negative, oxidase-positive diplococci, which is also called Branhamella catarrhalis. It causes bronchitis or pneumonia in children or adults with underlying lung pathology. It also occasionally causes bacteremia and meningitis, especially in immunocompromised individuals. As most strains produce beta-lactamase, penicillin, ampicillin, or amoxicillin will only be effective against the strains not producing beta-lactamase. However, since resistance to both ciprofloxacin and tetracycline has been reported, this pneumonia will require treatment with the most effective agent. Amoxicillin-clavulanic acid, 2nd and 3rd-generation oral cephalosporins, trimethoprim, and sulfamethoxazole are all effective. Therefore, the appropriate choice is the combination of amoxicillin and clavulanic acid, which will suppress the M. catarrhalis lactamase.
A 30-year-old woman with no significant past medical history is evaluated for a 3-month history of exertional dyspnea and increased fatigue; it is associated with chest discomfort and lightheadedness. Her physical exam reveals a right ventricular heave, a widely split S2, an accentuated pulmonic component of S2, a pulmonary ejection click, an S3 gallop, and a tricuspid regurgitation murmur. There is jugular vein distention and liver enlargement, but no peripheral edema. Pulmonary auscultation is normal. An EKG is performed; the tracing is attached. An inhaled vasodilator challenge results in a favorable response. What is the most appropriate initial therapy in this case? 1. Digoxin 2. Epoprostenol 3. Ambrisentan 4. Metoprolol 5. Nifedipine
Explanation This patient's most likely diagnosis is (IPAH). Calcium channel blockers, such as nifedipine, have been used to treat IPAH. Calcium channel blockers (CCBs) are believed to act on the vascular smooth muscle, dilating the pulmonary resistance vessels and lowering the pulmonary artery pressure. Several studies report clinical and hemodynamic benefits from the use of long-term calcium channel blockade. Long-term treatment improves the quality of life and survival rate in patients who have a proven response to such therapy. In general, CCBs are used at high doses in patients with IPAH. Cardiac glycosides are used for prevention and treatment of supraventricular arrhythmias associated with SPAH and for patients who have concomitant left-sided heart failure. Digoxin is not useful in the treatment of right-sided ventricular failure. Only patients with an acute vasodilator response to an intravenous or inhaled pulmonary vasodilator challenge (e.g., with inhaled nitric oxide at 10 to 20 parts per million, intravenous epoprostenol (2 to 12 ng/kg/min), intravenous adenosine (50 to 350 mg/min), or inhaled iloprost [5 mg]) derive any long-term benefit from CCBs. Parenteral vasodilators such as epoprostenol are used for patients whose IPAH fails to respond to calcium channel blockers or who cannot tolerate these agents and who have New York Heart Association (NYHA) type III or IV right-sided heart failure. Ambrisentan is an endothelin receptor antagonist indicated for pulmonary arterial hypertension in patients with WHO class II or III symptoms. It improves exercise ability and decreases progression of clinical symptoms. It inhibits vessel constriction and elevation of blood pressure by competitively binding to endothelin-1 receptors ETA and ETB in endothelium and vascular smooth muscle. Because of the risks of hepatic injury and teratogenic potential, this agent is available only through the Letairis Education and Access Program (LEAP). Beta-blockers, such as metoprolol, have no established role in the treatment of IPAH.
A 61-year-old male presents with a recent history of increased fatigue with mildly increased exertional dyspnea. Patient denies any significant past medical history but states that he had some heart problems as a child, though he was never clear as to what was the problem. On cardiac examination, you hear an early diastolic, soft decrescendo murmur with a high pitch quality, especially when patient is sitting and leaning forward. No thrill is felt. Based on this patient's presentation, you expect the patient to have... 1. Tricuspid stenosis 2 Aortic regurgitation 3 Mitral stenosis 4 Mitral valve prolapse 5 Pulmonic stenosis
The correct answer is aortic regurgitation, as it presents as a soft, early diastolic, high-pitched murmur heard best when sitting and leaning forward. It is often a result of rheumatic heart disease, which may be inferred by the patient's history.
A 41-year-old woman presents for follow-up regarding elevated blood pressure. This is her third visit to the office, and her blood pressure has been elevated on multiple readings at each visit. She has a history of eczema but is otherwise healthy. Labs reveal the following: WBC: 14.5 k/uL Hgb: 13.5 g/dL HCT: 41% PLT: 152 k/uL Na: 135 mmol/L K: 2.8 mmol/L Cl: 99 mmol/L CO2: 32 mmol/L BUN: 10 mmol/L Cr: 1.02 mmol/L What lab abnormality is most likely to be causing her secondary htn?
*Hypokalemia is correct.* Patients with primary hypertension should not have abnormalities in serum electrolytes. The patient's hypokalemia suggests a secondary cause of hypertension. *The most common cause of secondary hypertension is renal artery stenosis caused by fibrosmuscular dysplasia.* If the adult is middle-aged, then the most common secondary cause of hypertension would be *aldosteronism and a highly recommended initial diagnostic test would be an aldosterone/renin ratio.* Age is incorrect. This patient is 41 years old. Secondary hypertension is more likely in patients under 30 years old or over 50 years old. Female gender is incorrect. Primary hypertension is more common in middle-aged males than middle-aged females. The gender distribution for secondary hypertension varies with the specific cause of secondary hypertension. Hypokalemia is the most suggestive risk factor of secondary hypertension in this patient. Eczema is incorrect. Eczema is not a risk factor for secondary hypertension Leukocytosis is incorrect. Leukocytosis is not associated with secondary hypertension.
A 20-year-old woman presents for counseling after being diagnosed as a carrier of Emery-Dreifuss muscular dystrophy. She manifests a mild form of the disease, with only contractures of the Achilles' heels and elbows. Both her brother and her father have been diagnosed with the disease. What test will help to change the course of the disease in this young woman? 1 Electrocardiography 2 Creatine kinase 3 Electromyography 4 Muscle biopsy 5 Antibodies to emerin
An electrocardiogram (ECG) should be obtained in all Emery-Dreifuss muscular dystrophy patients, including female carriers. Conduction defects may occur even with minimal musculoskeletal and joint involvement. It is estimated that 10 - 20% of female carriers have atrial arrhythmias or conduction defects and need to be monitored with yearly ECG to try to prevent sudden cardiac death. Early ECG changes include low amplitude P waves and a prolonged PR interval that progress to bradycardia, absent P waves, irregular atrial rhythm, atrial fibrillation/flutter, AV-conduction defects, and a late cardiomyopathy. There is no electrical and mechanical activity of the atria; the myocardium, not the conduction system, is affected. No specific treatment for Emery-Dreifuss muscular dystrophy exists. Supportive care to preserve muscle activity and functional ability, as well as the treatment of cardiac and/or pulmonary complications, are currently the only options. Early pacemaker implantation can change the course of the disease.
While doing rounds one morning, you come upon a 42-year-old man suspected of having an infective endocarditis and currently undergoing an extensive workup. Which of the following represents the most definitive diagnosis of Infective Endocarditis based on Modified Duke Criteria? 1. 1 positive blood culture with Staphylococcus aureus with Osler's nodes and Roth spots 2. 2 positive blood cultures with Streptococcus pneumoniae with cutaneous hemorrhages and glomerulonephritis 3. Evidence of endocardial vegetation on echocardiography with Osler's nodes 4. 2 positive blood cultures with Staphylococcus aureus and development of a new regurgitant murmur 5. Fever >100.4 degrees Fahrenheit (38 degrees Celsius) with evidence of endocardial vegetation on echocardiography and glomerulonephritis
Clinical criteria is also known as the Modified Duke criteria and is widely utilized to establish the diagnosis of endocarditis. The criteria are classified as either Major criteria (two positive blood cultures for a microorganism that is typical to cause endocarditis; evident of endocardial involvement via an echocardiogram ((vegetation, abscess)); development of a new regurgitant murmur. Minor criteria include: vascular phenomena (skin hemorrhages, emboli, aneurysms, or pulmonary infarction), fever >100.4 degrees Fahrenheit (38 degrees Celsius), immunologic phenomenon (glomerulonephritis, Osler's nodes, Roth spots, rheumatoid factor), and positive blood cultures that do not meet the specifics of the major criteria. The correct answer is 2 positive blood cultures with Staphylococcus aureus and development of a new regurgitant murmur. Since two Major criteria are identified, a definitive diagnosis of infective endocarditis can be made with 80% accuracy. The presence of one major criterion and three minor criteria or even if there are five minor criteria listed can also qualify in this 80% accuracy diagnosis range. The diagnosis is possible but not highly likely to be infective endocarditis if the patient displays the following: one major and one minor criterion or three minor criterions are met. Any less than these should lead a healthcare provider to suspect a different diagnosis. Choice A only has one minor criterion; Choice B has one major and only two minor criteria; Choice C has only one major and one minor criterion and finally Choice D only has two minor criteria.
A 3-year-old girl presents with her mom for an acute visit. Mom states that the chief complaint is a persistent cough. About four weeks ago, she had a low-grade fever with rhinorrhea and cough for several days. The rhinorrhea and fever has since resolved but the cough has continued. It bothers her primarily during the night and at times when she wakes first thing in the morning. Her medical history is significant only for mild seasonal allergies that appeared after she turned 3. Her vaccines are current and her parents deny recent travel. Mom described the patient as a thriving, happy child who attends preschool 3 days a week. For the past few weeks, she has been tired and fussy during the day because the cough keeps her up at night. On exam, she appears overall healthy. She is in no respiratory distress and her lungs are clear to auscultation. What intervention will most likely cut down on the coughing and improve the girl's sleep? 1. Give diphenhydramine before bed 2. Encourage extra fluids and humidifier for the room 3. Discourage milk consumption late in the day 4. Begin daily leukotriene 4mg granule packets 5. Give Robitussin as needed
Correct answer: Begin daily leukotriene 4mg granule packets Explanation This girl's symptoms and history of allergic rhinitis are consistent with cough-variant asthma. Respiratory symptoms are characteristically worse at night and often triggered by respiratory (likely viral) infections. Since this patient has a history of allergic rhinitis, chances are that she is being exposed to allergens that are accentuating her symptoms. For this reason it would be more beneficial to initiate treatment that would be both alleviating signs and symptoms of the suspected allergic rhinitis and potential asthma. The most effective option would be montelukast 4mg granule packets. Montelukast is the best option because it is leukotriene inhibitor which is prescribed to not only treat asthma, prevent bronchospasms, but also to treat perennial allergies and children as young as 6 months old and adults as well as seasonal allergies in kids as young as 2 years old. Diphenhydramine, Robitussin, and cutting down on milk are unnecessary and not appropriate choices. They are ineffective in treating a cough caused by asthma. Fluids and humidification are always a good recommendation, but in this case they will not directly cut down on the cough if being contributed by seasonal and/or perennial allergies.
A 63-year-old man with a past medical history of hyperlipidemia presents with an 8-month history of intermittent flushing and warmth sensation to his face and neck areas, alternating watery, frothy, and bulky stools associated with steatorrhea, weight loss, and wheezing. His urine demonstrated increased 5-hydroxyindoleacetic acid (HIAA) levels. He had subsequently been referred for an octreotide scintigraphy, which revealed innumerable foci of intense activity scattered throughout the liver. What agent is highly effective at reducing symptoms? 1. Sandostatin (Octreotide) 2. Danocrine (Danazol) 3. Cytarabine (Depocyt) 4. Imatinib (Gleevec) 5. Gemcitabine (Gemzar)
Correct answer: Sandostatin (Octreotide) This patient's diagnosis is consistent with carcinoid tumor. Carcinoid tumors are of neuroendocrine origin and derived from primitive stem cells in the gut wall, but they can be seen in other organs, including the lungs,mediastinum, thymus, liver, pancreas, bronchus, ovaries, prostate, and kidneys. Carcinoid tumors have high potential for metastasis. Neuroendocrine tumors contain somatostatin receptors. This feature allows for the localization of primary tumors and tumor metastases by scintigraphy with the radiolabeled somatostatin analog octreotide. It can be used to detect certain types of cancer arising from the neuroendocrine system. As a somatostatin analog, sandostatin is highly effective in reducing symptoms. Survival advantage with the use of this drug has not yet been proven. Sandostatin reduces the amount of the growth factor produced, thus theoretically impairing growth. Danocrine suppresses pituitary-ovarian axis by inhibition of pituitary gonadotropin output. It is indicated in the treatment of endometriosis, fibrocystic breast disease, and hereditary angioedema. Cytarabine is an antimetabolite chemotherapeutic agent that inhibits DNA polymerase during the S phase; it is indicated in the treatment of acute myelogenous leukemia and acute lymphocytic leukemia. Imatinib is a tyrosine kinase inhibitor used in the treatment of acute lymphoblastic leukemia, myelodysplastic, or myeloproliferative diseases, hypereosinophilic syndrome, eosinophilic leukemia, and chronic myeloid leukemia. Gemcitabine is indicated as first-line treatment for locally advanced (nonresectable Stage II or Stage III) or metastatic (Stage IV) adenocarcinoma of the pancreas.
A 65-year-old man presents with a 25-pound unexplained weight loss. He also has noticed a change in his usual cough. He sees his family doctor. Upon questioning for the medical history, the doctor discovers that he has a 60-pack/year history of smoking as well as dyspnea. On physical examination, he appears to have Cushing's syndrome. X-rays show a central lesion with no cavitation. A hilar mass is seen on chest X-ray. What is the most likely diagnosis? 1. Small cell carcinoma 2. Adenocarcinoma 3. Metastatic Wilms tumor 4. Large cell carcinoma 5. Squamous cell carcinoma
Correct answer: Small cell carcinoma Small cell carcinoma is most frequently a central lesion not associated with cavitation. A hilar mass is associated with small cell carcinoma. Small cell carcinoma is associated with paraneoplastic syndromes. Cushing's syndrome secondary to ectopic ACTHproduction can sometimes be seen with small cell carcinoma, as can syndrome of inappropriate antidiuretic hormone production (SIADH). An adenocarcinoma is usually seen peripherally on X-ray. An adenocarcinoma is characterized pathologically by mucus production. On the rare occasion when lung cancer occurs in somebody who has never smoked, it is most often classified as an adenocarcinoma. Adenocarcinomas may develop in areas of the lung where there is scarring or fibrosis. For example, a cause of the scarring can be a granulomatous infection, such as tuberculosis. For this reason, adenocarcinoma is sometimes referred to as "scar carcinoma". Wilms tumor is a pediatric malignancy of the kidney. It can metastasize to the lungs. Large cell carcinoma is usually seen peripherally on X-ray. Large cell carcinoma shows poorly differentiated cells. It does not show squamous features or glandular features. Large cell carcinoma does not react to mucicarmine staining. Large cell carcinoma is also called undifferentiated carcinoma or anaplastic carcinoma. The cell of origin is unknown. Squamous cell carcinoma is centrally located. Cavitation can also sometimes be seen with X-ray. Keratin formation, keratin pearl formation, and intercellular bridges can be seen with squamous cell carcinoma.
A 57-year-old woman presents to the hospital complaining of a 2-week history of feeling weak. She is an artist who has been chain smoking for 30 years. On examination, she is kempt and wasted, but not dehydrated. Her BP is 116/70 mm Hg. Laboratory investigations reveal plasma sodium of 122 mEq/L with a high urine osmolality. A chest radiograph reveals a rounded opacity in the right lung field. What is the most likely diagnosis? 1. Lambert-Eaton myasthenic syndrome 2. Hypertrophic pulmonary osteoarthropathy 3. Horner's syndrome 4. Ectopic adrenocorticotropic hormone (ACTH) secretion 5. Syndrome of inappropriate antidiuretic hormone secretion (SIADH)
Correct answer: Syndrome of inappropriate antidiuretic hormone secretion (SIADH) The syndrome of inappropriate antidiuretic hormone secretion (SIADH) is a paraneoplastic syndrome associated with bronchogenic carcinoma. There is excessive water reabsorption by the kidneys due to the elevated antidiuretic hormone (ADH). Patients may complain of feeling weak and lethargic. Other symptoms include confusion and coma. Laboratory tests reveal hyponatremia, high urine sodium, low serum osmolality, and high urine osmolality in a euvolemic patient. In hypertrophic pulmonary osteoarthropathy (HPO), there is formation of new subperiosteal cancellous bone at the distal ends of long bones. Patients present with periarticular pain, polyarthralgia, and painful swellings at the wrists, knees, elbows, and ankles. On examination, there is clubbing of the fingers and toes. There may also be localized articular erythema, tenderness, swelling, and even effusions. X-rays may reveal subperiosteal bone formation with periosteal elevation seen as thickening and detachment of the periosteum. Lambert-Eaton myasthenic syndrome is an immune-mediated disorder of neuromuscular transmission resulting in impaired release of acetylcholine from nerve terminals. Patients complain of weakness of the scapular and pelvic girdles musclesand the resultant difficulties in performing activities such as climbing stairs. Other symptoms such as dry mouth, paresthesias, and sexual impotence due to autonomic dysfunction may also be present. On examination, there is proximal muscle weakness. Deep tendon reflexes are decreased or absent. Horner's syndrome is due to a bronchogenic carcinoma tumor mass extending to the sympathetic chain. Horner's syndrome symptoms include miosis, ptosis, enophthalmos, and hemianhidrosis. In addition, the patients may also have atrophy of hand muscles. Cushing's syndrome may develop as a result of the small cell carcinomas secreting ectopic adrenocorticotropic hormone (ACTH). Patients may complain of bruising easily. On examination, they may have red-purplish striae, especially over their abdomen, in addition to the typical moon facies and truncal obesity. They may also have muscle wasting, especially of the proximal limb girdle muscles. Laboratory investigations reveal hypokalemia and high plasma ACTH, as well as increased serum and urine cortisol.
A 3-year-old girl presents with a rapid onset of high fever and noisy breathing that developed in the last 8 hours. The patient has missed standard immunizations. The child denies having a sore throat, but she tells you that she cannot eat or drink anything because of pain; when she tells you this, she points with her finger to a spot deep inside her mouth. On examination, the child has a muffled voice, drooling, fever, tachycardia, tachypnea, and she adopts a leaning forward position with the neck extended. Stridor and retractions of the chest wall are noted on inspirations, but the lungs are clear on auscultation. During the examination, the child becomes cyanotic and prostrated. What is the most appropriate next step in patient management?? Aerosol ribavirin 2. Lateral neck X-ray 3 Immediate intubation 4. Nebulized racemic epinephrine 5 Throat examination
Epiglottitis, an infection most commonly caused by Haemophilus influenzae, is characterized by inflammation and rapidly progressive edema of epiglottis and contiguous tissue. Children of ages 2 to 7 years who missed some immunizations are prone to this infection during winter. The use of H. Influenzae vaccine caused a marked decrease in the incidence of this disease. Drooling, hoarseness, high fever, sore throat, the characteristic "sniffing dog" position and the rapidly progressive respiratory obstruction make the diagnosis a clinical one. Because of the rapid evolution to airway occlusion, a patient with suspicion of epiglottitis should receive immediate intubation under anesthesia in the operating room. Emergency cricothyrotomy is the alternative if there are no conditions to ensure a patent endotracheal airway. Intravenous ceftriaxone or cefuroxime provide an empiric antibiotic coverage.
A 33-year-old man presents with shortness of breath, wheezing, mild fever, and fatigue. He has had several similar episodes in the past, and each previous episode began after a cold that moved into his chest. Over the past several weeks, he has had a productive cough most mornings. He smokes on a social basis. What is the most likely diagnosis? 1. Chronic emphysema 2. Chronic bronchitis 3. Cor pulmonale 4. Acute asthmatic bronchitis 5. Bronchiectasis
Explanation The clinical picture is suggestive of acute asthmatic bronchitis. Typical symptoms include persistent cough with mucus that can become thicker and more profuse, dyspnea, mild fever, and chest pains. It is commonly caused by a virus infecting the lining of the bronchial tree. Wheezing can occur for several weeks. In this particular patient, the cold (viral infection) moved to his lungs and induced acute asthmatic bronchitis. Productive cough is not seen in chronic emphysema patients. Chronic bronchitis is defined as a clinical history of productive cough for 3 months of the year for 2 consecutive years, which is not described in this patient. Cor pulmonale has similar symptoms to bronchitis and emphysema, but it also has elevated jugular venous pressures, parasternal lift, edema, hepatomegaly, and ascites, which are not seen in this patient. Bronchiectasis presents with dyspnea, wheezing, chronic cough with copious mucous production, hemoptysis, and pleuritic chest pain. Hemoptysis and pleuritic chest pain are not present in this patient.
A 78-year-old man with known left-sided congestive heart failure presents with a complaint of cough, worsening dyspnea with exertion, and orthopnea. What is the most direct cause of his symptoms? 1. Tricuspid insufficiency 2. Left ventricular hypertrophy 3 .Decreased peripheral vascular resistance 4. Increased pulmonary venous pressure 5. Mucus plugging
Increased pulmonary venous pressure In left-sided congestive heart failure, the predominant feature is *low cardiac output and elevated pulmonary venous pressure, resulting in dyspnea.* As dyspnea worsens, the patient will also begin to experience shortness of breath at rest, which is worsened in the supine position. Tricuspid function is not related to CHF. Peripheral vascular resistance typically increases in CHF, which is designed to help maintain perfusion to vital organs. Mucus plugging is not associated with CHF.
A 48-year-old man is brought to the ER complaining of difficulty breathing, fatigue, and intermittent chest pain for the past month. On further questioning, he states that the breathing seems to worsen when lying down. On physical exam, you note elevated respiratory and heart rates and pale, sweaty skin. On auscultation, rales are noted as well as a 3rd heart sound. Which of the following is the most likely diagnosis? 1 Right Ventricular failure 2 Pulmonary Embolism 3 Mitral Valve Stenosis 4 Left Ventricular failure 5 Chronic Obstructive Pulmonary Disease (COPD)
Left Ventricular failure The clinical picture is suggestive of left ventricular failure (LVF). Clinical presentation includes dyspnea, orthopnea, and paroxysmal nocturnal dyspnea. The patient may also have hemoptysis, chest pain, fatigue, nocturia, and confusion. On physical exam, the patient may present with cold, pale sweaty skin, tachypnea and tachycardia, rales, and 3rd and 4th heart sounds. This is diagnostically different from right ventricular failure. Right ventricular failure (RVF) has a clinical picture of shortness of breath, pedal edema, and abdominal pain. On PE, RVF will present with a S3, jugular venous distention (JVD) and may have signs of LVF.
A 32-year-old man with no significant past medical history presented with dyspnea, palpitations, feelings of anxiety, and dizziness, all of which occurred earlier in the morning following a brisk walk. He denied any prior episodes, illicit drug use, alcohol or cigarette use, skipping meals, or caffeine intake. He further denied fever, chills, chest pain, history of murmurs, cough, edema, rashes, syncope, headache, psychiatric, or focal neurological complaints. The physical examination demonstrated a fast, regular pulse with a constant-intensity first heart sound, but was otherwise normal. An EKG was performed, which revealed a short PR interval plus a slurred upstroke at the beginning of the QRS complex. What is the most likely mechanism responsible for this patient's presentation? 1. Conduction delay in the proximal part of the right or left branches 2. Pre-excitation occurring via an atrio-His bundle 3. Spontaneous ectopy from muscular sleeves of pulmonary veins 4. Early excitation due to accessory pathways between the atria and ventricles 5. Inappropriately enhanced automaticity of sinus node pacemaker cells
Patients with Wolff-Parkinson-White syndrome have an additional aberrant muscular or nodal tissue connection (bundle of Kent) between the atria and ventricles. This conducts more rapidly than the slowly conducting AV node, and one ventricle is excited early. The manifestations of its activation merge with the normal QRS pattern, producing a short PR interval and a prolonged QRS deflection slurred on the upstroke, with a normal interval between the start of the P wave and the end of the QRS complex ("PJ interval"). The QRS complexes show an abnormal morphology with a width greater than the baseline QRS complex (often >0.11 second), with the characteristic initial slurring referred to as a delta wave. The paroxysmal atrial tachycardias seen in this syndrome often follow an atrial premature beat. This beat conducts normally down the AV node but spreads to the ventricular end of the aberrant bundle, and the impulse is transmitted retrograde to the atrium. A circus movement is thus established. Less commonly, an atrial premature beat finds the AV node refractory but reaches the ventricles via the bundle of Kent, setting up a circus movement in which the impulse passes from the ventricles to the atria via the AV node.
A 45-year-old man is admitted to the hospital with fever, weakness, weight loss, muscle and testicular pain, and a rash on his legs. He states that his symptoms began about 1 week ago. The testicular pain began about 1 day ago and has increased significantly over the last 24 hours. He denies recent illness or injury and states that has been in good health for as long as he can remember. On physical exam, the patient was well-developed, well-nourished, and in mild physical distress. His blood pressure was elevated at 152/94 mm Hg, and a chest radiograph was negative. Laboratory analysis revealed an elevated sedimentation rate and C-reactive protein, elevated BUN, and creatinine. His red blood cell count was decreased, and his ANCA was negative. An arteriogram showed diffuse arterial saccular aneurysms and narrowing of the arteries. What is the most likely cause of this patient's symptoms? Systemic lupus erythematosus 2 Polyarteritis nodosa 3 Rheumatoid arthritis 4 Sjögren syndrome 5 Kawasaki disease
Polyarteritis nodosa is a rare autoimmune disease that can affect medium-sized arteries in any organ of the body. The cause is unknown. It commonly affects nerves, skin, intestines, muscles, and joints. It is seen more in middle-aged people and results in decreased blood supply to affected body parts. Men and women are affected approximately equally. There has been some association made with hepatitis B. 3 criteria need to be met out a list of possible symptoms before the diagnosis of polyarteritis nodosa can be made. These include: muscle pain, a positive hepatitis B surface antigen or antibody test, elevated BUN or creatinine, livedo reticularis (a mottled purplish skin discoloration over the extremities or torso), an elevated diastolic blood pressure (greater than 90 mm Hg), weight loss of 4kg or more, a biopsy positive for vasculitis, dilated arteries or constricted blood vessels (seen by arteriogram), testicular pain or tenderness, or nerve disease. It is diagnosed by examining the tissues. This is usually achieved through a biopsy. Histological analysis usually shows focal necrotizing arteritis and a mixed cellular infiltrate within the walls of the affected blood vessels. Testing for an elevated sedimentation rate and C-reactive protein will also help with the diagnosis. Patients often have an elevated white blood cell count and a decreased red blood cell count. Urinalysis may show proteinuria and hematuria. Polyarteritis nodosa is usually treated with high doses of cortisone and immunosuppressive drugs such as cyclophosphamide or azathioprine. If the patient also has hepatitis B, antiviral medications such as interferon-alpha may be included in treatment.
A 21-year-old woman presents with a severe headache; it is accompanied by sweating and palpitations. She was seen by her primary care physician earlier in the day, and he admitted her from the office when her blood pressure was recorded several times at 210/98 mm Hg. The patient has no known medical or surgical history, and she is currently on a daily multivitamin. She states that she has been having her symptoms episodically over the last 2 weeks, but this is the first time she has been seen by a physician. The patient is found to have elevated urinary and plasma metanephrines and catecholamines. What antihypertensive agent should be used for this patient? Lisinopril 2 Phenoxybenzamine 3 Metoprolol 4 Lasix 5 Hypertension should not be treated at this time
The correct answer is phenoxybenzamine. The patient's young age suggests that she is suffering from a form of secondary hypertension. Given her paroxysmal headaches, diaphoresis, and palpitations accompanied by hypertension, she is likely suffering from a pheochromocytoma. Pheochromocytomas are benign tumors that arise from the adrenal glands. The tumors secrete norepinephrine, which leads to hypertension, tachycardia, headache, and diaphoresis. Diagnosis is made based on elevation in urinary and plasma metanephrines and catecholamines. and it is followed by tumor location on CT or MRI. The patient requires removal of the pheochromocytoma, but preoperative control of her blood pressure is necessary. *Alpha blockers, such as phenoxybenzamine or prazosin, are the preferred antihypertensive agents in pheochromocytoma.*
A patient is presenting with substernal crushing chest pain and shortness of breath for just 15 minutes prior to arriving to the emergency department. An EKG is obtained and appears suspicious for an acute myocardial infarction. What would most likely be seen in this patient's EKG? 1. S-T segment depression 2 T-wave changes 3 Ventricular bigeminy 4 Q-wave elongation 5 Hyperacute T-wave
The earliest presentation of an acute myocardial infarction is the hyperacute T wave. If found, this is treated the same as an ST segment elevation type of infarction (STEMI). The hyperacute T wave is considered very rare in clinical practice, as it only exists for 2-30 minutes after the onset of an infarction. Hyperacute T waves must also be distinguished from the characteristic peaked T waves, which commonly are seen with hyperkalemia.
A 55-year-old man presents with severe central chest pain. He is a farmer and describes the pain as tearing, saying it started suddenly and is radiating. He is only able to lie comfortably on his side. He denies any previous symptoms. He feels nauseous but has not vomited. He has no major illnesses and knows of none that run in his family. He does not use alcohol, tobacco, or illicit substances. He is allergic to sulfa drugs. On physical exam he appears to be in extreme pain, despite lying on his side. His temperature is 37.0°C, heart rate is 110 BPM, blood pressure is 180/105 mm Hg, and his respiratory rate is 20/min. Cardiac exam reveals normal S1 and S2 without rubs or gallop. The top of his internal jugular venous column is present at 2 to 3 cm above the sternal notch. Chest auscultation reveals no abnormalities. He has normal active bowel sounds tympanic to percussion. Extremity exam was normal and the lower motor and sensory function is intact. ECG shows left ventricular hypertrophy and chest X-ray shows widened mediastinum. Labs show: Hemoglobin 13.5 g/dL Leukocyte count 5,000 cells/dL Platelet count 190.000 cells/mL Urea 70 mg/dL Creatinine 2.5 mg/dL Sodium 139 mmol/dL Potassium 4.8 mmol/dL Calcium 8.9 mg/dL Bicarbonate 30 mEq/L Troponin T <0.03 Creatinine Kinase 150 IU What is the best test for diagnosing this patient's condition? 1 CT angiography 2 Magnetic resonance angiography (MRA) 3 Transesophageal echocardiography 4 Troponin 5 Chest X-ray
This patient has an aortic dissection. Aortic dissections are more common in patients with hypertension, connective tissue disorders, congenital aortic stenosis or bicuspid aortic valve, and in those with first-degree relatives with history of thoracic dissections. Chest pain is the most common presenting symptom. The pain is usually sudden and severe and described as ripping or tearing. Magnetic resonance angiography (MRA) is currently the best test for the detection and assessment of an aortic dissection. It will produce a 3-D reconstruction of the aorta, allowing the physician to determine the location of the intimal tear, the involvement of branch vessels, and locate any secondary tears. It is a non-invasive test, does not require the use of iodinated contrast material, and can detect and quantify the degree of aortic insufficiency.
A 5-day-old male infant has subtle, unusual facial features (i.e., a triangular face, hypertelorism, and down-slanting eyes). He also has a webbed neck and low-set ears. Suspecting a congenital disorder, you order a complete work-up, including a CBC, coagulation profile, cardiac evaluation, karyotyping, and mutation analysis. PTPN11 (protein-tyrosine phosphatase, nonreceptor-type 11) mutations are detected. Echocardiography detects a cardiac defect. What's most likely to be found on echo?
This neonate most likely has Noonan syndrome (NS). Pulmonary stenosis is the most common cardiac defect in this condition. Noonan syndrome is a sporadic, or autosomal dominant, congenital disorder with typical phenotypical features that may not be visible to the casual onlooker. The most common facial features include hypertelorism and low-set, backward-rotated ears with a thick helix. The philtrum is deeply grooved in more than 90% of cases. Congenital cardiac defects, bleeding disorders, mental retardation, webbed neck, and a short stature are other features.
A 75-year-old African-American man presents with a 5-month history of gradually progressive dyspnea that is especially pronounced when climbing stairs. He also has been noticing that his ankles and lower legs have "gotten larger" over roughly the same time period, which no longer allows him to fit into his sneakers. He denies fever, chills, chest pain, palpitations, cough, pleurisy, calf pain, abdominal complaints, sick contacts, or travel. His psychosocial history is noteworthy for chronic alcohol use. His physical exam reveals bibasilar rales, JVD of 5cm, an S3 gallop, a holosystolic murmur at the apex that radiates to the left axilla, and 2+ pitting edema to the level of the mid-calves bilaterally. A bedside echocardiogram was remarkable for biventricular enlargement. What additional physical exam finding would be expected in this patient? 1 Tachycardia 2 Fever 3 Asymmetric upper extremity blood pressures 4 Warm, moist skin 5 Acanthosis nigricans
This patient's presentation is significant for dilated cardiomyopathy. Dilated cardiomyopathy occurs more often in African Americans than Caucasians, and it occurs in men more frequently than women. Often no cause can be identified, but chronic alcohol abuse, major catecholamine discharge, myocarditis, postpartum, and doxorubicin are frequent causes. Chronic tachycardia may also precipitate a dilated cardiomyopathy that may improve over time if rate control can be achieved. Amyloidosis, sarcoidosis, hemochromatosis, and diabetes may rarely present as dilated cardiomyopathies, as well as the more classic restrictive picture.
A 32-year-old man with no significant past medical history presents to his primary care provider with a 2-month history of increased dyspnea upon exertion, which becomes apparent following walking 10 city blocks. He denies any other associated symptoms such as fever, chills, changes in weight, chest pain, abdominal pain, nausea, or vomiting. He further denies any history of cigarette smoking, occupational risk factors, sick contacts, or recent travel. His physical exam revealed normal vital signs and no distension of his jugular vein. However, there was a prominent right ventricular impulse along the lower-left sternal border associated with a palpable pulmonary artery and a midsystolic ejection murmur at the upper left sternal border that does not vary in intensity with respiration. There is a fixed split second heart sound. The remainder of his examination is normal. Following diagnostic testing, this patient was referred for surgical repair. What is the major long-term complication that requires monitoring following surgical repair? 1 Hypertension 2 Myocardial infarction 3 Mitral valve prolapse 4 Supraventricular arrhythmia 5. TIA
This patient's presentation represents an atrial septal defect. The major long-term complication following surgical transcatheter device closure of ASD is the development of supraventricular arrhythmias, although the risk is lowered when the ASD is closed in childhood. The persistence of this risk despite relief of right-sided volume overload is thought to be related to incomplete atrial remodeling or due to the presence of the atriotomy scar. Longer follow-up is required to determine whether device closure alters the risk of atrial dysrhythmias.
In order to test for orthostatic changes, blood pressure and pulse are measured with the patient first supine then standing. What are the criteria for a positive orthostatic change (going from supine to standing)?
When an individual experiences a loss of blood volume, for whatever reason, he or she may become orthostatic. Causes include antihypertensive and other medications, severe dehydration or anemia, prolonged bedrest, and autonomic instability. When such a person has their blood pressure and pulse measured first in the supine position, then again while standing, both the systolic and diastolic blood pressure drop, and the pulse increases. In order to be considered significant, the systolic pressure should drop by at least 20 mm Hg, the diastolic pressure should drop by at least 10 mm Hg, and the pulse should increase by at least 20 beats per minute.
A 35-year-old woman presents with 5-hour history of progressive shortness of breath, cough, and wheezing. This morning she felt that she was "catching a cold" because of sore throat and thin purulent rhinorrhea, for which she took aspirin. Her past medical history is significant for persistent rhinitis resistant to therapy. What should your patient do to prevent future asthma attacks? 1 Avoid aspirin 2 Take antihistamine 3 Inhale cromolyn 4 Take albuterol 5 Get influenza vaccine
Your patient most probably has aspirin-exacerbated respiratory disease. It is chronic sinusitis characterized by nasal polyposis, nonallergic induced asthma, and aspirin sensitivity. The condition is sometimes called aspirin triad [SAMTER TRIAD]. It commonly starts in the third decade. Clinical symptoms of aspirin-sensitive patients are characterized by mucosal inflammation and rhinitis, severe asthma precipitated by aspirin ingestion, and aggressive bilateral nasal polyposis. The rhinitis is persistent and difficult to manage, and the rhinorrhea is thin and nonpurulent (nonallergic rhinitis with eosinophilia syndrome). Asthma usually appears an average of 2 years after rhinitis, followed by the intolerance to aspirin and the co-occurrence of nasal polyps. Severe acute asthma attack can occur within a few minutes and up to 3 hours after ingestion of aspirin. Aspirin challenge can be used to confirm a diagnosis of aspirin sensitivity in these patients.
What is not a common cause of aortic stenosis? 1 Rheumatic heart disease 2 Chronic intravenous drug abuse 3 Congenital bicuspid aortic valve 4 Monckeberg senile calcific changes 5 Age greater than 60 years
Correct answer: Chronic intravenous drug abuse Explanation Intravenous drug abuse has not been described as a common cause of aortic stenosis. Common causes of aortic stenosis are: (1) Infants, children and adolescents (a) congenital aortic stenosis (b) congenital subvalvular aortic stenosis (c) congenital supraclavicular aortic stenosis (2) Young adults to middle age: (a) calcification and fibrosis of congenitally bicuspid aortic valve (b) rheumatic aortic stenosis (3) Middle aged to elderly (a) calcification of bicuspid valve (b) senile degenerative aortic stenosis (c) rheumatic aortic stenosis
A 40-year-old man presents with a 5-day history of cough and purulent sputum. The patient has had recurrent attacks of cough with sputum production since his childhood. There are no other systemic complaints. The patient is febrile and has grade III finger clubbing. Rales are present all over the chest on auscultation. The chest X-ray shows a characteristic honeycomb appearance. What is the most likely diagnosis? 1. COPD 2. Tuberculosis 3. Bronchiectasis 4. Pneumonia 5. Cystic fibrosis
Correct answer: Bronchiectasis Explanation The clinical picture along with clubbing and the honeycomb appearance on chest X-ray is diagnostic of bronchiectasis. Bronchiectasis is a disease caused by irreversible dilatation of the bronchial tree. Pathogenesis can be obstruction, congenital disorder, and infection. It can develop as a result of severe bacterial infections in childhood, often as a complication of whooping cough or measles. It can be cylindrical, varicose, or cystic, and it corresponds to the severity of degree of bronchiectasis. Typically, the patient has recurrent respiratory infections, usually cough with lots of sputum production. There will be clubbing of fingers and cyanosis (depending on the severity), and it presents at a later age compared to cystic fibrosis. Chest X-ray shows a typical honeycomb appearance. Diagnosis can be confirmed by CT scan or bronchography. Treatment is by antibiotics and assisting postural drainage. Cystic fibrosis presents with associated symptoms of malabsorption and infertility. COPD is associated with a history of smoking. It will usually have a normal X-ray, as in bronchitis, or be seen with associated features of emphysema (e.g., bullae and hyper-translucent lung field with loss of peripheral vascular markings). Pneumonia will have an acute history, and recurring infections are not characteristic. X-ray will be diagnostic. Tuberculosis will also be diagnosed on the X-ray. Bronchiectasis may develop secondary to a tuberculous hilar lymph node obstructing a major bronchus.
A 25-year-old man presents with acute onset shortness of breath associated with right-sided chest pain. The pain is unaffected by position and is worse with inspiration. He was grocery shopping when it started. He denies chest trauma. He had an upper respiratory infection earlier in the month that had resolved without incident. He smokes 1 pack of cigarettes per day, and he has no significant past medical history. On examination, he is afebrile and BP is138/80 mm Hg; pulse is 124, respiratory rate is 24, and pulse oximetry is 94% on room air with mild respiratory distress. Trachea is midline. He has increased resonance to percussion with no breath sounds on the right anterior apex; the other lung fields are clear to auscultation. Heart is tachycardic with normal S1 and S2; no murmur, rubs, or gallops are present. What is the imaging of choice to make the diagnosis? Chest computed tomography (CT) 2 Chest radiograph 3. Chest ultrasound 4 Electrocardiogram (ECG) 5 Spiral chest computed tomography (CT)
Correct answer: Chest radiograph Explanation The patient has a primary spontaneous pneumothorax (PSP). There is a higher incidence in men, and it usually occurs between 20 - 40 years of age. PSP is heavily associated with smoking. Patient presentation and physical exam findings will vary depending on the size of the pneumothorax. Patients typically present with unilateral chest pain, dyspnea, and cough, which occur with minimal activity, as well as tachycardia and tachypnea. A large PSP may become a tension pneumothorax, in which case the patient may have a deviated trachea, no breath sounds on the affected side with increased resonance to percussion, as well as respiratory distress and shock. Imaging of choice for a PSP is a chest radiograph, usually an upright inspiratory film. Supine or lateral decubitus views can also be used. A chest CT can be used to visualize a pneumothorax, but it is not indicated unless additional pathology is suspected. Ultrasound is user dependent, and it is not superior to a plain film in the diagnosis of PSP. A spiral CT is used to evaluate for pulmonary embolism, and an ECG is used to evaluate for pericarditis, both of which are differentials for this patient presentation, and they may have been indicated had the chest radiograph been normal.
A 46-year-old man is a known asthmatic and has been on a carefully titrated dose of theophylline for 14 years without any complications. Recently, he sought medical attention for malaise, persistent cough, and weight loss. He was diagnosed with tuberculosis. During visits to the clinic for diagnosis and initiation of treatment for tuberculosis, he was also discovered to be hypertensive and to have acid peptic disease. He was put on medication for all 3 conditions. After 2 weeks of therapy, the patient had a seizure and was rushed to the emergency department. Reports showed a tearing of the bridging veins between the dural sinuses and an increase in the plasma theophylline concentration in the large cerebral veins. What drug is most likely to have caused the theophylline toxicity? 1. Atenolol 2. Cimetidine 3 Hydrochlorothiazide 4 Prazosin 5 Rifampin
Correct answer: Cimetidine Explanation Theophylline is metabolized via the cytochrome P450 oxidative drug metabolizing system. Cimetidine is the most likely cause of the theophylline toxicity because it inhibits cytochrome P450. Theophyllines metabolism would be impaired, leading to toxic plasma levels. As the case clearly describes a supra therapeutic level of theophylline, cimetidine is the best answer as it relates to this complication.
A 52-year-old woman with a history of untreated bronchogenic carcinoma presents with a 1-week history of dyspnea, facial edema, and marked neck vein distention; the distention has progressed over the past 36 hours. A chest X-ray confirms a right hilar mass with a small pleural effusion in the right hemithorax. Which of the following steps is most appropriate? 1. Consult a cardiologist for pericardiocentesis 2. Consult an oncologist for radiation therapy 3. Consult a cardiothoracic surgeon for a mediastinoscopy 4. Consult a cardiovascular surgeon for exploration thoracotomy 5. Prepare the patient for a pleurocentesis in the office
Correct answer: Consult an oncologist for radiation therapy Explanation The patient is presenting with signs and symptoms of Superior Vena Cava (SVC) syndrome, caused by obstruction of blood flow through the SVC due to compression, thrombosis, or infiltration from a right upper hilar malignancy. Obstruction of venous drainage at the upper thorax causes venous distention. Morbidity may result from irreversible thrombosis, central nervous system damage, or pulmonary complications. The most common cause of obstruction is malignant disease, with bronchogenic carcinoma at the top of the list. Signs and symptoms of SVC syndrome include swelling and cyanosis in the face and upper trunk, dyspnea, dysphagia, neck vein distension, and headache. Cerebral edema is a potential complication. In patients who present with serious airway or central nervous system systems, the preferred management strategies are SVC stenting and/or radiation therapy. Diuretics, steroids, and thrombolytics, are sometimes used as temporizing measures, but their effectiveness is questionable. The diagnosis of pericardial effusion is not supported by the history, physical exam, and chest film; therefore, pericardiocentesis is not indicated. Direct surgical interventions are ineffective in this case. The small pleural effusion present on chest x-ray is unlikely to account for the patient's symptoms, and an in-office pleurocentesis would not be advisable.
A 69-year-old man presents with dyspnea on exertion that has been slowly progressive over the course of the last year. He notes impairment in climbing stairs and walking short distances. His review of systems is positive for fatigue, palpitations, intermittent retrosternal chest pain, swelling of his lower extremities, dizziness, and "feeling faint;" his associated symptoms are also known to occur upon exertion. He denies any fever, chills, weight changes, cough, abdominal pain, early satiety, nausea, vomiting, diarrhea, changes to his urine color or odor, flank pain, hematuria, or dysuria. He denies any cigarette, alcohol, or drug use. His cardiac exam is remarkable for an increased pulmonic component of the second heart sound (P2), wide, inspiratory splitting of S2 over the cardiac apex, right-sided S3 and S4 gallops, a left parasternal lift, a loud diastolic murmur that increases with inspiration and diminishes with the Valsalva maneuver, prominent "A" waves in jugular venous pulsations, and increased JVD. He has an enlarged liver with hepatojugular reflux, peripheral edema, and ascites. A bedside EKG analysis revealed peaked P waves, rightward axis deviation, and prominent R waves in the early V leads. What is the most likely diagnosis? 1. Myocardial infarction 2. Cor pulmonale 3. Primary biliary cirrhosis 4. Left ventricular heart failure 5. Pulmonary embolism
Correct answer: Cor pulmonale Explanation Myocardial infarctions occur at rest and most commonly in the early morning. The pain is similar to angina in location and radiation, but it may be more severe and builds up rapidly or in waves to maximum intensity over a few minutes or longer. Associated symptoms include diaphoresis, weakness, apprehension, and a feeling of impending doom; patients may move about, seeking a position of comfort, preferring not to lie quietly. Light-headedness, syncope, dyspnea, orthopnea, cough, wheezing, nausea and vomiting, or abdominal bloating may occur. Physical exam findings may include fever, anxiousness, diaphoresis, bradycardia or tachycardia, low cardiac output, or arrhythmia. There may be hypertension (in hypertensive patients) or low in patients with shock. Respiratory distress, jugular venous distention a Kussmaul sign, soft heart sounds, and atrial gallops (S4) or ventricular gallops (S3) usually indicate heart failure. Primary biliary cirrhosis is most common in middle-aged females and is characterized by fatigue, pruritus, hepatosplenomegaly, xanthomatous lesions on the skin, eyelids, and tendons, jaundice, and steatorrhea. Signs of portal hypertension are late findings. Other findings include orthostatic hypotension and cognitive dysfunction. Left-sided or forward failure may account for many of the clinical manifestations of heart failure, such as mental confusion from decreased cerebral perfusion, fatigue and weakness from decreased skeletal muscle perfusion, and sodium and water retention with secondary venous congestion from decreased renal perfusion. Isolated left-side heart failure is associated with dyspnea, fatigue, weakness, cough, paroxysmal nocturnal dyspnea, and orthopnea in the absence of peripheral edema, jugular venous distention (JVD), or hepatojugular reflux. Chest pain, dyspnea, and tachypnea are the most frequent signs and symptoms of pulmonary embolism. Other manifestations may include tachycardia, pleurisy, low-grade fever, apprehension, and productive cough with blood-tinged sputum. Massive PE may manifest as sudden collapse, crushing substernal chest pain, shock, diaphoresis, hypotension, distended neck veins, and loss of consciousness.
A 69-year-old man presents with a 2-hour history of right upper extremity hemiparesis, which resolves upon presentation to the hospital. A cerebral MRI demonstrates an isolated intracranial lesion, which is consistent with metastatic disease. A chest X-ray demonstrates a right hilar mass with right lower lung atelectasis. A CT is shown below; it demonstrates right lower and middle lobe atelectasis secondary to an endobronchial cancer without lymphadenopathy. PET scan demonstrates uptake in the right lung and brain alone. Bronchoscopy demonstrates complete occlusion of the right lower lobe secondary to adenocarcinoma. The tumor encroaches upon the right middle lobe but does not occlude the orifice. What should the treatment consist of for this patient? 1. Chemotherapy alone 2. Lung resection followed by chemotherapy 3. Cranial resection followed by lung resection 4. Lung resection followed by cranial resection 5. Radiation therapy alone
Correct answer: Cranial resection followed by lung resection Explanation Occasionally, patients present with resectable localized lung cancer and evidence of solitary metastasis. These patients should be considered for a combined approach in which the primary tumor and solitary metastasis are resected. Locations that are amenable to a combined approach include adrenal glands, brain, lung, and bone. Approximately 11% of asymptomatic and 33% of symptomatic patients with bronchoalveolar lung cancer have brain metastasis. The overall histologic incidence in order of decreasing frequency is adenocarcinoma, small cell carcinoma, squamous cell carcinoma, and large cell carcinoma. Cranial resection is done first and thoracotomy thereafter, unless symptoms from the primary tumor site dominate the clinical picture. The overall survival with this combined approach is 37.5 at 24 months with complete resections. PRIMARY TUMOR (T) TIS: carcinoma in situ T1: Tumor <3 cm in greatest dimensions, surrounded by visceral pleura, without bronchoscopic evidence of invasion more proximal than the lobar bronchus T2: Tumor with any of the following features of size or extent: >3 cm, involves main bronchus >2 cm distal to the carina, invades the visceral pleura, associated with atelectasis or obstructive pneumonitis that extends to the hilar region but does not involve the entire lung T3: Tumor of any size that directly invades any of the following: chest wall, tumor in the main bronchus <2 cm distal to the carina but without involvement of the carina, or associated atelectasis or obstructive pneumonitis of the entire lung T4: Tumor of any size that invades any of the following: Mediastinum, heart, great vessels, trachea, esophagus, vertebral body, carina; or tumor with a malignant pleural effusion or pericardial effusion, or tumor with satellite tumor nodules within the ipsilateral primary-tumor lobe of the lung REGIONAL LYMPH NODES N0: No regional lymph node metastasis N1: Metastasis to ipsilateral peribronchial or ipsilateral hilar lymph nodes and intrapulmonary lymph nodes involved by direct extension of the primary tumor N2: Metastasis to ipsilateral mediastinal or subcarinal lymph node (s) N3: Metastasis to contralateral mediastinal, contralateral hilar, ipsilateral or contralateral scalene or supraclavicular lymph node (s) DISTANT METASTASIS M0: No distant metastasis detected M!: Distant metastasis present including a different lobe from primary-tumor lobe STAGES Stage IA T1N0MO IB T2NOMO Stage IIA T1N1MO IIB T2N1MO T3N0M0 Stage IIIA T3N1MO T3N2MO T2N2MO T1N2M0 Stage IIIB T4NOMO T4N3MO Stage IV T1-4N1-3M1
A 55-year-old female presents with a several-month history of increasing cough and dyspnea. She also has increased serum urea, nitrogen, and serum creatinine. A chest X-ray shows multiple bilateral small nodules. A renal biopsy shows a focal necrotizing vasculitis; her antineutrophil cytoplasmic autoantibody (ANCA) test is positive at 1:160. What additional finding would be most likely to occur? 1. Angina 2. Hemorrhagic pericarditis 3. Endocarditis 4. Hemoptysis 5. Hemothorax
Correct answer: Hemoptysis Explanation The history of upper airway involvement (cough with dyspnea), the nodular pattern on X-ray, and the renal biopsy showing focal necrotizing vasculitis suggests the diagnosis of Wegener's granulomatosis. Pulmonary (as well as upper respiratory tract) involvement with Wegener's granulomatosis commonly leads to hemorrhage, which manifests as hemoptysis. Although the etiology is unknown, it is believed to represent an antigen-triggered immunologic reaction. Pulmonary manifestations include cough, dyspnea, chest discomfort, and hemoptysis. In most cases, the extrarenal disease precedes the onset of renal disease. Although the heart may be involved with Wegener's granulomatosis, it does not usually lead to symptoms of ischemia (angina). The vasculitis does not typically involve the pericardium extensively; therefore, hemorrhagic pericarditis would not be a finding. The renal failure could lead to a fibrinous pericarditis. Endocarditis is not a feature of Wegener's granulomatosis. The vasculitis of Wegener's granulomatosis involves very small arteries and capillaries. Hemothorax is more typical of trauma.
You have just taken over the management of a 55-year-old man with COPD who was admitted 3 days earlier for community-acquired pneumonia. He currently feels somewhat better, and he has been afebrile for the last 24 hours. Prior to your discharging him, you decide to review his laboratory values, which are shown in the table. Leukocytes count: 5,600/μL Segmented neutrophils: 75% Hemoglobin: 19g/dL Platelets: 245,000/μL Arterial blood gas: pH 7.25 PCO2 55 PO2 57 HCO3 29 O2 saturation 88% Others are serum glucose 106 mg/dl, sodium 138mmol/L, chloride 102mmol/L, potassium 4.2mmol/L, bicarbonate 29mmol/L, BUN 18mmol/L, and creatinine 1.0mmol/L. What has been shown to improve life expectancy in a patient like this? 1. Antibiotics 2. Bronchodilator therapy 3. Home oxygen 4. Inhaled steroids 5. Acetazolamide
Correct answer: Home oxygen Explanation The only therapy that has been shown to decrease mortality and morbidity in COPD patients with chronic hypoxemia is oxygen therapy. It is indicated in patients with COPD with PO2 or O2 saturation of less than 55 and 88 respectively or PO2 between 56 to 59 and O2 saturation of 89 if there is evidence of cor pulmonale, signs of congestive heart failure, or hematocrit of 56 or more. The patient has COPD and a PO2 57. His polycythemia qualifies him for home O2 therapy. Antibiotics are only used in acute exacerbations to treat superimposed infections, especially when there is a change in sputum quantity or color. Its long-term use has not been shown to improve mortality or morbidity. Likewise, inhaled glucocorticoids and bronchodilators, such as β-agonist and theophylline, may provide symptomatic relief to these patients but do not improve life expectancy. Acetazolamide is a diuretic that is not indicated in the management of COPD.
A 27-year-old accident victim with a head injury is admitted to the ICU and kept on mechanical ventilatory support. On the 7th day after admission, he is clinically diagnosed with pneumonia. Blood samples and lower respiratory secretions are submitted to the laboratory for culture, and empiric antimicrobial therapy is started. What is the most likely etiologic agent of pneumonia in this patient? 1. Streptococcus pneumoniae 2. Klebsiella pneumoniae 3. Mycoplasma pneumoniae 4. Moraxella catarrhalis 5. Haemophilus influenzae
Correct answer: Klebsiella pneumoniae Explanation The correct response is Klebsiella pneumoniae. The patient has been under mechanical ventilation for >5 days when he develops pneumonia; therefore, he is having nosocomial late-onset ventilator-associated pneumonia (VAP). The main etiological agents of late-onset VAP consist of antibiotic-resistant exogenous organisms (e.g., multi-drug resistant Gram-negative bacilli and methicillin-resistant Staphylococcus aureus). Often more than 1 organism may be involved. The most common Gram-negative bacilli associated with late-onset VAP are Pseudomonas, Klebsiella, Enterobacter, Serratia, and Acinetobacter species. From the given list of bacteria, the most likely etiological agent of pneumonia in this patient is Klebsiella pneumoniae. Multi-drug resistant strains of Klebsiella pneumoniae occur in the hospital environment and are known to be important agents of nosocomial infections, including late-onset VAP. Streptococcus pneumoniae, Haemophilus influenzae, and Moraxella catarrhalis are generally associated with community-acquired pneumonia; however, colonization and infection by these endogenous bacteria can occur within the initial 4 - 5 days of ventilatory support (resulting in early-onset VAP). Mycoplasma pneumoniae is an important agent of community-acquired pneumonia. Recently, the possible involvement of this organism in early-onset VAP has been suggested.
A 69-year-old man presents with a 7 - 10 day history of increasing dyspnea and inspiratory chest pain. He has a 2-pack-per-day smoking history and abuses alcohol. He has hypertension, diabetes, coronary artery disease, and chronic kidney disease. Upon further questioning, you discover that he was discharged from the hospital 2.5 weeks ago; he was diagnosed with congestive heart failure after presenting with similar symptoms. The patient's breathing appears labored, and you note diminished breath sounds on auscultation and dullness to percussion in the lower half of the lung fields bilaterally. A chest X-ray reveals bilateral moderate pleural effusions. Thoracentesis reveals pleural fluid with the following characteristics: (1) turbid in appearance; (2) 2500 white blood cells/microliter; (3) glucose equal to serum levels; (4) ratio of pleural fluid protein to serum protein of 0.75; and (5) ratio of pleural fluid LDH to serum LDH of 0.72. Based on the information above, what is the most likely etiology of the pleural effusions? 1. Congestive Heart Failure 2. Nephrotic Syndrome 3. Cirrhosis 4. Malignancy 5. Bacterial pneumonia
Correct answer: Malignancy Explanation Malignancy is the correct response. A pleural effusion is an abnormal accumulation of fluid in the pleural space. Patients with pleural effusions most often report dyspnea, cough, and pleuritic chest pain. Large pleural effusions are more likely to be symptomatic than smaller effusions. Physical findings are usually absent in small effusions. Larger effusions may present with dullness to percussion and diminished or absent breath sounds over the affected area. A diagnostic thoracentesis should be performed whenever there is a new pleural effusion and no clinically apparent cause, when there is an atypical presentation, or when an effusion fails to resolve as expected. Sampling allows visualization of the fluid in addition to chemical and microbiologic analyses to help identify the etiology of the effusion. Pleural samples should be sent for measurement of protein, glucose, and LDH, in addition to total and differential white blood cell counts. These chemistry tests are used to classify effusions as a transudate or an exudate. This distinction is important because the differential diagnosis for each is entirely different. A pleural exudate is an effusion that has 1 or more of the following laboratory features: (1) ratio of pleural fluid protein to serum protein > 0.5; (2) ratio of pleural fluid LDH to serum LDH > 0.6; and (3) pleural fluid LDH greater than 2/3 the upper limit of normal serum LDH. Additionally, samples with low glucose levels usually indicate a bacterial or significant inflammatory etiology. In contrast, transudative effusions have none of these features. Transudates are also distinguished by fewer than 1000 white blood cells/microliter and a pleural glucose level equal to serum. These types of effusions occur in the setting of normal capillary integrity and also suggest the absence of local pleural disease. The characteristics of the pleural fluid from the patient in the clinical scenario presented above indicate an exudative pleural effusion. Malignancy is the most likely etiology in this patient, as a bacterial cause would typically present with a decreased glucose level in the pleural fluid compared with the serum; there is an equal glucose level in this case. Congestive heart failure accounts for >90% of transudative pleural effusions. Nephrotic syndrome and cirrhosis with ascites can also lead to transudative pleural effusions. Bacterial pneumonia and cancer are the most common causes of exudative effusions.
A 16-year-old boy has started smoking cigarettes. His father insists that you explain the dangers of smoking and the harmful effects that it can have to his body. You discuss cancer, heart disease, respiratory disease, and peripheral vascular disease. You then comment that 1 of the findings in smokers is the replacement of 1 adult tissue type with another adult tissue type within the respiratory tubes. What is this phenomenon called? 1. Metaplasia 2. Hyperplasia 3. Hypertrophy 4. Dysplasia 5. Anaplasia
Correct answer: Metaplasia Metaplasia is the replacement of 1 adult tissue type with another adult tissue type. In this scenario, the doctor was referring to the replacement of pseudostratified ciliated columnar epithelium with squamous epithelium. This would be called "squamous metaplasia". Hyperplasia is an increase in the number of cells in the tissue. Hypertrophy is an increase in the organ size. Hypertrophy is not due to an increase in the number of cells; instead, it is due to an increase in the size of the cells. Dysplasia is a disorderly arrangement to the tissue. Anaplasia is characteristic of malignancy. It refers to cancerous cells being undifferentiated.
An 18-year-old man has been diagnosed with asthma since he was 12 years old. The patient states he has been controlled using his rescue inhaler but for some reason his allergies this year are really acting up. You ask him how many times a week he has been using his inhaler. The patient responds "4 times a week." He also wakes up multiple times a month due to shortness of breath. He wheezes on a regular basis. His FEV1 is above 80% predicted. You determine you need to add additional therapy at this time. Based on the stepwise approach for asthma, what should you prescribe? 1. Mometasone furoate 2. Advair 3. Serevent Diskus 4. Omalizumab 5. Prednisone
Correct answer: Mometasone furoate Explanation Momemtasone furoate is an inhaled low-dose corticosteroid, which is the preferred treatment for mild persistent asthma. This is a daily maintenance treatment that can be used up to twice a day to help control symptoms of asthma. Advair is a combined low-dose inhaled corticosteroid and long acting beta agonist. This would be used if therapy has not worked with a low-dose inhaled corticosteroid alone or for moderate persistent asthma. Serevent Diskus is a long-acting beta agonist. This would be used in addition to an inhaled corticosteroid. This would be used if therapy has not worked with a low-dose inhaled corticosteroid alone or for moderate persistent asthma. Omalizumab is used for severe asthma and reserved for those that have failed traditional therapies. This inhibits IgE binding to mast cells and basophils. Therapy cost can range from $6,000-$24,000 dollars a year. Oral prednisone is also used to treat severe asthma symptoms. Long term use is not recommended due to prolong osteoporosis, dermatologic changes, and adrenocortical and pituitary unresponsiveness.
A 44-year-old healthy man, who is a non-smoker, has a 3-cm coin lesion in the right upper lobe. The lesion was revealed on his chest radiograph. The patient did not have any clinical symptoms. The fine-needle aspiration did not reveal any cells. What is the most likely tumor in this case? 1. Pulmonary hamartoma 2. Bronchial carcinoid 3. Mesothelioma 4. Metastatic adenocarcinoma 5. Large cell undifferentiated carcinoma
Correct answer: Pulmonary hamartoma Explanation A peripheral 'coin lesion' could be a granuloma, carcinoma, or a hamartoma. The most likely malignancy in a non-smoker would be adenocarcinoma. Pulmonary hamartoma is the most common benign lung tumor, although it is a relatively uncommon lesion. It is likely an acquired lesion from peribronchial mesenchyme. The patients are generally asymptomatic. The lesion is almost always solitary and is usually peripheral; hence, it does not reveal any cells on fine-needle aspiration. Histologically, hamartomas are composed of both mesenchymal and epithelial elements. A bronchial carcinoid is (usually) a central benign tumor with some of the properties of malignant growth. Mesothelioma is a bulky pleural mass. It is rare, even in smokers with asbestos exposure. Metastatic adenocarcinoma usually produces multiple lung nodules. Large cell undifferentiated carcinoma is a large peripheral mass.
A 68-year-old man with a history of cirrhosis presents due to weight gain, increased girth, and shortness of breath. He denies fever or chills, cough, melena, hematemesis, hemoptysis, and confusion. He takes furosemide on a daily basis and has not missed any doses. On physical exam, his blood pressure is 120/80; pulse is 78, and respiratory rate is 18. He is alert and cooperative. Heart exam: regular rate and rhythm without murmurs. Lung exam: decreased breath sounds on right side. Abdominal exam: positive fluid wave, no tenderness to palpation. There is no hepatosplenomegaly appreciated. Chest X-ray shows a right-sided pleural effusion; it is free flowing, as is evidenced on a decubitus film. What is the etiology of his pleural effusion? 1. Altered permeability of pleural membrane 2. Reduced intravascular oncotic pressure 3. Increased hydrostatic pressure 4. Decreased lymphatic drainage 5. Reduced pressure in pleural space
Correct answer: Reduced intravascular oncotic pressure Explanation Pleural effusions, an abnormal collection of fluid in the pleural space, is related to an underlying pathology causing excess fluid production or decreased absorption. Effusions are classified as transudative or exudative depending on the characteristics of the fluid. Patients with liver disease/cirrhosis have decreased protein and albumin production, leading to decreased oncotic pressure of the intravascular fluid. This contributes to ascites and dependent edema. Peritoneal ascites may travel across the diaphragm and lead to pleural effusions (hepatic hydrothorax), most commonly on the right side. Pleural effusions caused by decreased oncotic pressure are transudative. Treatment is aimed at reducing the ascitic fluid using diuretics (loops and/or spironolactone). Severe cases may require treatment with a transjugular intrahepatic portosystemic shunt (TIPS) or liver transplant. Other causes of hypoalbuminemia, such as nephrotic syndrome, also cause pleural effusions due to reduced intravascular pressure. Altered permeability of the pleural membrane causing a pleural effusion is seen with localized inflammation, malignancy, or pulmonary embolism. Decreased lymphatic drainage is commonly seen in malignancy (lung, breast, or lymphoma). The effusion is exudative and almost always unilateral on the side with the associated pathology. Pleural effusions caused by congestive heart failure are due to increased hydrostatic pressure. They are usually bilateral and transudative. Reduced pressure in the pleural space prevents full lung expansion, allowing pleural fluid accumulation. Causes of this include atelectasis (transudative) or mesothelioma (exudative).
A 54-year-old Caucasian man presents with sudden severe shortness of breath. The patient is well known to you due to a 5-year history of severe emphysema. He states that he has been a basketball player all his life; he was practicing about 1 hour prior to presentation, at which point he experienced sudden chest pain and immediate shortness of breath. It is still bothering him. He describes the chest pain in the middle of the chest, more so on the right anterior side. The patient admits to smoking 2.5 packs of cigarettes daily. The patient has had at least 5 episodes similar to this one since he was 19 years old. Physical examination reveals a tall, thin, well-developed man in moderate distress. Other abnormalities include mild tachycardia (120 beats per minute) and diminished breath sounds in the posterior right lower lobe. There is noted wheezing and crackles heard throughout the rest of the lung fields in both the inspiratory and expiratory phases of breathing. Based upon the most likely diagnosis at this time, what pharmaceutical option may be used to help decrease this occurrence in the future? 1. Inhaled corticosteroids 2. Intravenous antibiotics 3. Talc sclerotherapy 4. Inhaled short-acting β2-agonist 5. Oral antibiotics
Correct answer: Talc sclerotherapy Explanation The correct response is Talc sclerotherapy. The clinical scenario is highly suggestive of a (recurrent) secondary pneumothorax. Components that lead to this diagnosis include unilateral, sudden chest pain with dyspnea and minimal physical exam findings, in this case the mild tachycardia and diminished breath sounds. The above scenario most likely fits what is considered a secondary pneumothorax, which usually occurs as a complication of various pathologies, including COPD, asthma, cystic fibrosis, tuberculosis, Pneumocystis pneumonia, and various interstitial lung diseases. Diagnosis is usually confirmed with chest radiograph. Treatment is typically observation and potential surgical intervention after about 5 days if the pneumothorax fails to resolve itself. However, the ability of the patient to withstand a surgical intervention also must be considered. There are various thought processes in terms of management of recurrent pneumothorax especially. The correct option out of the above is the talc sclerotherapy; this is commonly reserved for patients who may not necessarily be surgical candidates. Intravenous antibiotics would be correct if the pneumothorax were actually caused by the sequelae of staphylococcal pneumonia. Oral antibiotics, short-acting β2-agonist, and inhaled corticosteroids are inappropriate in the management of a secondary pneumothorax.
A 3-month-old male infant presents with history of noisy breathing since birth; the noise is gradually increasing. There is no history of fever, cough, or running nose. Physical examination reveals a low-pitched, inspiratory wheeze; it is more prominent over the central airways and loudest over the trachea. Wheezing increases during crying, feeding, and when the infant is laid in supine position. There is no cyanosis, subcostal or intercostal retraction, or hoarseness of voice. Wheezing has not shown any response to bronchodilators. 1. Bronchiolitis 2. Congenital subglottic stenosis 3. Tracheomalacia 4. Tracheoesophageal fistula 5. Vocal cord paralysis
Correct answer: Tracheomalacia Explanation Tracheomalacia is a common cause of persistent wheezing in early infancy, with male preponderance in the ratio of 2:1. In primary tracheomalacia, there is insufficient cartilage to maintain the patency of the airway throughout the respiratory cycle. It commonly occurs in premature infants. Secondary tracheomalacia occurs when trachea is compressed by structures like vascular rings or deficient cartilage due to tracheoesophageal fistula. The dominant finding is a monophonic, low-pitched wheeze most prominent over central airways. There is persistent wheeze even in the absence of any viral respiratory infection, and it is loudest over the trachea. Subcostal retractions are absent unless there is asthma or some other cause of small airway obstructions. Wheeze does not respond to bronchodilators. Diagnosis is confirmed by flexible or rigid bronchoscopy. Bronchiolitis is usually caused by viral infection. More than 50% of cases are caused by respiratory syncytial virus (RSV). Other viruses include parainfluenza virus, adenovirus, and mycoplasma. It commonly occurs in male infants who have not been breast-fed. The infant first develops a mild upper respiratory tract infection with sneezing and rhinorrhea, which is accompanied by fever up to 101 - 102 degrees Fahrenheit. Respiratory distress, cough, and wheezing appear gradually; symptoms may interfere with feeding. Apnea is more common in infants less than 2 months of age. The infant is tachypneic with nasal flaring and chest retractions. Auscultation may reveal fine crackles and wheeze. A trial with bronchodilators, albuterol, or epinephrine may be considered and continued only if there is a clinical improvement with epinephrine used in an inpatient setting. Congenital subglottic stenosis is the 3rd most common congenital anomaly of the larynx causing stridor, which is usually biphasic or primarily inspiratory. Recurrent or persistent croup occurs in these infants. The 1st symptom often occurs during an episode of respiratory infection, as edema and thick secretions cause narrowing of the already compromised airway. Esophageal atresia is a frequent congenital anomaly affecting 1/4000 neonates. Of these, over 90% have associated tracheoesophageal fistula. It typically manifests with frothing and bubbling at the mouth and nose and episodes of coughing, respiratory distress, choking, and cyanosis. These symptoms are exacerbated by feeding. Regurgitation of gastric contents through the distal fistula causes more damaging pneumonia than aspiration of pharyngeal secretions from the blind upper pouch of esophagus. Vocal cord paralysis is the 2nd most common cause of neonatal stridor. It is often associated with central nervous system lesions like meningomyelocele, Arnold-Chiari malformation, and hydrocephalus. Bilateral vocal cord paralysis typically produces airway obstruction manifested by high-pitched inspiratory stridor. Unilateral paralysis causes aspiration, coughing, and choking. Cry is weak. Other symptoms of airway obstruction are less common. Diagnosis of vocal cord paralysis is made by awake flexible laryngoscopy with ultrasonography, which is a useful adjunctive examination.
You are called to the emergency department at 2 P.M. to see a 44-year-old male patient. He is a 3-pack-a-day, unfiltered cigarette smoker with crushing chest pains. He has a wide-complex, rapid, regular tachyarrhythmia at 160 beats per minute. When you reach his examination room, you note his monitor also reveals evidence of "P" waves at 75 beats per minute. What type of rhythm do his symptoms show? Paroxysmal supraventricular tachycardia 2 Sinus tachycardia 3 Ventricular tachycardia 4. Ventricular fibrillation 5 Asystole
Correct answer: Ventricular tachycardia Explanation The observation of a wide-complex tachyarrhythmia at a rate of 160 beats per minute associated with P waves at a rate of 75 beats per minute represents a total dissociation (AV Dissociation) between the atria (upper portions of the heart) and the ventricles (lower portions of the heart). A prudent provider will recognize that, in the presence of a rapid, wide complex tachyarrhythmia, AV Dissociation is the classic criterion of ventricular tachycardia (V.T.). Having diagnosed ventricular tachycardia, you will endeavor to obtain a more complete history and reasonably complete medical physical examination. It is essential that you provide timely placement of this patient in a medical intensive care unit or a coronary care unit with placement on the critically ill list and advise out-of-town family to travel to visit him. Should the patient survive, then his healthcare providers will need to document his verbalized understanding of their medical professional advice in his chronological record of medical care, which includes the recommenadation that he stop smoking cigarettes immediately and abstains from smoking cigars and pipes as well as from chewing tobacco and dipping snuff. Paroxysmal supraventricular tachycardia (PSVT) is a narrow-complex tachyarrhythmia. Sinus tachycardia is also a narrow-complex tachyarrhythmia. Ventricular fibrillation (V.F.) is a very chaotic tachyarrhythmia with no readily discernible P waves and no readily discernible QRS complexes. Asystole represents a total absence of cardiac contractions and is seen as a flat line on the monitor. Atrial fibrillation is both irregular and narrow-complex. These answers, therefore, are incorrect.
A 58-year-old woman with a past medical history of hypertension, hyperlipidemia, breast cancer, hip fractures, and coronary artery disease is being evaluated for acute-onset, severe left-sided pleuritic chest pain over the course of the last 2 hours. The pain is associated with feelings of anxiety, hemoptysis, shortness of breath, and nausea. She "feels warm" but denies chills, palpitations, wheezing, cough, edema, vomiting, abdominal pain, abnormal bowel habits, or dietary intolerances. She endorses a 30 pack/year smoking history, but denies drug or alcohol use. Upon physical exam, she is found to be febrile, hypotensive, tachycardic, tachypnic, diaphoretic, and in acute painful distress. There is perioral cyanosis and a pleural friction rub to the left lung fields; the remainder of the exam is normal. What is the most appropriate therapeutic intervention for this patient at this time? 1. Indomethacin 2. Doxycycline 3. Heparin 4. Prednisolone 5. Albuterol
Correct answer: Heparin This patient's presentation is significant for a pulmonary embolism. Predisposing underlying conditions are almost always present; venous thrombosis may result from a generalized hypercoagulable state, venous endothelial injury, or local stasis (Virchow triad). Most commonly, the initial manifestations of pulmonary embolism include an abrupt dyspnea and chest pain. Tachycardia and hypoxia are the most common clinical signs. Associated manifestations include fever, hypotension, cyanosis, pleural friction rub, and findings consistent with pulmonary consolidation. Anticoagulation is the foundation for successful treatment of DVT and PE. Unfractionated heparin, low-molecular-weight heparin, or fondaparinux all achieve effective anticoagulation immediately.
A 15-year-old girl with a history of mild asthma has had worsening episodes of cough, wheezing, and increasing bloody sputum over the past 5 months. She denies any weight loss, decreased appetite, lethargy, or unusual travel. She has increased her bronchodilator use, but she has not sought further care until now. Her mother has also noted occasional facial flushing with sweating that sometimes appears when she feels stressed, but not always. They were of brief duration at first, but seem to be lasting longer now. On exam, her respiratory rate is 32 breaths/min. with obvious respiratory distress, temperature is 98.6, heart rate 84 bpm and BP 114/76, oxygen saturation is 94%. Her throat is clear and on auscultation, breath sounds over the left hemithorax are diminished without retractions or wheezes, there are few fine crackles at the base. Right side is clear. The remainder of the exam is normal. Chest X-ray reveals a round area of increased opacification near the right hilar region. Complete blood count shows a normal white count and differential. Based on this information, what is the most likely diagnosis? 1. Pulmonary embolism 2. Bacterial pneumonia 3. Bronchial carcinoid tumor 4. Pulmonary hemosiderosis 5. Unrecognized vascular malformation
Correct answer: Bronchial carcinoid tumor Bronchial carcinoid tumors, though rare in children, are the most common primary malignant lung tumor in children. They tend to arise in the perihilar region. Carcinoid tumors are rare neuroendocrine tumors occurring most often in the GI tract, especially the appendix. They can be associated with systemic symptoms due to the release of a variety of hormones. One of these hormones, serotonin, is thought to cause carcinoid syndrome. This consists of episodic flushing, wheezing and diarrhea, and it is noted in less than 1% of patients with bronchial carcinoid tumors. It is more common for midgut primary tumors. Patients may also present with recurrent pneumonia, cough, hemoptysis, wheezing, asthma, or chest pain. Metastasis, if present, usually occurs in mediastinal lymph nodes, the liver, bone, or skin. Resection is the preferred treatment for localized tumors; survival rates, when localized, are greater than 90%. Pulmonary embolism is extremely rare in pediatric patients, but it may be considered in a young female patient using oral contraceptives or in one who has had a recent abortion. It is also possible in a young male with recent leg trauma. Presenting symptoms include dyspnea, fever, pleuritic pain, cough, and hemoptysis. Bacterial pneumonia patients typically present with high temperatures and associated pleural effusions, along with a high percentage of band forms on lab study. Most commonly, it is secondary to acute viral bronchitis occurring during minor upper respiratory infection, or it is complicating an underlying chronic illness. Pulmonary hemosiderosis can occur as primary lung disease, or it can be secondary to cardiovascular or systemic disease. It is characterized by repeated episodes of intra-alveolar bleeding leading to abnormal accumulation of iron as hemosiderin in alveolar macrophages with subsequent fibrosis and severe anemia. Nonspecific recurrent or chronic pulmonary symptoms such as cough, dyspnea, tachypnea, and wheezing are observed most often. Hemoptysis may or may not occur in children. Vascular malformations include congenital structural anomalies between arteries and veins within blood vessels of the lungs, resulting in blood shunting and poor oxygenation. Many patients have no symptoms; others have dyspnea, shortness of breath with exertion, or bloody sputum. Long-term changes include clubbing, low oxygen saturation, and elevated hematocrit. Chest X-ray will usually reveal abnormal blood vessels. In addition, a murmur may be heard on auscultation.